MCAT- Biological and Biochemical Foundations of Systems

Réussis tes devoirs et examens dès maintenant avec Quizwiz!

At which pH can protein A best be obtained through electrophoresis? Protein. pI MM A 4.5 25,000 B 6.0 10,000 C 9.5 12,000 A. 2.5 B. 3.5 C. 4.5 D. 5.5

D. In most electrophoresis experiments, we attempt to separate out one component from the others. Because we are attempting to isolate protein A only a pH that causes protein A to be negative while proteins B and C are neutral or positive will be best.

What are the major waste products excreted in urine?

Dump the HUNK H+ Urea NH3 K+

Glycine: 3 letter abbreviation, 1 letter abbreviation, polar or non polar

Gly. G. Non-polar

Which of the following factors determine an enzyme's specificity? A. the 3D shape fo the active site B. The Michaelis constant C. the type of cofactor required for the enzyme to be active D. the prosthetic group on the enzyme

A

[S] (mM). (v mol/s) 0.01. 1 0.05 9.1 0.1 17 0.5 50 1 67 5 91 10 95 50 99 100 100 Km for this enzyme is approx: A. 0.5 B. 1 C. 10 D. 50

A

Leucine: 3 letter abbreviation, 1 letter abbreviation, polar or non polar

Leu. L. Non-polar

nonsense mutation

A point mutation that changes an amino acid codon to one of the three stop codons, resulting in a shorter and usually nonfunctional protein.

What is irreversible inhibition?

1) Something covalently bonds to active site. 2) Cannot be removed. A few bind non-covalently.

What is binary fission?

A form of asexual reproduction seen in prokaryotes in which the parent divides into two approximately equal parts. The circular chromosome attaches to the cell wall and replicates while the cell continues to grow in size. Eventually, the plasma membrane and cell wall begin to grow inward along the midline of the cell to produce two identical daughter cells.

Which of the following methods would be best to separate large quantities of the following proteins? Protein pI MM A 6.5 28,000 B 6.3 70,000 C 6.6 200,000 A. Ion-exclusion chromatography B. Size-exclusion chromatography C. Isoelectric focusing D. Native PAGE

B

Which of the following is true about glycerophospholipids? A. Glycerophospholipids can sometimes be sphingolipids, depending on the bonds in their head groups. B. Glycerophospholipids are merely a subset of phospholipids C. Glycerophospholipids are used in ABO blood typing system D. Glycerophospholipids have one glycerol, one polar head group, and one fatty acid tail

B Glycerophospholipids are a subset of phospholipids, as are sphingomyelins. They are never sphingolipids because they contain a glycerol backbone (rather than a sphingosine or a sphingoid backbone). Sphingolipids are used in the ABO blood typing system Glycerophospholipids have a polar head group, glycerol, and two fatty acid tails

Iodine deficiency may result in: A. galactorrhea B. cretinism C. gigantism D. hyperthyroidism

B Inflammation of the thyroid or iodine deficiency can cause hypothyroidism. Hypothyroidism in newborns can cause cretinism, characterized by poor neurological and physical development.

How does the gel for isoelectric focusing differ from the gel for traditional electrophoresis? A. Isoelectric focusing uses a gel with much larger pore sizes to allow fro complete migration B. isoelectric focusing uses a gel with SDS added to encourage a uniform negative charge C. isoelectric focusing uses a gel with a pH gradient that encourages a variable charge D. The gel is unchanged in isoelectrical focusing: the protein mixture is treated before loading.

C

A patient has a very high TSH level. Which of the following would NOT cause a high TSH level? A. Autoimmune destruction of thyroid cells that produce T3 and T4 B. A tumor in the hypothalamus that secretes high levels of TRH C. High levels of T4 from thyroid replacement medications D. Cancerous growth of parafollicular cells in the thyroid, destroying other cell types in the organ

C High levels of T4 would cause too much negative feedback and lower TSH levels

Which of the following hormones increases feeding behavior? A. Leptin B. Cholecystokinin C. Ghrelin D. Gastrin

C ghrelin promotes a sensation of hunger, increasing feeding behavior. Both leptin and cholecystokinin promote satiety decreasing feeding behavior-eliminating A and B. Gastrin increases acid production and gastric motility, but does not have any significant relationship with feeding behavior

The conversion of ATP to cyclic AMP and inorganic phosphate is most likely catalyzed by which class of enzyme? Ligase Hydrolase Lyase Transferase

C lyases are responsible for the breakdown of a single molecule into 2 molecules without water or transfer of electrons. they often form cyclic compounds or double bonds in the products to accommodate this. water was not a reactant, no cofactor was mentioned, so C is best choice

What is the main function of the nucleolus? A. ribosomal RNA synthesis B. DNA replication D. cell division D. chromosome assembly

A. the nucleolus is a dense structure where rRNA is synthesized

Which of the following is true regarding pancreatic somatostatin? A. Its secretion is increased by low blood glucose B. It is always inhibitory C. It is regulated by cortisol levels D. It stimulates insulin and glucagon secretion

B

What is the function of the loop of Henle?

Filtrate from the proximal convoluted tubule enters the descending loop of Henle, which dives deep into the medulla before turning around to become the ascending limb of the loop of Henle. The descending limb is permeable only to water and the medulla has an ever-increasing osmolarity as the descending limb travels deeper into it. As the descending limb traverses deeper into the medulla, the increasing interstitial [ ] favors the outflow of water from the descending limb, which is reabsorbed into the vasa recta. The kidney is capable of altering the osmolarity of the interstitium. This creates a gradient that, coupled with selective permeability of the nephron, allows maximal reabsorption and conservation of water. In the normal physiological state, the osmolarity in the cortex is approximately the same as that of the blood and remains at that level. Deeper in the medulla, the osmolarity in the interstitium can range from isotonic with blood (when trying to excrete water) to four times as [ ]ed (when trying to conserve water). If the [ ] is the same as the tubule and in the interstitium, there is no driving force and the water will be lost in urine. If the interstitium is more [ ]ed, then water will move out of the tubule, into the interstitium, and eventually back into the blood. As the descending limb transitions to become the ascending limb of the loop of Henle, a change in permeability occurs. The ascending limb is only permeable to salts and is impermeable to water. So while the descending limb maximizes water reabsorption by taking advantage of increasing medullary osmolarity, the ascending limb maximizes salt reabsorption by taking advantage of decreasing medullary osmolarity. At the transition from the inner to outer medulla, the loop of Henle becomes thicker in what is called the diluting segment. This is because the cells lining the tube are larger, not because the lumen within the tube has enlarged. These cells contain large amounts of mitochondria, which allow the reabsorption of sodium and chloride by active transport. Indeed, because so much salt is reabsorbed while water is stuck in the nephron, the filtrate actually becomes hypotonic compared to the interstitium. While we tend to focus on the concentrating abilities of the nephron, this segment is noteworthy because it is the only portion of the nephron that can produce urine that is more dilute than the blood. at the beginning of the loop of Henle, the filtrate is isotonic to the interstitium. Thus, from the beginning of the loop of Henle to the end, there is a slight degree of dilution. Far more important, however, is the fact that the volume of the filtrate has been significantly reduced, demonstrating a net reabsoption of a large volume of water.

The most important glucocorticoid in humans is cortisol. Cortisol has pleiotropic effects across cardiovascular, metabolic, immunologic and homeostatic systems. All of the following are likely consequences of an exogenous administration of cortisol EXCEPT: A. Stimulation of gluconeogenesis B. Inhibition of glucose uptake in adipose tissue C. Down-regulation of anti-inflammatory proteins D. Increased arousal during periods of moderate stress

For the MCAT, you only need to remember a few basic ideas about cortisol: it's the "stress hormone", it raises blood sugar, and it suppresses the immune system. Here, (C) is false because cortisol would up-regulate anti-inflammatory agent in order to suppress the immune system. Since this is an EXCEPT question, (C) is the right answer. A: Stimulating gluconeogenesis is one way that cortisol helps raise blood sugar. B: Preventing adipose tissue from pulling glucose out of the blood is one way that cortisol helps to raise blood sugar. D: In response to moderate stress, moderate levels of cortisol help cognition by increasing arousal levels, vigilance levels, and memory formation.

Explain basically what causes HIV/AIDS.

The loss of these helper T cells occurs in HIV infection, prevents the immune system from mounting an adequate response to infection; in advanced HIV infection, also called AIDS, even weak pathogens can cause devastating consequences as opportunistic infections.

What causes a left and right shift in the oxyhemoglobin dissociation curve?

Right (exercise): exercise is the right thing to do - Inc CO2 -Inc H+ (dec pH) - Inc temp -Inc 2,3 BPG Left -dec CO2 - dec H+ (inc pH) -dec temp -dec 2,3-BPG -presence of fetal hemoglobin

Surrounding tissues get a developing cell to become a particular type via __________. The cell that is acted on is called a __________; to be acted on, the cell must be _________ (or be able to do what).

Surrounding tissues induce a developing cell to become a particular cell type via inducers; the term inducer may also refer to the cell secreting the signal. The cell that I induced is called a responder; to be induced, a responder must be competent, or able to respond to the inducing signal.

what are enzyme-linked receptors? what are the three primary protein domains? What is their function?

membrane receptors that display catalytic activity in response to ligand binding a membrane-spanning domain, a ligand-binding domain, and a catalytic domain. The membrane-spanning domain anchors the receptor in the cell membrane. The ligand-binding domain is stimulated by the appropriate ligand and induces a conformational change that activates the catalytic domain. This often results in the initiation of a second messenger cascade.

what are Merkel cells (discs)?

sensory receptors present at the epidermal-dermal junction. Connected to sensory neurons and are responsible for deep pressure and texture sensation within the skin.

What is SDS-Page?

separates proteins on the basis of relative molecular mass alone. The SDS-PAGE technique starts with the premise of PAGE but adds SDS, a detergent that disrupts all non covalent interactions. It binds to proteins and creates large chains with net negative charges, thereby neutralizing the protein's original charge and denaturing the protein.

speciation

Formation of new species through evolution

What is Mendel's first law of segregation? What proves it?

also known as the law of segregation. the alleles of a given locus segregate into separate gametes. proven by disjunction. During disjunction, each chromosome of paternal origin separates from its homologue of maternal origin, and either chromosome can end up in either daughter cell. Thus, the distribution of homologous chromosomes to the two intermediate daughter cells is random with respect to parental origin. This separating of the two homologous chromosomes is referred to as segregation.

What is the G1/S checkpoint? What is the main control of this?

also known as the restriction point. it controls whether the cell will go from G1 to S phase in the cell cycle, it also checks to make sure all the requirements have been met (are all its DNA molecules intact? are there enough nutrients? is the cell big enough? has the cell been stimulated by growth factors?) The main protein in control of this is p53

peptides are composed of what?

amino acid subunits, sometimes called residues

which amino acids are hydrophilic?

amino acids with charged side chains: histidine, arginine ,lysine, glutamate and aspartate, as well as the amides: asparginine and glutamine

What is the difference between an endergonic and exergonic reaction?

an endergonic reaction is one that requires energy input (delta G > 0) an exergonic reaction is one in which energy is given out (delta G < 0)

How does the hypothalamus control the anterior pituitary? Vague. How is the release of hormones by the hypothalamus regulated?

because they're so close to each other, the hypothalamus exerts control through paracrine release of hormones into a portal system (the hypophyseal portal system). negative feedback

Bone's characteristic strength comes from ______ bone. The other type of bone structure is what type of bone? The ____ structure of this bone is visible under microscopy and consists of what? The cavities between these are filled with what?

compact bond. The other type is spongy or cancellous bone. It consists of bony spicules (points) known as trabeculae. The cavities between trabeculae are filled with bone marrow, which may be either red or yello.

What are amino-acid derivative hormones? Exs?

epi, norepi, triiodothyronine, thyroxine. they are derived from 1 or 2 animo acids, usually with a few additional modifications. The chemistry of this family of hormones is considerably less predictable and is one of the few instances where overt memorization may be the best strategy.

What is insulin excess called and what are the downstream effects?

hypoglycemia, characterized by low blood glucose [ ].

Is ATP a low, mid, or high-level energy carrier? Why is this?

mid-level think about your wallet. if you never had the ability to get change back after a purchase, what type of bill would you want in abundance? one dollar bills, duh. ATP cannot get back the "leftover" free energy after a reaction, so its best to use a carrier with a smaller free energy.

what is a niche

a specific environment, including habitat, available resources, and predators, for which a species is specifically afapted.

What is a mental set?

a tendency to approach a problem in one particular way, often a way that has been successful in the past

what is an adductor vs an abductor?

an abductor moves a part of the body away from the midlin. an adductor moves a part of the body towards the midline.

all steroids are derived from what?

cholesterol

Bone is _____ tissue derived from what?

connective tissue derived from embryonic mesoderm

What are the 2 molecules responsible for the cell cycle? Explain them.

cyclins and cyclin-dependent kinases (CDK). CDKs require the presence of the right cyclins to be activated. During the cell cycle, concentrations of the various cyclins increase and decrease during specific stages. These cyclins bind to CDKs, creating an activated CDK-cyclin complex. This complex can then phosphorylate transcription factors, which promote transcription of genes required for the next stage of the cell cycle

how are fingernails and hair formed?

from keratin and are produced by specialized cells in the skin

thermoregulation is achieved by what 4 things?

sweating, piloerection, vasodilation, and vasoconstriction

how does food leave the stomach?

through the pyloric sphincter and enters the duodenum.

Explain triacylglycerol (triglyceride) synthesis

triacylglycerols, the storage form of fatty acids, are formed by attaching three fatty acids (as fatty acyl-CoA) to glycerol. Triacylglycerol formation from fatty acids and glycerol 3-phosphate occurs primarily in the liver and somewhat in adipose tissue, with a small contribution directly from the diet as well. In the liver, triacylglycerols are packaged and sent to adipose tissue as VLDLs, leaving only a small amount of stored triacylglycerols.

What happens during the second trimester of pregnancy?

The fetus grows a ton. - It begins to move within the amniotic fluid, its face takes on a human appearance and its toes and fingers elongate. - By the end of the 6th month the fetus is 30-36 cm long

Aspartic acid: 3 letter abbreviation, 1 letter abbreviation

Asp. D

What in gene linkage?

The tendency for genes to be inherited together. genes that are located farther from each other physically are less likely to be inherited together, and more likely to undergo crossing over

Explain metaphase I.

2nd phase of Meiosis homologous pairs (tetrads) align at the metaphase plate, and each pair attaches to a separate spindle fiber by its kinetochore.

Which of these amino acids has a side chain that can become ionized in cells? A. histidine B. leucine C. proline D. threonine

A. histidines imidazole ring has a nitrogen atom that can be protonated.

Alanine: 3 letter abbreviation, 1 letter abbreviation, polar or non polar

Ala, A, nonpolar

At pH 7, the charge on a glutamic acid molecules is: A. -2 B. -1 C. 0 D. +1

B

What are the irreversible steps of glycolysis?

How Glycolysis Pushes Forward the Process: Kinases Hexokinase Glucokinase PFK-1 Pyruvate Kinase

What is the basic pathway of sperm through the male reproduction system?

SEVE(N) UP Seminiferous tubules Epididymis Vas deferens (also called the ductus deferent) Ejaculatory duct (Nothing) Urethra Penis

What are the other names for the citric acid cycle? Where does it occur? What is its main function?

The Krebs cycle or the tricarboxylic acid (TCA) cycle Occurs in the mitochondria. Main function is the oxidation of acetyl-CoA to CO2 and H2O. In addition, the cycle produces the high-energy electron-carrying molecules NADH and FADH2.

what is transcription?

The creating of mRNA from a DNA template

Explain step 7 of the citric acid cycle, Malate formation.

The enzyme fumarase catalyzes the hydrolysis of the alkene bond in fumarate, thereby giving rise to malate. Although two enantiomeric forms are possible, only L-malate forms in this reaction.

What is a Bradford protein assay?

a means of determining the concentration of ONE SPECIFIC type of protein by looking at the way a blue dye binds to the molecules.

What is cardiac output? How do you calculate it?

cardiac output is the total blood volume pumped by one ventricle in a minute. CO=HR x SV SV=stroke volume-volume of blood pumped/beat for humans, CO is 5L/min

explain competitive inhibition how can it be overcome does it alter Vmax? does it alter Km?

competitive inhibitors bind to the active site. b/c the active site is occupied, so substrates can't access enzymatic binding sites can be overcome by adding more substrate so that the substrate-to-inhibitor ration is higher adding a competitive inhibitor does not alter value of Vmax b/c if enough substrate is added, it will outcompete the inhibitor and be able to run the reaction at max velocity does inc Km. because the substrate [ ] has to be higher to reach half the max velocity in the presence of the inhibitor

what are the four types of reversible inhibition in enzymes?

competitive, noncompetitive, mixed, uncompetitive comp and noncomp are most encountered on MCAT

Lysosomes often function in conjunction with ______, which do what?

endosomes, which transport, package, and sort cell material traveling to and from the membrane.

with the exception of the three portal systems, all systemic capillary beds are what? meaning what?

in parallel with each other. Therefore, opening capillary beds will decrease vascular resistance and assuming the body can compensate, increase cardiac output.

What are frameshift mutations. what are the subcategories?

insertions or deletions of base pairs that cause the whole later sequence to be off, shifted by one or more. Easier to catch than substitutions. insertion or deletion mutations.

How is the inner mitochondrial membrane assembled?

into folds called christae, which maximize surface area

Explain heroin

opioid. Once injected, the body rapidly metabolized heroin to morphine.

prosthetic groups can be what types of things?

organic molecules like vitamins or metal ions like iron

What is native PAGE?

polyacrylamide gel electrophoresis (PAGE) is a method for analyzing proteins in their native states. useful to compare the molecular size or the charge of proteins known to be similar in size

Proteins are made up of what?

polypeptides that range from a few to thousands of amino acids

Lipids are the major component of what?

the phospholipid bilayer

species

the largest group of organisms capable of breeding to form fertile offspring

in alpha (a) amino acids, are the amino and carboxyl group are bonded to the same carbon or opposite carbons?

the same carbon

how do enzymes catalyze peptide hydrolysis?

they break apart the amide bond by adding a H atom to the amide nitrogen and an OH group to the carbonyl carbon.

What are the 3 recombination processes for bacteria genetic recombination?

transformation, conjugation, and transduction

What is the base rate fallacy?

using prototypical or stereotypical factors while ignoring actual numerical information

what is expressivity? explain constant vs variable expressivity

varying phenotypes despite identical genotypes. if expressivity is constant then all individuals with a given genotype express the same phenotype. However, if expressivity is variable, then individuals with the same genotype may have different phenotypes.

Explain the structure of a male sperm

very compact. They have a head (containing genetic material), a mid piece (which generates ATP from fructose), and a flagellum (for motility) -the mid piece is filled with mitochondria, which generate energy for swimming through the female reproductive tract -each sperm head is covered by a cap called an across. This structure is derived from the Golgi apparatus and is necessary to penetrate the ovum. ]"

Explain Oxidoreductases

one of the 6 classes of enzymes. they catalyze oxidation-reduction reactions (the transfer of electrons between molecules). They often have a cofactor that acts as an electron carrier, such as NAD+ or NADP+. In reactions paralyzed by oxidoreductases, the electron donor is the reductant, the electron acceptor is the oxidant . Enzymes with dehydrogenase or reductase in their names are usually oxidoreductases Enzymes in which oxygen is the final electron acceptor often include oxidase in their names

What are the three stages a cell goes through to specialize?

specification, determination, and differentiation

What are trioses, tetroses, pentoses, and hexoses?

monosaccharides (simplest carbohydrate) containing 3, 4, 5, and 6 carbon atoms, respectively

Phospholipids with unsaturated fatty acid tails make up what kind of regions of the phospholipid bilayer?

more fluid regions unsaturated fatty acids have one or more double bonds. Double bonds introduce kinks into the fatty acid chain, which makes it difficulty for them to stack and solidify. Therefore , they are usually liquid at room temp.

What is the most prominent type of protein found in the immune system? What do they do?

the antibody, also called immunoglobulins (Ig) they are proteins produced by B-cells that function to neutralize targets in the body such as toxins and bacteria and then recruit other cells to help eliminate the threat.

How does size exclusion chromatography work?

the beads used in the column contain tiny pores of varying sizes. These tiny pores allow small compounds to enter the beads, thus slowing them down.

peptides are joined by what? which forms between what 2 things?

peptide bonds, which forms between the -COO- group of one amino acid and the NH3+ group of another amino acid

What is delirium?

rapid fluctuation in cognitive function that is reversible and caused by medical (nonphysiological) causes ex: electrolyte and pH disturbances, malnutrition, alcohol withdrawal, pain, etc

What is tautomerization?

rearrangement of bonds in a compound, while preserving the molecular formula (usually by moving a hydrogen and forming a double bond)

What are the two types of bone marrow and what are the differences?

red or yellow marrow. Red marrow is filled with hematopoietic stem cells, which are responsible for the generation of all the cells in our blood; yellow marrow is composed primarily of fat and is relatively inactive.

Explain depressants

reduce nervous system activity, resulting in a sense of relaxation and reduced anxiety. Alcohol is the most common.

What type of replication does DNA undergo? Explain what this means.

semi-conservative replication parental strands serve as templates for the generation of new daughter strands. The replication process is termed semiconservative because one parental strand is retained in each of the two resulting identical double-stranded DNA molecules.

Explain B-DNA vs Z-DNA. which is more common?

the double helix of most DNA is a right-handed helix, forming B-DNA Z-DNA is a left handed helix. no biological activity has been attributed to Z-DNA because it is unstable and difficult to research

What is saponification? explain the chemical process.

the ester hydrolysis of triacylglycerols using a strong base traditionally, the base used is lye, the common name for NaOH or KOH. The result is the basic cleavage of the fatty acid, leaving the sodium salt fo the fatty acid and glycerol. The fatty acid salt is soap.

insertion mutation

occurs when a segment of DNA is moved from one chromosome to another. small insertion mutations (including those where the inserted DNA is not from another chromosome) are considered frameshift mutations.

What are the gonads in females? What hormones do they produce? Where are the gonads located? Each gonad consists of thousands of what? What are these things?

ovaries- estrogen and progesterone. located in the pelvic cavity. each consists of thousand of follicles, which are multilayered sacs that contain, nourish, and protect immature ova (eggs).

What is parathyroid hormone and how does it affect bone metabolism?

parathyroid hormone is a peptide hormone released by the parathyroid glands in response to low blood calcium, it promotes resorption of bone, increasing the [ ] of calcium and phosphate in the blood.

What are thrombocytes and what do they do?

platelets. they are cell fragments or shards released from cells in bone marrow known as megakaryocytes. Their function is to assist I blood clotting and they are present in high [ ]s.

What are the two types of point mutations? explain each. what is another name for point mutations?

point mutations also called expressed mutations because they can affect the primary amino acid sequence of the protein. Missense mutation- a mutation where one amino acid substitutes for another Nonsense mutation- a mutation where the codon now encodes for a premature stop codon (also called a truncation mutation)

What is a point mutation? What the subcategories?

point mutations occur when one nucleotide in DNA is swapped for another. Silent, missense, or nonsense mutations.

A disease results in the death of Schwann cells. Which portion of the nervous system is NOT likely to be affected? A. CNS B. PNS C. Autonomic nervous system D. Parasympathetic nervous system

A

A stroke patient comprehends speech but cannot properly move her mouth to form words. Which of the following brain areas is likely affected? A. Broca's area B. Wernicke's area C. Arcuate fasciculus D. Superior temporal gyrus

A

At what point are two population descended from the same ancestral stock considered to be separate species? A. When they can no longer produce viable, fertile offspring B. When they look significantly different from each other C. When they can interbreed successfully and produce offspring D. When their habitats are separated by a significantly large distance so they cannot meet.

A

EEG waveforms during REM sleep most resemble which of the following states of consciousness? A. Alertness B. Slow-wave sleep C. Stage 1 sleep D. Meditation

A

Enzymes inc the rate of a reaction by: A. decreasing the activation energy B. decreasing the overall free energy change of the reaction C. inc the activation energy D. inc the overall free energy change of the reaction

A

Neurofibromatosis type 1 is a disorder that causes formation of tumors in multiple nervous system structures as well as the skin. While all cells carry the same mutation on chromosome 17, selective transcription of the genome appears to cause the most significant tumorigenesis in which of the following primary germ tissue layers? A. Ectoderm B. Mesoderm C. Endoderm D. Notochord

A

Explain the difference between autocrine, paracrine, juxtacrine, and endocrine signals.

Autocrine signals act on the same cell that secreted the signal in the first place. Paracrine signals act on cells in the local area. Juxtacrine signals do not usually involve diffusion, but involve a cell directly stimulating receptors of an adjacent cell. Endocrine signals involve secreted hormones that travel through the bloodstream to a distant target tissue.

A protein collected through affinity chromatography displays no activity even though it is found to have a high concentration using the Bradford protein assay. What best explains this finding? A. The bradford reagent was prepared incorrectly B. The active sit is occupied by a free ligand C. The protein is bound to the column D. The protein doe not catalyze the reaction of interest

B

During saponifocaiton: A. triacylglycerols undergo a condensation reaction B. triacylglycerols undergo ester hydrolysis C. fatty acid salts are produced using a strong acid D. fatty acid salts are bound to albumin

B

The pH inside the stomach is very closely regulated. Which of the following is a major function for the pH in the stomach? A. Neutralizing the acidity of the chyme entering it B. Denaturing proteins to allow for digestive processes C. Activating trypsinogen into the active form D. Establishing a proton gradient for ATP production

B A- the chyme is the substance that leaves the stomach, not enters it C-trysinogen is found in the small intestine, not the stomach D-A proton gradient is used for ATP production in the mitochondria, not the lumen of the stomach

where does B-oxidation of fatty acids occur within the cell? A. Cytosol B. Mitochondria C. Smooth ER D. Plasma membrane

B B-Oxidation occurs within the mitochondria, along with the electron transport chain. In contrast, fatty acid synthesis occurs in the cytosol. Fatty acyl carriers like the carnitine shuttle allows entry into the mitochondrion for breakdown.

Which complex does NOT contribute to the proton-motive force? A. Complex I B. Complex II C. Complex III D. Complex IV

B Complex II is the only complex of the ETC that does not contribute to the proton gradient. Complexes I and III each add 4 protons to the gradient; Complex IV adds two protons to the gradient

A patient has been exposed to a toxic compound that increases the permeability of mitochondrial membranes to protons. Which of the following metabolic changes would be expected in this patient? A. Increased ATP levels B. Increased oxygen utilization C. Increased ATP synthase activity D. Decreased pyruvate dehydrogenase activity

B The increased permeability of the inner mitochondrial membrane allows the proton-motive force to be dissipated through locations besides the F0 portion of ATP synthase. Therefore, ATP synthase is less active and is forming less ATP, invalidating A and C. The body will attempt to regenerate the proton-motive force by increasing fuel catabolism, eliminating D. This increase in fuel use requires more oxygen utilization in the electron transport chain.

Which of the following correctly pairs the stage of development of an egg cycle with the relevant point in a woman's life cycle? A. from birth to menarche- prophase II B. At ovulation- metaphase I C. at ovulation- metaphase II D. At fertilization- prophase II

C

Which of the following is NOT a difference that would allow one to distinguish a prokaryotic and eukaryotic cell? A. ribosomal subunit weight B. presence of a nucleus C. presence of a membrane on the outside surface of the cell D. presence of membrane-bound organelles

C

Which of the following is likely to be found in maternal blood during pregnancy? A. Immunoglobulins produced by the fetus B. Fetal hemoglobin released form fetal red blood cells C. Progesterone produced by placental cells D. carbon dioxide exhaled from fetal lungs

C

Which of the following is not a necessary condition for Hardy-Weinberg equilibrium? A. large population size B. no mutations C. monogamous mating partners D. no migration into or out of the population

C

When a sensory neuron receives a stimulus that brings it to threshold, it will do all of the following EXCEPT: A. become depolarized B. transduce the stimulus to an action potential C. inhibit the spread of the action potential to other sensory neurons D. cause the release of neurotransmitters onto cells in the CNS

C If the receptor is stimulated, it will promote the spread of the action potential to postsynaptic sensory neurons in the spinal cord, which can send the signal towards the brain.

Which of the following statements concerning growth hormone is NOT true? A. Overproduction of growth hormone in adults results in acromegaly B. It promotes growth of bone and muscle C. It is produced by the hypothalamus but secreted by the pituitary D. A childhood deficiency in growth hormone results in dwarfism

C GH is a direct hormone secreted by the ant pit. It GH is synthesized and secreted in the anterior pituitary, not the hypothalamus

What is the response of the immune system to downregulation of MHC molecules on somatic cells? A. B-cells are activated and antibodies are released B. T-cells are activated, resulting in a cytotoxic response C. Natural killer cells induce apoptosis of affected cells D. Macrophages engulf the pathogen and display its antigens

C Healthy cells exhibit MHC class I molecules. Natural killer cells monitor the expression of MHC molecules on the surface of cells. Viral infection and cancer often cause a reduction in the expression of MHC class I molecules on the cell surface. Natural killer cells detect this lack of MHC and induce apoptosis in the affected cells.

adding heat to a closed biological system will do all of the following except: A. increase the internal energy of the system B. increase the average of the vibrational, rotational, and translational energies C. cause the system to do work to maintain a fixed internal energy D. increase the enthalpy of the system

C In a closed biological system, enthalpy, heat, and internal energy are all directly related because there is no change in P or V. Because P and V are fixed, work cannot be done.

2,4-Dienoyl-CoA reductase is used in the oxidation of A. saturated fatty acids B. monounsaturated fatty acids C. polyunsaturated fatty acids D. cholesterol

C In order for the enzymes of fatty acid oxidation to operate, there can be, at most, one double bond in the area of enzyme activity, and it must be oriented between carbons 2 and 3. In order to accomplish this in monounsaturated fatty acids, an isomerase is employed. When there are multiple double bonds that fall within the enzymatic binding site, both an isomerase and 2,4-dienoyl-CoA reductase are required for the oxidative enzymes to act on the fatty acid. For this question, simply recognizing that dienoyl refers to having multiple double bonds is sufficient to arrive at the answer.

Which of the following correctly lists two organs in which proteins are digested? A. Mouth and stomach B. Stomach and large intestine C. Stomach and small intestine D. Small intestine and large intestine

C Protein digestion begins in the stomach, where pepsin (secreted as pepsinogen) hydrolyzes specific peptide bonds. Protein digestion continues in the small intestine as trypsin (secreted as trypsinogen), chymotrypsin (secreted as chymotrypsinogen), carboxypeptidases A and B (secreted as procarboxypeptidases A and B), aminopeptidase, and dipeptidases hydrolyze specific parts of the peptide. No protein digestion occurs in the mouth or large intestine.

A person has a heart attack that primarily affects the wall between the two ventricles. Which portion of the electrical conduction system is most likely affected? A. AV node B. SA node C. bundle of His D. left ventricular muscle

C The cardiac conduction system starts at the SA node, which is located near the top of the right atrium, and continues down to the AV node, which is located between the two AV valves. The bundle of His is located within the wall between the ventricles and is likely to be affected if the wall between the ventricles has been damaged by a heart attack. This may affect the left ventricle, but the elft ventricular muscle itself is not part of the cardiac conduction system

Hemoglobin's affinity for O2 A. increases in exercising muscle tissue B. decreases as blood PaCO2 decreases C. decreases as blood pH decreases D. is higher in maternal blood than in fetal blood

C according to the Bohr effect, decreasing the pH in the blood dec hemoglobin's affinity for O2. The affinity is generally lowered in exercising muscle to facilitate unloading of oxygen to tissues, eliminating A. A decrease in the PaCO2 would cause a decrease in [H+] or inc pH-which increases hemoglobin's affinity for O2. Finally, D is incorrect because hemoglobin's affinity for O2 is higher in fetal blood that in adult blood

Which theory of dreaming states that dreams and thoughts during wakeful periods use the same stream-of-consciousness system? A. Activation-synthesis theory B. Problem-solving theory C. Cognitive process theory D. Neurocognitive theory

C cognitive theorists proposed in the cognitive process dream theory that wakeful and dreaming states use the same mental systems within the brain, particularly stream-of-consciousness.

Which of the following is NOT a cell-cell junction in animals? A. desmosomes B. gap junctions C. plasmodesmata D. tight junctions

C plasmodesmata are cell-cell junctions found in plants

Which of the following RNA molecules or proteins is NOT found in the spliceosome during intron excision? A. snRNA B. hnRNA C. shRNA D. snRNPs

C shRNA (short hairpin RNA) is a useful biotechnology tool used in RNA interference. It is not, however, produced in the nucleus for use in the spliceosome. It targets mRNA to be degraded in the cytoplasm; it is not utilized in splicing of the hnRNA (heterogenous nuclear RNA). snRNA (small nuclear RNA) and snRNPs (small nuclear ribonucleoproteins), however, do not bind to the hnRNA to induce splicing.

A particular a-helix is known to cross the cell membrane. Which of these amino acids is most likely to be found in the transmembrane portion of the helix? A. glutamate B. lysine C. phenylalanine D. aspartate

C. An amino acid likely to be found in a transmembrane portion of an a-helix will be exposed to a hydrophobic environment, so we need an amino acid with a hydrophobic side chain

Consider a biochemical reaction A--> B, which is catalyzed by A-->B dehydrogenase. Which of the following statements is true? A. the reaction will proceed until the enzyme [ ] decreases B. the reaction will be most favorable at 0 degrees Celsius C. A component of the enzyme is transferred from A to B D. The free energy change (deltaG) of the catalyzed reaction is the same as for the uncatalyzed reaction

D

In the equation below, substrate C is an allosteric inhibitor to enzyme 1. Which of the following is another mechanism necessarily cause by substrate C? A ----------> B ----------> C (enzyme 1). (enzyme 2) A. competitive inhibition B. irreversible inhibition C. feedback enhancement D. negative feedback

D

Which of the following statements concerning peptide bonds is FALSE? A. Their formation involves a reaction between an amino group and a carboxyl group B. They are the primary bonds that hold amino acids together C. They have partial double bond character D. Their formation involves hydration reactions

D

Whihc of the following amino acids will provide the most energy when degraded? A. Glycine B. Alanine C. Valine D. Isoleucine

D The energy contribution of an amino acid depends on its ability to be turned into glucose through gluconeogenesis (glucogenic amino acids), ketone bodies (ketogenic amino acids), or both. All of the amino acids listed in the answer choices are glucogenic; isoleucine is also ketogenic. The energy acquired form an amino acid will also depend on the number of carbons it can donate to these energy-creating processes, which depends on the size of its side chain. Isoleucine has the largest side chain of the answer choices, and will thus contribute the most energy per molecule.

the majority of triacylglycerols stored in adipocytes originate from: A. synthesis in the adipocyte B. dietary intake C. ketone bondies D. synthesis in the liver

D The liver is the major metabolic organ in the body and is responsible for much of the synthesis and interconversion of fuel sources. Most of the triacylglycerols that are synthesized in the liver are transported as VLDL to adipose tissue for storage. Both the adipocytes and dietary intake constitute a minor source of triacylgylerols.

Which of the following tissues is most dependent on insulin? A. Active skeletal muscle B. Resting skeletal muscle C. Cardiac muscle D. Smooth muscle

B Adipose tissue and resting skeletal muscle require insulin for glucose uptake. active skeletal muscle uses creatine phosphate and glycogen (regulated by epi and AMP) to maintain its energy requirements

Arginine: 3 letter abbreviation, 1 letter abbreviation

Arg. R

In which neural structure are ribosomes primarily located? a) dendrites b) soma c) axon hillock d) axon

B

Which of the following best describes the structure of steroids? A. 3 cyclopentane rings, one cyclohexane ring B. 3 cyclohexane rings, one cyclopentane ring C. 4 carbon rings, differing in structure for each steroid D. 3 cyclic carbon rings and a functional group

B

Which of the following INCORRECTLY pairs a metabolic process with its site of occurrence? A. Glycolysis-cytosol B. Citric acid cycle-outer mitochondrial membrane C. ATP phosphorylation-cytosol and mitochondria D. Electron transport chain- inner mitochondrial membrane

B The citric acid cycle takes place in the mitochondrial matrix, not the outer mitochondrial membrane. WHile most citric acid cycle enzymes are located within the matrix, succinate dehydrogenase is located on the inner mitochondrial membrane.

In an experiment, enteropeptidase secretion was blocked. As a direct result, levels of all of the following active enzymes would likely be affected EXCEPT: A. trypsin B. aminopeptidase C. chymotrypsin D. carboxypeptidase A

B aminopeptidase is a brush-border peptidase secreted by the cells lining the duodenum; it does not require enteropeptidase for activation. Both trypsinogen and procarboxypeptidases A and B are activated by enteropeptidase, eliminating A and D. Once activated, trypsin can activate chymotrypsinogen; if trypsinogen cannot be activated, then chymotrypsinogen will not be activated either.

Which of the following associations correctly matches a gastric cell with a compound it secretes? A. G-cells ---HCl B. Chief cells --- pepsinogen C. Parietal cells --- alkaline mucus D. Mucous cells --- intrinsic factor

B. Chief cells secrete pepsinogen, a pentose secreted as a zymogen that is activated by the acidic environment of the stomach. G-cells secrete gastric, parietal cells secrete hydrochloric acid and intrinsic factor, and mucous cells secrete alkaline mucus, eliminating the other answer choices.

what are baroreceptors? where are they located?

Baroreceptors are located in the walls of the vasculature. They are specialized neurons that detect changes in the mechanical forces on the walls of the vessel. When the BP is too low, they can stimulate the sympathetic NS, which causes vasoconstriction and inc BP.

what do free nerve endings, Meissner's corpuscles, Ruffini endings, and Pacinian corpuscules respond to , respectively?

free nerve endings respond to pain Meissner's corpuscles respond to light touch Ruffini endings respond to stretch Pacinian corpuscles respond to deep pressure and vibration

what is a deleterious mutation?

individual who carries the mutation has a lower fitness (produces lower successful descendants). opposed to an advantageous mutation

what do keratins do? What are they?

structural proteins. they are intermediate filament proteins found in epithelial cells. Keratins contribute to the mechanical integrity of the cell and also function as regular proteins. Keratin is the primary protein that makes up hair and nails

What are lysosomes and what do they do?

Lysosomes are vesicles containing hydrolytic enzymes that digests things like food and viral/bacterial particles. (break down substrates) Things you want to digest gets into a vacuole by endocytosis or phagocytosis, and then the vacuole fuses with the lysosome. Anything inside gets digested by the hydrolytic enzymes.

How do sphingolipids relate to phospholipids?

Many sphingolipids are also phospholipids because they contain a phosphodiester linkage. However, other sphingolipids contain glycosidic linkages to sugars; any lipid linked to a sugar can be termed a glycolipid.

Explain the function and purpose of prolactin

One of 7 hormones released by the anterior pituitary. Direct hormone. - more important in females than males; stimulates milk production in the mammary glands. - the release of dopamine from the hypothalamus decreases its secretion

What is the lysogenic cycle of a virus?

If the virus doesn't lyse its host cell, it can integrate into the host genome as a provirus or prophage, initiating the lysogenic cycle. A virus will be replicated as the bacterium reproduces b/c it is now a part of the host's genome.

With prolonged fasting, the brain can turn to which alternative fuel for energy? A. Fructose B. Lactate C. Ketone bodies D. Fatty acids

C The brain is almost exclusively dependent on glucose for energy; however, in a prolonged fast, ketone bodies can be used for up to 2/3 of the brain's energy requirements.

explain metaphase

Second phase of mitosis. the centriole pairs are now at opposite ends of the cell. The kinetochore fibers interact with the fibers of the spindle apparatus to align the chromosomes at the metaphase plate (equatorial plate), which is equidistant from the two poles of the cell

Explain proton motive force

Take a step back and look at the proton gradient that formed as electrons were passed along the ETC (electron transport chain). As [H+] increases in the intermembrane space, two things happen simultaneously: pH drops in the intermembrane space, and the voltage difference between the intermembrane space and matrix increases due to proton pumping. Together, these two changes contribute to what is referred to as an electrochemical gradient: a gradient that has both chemical and electrostatic properties. Because it is based on protons, we refer to the electrochemical gradient across the inner mitochondrial membrane as the proton-motive force. Any electrochemical gradient stores energy, a and it will be the responsibility of ATP synthase to harness this energy to form ATP from ADP and an inorganic phosphate.

_______ are a class of lipids specifically used for energy storage. Why are lipids considered a fantastic way to store energy?

Triacylglycerols 1. The carbon atoms of fatty acids are more reduced than those of sugars, which contain numerous OH groups. The result of this is that the oxidation of triacylglycerols yields 2x the energy per gram as carbs, making this a far more energy-dense storage mechanism compared to polysaccharides like glycogen. 2. Triacylglycerols are hydrophobic. They do not draw in water and do not require hydration for stability. This helps decrease their weight, esp in comparison to hydrophilic polysaccharides.

What are glucocorticoids and their function? Where are they secreted from? Which are some classic examples of glucocorticoids and what do they do in the body?

a class of corticosteroids, secreted from the adrenal cortex they are steroid hormones that regulate glucose levels. They also affect protein metabolism. Two most common are cortisol and cortisone. These hormones raise blood glucose by increasing gluconeogenesis and decreasing protein synthesis. -They can also decrease inflammation and immunologic responses. (remember cortisone injections and cortisone cream?) -cortisol is known as a stress hormone because it is released in times of physical or emotional stress. This inc blood sugar and provides a ready source of fuel in case the body must react quickly to a dangerous situation. Glucocorticoid release is under the control of ACTH. CRF --> ATCH-->glucocorticoids from the adrenal cortex

Valine: 3 letter abbreviation, 1 letter abbreviation, polar or non polar

Val. V. Non-polar

How do monozygotic twins form? What about conjoined twins?

When a single zygote splits into two. Because the genetic material is identical, the genomes of the offspring will be too. As more gestational structures are shared, there are more risks as the fetuses grow and develop. If division is incomplete, conjoined twins may result.

what is erythropoietin and what secretes it? what does it do? What is it secreted in response too?

a hormone produced by the kidneys. stimulates bone marrow to inc production of erythrocytes (RBCs). It is secreted in response to low oxygen levels in the blood.

what is inclusive fitness?

a measure of an organism's success in the population based on the number of offspring, success in supporting offspring, and the ability of the offspring to then support others.

what is homeostasis?

a physiological tendency toward a relatively stable state that is maintained and adjusted, often with the expenditure of energy.

Which of the following terms describes how existing schemata are modified to incorporate new information? A. Assimilation B. Adaptation C. Affirmation D. Accommodation

D

what are the EEG patterns correlated with different stages of waking and sleeping

4 characteristic EEG patterns correlated with different stages of waking and sleeping: beta, alpha, theta, and delta waves. There is a fifth wave that corresponds to REM sleep, which is the time during the night when we have most of our dreams.

What is the structure of ATP and how is it generated? How is it consumed? How are ADP and AMP produced from ATP?

ATP consists of an adenosine molecule attached to three phosphate groups, and is generated from ADP and Pi with energy input from an exergonic reaction or electrochemical gradient. It is consumed either through hydrolysis or the transfer of a phosphate group to another molecule. In one phosphate group is removed, adenosine diphosphate (ADP) is produced; it two phosphate groups are removed, adenosine monophosphate (AMP) is produced.

How are amino acids classified? what is the difference between glucogenic amino acids and ketogenic amino acids?

Amino acids are classified by their ability to turn into specific metabolic intermediates: glucogenic amino acids (all but leucine and lysine) can be converted into glucose through gluconeogenesis; ketogenic amino acids (leucine and lysine as well as isoleucine, phenylalanine, threonine, tryptophan, and tyrosine, which are also glucogenic as well) can be converted into acetyl-CoA and ketone bodies.

Explain cell death. What is the process? Is this the same thing as necrosis?

Apoptisis, or programmed cell death, occurs at various times in development. Ex: the fingers are originally webbed during development of the hand. The cells of the webbing later undergo apoptosis, resulting in separation of each individual finger and toe. During the process of apoptosis, the cell undergoes changes in morphology and divides into may self-contained protrusions called apoptotic blebs, which can then be broken apart into apoptotic bodies and digested by other cells. This allow recycling of materials. Because the blebs are contained by a membrane, this also prevents the release of potentially harmful substances into the extracellular environment. - this is different from necrosis, which is a process of cell death in which the cell dies as a result of injury. In necrosis, internal substances can be leaked, causing irritation of nearby tissues or even an immune response.

Asparginine: 3 letter abbreviation, 1 letter abbreviation, polar or non polar

Asn. N. Polar

A neuron only fires an action potential if multiple presynaptic cells release neurotransmitter onto the dendrites of the neuron. This is an example of: a) saltatory conduction b) summation c) a feedback loop d) inhibitory transmission

B

Describe posttranscriptional processing.

Before the hnRNA (heterogenous nuclear RNA)/ primary transcript can leave the nucleus and be translated to protein, it must undergo three specific processes to allow it to interact with the ribosome and survive the conditions of the cytoplasm. You can think of the nucleus as the happy home of the cell; the DNA strands are the parents, and the hnRNA as their child. The child must mature if he or she is to survive. 3 Processes: intron/exon splicing 5' cap 3' polyA tail

what is bile? what is it secreted by and stored in?

Bile contains bile salts, pigments, and cholesterol. Bile is secreted by the liver and stored in the gallbladder.

Explain beta and alpha waves.

Both characterize brain wave activity when we are awake. Beta waves have a high frequency and occur when the person is alert or attending to a mental task that requires concentration Alpha waves occur when we are wake but relaxing with our eyes closed, and are somewhat slower and more synchronized than beta waves.

What are the vasa recta?

Capillary beds around the loops of henle. The blood flow of the VR is in the opposite dirxn of the filtrate flow and allows for the maintenance of the conc of the medulla.

Which of the following is true regarding prophase? A. The chromosomes separate and move to opposite poles of the cell B. The spindle apparatus disappears C. The chromosomes uncoil D. The nucleoli disappear

D

What are the steps of the viral life cycle?

infection, translation, progeny assembly, progeny release

Cysteine: 3 letter abbreviation, 1 letter abbreviation, polar or non polar

Cys. C. Polar

A tumor is removed from a patient, and when investigated by pathologists, appears to contain tissue resembling placenta, hair, thyroid tissue, and cardiac muscle. What was the likely potency of the cells from which this tumor originated? A. Nonpotent B. Multipotent C. Pluripotent D. Totipotent

D

What is the third stage of cell specification? Explain it

Differentiation After a cells fate has been determined, the cell must begin to undertake changes that cause the cell to develop into the determined cell type. This includes changing the structure, function, and biochemistry of the cell to match the cell type (differentiation). -Cells that have not yet differentiated or that give rise to other cells that will differentiate are celled stem cells. The potency of the stem cell determines how may different cell types a stem cell can become. As cells become more differentiated, the potency of the cell decreases (from totipotent to pluripotent to motipotent). -Stem cells exist in embryonic tissues as well as adult tissues

Howard Gardner's Theory of Multiple Intelligences

Eight intelligences in problem solving: namely linguistic, musical, logical-mathematical, visual-spatial, bodily-kinesthetic, naturalistic, interpersonal, and intrapersonal. Possible ninth is existential.

convergent evolution

Evolution toward similar characteristics in unrelated species

What are triacylglycerols (triglycerides)?

Fatty acid triesters of glycerol. Most contain two or three different types of fatty acid residues. It is rare for all 3 fatty acids to be the same - overall these compounds are non polar and hydrophobic (insoluble in water) Triacylclyerol deposits can be observed in cells as oily droplets in the cytosol. They serve as depots of metabolic fuel that can be recruited when the cell needs additional energy to divide or survive when other fuel supplies are low.

What are the four stages of the cell cycle? What happens to cells that don't divide?

G1, S, G2, Mitosis (M) The first 3 stages are collectively known as interphase. Cells spend about 90% of their time here. They spend all their time in an offshoot of G1 called G0. During G0, the cell is simply living and carrying out its function without preparing for division

What are the rate-limiting enzymes for glycolysis, fermentation, glycogenesis, glycogenolysis, gluconeogenesis, and the pentose phosphate pathway?

Glycolysis: phosphofructokinase-1 Fermentation: lactate dehydrogenase Glycogenesis: glycogen synthase Glycogenolysis: glycogen phosphorylase Gluconeogenesis: fructose-1,6-bisphosphate Pentose Phosphate Pathway: glucose-6-phosphate dehydrogenase

What are the groups of terpenes?

Grouped according to the number of isoprene units present; a single terpene units contains two isoprene units. Monoterpenes (C10H16) are abundant in essential oils and turpentine; contain 2 isoprene units. Sequiterpenes (sequesi means one and a half) contains 3 isoprene units Diterpenes contain 4 isoprene units Triterpenes contain with 6 isoprene units tetraterpenes have 8 isoprene units

explain menopause

Happens gradually between the ages of 45-55 years of age. as a woman ages her ovaries become less sensitive to FSH and LH, resulting in ovarian atrophy. As estrogen and progesterone levels drop, the endometrium also atrophies, and menstruation stops. Also, because the negative feedback on FSH and LH is removed, the blood levels of these 2 hormones rise. Profound physical and physiological changes usually accompany this process including flushing, hot flashes, bloating, headaches, and irritability.

Histidine: 3 letter abbreviation, 1 letter abbreviation

His. H.

How do you draw a Fisher Projection?

Horizontal lines are wedges and vertical lines are dashes

During fatty acid mobilization, which of the following occurs? I. HSL is activated II. Free fatty acids are released III. Gluconeogenesis proceeds in adipocytes

I and II During fatty acid mobilization, there is a breakdown of triacylglycerols in adipocytes by hormone-sensitive lipase (HSL). This breakdown results in the release of three fatty acids and a glycerol molecule. the glycerol may be used by the liver for gluconeogenesis, but adipocytes do not have the ability to carry out gluconeogenesis

How does insulin affect metabolism of fats?

Insulin exhibits a significant impact of the metabolism of fats, especially in the liver and adipocytes. Insulin increases: -glucose and triacylglycerol uptake by fat cells -Lipoprotein lipase activity, which clears VLDL and chylomicrons form the blood -Triacylglycerol synthesis (lipogenesis) in adipose tissue and the liver from acetyl-CoA Insulin decreases - Triacylglycerol breakdown (lipolysis) in adipose tissue - Formation of ketone bodies by the liver

What are the 6 categories of enzymes?

LIL HOT Lyases Isomerases Ligases Hydrolyses Oxidoreductases Transferases

What are ependymal cells?

Line ventricles and produce CSF, which physically supports the brain and serves as a shock absorber

Lysine: 3 letter abbreviation, 1 letter abbreviation

Lys. K.

What does the Na+/K+ ATPase do?

Maintains potential of concentration gradient by pumping out Na+ and pumping in K+. Pumps them against their [ ] gradients. To remember the direction of ion movement, think pumpKin

inversion mutation

Mutation in which a chromosome piece reattaches to original chromosome but in reverse orientation

What is natural selection and its relation to evolution?

NS is survival of the fittest and it is NOT evolution but is a mechanism of evolution. certain characteristics or trains possessed by individuals within a species may help those individuals have greater reproductive success, thus passing on those traits to offspring

What are glomeruli?

Networks of capillaries that filter wastes from blood. highly convoluted capillary tufts derived from afferent arterioles.

duplication mutations

Occur when a segment of DNA is copied multiple times in the genome.

What is a P generation? F1 generation? F2 generation?

P is parent, F1 is first filial, f2 is second filial.

Why does the initiation of the antibody response take so long?

Part of the reason is that each B cell undergoes hypermutation of its antigen-binding region, trying to find the best match for the antigen. Only those B-cells that can bind the antigen with high affinity survive, providing a mechanism for generating specificity called clonal selection.

what is punctuated equilibrium?

Pattern of evolution in which long stable periods are interrupted by brief periods of more rapid change

Almost all chiral amino acids are in the R or S configuration? What is the one exception?

S. Except cysteine which is R

Serine: 3 letter abbreviation, 1 letter abbreviation, polar or non polar

Ser. S. Polar

What is residual volume (RV)?

The volume of air remaining in lungs after maximum exhalation.

What are peroxisomes? What do they do-simply?

They are enzymes that contain hydrogen peroxide. Break down ethanol (very long chain fatty acids via B-oxidation)

What are the two standard reagents used to detect the presence of reducing sugars? Explain them

Tollens' reagent and Benedict's Reagent Tollens'-reduced to produce a silvery mirror when aldehydes are present Benedict's-indicates an aldehyde by a red precipitate (solid Cu2O)

Explain cell migration.

cells must be able to disconnect from adjacent structures to migrate to their correct location. Ex: the ant pit originates from a segment of oral ectoderm and must migrate from the top of the mouth to its final location below the hypothalamus.

What is the path of blood deoxygenated blood into the heart until oxygenated blood leaves the heart?

deoxygenated blood enters the superior and inferior vena cava--> right atrium -->tricuspid value-->right ventricle-->pulmonary semilunar valve-->lungs-->pulmonary veins-->left atrium-->bicuspid (mitral) valve--> left ventricle-->aorta-->systemic circulation via arteries pneumonic LAB RAT or LAMB RAT

Explain the process of ketolysis in the brain

during a prolonged fast (longer than 1 week), the brain begins to derive up to two-thirds of its energy from ketone bodies. In the brain, when ketones are metabolized to acetyl-CoA, pyruvate dehydrogenase is inhibited. Glycolysis and glucose uptake in the brain decreases. This important switch spates essential protein in the body, which otherwise would be catabolized to form glucose by gluconeogenesis in the liver, and allows the brain to indirectly metabolized fatty acids as ketone bodies.

What happens during systole and diastole?

during systole, ventricular contraction and closure of the AV valves occurs and blood is pumped out of the ventricles. During diastole, the ventricles are relaxed, the semilunar valves are closed, and blood from the atria fills the ventricles.

Why is ATP such a good energy carrier? Explain.

its high energy phosphate bonds. The negative charges on the phosphate groups experience repulsive forces with one another, and the ADP and Pi molecules that form after hydrolysis are stabilized by resonance.

Orexin

its secreted is stimulated by Ghrelin. further increases appetite and is also involved in alertness and the sleep-wake cycle. hypoglycemia is also a trigger for orexin release.

What are steroids?

metabolic derivates of terpenes LIPIDS, though they are very different than traditional lipids, both in structure and function Steroids are characterized by having 4 cycloalkane rings fused together: three cyclohexane and one cyclopentain. Steroid functionality is determined by the oxidation status of these rings, as well as the functional groups they carry. NONPOLAR due to large # of carbons and hydrogens

To leave the body, urine must pass through what things? Explain this process.

pass through two sphincters-the internal and external urethral sphincter. The internal urethral sphincter, consisting of smooth muscle, is contracted in its normal state. Because the internal sphincter is made of smooth muscle, it is under involuntary control. The external urethral sphincter consists of skeletal muscle and is under voluntary control. When the bladder is full, stretch receptors convey to the nervous system that the bladder requires emptying. This causes parasympathetic neurons to fire, and the detrusor muscle contracts. This contraction also causes the internal sphincter to relax. This reflex is known as the micturition reflex. The next step is up to the individual. The person can choose to relax the external sphincter to urinate or can maintain the tone of the external sphincter to prevent urination. Urination itself is facilitated by the contraction of the abdominal musculature, which increases pressure within the abdominal cavity, resulting in compression of the bladder and increased urine flow rate.

What is the path of electrical conduction in the heart (basic)?

sinoatrial (SA) node-->atrioventricular (AV) node-->bundle of His (AV bundle)-->bundle branches --> Purkinje fibers

what is denaturation?

when a protein loses its tertiary structure

Which of the following would NOT be seen during pregnancy? A. high levels of hCG in the first trimester B. High levels of progesterone throughout the pregnancy C. low levels of FSH in the first trimester D. high levels of GnRH throughout the pregnancy

D

Tryptophan: 3 letter abbreviation, 1 letter abbreviation, polar or non polar

Trp. W

what is a zwitterion?

a zwitterion is a species with both a positive and a negative charge

Which of the following is NOT involved in cell migration? A. Dynein B. Flagella C. Actin D. Centrioles

D centrioles are composed of microtubules, but are involved in mitosis, not cell migration

Which of the following is NOT a type of glycolipid? A. Cerebroside B. Globoside C. Ganglioside D. Sphingomyelin

D sphingomyelin is a phospholipid

What are nuclear pores?

holes in the nuclear envelope where things can pass into and out of the nucleus. Transcription occurs in the nucleus, and those transcribed RNA need to pass out of the nucleus. Things like transcription factors need to pass into the nucleus where they can access the DNA to be transcribed.

Explain step 2 of the citric acid cycle, citrate isomerized to isocitrate

achiral citrate is isomerized to one of four possible isomers of isocitrate. First, citrate binds ta three points to the enzyme aconitase. Then water is lost from citrate, yielding cis-aconitate. FInally, water is added back to form isocitrate. The enzyme is a metalloprotein that requires Fe2+. This results in a switching of a hydrogen and hydroxyl group. Overall, this step is necessary to facilitate the subsequent oxidative decarboxylation.

At very acidic pH values, amino acids tend to be _______charged. At very basic pH values, amino acids tend to be _______charged.

acidic. + charge basic. - charge

what is the role of fatty acid synthase? what is it?

aka palmitate synthase because palmitate is the only fatty acid that hymans can synthesize de novo. Fatty acid synthase is a large multienzyme complex found in the cytosol that is rapidly induced in the liver following a meal high in carbohydrates because of elevated insulin levels. The enzyme complex contains an acyl carrier protein (ACP) that requires pantothenic acid (Vitamin B5). NADPH is also required to reduce the acetyl groups added to the fatty acid. Eight acetyl-CoA groups are required to produce palmitate (16:0). Fatty acyl-CoA may be elongated and desaturated, to a limited extent, using enzymes associated with the smooth ER.

The protein coat of a virus is known as a _______, which may be surrounded by what?

capsid, by an envelope composed of phospholipids and virus-specific proteins

What are aldoses and ketoses?

carbohydrates that contain an aldehyde or ketone (respectively) group as their most oxidized functional group

How is the backbone of DNA held together?

composed of alternating sugar and phosphate groups. always read from 5' to 3' It is formed as nucleotides are joined by 3'-5' phosphodiester bonds. That is, a phosphate group links the 3' carbon of one sugar to the 5' phosphate group of the next incoming sugar in the chain.

What are prions?

infections proteins. non-living. They cause disease by triggering misfolding of other proteins, usually through the conversion of a protein from an a-helical structure to a B-pleated sheet. This reduces the solubility of the protein as well as the cells ability to degrade the protein

What are IDLs? What is their function?

intermediate density lipoproteins. they are the third lowest density form of lipoprotein. after chylomicrons and VLDLs and before LDLs. Once a triacylglycerol is removed from VLDL, the resulting particle is either a VLDL remnant or IDL. Some IDL is reabsorbed by the liver by apolipoproteins on its exterior, and some is further processed in the bloodstream. Ex: some IDL picks up cholesteryl esters from HDL to become LDL. IDL thus exists as a transition particle between triacylglycerol transport (associated with chylomicrons and VLDL) and cholesterol transport (associated with LDL and HDL)

dipeptides consist of what? tripeptides? oligopeptides? polypeptides?

dipeptide- 2 amino acid residues tripeptide- 3 amino acid residues oligopeptides- 4-20 residues polypeptides- 21+ residues

What is the difference between diploid (___n) and haploid (___n)? What kinds of cells are diploid vs haploid?

diploid (2n)- autosomal cells- contain two copies of each chromosome. haploid (n)- germ cells- contain only one copy of each chromosome

explain the effects of temperature on enzyme-catalyzed reactions

enzyme-catalyzed reactions tend to double in velocity for every 10 degrees Celsius inc in temp until the optimum temp is reached, for the human body this is 37C, 98.6F, 310K. after this, activity falls off sharply, as the enzyme will denature at higher temps.

what is alternative splicing?

for some genes in eukaryotic cells, the primary transcript of hnRNA may be spliced together in different ways to produce multiple variants of proteins encoded by the same original gene. By using alternative splicing, an organism can make many more different proteins from a limited number of genes. By utilizing alternative splicing, an organism can make many more different proteins from a limited number of genes.

How do dizygotic twins occur?

fraternal. fertilization of two different eggs released b=during one ovulatory cycle by two different sperm. Each zygote will implant in the uterine wall, and each develops its own placenta, chorion, and amnion. If the zygotes implant close together the placentas may grow onto each other. Fraternal twins are no more genetically similar than any other pair of siblings.

Most fatty acid catabolism proceeds via what that occurs where?

B-oxidation that occurs in the mitochondria.

An a-helix is most likely held together by: A. disulfide bonds B. hydrophobic effects C. hydrogen bonds D. ionic attractions between side chains

C

During which phase of the menstrual cycle does progesterone [ ] peak? A. follicular phase B. ovulation C. luteal phase D. menses

C

Hormones are found in the body in very low concentrations, but tend to have a strong effect. What type of receptor are hormones most likely to act on? I. Ligand-gated ion channels II. Enzyme-linked receptors III. G protein-coupled receptors A. I only B. III only C. II and III only D. I, II, and III

II and III

Isoleucine: 3 letter abbreviation, 1 letter abbreviation, polar or non polar

Ile. I. Non-polar

What is Mendel's second law? What explains it?

Law of independent assortment. states that the inheritance of one allele has no effect on the likelihood of inheriting certain alleles for other genes. explained by crossing over during prophase I of Meiosis

where are the testes located? Why?

They are located in the scrotum, an eternal pouch that hangs below the penis, a position that allows it to maintain a temp 2-4C lower than the body.

What is intuition

ability to act on perceptions that may not be supported by available evidence

What are microglia?

phagocytic cells that ingest and break down waste products and pathogens in the CNS

What is the initial stage of cell specialization? Explain it.

it is specification. the cell is reversibly designated as a specific cell type

what is the stratum basale and what is its importance?

it is the deepest layer of the epidermis. It contains stem cells and is responsible for proliferation of keratinocytes, the predominant cells of the skin, that produce keratin.

Explain spermatogenesis. Where does it occur?

it is the formation of haploid sperm through meiosis. it occurs in the seminiferous tubules. In males, the diploid stem cells are known as spermatogonia. After replicating their genetic material (S stage), they develop into diploid primary spermatocytes. The first meiotic division will result in haploid secondary spermatocytes, which then undergo meiosis II to generate haploid spermatids. Finally, the spermatids undergo maturation to become mature spermatozoa.

what is the rate limiting enzyme of glycogenolysis? What is glycogenolysis?

it is the process of breaking down glycogen. rate limiting enzyme is glycogen phosphorylase.

What is ATP cleavage? What does this generally do to the target molecule?

it is the transfer of a high-energy phosphate group from ATP to another molecule. Generally, this inactivates or activates the target molecule. With these phosphoryl group transfers, the overall free energy of the reaction will be determined by taking the sum of the free energies of the individual reactions.

Explain Lev Vygotsky's perspective of cognitive development.

sociocultural perspective proposed that the engine driving cognitive development is the child's internalization of his or her culture, including interpersonal and societal rules, symbols, and language. As a child develops, their skills and abilities are still in formative stages. With help from adults or other children, those skills can develop further. That help may come in the form of instruction from a teacher or even watching another child perform the skill.

What is the job of rRNA (ribosomal RNA)? where is it synthesized? what are ribozymes?

synthesized in the nucleolus and functions as an integral part of the ribosomal machinery used during protein assembly in the cytoplasm. Many rRNA molecules function as ribozymes; that is, enzymes made of RNA molecules instead of peptides. rRNA helps catalyze the formation of peptide bonds and is also important in splicing out its own introns within the nucleus.

What is the stratum corneum? What is its importance?

the most superficial layer of the epidermis. contains up to several dozen layers of flattened keratinocytes, forming a barrier that prevents invasion by pathogens and that helps to prevent loss of fluids and salt. Hair projects above the skin, and there are openings for sweat and sebaceous glands.

What are self-antigens? What do they do normally?

the proteins and carbohydrates present on the surface of every cell of the body. Under normal circumstances, these self-antigens signal to immune cells that the cell is not foreign and should not be attacked. However, when the immune system fails to make the distinction between self and foreign, it may attack cells expressing particular self-antigens, a condition known as autoimmunity.

What are some mechanisms that the respiratory system has to prevent pathogens from entering the body?

the respiratory passages are mucous membranes, lined with cilia to trap particulate matter and push it up toward the oropharynx, where it can be swallowed or expelled. While mucus helps to trap particulates like smoke and dirt, it also helps to prevent bacteria and viruses from gaining access to the lung and tissue below. Several other mucous membranes, including those around the eye and in the oral cavity, produce a nonspecific bacterial enzyme called lysozyme, which is secreted in tears and saliva, respectively

What is the tidal volume (TV)?

volume of air inhaled or exhaled in a normal breath

Which of the following does NOT contain tubulin? A. cilia B. flagella C. microfilaments D. centrioles

C

With which of the following molecules does Ca2+ bind after its release from the sarcoplasmic reticulum to regulate muscle contraction? A. Myosin B. Actin C. Troponin D. Tropomyosin

C Calcium is released form the SR into the sarcoplasm. It binds troponin molecules on the thin filaments, causing the strands of tropomyosin to shift, thereby exposing the myosin-binding sites on the thin filaments.

What are the layers of the skin (epidermis)? From superficial to deep.

Come, Lets, Get Sun Burned Stratum corneum stratum lucidum Stratum granulosum Stratum spinosum Stratum basale

What is an enterogastrone? Give an example. Why are they important?

It is a hormone that slows motility through the digestive tract, ex: secretin. Slowing of motility allows increased time for digestive enzymes to act on chime- especially fats.

What is the role of dihydroxyacetone phosphate (DHAP) in glycolysis?

It is an intermediate used in hepatic and adipose tissue for triacylglycerol synthesis. DHAP is formed from fructose 1,6-bisphosphate. It can be isomerized to flycerol 3-phosphate, which can then be converted to glycerol, the backbone of triacylglycerols.

Explain the concept of leading and lagging strands in DNA synthesis

The leading strand in each replication fork is the strand that is copied in a continuous fashion, in the same direction as the advancing replication fork, This parental strand will be read 3' to 5' and. its complement will be synthesized in a 5' to 3' manner. The lagging strand is the strand that is copied in a direction opposite the direction of the replication fork. On this side of the replication fork, the parental strand has 5' to 3' polarity. Because DNA polymerase can only synthesize int he 5' to 3' direction from a 3' to 5' template, small strands called Okazaki fragments are produced. As the replication fork continues to move forward, it clears additional space that DNA polymerase must fill in.

What is an amphipathic molecule?

a molecule with both hydrophilic and hydrophobic regions

What are the 3 other ways that 5'-ATG-3' can be written?

backwards: 3'-GTA-5' position of phosphates shown: pApTpG "d" used as shorthand for deoxyribose: dAdTdG

Explain barbiturates and Benzodiazepines

barbiturates were historically used as anxiety-reducing and sleep medications, but have mostly been replaced by benzodiazepines, which are less prone to ovredose. Barbituates include amobarbital and phenobarbital. Benzodiazepines include alprazolam, lorazepam, diazepam, and clonazepam. These drugs increase GABA activity, causing a sense of relaxation.

What type of things does the duodenum secrete?

brush border enzymes to break down dimers and trimers of biomolecules into absorbable monomers. enteropeptidase, which is involved in the activation of other digestive enzymes from the accessory organs of digestion. hormones like secreted and CCK into the bloodstream

the side chains (R groups) of amino acids determine what?

their chemical properties

Explain memory T cells.

these can be generated. similar to memory B cells, these cells lie in wait until the next exposure to the same antigen. When activated, they carry out a more robust and rapid response.

what are amphoteric species?

they can either accept or donate a proton (such as an amino acid). how they react depends on the pH of their environment

Peptide hormones -what are they made up of -what size are they -how are they derived and how do they get released into the bloodstream?

made up of amino acids range in size from very small (ADH) to large (insulin) derived from larger precursor polypeptides that are cleaved during posttranslational modification. These smaller units are transported to the Golgi apparatus for further modifications that activate the hormones and direct them to the correct locations in the cell. Such hormones are released by exocytosis after being packaged into vesicles

What is the main pathway that produces acetyl-CoA? What other pathways form acetyl-CoA?

main pathway is glycolysis. other pathways include fatty acid oxidation (B-oxidation), amino acid catabolism, ketones and alcohol.

Dietary fat consists mainly of ____________, with the remainder comprised of what?

mainly triacylglycerols, with the remainder comprised of cholesterol, cholesteryl esters, phospholipids, and free fatty acids.

all chiral amino acids used in eukaryotes are?

L-amino acids

what is an operon?

a cluster of genes transcribed as a single mRNA, under the control of a single promoter

What is cognitive development?

development of one's ability to think and solve problems across the lifespan

Explain insomnia

difficulty falling asleep or staying asleeo. It is the most common sleep-wake disorder and may be related to anxiety, depression, medications, or disruption of sleep cycles and circadian rhythms.

what is biosignaling?

process in which cells receive & act on signals

What is the difference between oncogenes and tumor suppressor genes?

same outcome (cancer) different cause oncogenes promote the cell cycle while mutated tumor suppressors can no longer slow the cell cycle. oncogenes are like stepping on the gas and mutated tumor suppressors are like cutting the brakes

by both weight and size, what is the largest organ in our bodies?

the skin (integument)

What are the 5 histone proteins found in eukaryotic cells? How do they organize?

two copies of each of the histone proteins H2A, H2B, H3, H4 form a histone core and about 200 base pairs of DNA are wrapped around this protein complex, forming a nucleosome. The last histone, H1, seals off the SNA as it enters and leaves the nucleosome, adding stability to the structure.

Describe the mechanism of translation. How does this differ between prokaryotes and eukaryotes? Explain the 3 stages in prokaryotic vs eukaryotic translation. Where does it occur?

translation occurs in the cytoplasm in prokaryotes and eukaryotes. In prokaryotes, the ribosomes start translating before the mRNA is complete; in eukaryotes, however, transcription and translation occur at separate times and in separate locations within the cell. The process of translation occurs in 3 stages: initiation, elongation, and termination. Initiation: The small ribosomal subunit binds to the mRNA. In prokaryotes, the small subunit binds to the Shine-Dalgarno sequence in the 5' untranslated region of the mRNA. In eukaryotes, the small subunit binds to the f' cap structure. The charged initiator tRNA binds to the AUG start codon through base pairing with its anti-codon within the P site of the ribosome. The initial acid in prokaryotes is N-formylmethionine (fMet); in eukaryotes, its's methionine. The large subunit then binds to the small subunit, forming the completed initiation complex. This is assisted by initiation factors (IF) that are not permanently associated with the ribosome Elongation: Elongation is a three-step cycle that is repeated for each amino acid to the protein after the initiator methionine. During elongation, the ribosome moves in the 5' to 3' direction along the mRNA, synthesizing the protein from its amino (N-) to carboxyl (C-) terminus. The ribosome contains three very important binding sites: 1. The A site holds the incoming aminoacyl-tRHA complex. This is the next animo acid that is being added to the growing chain, and is determined by the mRNA codon within the A site. 2. The P site holds the tRNA that carries the growing polypeptide chain. It is also where the first amino acid (methionine) binds because it is starting the polypeptide chain. A peptide bond is formed as the polypeptide is passed from the tRNA in the P site to the tRNA in the A site. This requires peptidyl transferase, an enzyme that is part of the large subunit. GTP is used for energy during the formation of this bond. 3. The E site is where the now inactivated (uncharged) tRNA pauses transiently before exiting the ribosome. As the now-uncharged tRNA enters the E site, it quickly unbinds from the mRNA and is ready to be recharged. Elongation factors (EF) assist by locating and recruiting aminoacyl-tRNA along with GTP, while helping to remove GDP once the energy has been used. Some eukaryotic proteins contain signal sequences, which designate a particular destination for the protein. For peptides that will be secreted, such as hormones and digestive enzymes, a signal sequence directs the ribosome to move to the ER so that the protein can be translated directly into the lumen of the rough ER. From there, a protein can be sent to the Golgi apparatus an dbe secreted from a vesicle via exocytosis. Other signal sequences direct proteins to the nucleus, lysosomes, or cell membrane. Termination: When any of the three stop codons moves into the A site, a protein called release factor (RF) binds to the termination codon, causing a water molecule to be added to the polypeptide chain. The addition of this water molecule allows peptidyl transferase and termination factors to hydrolyze the completed polypeptide chain from the final tRNA. The polypeptide chain will then be released from the tRNA in the P site, and the two ribosomal subunits will dissociate.

What is the difference between how nerves cluster and the info they carry in the CNS and PNS?

Multiple neurons may be bundled together to form a nerve in the PNS. These nerves may be sensory, motor, or mixed, which refers to the type(s) of info they carry. mixed nerves carry both sensory and motor info. The cell bodies of neurons of the same type are clustered together into ganglia. In the CNS, axons may be bundled together to form tracts. Unlike nerves, tracts only carry one type of info. The cell bodies of neurons in the same tract are grouped into nuclei.

together the vasa recta and nephron create what system? explain

a countercurrent multiplier system. the flow of filtrate through the loop of Henle is the opposite direction from the flow of blood through the vasa recta. The filtrate is constantly being exposed to hypertonic blood, which allows maximal reabsorption of water.

what are the steps of aerobic respiration?

1. Glycolysis 2. Krebs Cycle 3. Electron Transport Chain

What are the isotypes of antibodies? Why are there different isotypes?

5 types. IgM, IgD, IgG, IgE, IgA. the different types can be used at different times during the adaptive immune response, for different types of pathogens, or in different locations.

What is osmosis?

Diffusion of water across a selectively permeable membrane. Water will move from a region of lower solute [ ] to a region of higher solute [ ]. That is, it will move from a region of higher water [ ] to a region of lower water [ ].

Explain step 1 of the citric acid cycle, citrate formation

First, acetyl-CoA and oxaloacetate undergo a condensation reaction to form citryl-CoA (an intermediate). Then the hydrolysis of citryl-CoA yields citrate and CoA-SH. This reaction is catalyzed by citrate synthease. Syntheases are enzymes that form new covalent bonds without needing significant energy. This second part of this step energetically favors the formation of citrate and helps the cycle revolve in the forward direction.

Explain gel electrophoresis and southern blotting.

Gel electrophoresis is a technique used to separate macromolecules, such as DNA and proteins, by size and charge. All molecules of DNA are negatively charged b/c of the phosphate group in their backbone, so all DNA strands will migrate toward the anode of an electrochemical cell. The preferred gel for DNA electrophoresis is agarose gel and- just like the proteins in polyacrylamide gel the longer the DNA strand, the slower it will migrate in the cell. Gel electrophoresis is often used while performing a Southern blot. A Southern blot is used to detect the presence and quantity of various DNA strands in a sample. DNA is cut by restriction enzymes and then separated by gel electrophoresis. The DNA fragments are then carefully transferred to a membrane, retaining their separation. The membrane is then probed with many copies of a single-stranded DNA sequence. The probe will bind to its complementary sequence and form double-stranded DNA. Probes are labeled with radioisotopes or indicator proteins, both of which can be used to indicate the presence of a desired sequence.

What does debranching enzyme do and how does it do it? Name it.

Glucosyl alpha-1,4:alpha-1,4 transferase and alpha-1,6 glucosidase it is a two-enzyme complex that deconstructs the branches in glycogen that have been exposed by glycogenphosphorylase. The two-step process by which this occurs is: Breaks an alpha-1,4 bond adjacent to the branch point and moves the small oligoglucose chain that is released to the exposed end of the other chain. Forms a new alpha-1,4 bond. Hydrolyzes the alpha-1,6, releasing the single residue at the branch point as free glucose. This represents the only free glucose produced directly in glycogenolysis (as opposed to the glucose produced from glucose 1-phosphate, which must be converted by a mutase to glucose 6-phosphate before it can be converted to glucose via the enzyme glucose-6-phosphatase)

Prior to puberty, the hypothalamus reduces production of what? What happens at the onset of puberty?

GnRH (gonadotropin-releasing hormone) the hypothalamus releases GnRH, triggers the anterior pituitary gland to synthesize and release FSH (follicle-stimulating hormone) and LH (luteinizing hormone). These hormones trigger the production of other sex hormones that develop and maintain the reproductive system

What are the differences between Gram-positive and gram-negative cell walls?

Gram positive cell walls consist of a thick layer of peptidoglycan, a polymeric substance made from amino acids and sugars. In addition to its structural and barrier functions, the cell wall may also aid a bacterial pathogen by providing protection from a host organism's immune system, In addition to peptidoglycan, the gram-positive cell wall also contains a lipoteichoic acid. Gram negative cell walls are very thin and also contain peptidoglycan, but it much smaller amounts. The peptidoglycan cell walls of these bacteria are adjacent to the cell membrane, and are separated from the membrane by the periplasmic space. In addition to the cell wall and cell membrane, gram-negative bacteria also have outer membranes containing phospholipids and lipopolysaccharides.

what are the three main types of G proteins and what are their functions?

Gs- Stimulates adenylate cyclase, which increases levels of cAMP in the cell Gi- Inhibits adenylate cyclase, which decreases levels of cAMP in the cell Gq-activates phospholipase C, which cleaves a phospholipid from the membrane to form PIP2. PIP2 is then cleaved in to DAG and IP3; IP3 can open calcium channels in the endoplasmic reticulum, increasing calcium levels in the cell Gs-stimulates Gi-inhibits Gq-mind your ps and qs "phospholipase C"

What are the five criteria for Hardy-Weinberg equilibrium to be met? What is it? What are the two key equations?

Hardy-Weinberg equilibrium is when the gene frequencies of a population are not changing, the gene pool is stable and evolution is NOT happening. Five Criteria 1. The population is very large (no genetic drift) 2. There are no mutations that affect the gene pool 3. Mating between individuals in the population is random (no sexual selection) 4. There is no migration of individuals into or out of the population 5. The genes in the population are all equally successful at being reproduced. Equations: p + q = 1 P^2 + 2pq + q^2 = 1 p-frequency of the dominant allele q-frequency of the recessive allele

Steroid hormones are steroids that: I. have specific high-affinity receptors II. travel in the bloodstream from endocrine glands to distant sites III. affect gene transcription by binding directly to DNA

I and II only steroid hormones are produced in endocrine glands and travel in the bloodstream to bind to high-affinity receptors in the nucleus. The hormone's receptor binds to DNA as part of the hormone-receptor complex, but the hormone itself does not

Which type(s) of muscle has/have myogenic activity? I. Cardiac muscle II. Skeletal muscle III. smooth muscle

I and III

Restriction endonuclease are used for which of the following? I. Gene therapy II. Southern blotting III. DNA repair

I, II, and III Endonuclease are enzymes that cut DNA. They are used by the cell for DNA repair. They are also used by scientists during DNA analysis, as restriction enzymes are endonuclease. Restricted enzymes are used to cleave DNA before electrophoresis and Southern blotting, and to introduce a gene of interest into a viral vector for gene therapy

In which of the following structures of a healthy human male would immotile sperm incapable of fertilization be found? I. Vas deferens II. Seminiferous tubules III. Urethra

II. This question is testing your understanding of sperm development within the male reproductive system. To answer this question, you must know the pathway sperm travels from the testes to ejaculation: seminiferous tubules -> epididymis -> vas deferens -> ejaculatory duct -> urethra -> penis. Furthermore, you must also know that the epididymis is the location where spermatozoa gain motility and fertilization capabilities. Therefore, immotile sperm that are incapable of fertilization may be found in the parts of the male reproductive system preceding the epididymis.

Which of the following is/are true about sphingolipids? I. They are all phospholipids II. They all contain a sphingosine backbond III. They can either have phosphodiester or glycosidic linkages to their polar head group

III only Sphingolipids can either have a phosphodiester bond, and therefore be phospholipids, or have a glycosidic linkage and therefore be glycolipids. Not all sphingolipids have a sphingosine backbone; some have related sphenoid compounds as backbones instead

What is frequency summation and tetanus?

If a muscle fiber is exposed to frequent and prolonged stimulation, it will have insufficient time to relax. The contractions will combine, become stronger and more prolonged. THis is known as frequency summation. If the contractions become so frequent that the muscle is unable to relax at all, this is tetanus. Note that tetanus (the disease) includes tetanus (the physiological phenomenon) as one of its primary clinical features, but that tetanic physiology also occurs under normal circumstances with multiple simple twitches in succession.

Explain an action potential in detail. What states can Na+ channels exist in?

If the cell is brought to threshold, voltage-gated Na+ channels open in the membrane. There is a strong electrochemical gradient that promotes the migration of Na+ into the cell. - From an electrical standpoint, the inside of the cell is more negative than the outside, which favors the movement of positively charged Na+ cations into the cell. - From a chemical standpoint, there is a higher [ ] of Na+ outside the cell, which also favors the movement of Na+ into the cell. As sodium passes through these ion channels, the membrane potential becomes more positive (the cell depolarizes rapidly). When Vm approaches +35mV, the Na+ channels are inactivated. Na+ channels can be closed (before the cell reaches threshold), open (from threshold to approx +35mV), or inactive (from approx +35mV to the resting potential) The positive potential inside the cell also triggers voltage-gated K+ channels open. Once Na+ has depolarized the cell, there is an electrochemical gradient favoring the efflux of K+ from the neuron. As K+ leaves, there will be a restoration of the negative membrane potential (repolarization). - The efflux of K+ causes an overshoot of the resting membrane potential, hyper polarizing the neuron. This hyper polarization makes the neuron refractory to further action potentials. There are 2 types of refractory periods. - During the absolute refractory period, no amount of stimulation can cause another action potential to occur. - During the relative refractory period, there must be greater than normal stimulation to cause an action potential because the membrane is starting from a potential that is more negative than its resting value.

What are the various growth phases of bacteria?

In a new environment, bacteria first adapt to the new local conditions during the lag phase. As the bacteria adapt, the rate of division increases, causing an exponential increase in the number of bacteria in the colony during the exponential phase (log phase). As the number of bacteria in the colony grows, resources are often reduced. The reduction of resources slows reproduction, and the stationary phase results. After the bacteria have exceeded the ability of the environment to support the number of bacteria, a death phase occurs, marking the depletion of resources.

What are some differences between meiosis and mitosis?

In contrast to mitosis, which consists of one round each of replication and division, meiosis consists of one round of replication followed by two rounds of division. Meiosis I results in homologous chromosomes being separated, generating haploid daughter cells (reductional division). Meiosis II is similar to mitosis in that it results in the separation of sister chromatids without a change in ploidy (equational division)

How does thermoregulation work in the respiratory system?

In order to maximize gas exchange, there is a tremendous surface area over which the alveoli and capillaries interact. Because the entire respiratory tract is highly vascular, it can also be used for thermoregulation, or the regulation of body temp. Heat is regulated via the body surfaces by vasodilation and vasoconstriction. As capillaries expand, more blood can pass through these vessels, and a larger amount of thermal energy (heat) can be dissipated. As capillaries contract, less blood can pass through them, conserving thermal energy. Nasal and tracheal capillaries are most frequently used for these purposes within the respiratory system. While these capillaries beds provide a mechanism for thermoregulation, humans predominantly regulate temp using capillaries and sweat glands in the skin, or rapid muscle contraction (shivering). the respiratory system can also transfer heat to the environment through evaporation of water in mucous secretions. Other animals, such as dogs, take advantage of this cooling mechanism by panting.

How does oxygen move in the body? Explain this phenomenon.

It moves by binding to hemoglobin in the blood. Hemoglobin is a protein composed of 4 cooperative subunits, each of which has a prosthetic heme group that binds to an oxygen molecule. The binding of oxygen occurs at the heme group's central iron atom. The binding or releasing of oxygen to or from the iron atom in the heme group is a redox reaction. In the lungs, oxygen diffuses into the alveolar capillaries. As the first oxygen binds to a heme group, it induces a conformational shift in the shape of hemoglobin from tout to relaxed. This shift increases the hemoglobin's affinity for oxygen, making it easier for subsequent molecules of oxygen to bind to the remaining three unoccupied heme groups. As other heme groups acquire an oxygen molecules, the affinity continues to increase, thus creating a positive feedback-like mechanisms. Once all of the hemoglobin subunits are bound to oxygen, the removal of one molecules of oxygen will induce a conformational shift, decreasing the overall affinity for oxygen and making it easier for the other molecules of oxygen to leave the heme group. This phenomenon is a form of allosteric regulation referred to as cooperative binding and results in the classic sigmoidal (S-shaped) oxyhemoglobin dissociation curve.

Explain the role of 3-phosphoglycerate kinase in glycolysis.

It transfers the high-energy phosphate from 1,3-bisphosphoglycerate to ADP, forming ATP and 3-phosphoglycerate. This type of reaction, in which ADP is directly phosphorylated to ATP using a high-energy intermediate, is referred to as substrate-level phosphorylation. In contrast to oxidative phosphorylation in mitochondria, substrate-level phosporylations are not dependent on oxygen, and are the only means of ATP generation in an anaerobic tissue.

What are kinesins and dyneins?

Kinesis and dyneins are the motor proteins associated with microtubules. They have two heads, at least one of which remains attached to tubulin at all times. Kinesis play key roles in aligning chromosomes during metaphase and depolymerizing microtubules during anaphase of mitosis. They move along microtubules in a stepping motion such that one or both heads remain attached at all times Dyneins are involved in the sliding movement of cilia and flagella. Both proteins are important for vesicle transport in the cell, but have opposite polarities: kinesis being vesicles toward the positive end of the microtubule, and dyneins bring vesicles toward the negative end.

Explain the formal operational stage of cognitive development

Last/4th stage starts around 11 and ends when the kid is able to think logically about abstract ideas. Generally coinciding with adolescence, this stage is marked by the ability to reason about abstract concepts and problem solve. The difference between this type of thought and concrete operations is illustrated by Piaget's pendulum experiment. Kids were given a pendulum in which they could vary the length of the string, the weight of the pendulum, the force of the push, and the initial angle of the swing. They were asked to find out what determined the frequency of the swing. Children in the concrete operational stage manipulated the variables at random and even distorted the data to fit preconceived hypothesis. Adolescents, on the other hand, were able to hold all variables but one constant at a given time, proceeding methodically to discover that only the length of the string affects the frequency

Explain the general structure of the lymphatic system and lymph nodes.

Made up of one way vessels that become larger as they move toward the center of the body. These vessels carry lymphatic fluid (lymph) and most join to form a large thoracic duct in the posterior chest, which then delivers the fluid into the left subclavian vein (near the heart). Lymph nodes are small, bean-shaped structures along the lymphatic vessels. Lymph nodes contain a lymphatic channel, as well as an artery and a vein. These lymph nodes provide a space for the cells of the immune system to be exposed to possible pathogens.

How does progeny release (viral life cycle) work? (3 ways)

Many ways. 1. The viral invasion may initiate cell death, resulting in spilling of viral progeny 2. the host cell may lyse as a result of being filled with tons of virions 3. a virus fuses with the plasma membrane (extrusion). this keeps the host cell alive, allowing for continued use of the host cell by the virus. (a virus in this state is in a productive cycle)

In the cytoplasm, what molecules act as high-energy electron carriers?

NADH, NADPH, FADH2, ubiquinone, cytochromes, and glytathione.

What are prostaglandins?

Named originally b/c we thought they prostate produced it, but they are actually produced by almost all cells in the body. These 20-Carbon molecules are unsaturated carboxylic acids derived from arachidonic acid and containing one 5-carbon ring. They act as paracrine or autocrine signaling molecules. In many tissues, they regulate the synthesis of cAMP, which mediates the actions of many other hormones.

Neurons are not actually in direct physical contact. There is a small space between the neurons called the ______ into which neurotransmitters are secreted. To clarify the terminology, the neuron preceding the synaptic cleft is called the ______ neuron; the neuron after is called the ______ neuron. If a neuron signals to a gland of muscle, rather than another neuron, the postsynaptic cell is termed a(n) ______. Most synapses are ______ in nature; they use small molecules referred to as ______ to send messages from one cell to the next.

Neurons are not actually in direct physical contact. There is a small space between the neurons called the synaptic cleft into which neurotransmitters are secreted. To clarify the terminology, the neuron preceding the synaptic cleft is called the presynaptic neuron; the neuron after is called the postsynaptic neuron. If a neuron signals to a gland of muscle, rather than another neuron, the postsynaptic cell is termed an effector. Most synapses are chemical in nature; they use small molecules referred to as neurotransmitters to send messages from one cell to the next.

Do all cells have the same relative distribution of organelles?

No. Form will follow function. Cells that required a lot of energy for locomotion (such as sperm cells) have high concentrations of mitochondria. Cells involved in secretion (such as pancreatic islet cells and other endocrine tissues) have high concentrations of RER and Golgi apparatuses. Other cells, such as red blood cells, which primarily serve a transport function, have no organelles at all.

What are the differences between nucleosides and nucleotides?

Nucleosides-composed of a five-carbon sugar (pentose) bonded to a nitrogenous base and are formed by covalently linking the base to C-1' of the sugar. Not that the carbon atoms in the sugar are labeled with a prime symbol to distinguish them from the carbon atoms in the nitrogenous base Nucleotides-formed when one or more phosphate groups are attached to C-5' of a nucleoside. Often these molecules are named according to the number of phosphates present. Adenosine di- and triphosphate (ADP and ATP), for example, gain their names from the number of phosphate groups attached to the nucleoside adenosine. These are high-energy compounds because of the energy associated with the repulsion between closely associated negative charges on the phosphate groups. Nucletides are the building blocks of DNA

What are the key regulators of oxidative phosphorylation? Explain this and respiratory control.

O2 and ADP. If O2 is limited, the rate of oxidative phosphorylation decreases, and the [ ]s of NADH and FADH2 increase. The accumulation of NADH, in turn, inhibits the citric acid cycle. The coordinated regulation of these pathways is known as respiratory control. In the presence of adequate O2, the rate of oxidative phosphorylation is dependent on the availability of ADP. The [ ]s of ADP and ATP are reciprocally related; an accumulation of ADP is accompanied by a decrease in ATP and the amount of energy available to the cell. Therefore, ADP accumulation signals the need for ATP synthesis. ADP allosterically activates isocitrate dehydrogenase, thereby increase the rate of the citric acid cycle and the production of NADH and FADH2. The elected levels of these reduced coenzymes, in turn, increase the rate of electron transport and ATP synthesis.

Explain the process of gastrulation and the primary germ layers.

Once the cell mass implants it can begin further developmental processes such as gastrulation, the generation of three distinct cell layers. Gastrulation begins with a small invagination in the blastula. Cells continue moving toward the invagination resulting in elimination of the blastocoel. In living things, the result of this process is called a gastrula. the membrane invagination into the blastocoel is called the archenteron (later develops into the gut). The opening of the archenteron is the blastopore. In deuterostomes, such as humans, the blastopore develops into the anus. In protostomes, it develops into the mouth. The outermost layer is the ectoderm and gives rise to the integument, including the epidermis, hair, nails, and the epithelia of the nose, mouth, and lower anal canal. The lens of the eye, nervous system, and inner ear are also derived from the ectoderm. The middle later is the mesoderm and develops into several different systems including the musculoskeletal, circulatory, and most of the excretory systems. Mesoderm also gives rise to the gonads as well as the muscular and connective tissue layers of the digestive and respiratory systems. The innermost layer is the endoderm and forms the epithelial linings of the digestive and respiratory tracts, inc the lungs. The pancreas, thyroid, bladder, and distal urinary tracts, as well as parts of the liver are derived fro the endoderm.

Explain cytotoxic T cells.

One of the 3 major types of T cells. (Tsubc, CTL, cytotoxic T-lymphocytes), also called CD8+ T-cells, are capable of directly killing virally infected cells by injecting toxic chemical that promote apoptosis into the infected cell. CD8+ T-cells respond to antigens presented on MHC-1 molecules. Because MHC-1 presents endogenous antigens, CD8+ T-cells are most effective against viral (and intracellular bacterial or fungal infections).

Explain the roles of the phosphofructokinases (PFK-1 and PFK-2) in glycolysis. Include the inhibitors and stimulators of both.

PFK-1 is the rate limiting enzyme and main control point in glycolysis. In this reaction, fructose 6-phosphate is phosphorylated to fructose 1,6-bisphosphate using ATP. PRK-1 is inhibited by ATP and citrate, and activated by AMP. This makes sense because the cell should turn off glycolysis when it has sufficient energy (high ATP) and turn on glycolysis when it needs energy (high AMP). Citrate is an intermediate of the citric acid cycle, so high levels of citrate also imply that the cell is producing sufficient energy. Insulin stimulates and glucagon inhibits PFK-1 in hepatocytes by an indirect mechanism involving PFK-2 and fructose 2,6-bisphosphate. Insulin activates PFK-2, which converts a tiny amount of fructose-6-bisphosphate to fructose 2,6-bisphosphate (F2,6MP). F2,6-BP activates PFK-1. on the other hand, glucagon inhibits PFK-2, lowering F2,6-BP and thereby inhibiting PFK-1. PFK-2 is found mostly in the liver. By activating PFK-1, it allows these cells to override the inhibition caused by ATP so that glycolysis can continue, even when the cell is energetically satisfied. The metabolites of glycolysis can thus be fed into the production of glycogen, fatty acids, and other storage molecules rather than just being burned to produce ATP.

what are night terrors?

Periods of intense anxiety that occur during slow-wave sleep most common in kids. Children will often thrash and scream during these terrors, and will show signs of sympathetic overdrive (high HR rapid breathing). B/c these usually occur during SWS, the child experiencing the episode is very difficult to wake, and usually does not remember the dream the next morning.

What are the two components of blood? Further divide both components. All blood cells are formed from what, which originates where?

Plasma (liquid component) and Cells' Plasma is an aqueous mixture of nutrients, salts, respiratory gases, hormones, and blood proteins. Plasma can be further refined via the removal of clotting factors into serum. The cellular portion of blood has 3 major categories: erythrocytes (RBCs), leukocytes (WBCs), and platelets. All blood cells are formed from hematopoietic stem cells, which originate in the bone marrow.

What is parathyroid hormone (PTH) and what does it do? How is it stimulated? What is its mechanism of action?

Produced by parathyroid glands -antagonistic hormone to calcitonin, raising blood calcium levels by 1. decreasing excretion of calcium by the kidneys 2. increasing absorption of calcium in the gut (via Vitamin D) 3. increasing bone reabsorption -promotes phosphorus homeostasis by increasing the resorption of phosphate from bone and reducing reabsorption of phosphate in the kidney (promoting its excretion in the urine) -activates Vitamin D, which is required for the absorption of calcium and phosphate in the gut. Overall effect: significant inc in blood calcium levels with little effect on phosphate (the absorption of phosphate in the gut and excretion in the kidney somewhat cancel each other) feedback inhibition. as levels of plasma calcium rise, PTH secretion is decreased.

Explain step 3 of the citric acid cycle, alpha-ketoglutarate and CO2 formation

Rate limiting step** Isocitrate is first oxidized to oxalosuccinate by isocitrate dehydrogenase. Then oxalosuccinate is decarboxylated to produce alpha-ketoglutarate and CO2. This is a very important step to know because isocitrate dehydrogenase is the rate limiting enzyme of the citric acid cycle. The first of the two carbons from the cycle is lost here. This is also the first NADH produced from intermediates in the cycle.

What is reciprocal development?

Reciprocal development-most tissues will be exposed to multiple inducers during the course of development

Explain cell regeneration explain complete vs incomplete regeneration. Which one do humans exhibit? Explain.

Regenerative capacity, or the ability of an organism to regrow certain parts of the body, varies from species to species. - some species like salamanders have an enhanced capacity to regenerate because they retain extensive clusters of stem cells within their bodies. complete regeneration-the lost or damaged tissues are replaced with identical tissues (salamanders) incomplete- the newly formed tissue is not identical instructor or function to the tissues that has been injured or lost (typically humans) Humans typically exhibit incomplete regeneration in response to injury. However, in humans regenerative capacity varies by the tissue type. Liver tissue has a high regenerative capacity. Living donors are often able to donate up to 50% of their liver tissue b/c their own livers will regenerate the missing portion. Unfortunately, the heart has little if any regenerative capacity. Scarring often results following an injury due to an event such as a heart attack.

Explain the last step of muscle contraction?

Relaxation. Ach is degraded in the synapse by the enzyme known as acetylcholinesterase. This results in termination of the signal at the neuromusclular junction and allows the sarcolemma to repolarize. As the signal decays, calcium release ceases, and the SR takes up calciujm from the sarcoplasm. The SR tightly controls intracellular calcium [ ]s so that muscles are contract only when necessary. ATP binds to the myosin heads, freeing them from actin. Once the myosin and actin disconnect, the sarcomere can return to its original width. Without calcium, the myosin-binding sites are covered by tropomyosin and contraction is prevented.

Explain smooth muscle. Responsible for what type of movement? Innervated by what? Where is smooth muscle found? How many nuclei does smooth muscle have? How does it appear when viewed microscopically? Why? Compare smooth muscle and skeletal muscle.

Responsible for involuntary action. Thus, smooth muscle is innervated by the autonomic nervous system. It is found in the respiratory tree, digestive tract, bladder, uterus, blood vessel walls, and many other places. Smooth muscle cells have a single nucleus located in the center of the cell. Just like skeletal muscle, smooth muscle cells contain actin and myosin, but the fibers are not as well organized, so striations cannot be seen. Compared to skeletal muscle, smooth muscle is capable of more sustained contractions; a constant state of low-level contraction, as may be seen in the blood vessels, is called tonus. Smooth muscle can actually contract without nervous system input in what is known as myogenic activity. In this case, the muscle cells contract directly in response to stretch or other stimuli.

Explain the structure and importance of sphingolipids. What are the classes?

Sphingolipids are important constituents of cell membranes. Although they don't contain glycerol, they are similar in structure to glycerophospholipids, in that they contain a hydrophilic region and two fatty acid-derived hydrophobic tails. The various classes of sphingolipds differ primarily in the identity of their hydrophilic regions. Classes of sphingolipids and their hydrophilic groups include ceramide, sphingomyelins, cerebrosides, and gangliosides.

Explain the sensorimotor stage of cognitive development

Stage 1 of cognitive development Birth-->2 years old A child learns to manipulate their environment to meet physical needs. Two different types of circular reactions begin. Primary circular reactions-repetition of a body movement that originally occurred by chance, such as sucking the thumb; usually repeated b/c the child finds it soothing. Secondary circular reactions-occur when manipulation is focused on something outside the body, such as repeatedly throwing toys from a high chair. These behavior are often repeated b/c a kid gets a response from the environment (parent picking up the toy) The key milestone that ends the sensorimotor stage is the development of object permanence. Object permanence marks the beginning of representational thought, in which the child has begun to create mental representations of external objects and events.

Explain the concrete operation stage of cognitive development

Stage 3 7-11 years the kid can understand conservation and consider the perspectives of others. Additionally, they are able to engage in logical thought as long as they are working with concrete objects or info that is directly available. These children have not yet developed the ability to think abstractly.

What are the stop and start codons?

Start: AUG Stop: UAG, UAA, UGA UAA- U Are Annoying UGA- U Go Away UAG- U Are Gone

Explain the conduction pathway of the cardiac muscle cells.

Starting at the sinoatrial (SA) node, depolarization spreads using conduction pathways to the atrioventricular (AV) node. From there, the depolarization spreads to the bundle of His and its branches, and then to the Purkinje fibers. The gap junctions allow for progressive depolarizations to spread via ion flow across the gap junctions between cells.

Explain the process of DNA replication

Step 1: lay down an RNA primer -DNA can't be synthesized de novo, meaning it needs another molecule to hook onto. -The primate synthesizes a short primer in the 5' to 3' direction to start replication on each strand. -These short RNA sequences are constantly being added to the lagging strand because each Okazaki fragment must start with a new primer. - In contrast, the leading strand requires only one. Step II: DNA polymerase III (prokaryotes) or DNA polymerases alpha, beta, and epsilon (eukaryotes) will then begin synthesizing the daughter strands of DNA in the 5' to 3' manner. The incoming nucleotides are 5' deoxyribonucleotide triphosphate: dATP, dCTP, dGTP, and dTTP. As the new phosphodiester bond is made, a free pyrophosphate (PPi) is released. Step III: RNA is removed -this is accomplished by the enzyme DNA polymerase I (prokaryotes) or RNase H (eukaryotes), which removes the RNA and then adds DNA nucleotides where the RNA primer had been. Step IV: DNA ligase seals the ends of the DNA molecules together, creating one continuous strand of DNA

Explain Complex II of the electron transport chain

Succinate-CoQ oxidoreductase succinate + FAD --> fumarate + FADH2 FADH2 + Fe-Soxidized --> FAD + Fe-Sreduced Fe-Sreduced + CoQ + 2H+ --> Fe-Soxidized + CoQH2 NET: succinate + CoQ + 2H+ --> fumarate + CoQH2 Just like complex I, complex II transfers electrons to coenzyme Q. While complex I received electrons from NADH, complex II actually receives electrons from succinate. Remember that succinate is a citric acid cycle intermediate, and that it is oxidized to fumarate upon interacting with FAD. FAD is covalently bonded to complex II, and once succinate is oxidized, it's converted to FADH2. After this, FADH2 gets reoxidized to FAD as it reduces an iron-sulfur protein. The final step reoxidizes the iron-sulfur protein as coenzyme Q is reduced. Because succinate dehydrogenase was responsible for oxidizing succinate to fumarate in the citric acid cycle, it makes sense that succinate dehydrogenase is also a part of complex II. It should be noted that no hydrogen pumping occurs here to contribute to the proton gradient.

Explain the difference between bacterial and eukaryotic DNA and what that means for replication

The bacterial chromosome is a closed, double-stranded circular DNA molecule with a single origin of replication. Thus, there are 2 replication forms that move away from each other in opposite directions around the circle. The two replication forks eventually meet, resulting in the production of two identical circular molecules of DNA. Eukaryotic replication must copy many more bases compared to prokaryotes and is a slower process. In order to duplicate all of the chromosomes efficiently, each eukaryotic chromosome contains one linear molecule of double-stranded DNA having multiple origins of replication. As the replication forms move toward each other and sister chromatids are created, the chromatids will remain connected at the centromere. `

Explain the implantation process of a blastocyst

The blastula moves through the fallopian tube to the uterus, where it burrows into the endometrium. The trophoblast cells are specialized to create an interface between the maternal blood supply and the developing embryo. These trophoblastic cells give rise to the chorion, an extraembryonic membrane that develops into the placenta. The trophoblasts form chorionic villi, which are microscopic fingerlike projections that penetrate the endometrium. As these develop into the placenta they support maternal-fetal gas exchange. The embryo is connected to the placenta by the umbilical cord, which consists of two arteries and one vein encased in a gelatinous substance. The vein carries freshly oxygenated blood rich with nutrients from the placenta to the embryo. The umbilical arteries carry deoxygenated blood and waste to the placenta for exchange. Until the placenta is functional, the embryo is supported by the yolk sac, which is also the site of early blood cell development. There are two other exraembryonic membranes that require discussion: the allantois and amnion. The allantois is involved in early fluid exchange between embryo and yolk sac. Ultimately, the umbilical cord is formed from the remnants of the yolk sac and the allantois. The allantois is surrounded by the ambion, a thin, tough membrane filled with amniotic fluid. This fluid serves as a shock absorber during pregnancy, lessening the impact of maternal motion on the developing embryo. In addition to forming the placenta, the chorion also forms an outer membrane around the amnion, adding an additional level of protection.

What is vital capacity (VC)?

The difference between the minimum and maximum volume of air in the lungs (TLC-RV)

What is the difference between an enveloped protein coat on a virus and a non-enveloped protein coat?

The envelope is very sensitive to heat, detergents, and desiccation, so they are easier to kill. Non-enveloped viruses are more resistant to sterilization and are likely to persist on surfaces for longer.

Explain step 8 of the citric acid cycle, oxaloacetate formed anew.

The enzyme malate dehydrogenase catalyzes the oxidation of malate to oxaloacetate. A third and final molecule of NAD+ is reduced to NADH. The newly formed oxaloacetate is ready to take part in another turn of the citric acid cycle, and we've gained all of the high-energy electron carries possible from one turn of the cycle.

How does the placental barrier help maintain fetal immunity?

The fetus is immunologically naive b/c it has not yet been exposed to any pathogens; however, accidental exposure can happen in utero. Thus, the crossing of antibodies across the placental membrane can serve a proactive function. The placenta also qualifies as an endocrine organ b/c it produces progesterone, estrogen, and human chorionic gonadotropin (hCG) which are all essential for maintaining pregnancy.

What are chylomicrons? What are they soluble in? What is their function? Where does their assembly occur and what does it result in?

The lowest density form of lipoprotein. Highly soluble in both lymphatic fluid and blood and function in the transport of dietary triacylglycerols, cholesterol, and cholesteryl esters to other tissues. Assembly of chylomicrons occurs in the intestinal lining and results in a nascent chylomicron that contains lipids and apolipoproteins.

What is oxygen saturation?

The percentage of hemoglobin molecules carrying oxygen- easily measured by a finger probe.

How does the hypothalamus interact with the posterior pituitary? What hormones does the posterior pituitary release and what do they do?

The posterior pit does not receive tropic hormones through the hypophyseal portal system. Neurons in the hypothalamus send their axons down the pituitary stalk directly into the post pit, which can then release oxytocin and antidiuretic hormone. Oxytocin stimulates uterine contractions during labor, as well as milk letdown during lactation. There is evidence that oxytocin is also involved in bonding behavior. ADH (vasopressin) increases reabsorption of water in the collecting ducts of the kidneys. ADH is secreted in response to inc plasma osmolarity or inc [ ] of solutes within the blood

What is the primary job of the kidneys? What are its two main goals? What are the three processes of the kidneys? Explain the basic function of each.

The primary job is to regulate blood volume and osmolarity. Two main goals: Keep what the body needs and lose what it doesn't and concentrate the urine to conserve water. Kidney function may be divided into three different processes: filtration (movement of solutes from blood to filtrate at Bowman's capsule), secretion (movement of solutes from blood to filtrate anywhere besides Bowman's capsule), and reabsorption (movement of solutes from filtrate to blood).

What do the seminal vesicles, prostate gland, and bulbourethral (Cowper's) glands do?

The seminal vesicles contribute fructose to nourish sperm, and both the seminal vesicles and the prostate gland give the fluid mildly alkaline properties so the sperm can survive in the relative acidity of the female reproductive tract. The bulbourethral glands produce a clear viscous fluid that cleans out any remnants of urine and lubricates the urethra during sexual arousal. The combination of sperm and seminal fluid is known as semen.

Explain the primary structure of proteins encodes what?

The sequence of amino acids in the polypeptide chain-the identity and specific order of amino acids. this alone encodes all the info needed for folding at all the higher structural levels.

What are the DNA polymerases in eukaryotic cells and what are their functions

There are 5 classic DNA polymerases in eukaryotic cells: alpha, beta, gamma, delta, and epsilon -DNA polymerase alpha, delta, and epsilon work together to synthesize both the leading and lagging strands; DNA polymerase delta also fills in the gaps left behind when RNA primers are removed - DNA polymerase gamma replicates mitochondrial DNA - DNA polymerase beta and epsilon are important to the process of DNA repair - DNA polymerases delta and epsilon are assisted by the PCNA protein, which assembles into a trimer to form the sliding clamp. The clamp helps to strengthen the interaction between these DNA polymerases and the template strand

Where are the adrenal glands? What does each adrenal gland consist of? What is the difference between the parts of the adrenal gland?

They are located on top of the kidneys (adrenal actually means next to the kidney) each gland consists of a cortex and a medulla each part of the gland is responsible for the secretion of different hormones. adrenal cortex-secretes corticosteroids (glucocorticoids, mineralocorticoids, and cortical sex hormones) adrenal medulla-a derivative of the nervous system; responsible for producing epi and norepi (catecholamines)

What are intermediate filaments?

They are one of the 3 elements of the cytoskeleton. They are a diverse group of filamentous proteins, which includes keratin, design, vimentin, and lamins. Many of them are involved in cell-cell adhesion or maintenance of the overall integrity of the cytoskeleton. They are able to withstand a tremendous amount of tension, increasing the structural rigidity of the cell. They also help anchor other organelles, including the nucleus. The identity of the intermediate filament proteins within a cell is specific to the cell and tissue type

Threonine: 3 letter abbreviation, 1 letter abbreviation, polar or non polar

Thr. T. Polar

Explain autoimmunity. How does the body strive to prevent autoimmune reactions? How are most autoimmune diseases treated?

Under normal circumstances, self-antigens (the proteins and carbohydrates present on the surface of every cell of the body), signal to immune cells that the cell is not foreign and should not be attacked. However, when the immune system fails to make the distinction between self and foreign, it may attack cells expressing particular self-antigens, a condition known as autoimmunity. T cells are educated in the thymus, where they mature. Part of this education involves elimination of T-cells that respond to self-antigens, called negative selection. Immature B-cells that respond to self antigens are eliminated before they leave the bone marrow. One common example is glucocorticoids (modified versions of cortisol) which have potent immunosuppressive qualities.

What is diabetes mellitus? What are the downstream effects Explain the 2 types.

Underproduction, insufficient secretion, or insensitivity to insulin all can result in diabetes mellitus, which is clinically characterized as hyperglycemia (excess glucose in blood). In the kidneys, excessive glucose in the filtrate will overwhelm the nephron's ability to reabsorb glucose, resulting in its presence in urine. Because it is an osmotically active particle and does not readily cross the cell membrane, the presence of glucose in the filtrate leads to excess excretion of water and an increase in urine volume. - Therefore, diabetics often report polyuria (inc frequency of urination) and polydipsia (inc thirst). There are 2 types. Type I (insulin-dependent) is caused by autoimmune destruction of the beta-cells of the pancreas, resulting in low or absent insulin production. -Type 1 diabetics require regular injections of insulin to prevent hyperglycemia and to permit uptake of glucose into cells. Type II (non insulin-dependent) is the result of receptor-level resistance to the effects of insulin. -partially inherited and partially due to environmental factors such as high carb diets and obesity.

Explain the process of exhalation.

Unlike inhalation, exhalation doesn't have to be an active process. Simple relaxation o the external intercostal muscles will reverse the processes in inhalation. As the diaphragm and external intercostals relax, the chest cavity decreases in volume. Pressure increases in the intrapleural space (per Boyle's law). Now pressure in the intrapleural space is higher than in the lungs, which is still at atmospheric pressure. Thus, air will be pushed out, resulting in exhalation. During active tasks, we can speed up this process by using the internal intercostal muscles and abdominal muscles, which oppose the external intercostals and pull the rib cage down, actively decreasing the volume of the thoracic carivity. Surfactant prevents the complete collapse of the alveoli during exhalation by reducing surface tension at the alveolar surface.

explain the differences between unsaturated and saturated fatty acids

Unsaturated fatty acids are regarded as "healthier" fats because they tend to have one or more double bonds and exist in liquid form at room temp; in the plasma membrane, these characteristics impart fluidity to the membrane. Humans can only synthesize a few of the unsaturated fatty acids; the rest come from essential fatty acids in the diet that are transported as triacylglycerols from the intestine inside chylomicrons. Two important essential fatty acids for humans are alpha-linolenic acid and linoleic acid. Saturated fatty acids are the main components of animal fats and tend to exist as solids at room temperature. Saturated fats are found in processed foods and are considered less healthy. When incorporated into phospholipid membranes, saturated fatty acids decrease the overall membrane fluidity.

Digestion requires exposure of lumen contents to the inner surface of the digestive tract. The inner surface of the digestive tract is increased in surface area by: A. relaxation of the pyloric sphincter B. stomach mucosal cells C. villi and convolutions D. absorptive flagella

Villi increase surface area in the small intestine by extending small "finger-like" projections from the epithelial lining into the lumen. Microvilli are also present on the villi, further increasing surface area for digestion. Convolutions are folds along the length of the digestive tract which further increase surface area in comparison with a large diameter linear geometry. A. relaxation of the pyloric sphincter, incorrect, Relaxation of the pyloric sphincter allows passage of chyme from stomach to duodenum. B. stomach mucosal cells, incorrect, Stomach mucosal cells secrete mucous and have no effect on digestive surface area. C. villi and convolutions, correct. D. absorptive flagella, incorrect, There is no such thing as an absorptive flagellum.

What is the difference between white and grey matter of the brain?

White matter consist of axons encased in myelin sheaths Grey matter is unmyelenated cell bodies and dendrites. White matter lies deeper than grey matter in the brain.

what is the structure of an antibody?

Y shaped molecule, four polypeptide chains: two heavy identical chains and two light identical chains held together by a disulfide bond, four constant regions (C), and four variable regions (V) Each body has an antigen-binding region at the tips of the "Y". Within this region, there are specific polypeptide sequences that will bind one, and only one, specific antigenic sequence. The remaining part of the antibody molecules is known as the constant region, which is involved in recruitment and binding of other cells of the immune system, such as macrophages.

What form of energy is in a readily available for the cell to use?

adenosine triphosphate (ATP)

What are cortical sex hormones and what is their function? Where are they secreted from?

a class of corticosteroids, secreted from the adrenal cortex they are androgens and estrogens because the testes already secrete large quantities of androgens, adrenal testosterone plays a relatively small role in male physiology. But because the ovaries secrete far smaller amounts of androgens, females are much more sensitive to disorders of cortical sex hormone production.

What is aphasia? What is the difference between Broca's aphasia, Wernicke's aphasia, and conduction aphasia?

a deficit of language production of comprehension Broca's (expressive) aphasia- can understand but can't speak Wernicke's (receptive) aphasia- can speak but cannot understand. Patient's believe they are speaking and understanding well, but people around them cannot comprehend what is being said. conduction aphasia-if the arcuate fasciculus (area connecting Broca's and Wernicke's areas) is damaged. The patient is unable to repeat something that has been said because the connection bdtween these two regions has been lost. Rare.

What is extensor vs a flexor muscle?

a flexor muscle decreases the angle across a joint (like the biceps brachii); an extensor increases or straightens this angle (like the triceps brachii).

Explain hypnosis

a state in which a person appears to be in control of his or her normal functions, but is in a highly suggestible state. Hypnosis starts with hypnotic induction, in which the hypnotist seeks to relax the subject and increase the subject's concentration. then, the hypnotist can suggest perceptions or action to the hypnotized person.

What is a gene? How is it expressed?

a unit of DNA that encodes a specific protein or RNA molecule, and through transcription and translation, that gene can be expressed

what are the three subunits that comprise the G protein? explain them.

alpha (a), beta (B), and gamma (Y). it its inactive form, the a subunit binds GDP and is in a complex with the B and Y subunits. When a ligand binds to the GPCR, the receptor becomes activated and, in turn, engages the corresponding G protein. Once GDP is replaced with GTP, the a subunit is able to dissociated from the B and Y subunits. The activated a subunit alters the activity of adenylate cyclase. If the a subunit is a (sub s), then the enzyme is activated; if the a subunit is a (sub i), then the enzyme is inhibited. Once GTP on the activated a subunit is dephosphorylated to GDP, the a subunit will rebind to the B and Y subunits, rendering the G protein inactive

What is bile and what are bile salts? Explain both of their importance in digestion.

bile is a complex fluid composed of bile salts, pigments, and cholesterol. Bile salts are derived from cholesterol. They are not enzymes and therefore do not directly perform chemical digestion (the enzymatic cleavage of chemical bonds). However, bile salts serve an important role in the mechanical digestion of fats and ultimately facilitate the chemical digestion of lipids. Bile salts have hydrophobic and hydrophilic regions, allowing them to serve as a bridge between aqueous and lipid environments. In fact, bile salts are much like the common soaps and detergents we use to wash our hands, clothes, and dishes. In the small intestine, bile salts emulsify fats and cholesterol into micelles. Without bile, fats would spontaneously separate out of the aqueous mixture in the duodenum and would not be accessible to pancreatic lipase, which is water-soluble. In addition, these micelles increase the surface area of the fats, increasing the rate at which lipase can act. Ultimately, proper fat digestion depends on both bile and lipase. Bile gets the fats into the solution and increases their surface area by placing them in micelles (mechanical digestion). Then lipase can come in to hydrolyze the ester bonds holding the lipids together (chemical digestion). the major pigment in bile is bilirubin. If the liver is unable to process or excrete bilirubin (due to liver damage, excessive red blood cell destruction, or blockage of the bile ducts), jaundice may occur.

what are binding proteins?

bind a specific substrate, either to sequester it in the body or hold its concentration at steady state includes hemoglobin, calcium-binding proteins, DNA-binding proteins (often transcription factors) and others. Each binding protein has an affinity curve for its molecule of interest; the oxyhemoglobin curve is one well-known example. The curve differs depending on the goal of the binding protein

In amino acids, the pKa for the carboxyl group is around _____. The pKa for the amino group is around _____.

carboxyl group: pKa=2 amino group: pKa= 9-10

Explain arteries structure and role in the cardiovascular system.

carry blood away from the heart. highly muscular and elastic, creating tremendous resistance to the flow of blood. This is one of the reasons why the left heart must generate much higher pressures: to overcome the resistance caused by systemic arteries. After arteries are filled with blood, the elastic recoil from their walls maintains a high pressure and forces blood forward.

what are cell-cell junction generally comprised of? what are these things?

cell adhesion molecules (CAM), which are proteins that allow cells to recognize each other and contribute to proper cell differentiation and developmen

What is isotype switching?

cells can change which isotype of antibody they produce when stimulated by specific cytokines

Most bacteria exist in 1 of 3 shapes. What are they?

cocci (spherical), bacilli (rod shaped), spirilli (spiral shaped)

Many enzymes require nonprotein molecules called ______ or ______ to be effective. They tend to be ______ (small/large) in size.

cofactors or coenzymes small

What is the difference between complete dominance, codominance, and incomplete dominance?

complete dominance is when only one dominant and one recessive allele exists for a given gene. In this case, the presence of one dominant allele will mask the recessive allele, if present. When more than one dominant allele exists for a given gene there is codomiannce. Ex: AB bloodtype. Incomplete dominance occurs when a heterozygote expresses a phenotype that is intermediate between the two homozygous genotypes. A classic example is the mating of red and white flower which gives pink.

What are clots? How are they formed? How are they broken down?

composed of both coagulation factors (proteins) and platelets, and they prevelt or at least minimize blood loss. When the endothelium of a blood vessel is damaged, it exposes the underlying connective tissue, which contains a collagen and protein called tissue factor. When platelets come into contact with exposed collagen, they sense this as evidence of injury. In response, they release their contents and begin to aggregate, or clump together. Simultaneously coagulation factors, most of which are secreted by the liver, sense tissue factors and initiate a complex activation cascade. The endpoint of the cascade is the activation of prothrombin to form thrombin by thromboplastin. Thrombin can then convert fibrinogen into fibrin. Fibrin ultimately forms small fibers that aggregate and cross-link into a woven structure, like a net, that captures RBCs and other platelets, forming a stable clot of the area of damage. Breaking down clots is accomplished by plasmin, which is generated from plasminogen.

what are optical isomers (stereoisomers)? What are enantiomers?

compounds that have the same chemical formula; these molecules differ from one another only in terms of the spatial arrangement of their component atoms. A special type of isomerism exists between stereoisomers that are nonidentical, nonsuperimposable mirror images of each other. These molecules are enantiomers. Any molecule that contains chiral carbons and no internal planes of symmetry has an enantiomer.

what is translation?

converting the mRNA transcript into a functional protein.

ATP hydrolysis is most likely to be encountered in what context?

coupled reactions. Many coupled reactions use ATP as an energy cource.

A secondary oocyte is ovulated from the follicle on approximately day ____ of the menstrual cycle. explain the process of fertilization.

day 14 The secondary oocyte travels into the fallopian tube, where it can be fertilized up to 24 hours after ovulation. Fertilization, usually occurs in the widest part of the fallopian tube, called the ampulla. When the sperm meets the secondary oocyte in the fallopian tube, it binds to the oocyte and releases acrosomal enzymes that enable the head of the sperm to penetrate the corona radiata and zona pellucida. The first sperm to come into direct contact with the secondary oocyte's cell membrane forms a tubelike structure known as the acrosomal apparatus, which extends to an penetrates the cell membrane. Its pronucleus may then freely enter the oocyte once meiosis II has come to completion. After penetration of the sperm through the cell membrane, the cortical reation, a release of calcium ions occurs. These calcium ions depolarize the membrane of the ovum, which serves two purposes: depolarization prevents fertilization of the ovum by multiple sperm cells, and the inc calcium [ ] inc the metabolic rate of the newly formed diploid zygote. The now depolarized and impenetrable membrane is called the fertilization membrane

What are terpenoids? What characteristics do they share with terpenes?

derivatives of terpenes that have undergone oxygenation or rearrangement of the carbon skeleton share similar characteristics with terpenes in terms of both biological precursor function and aromatic properties, contributing to steroid biosynthesis as well as the scenes of cinnamon, eucalyptus, camphor, turmeric, and lots of other compounds.

The bladder has a muscular lining known as the ______________. What does parasympathetic activity to to this thing?

detrusor muscle. causes the detrusor muscle to contract.

What is digestion? What are the two digestive processes and what is the difference between them?

digestion involves the breakdown of food into its constituent organic molecules: starches and other carbohydrates into monosaccharides, lipids (fats) into free fatty acids and glycerol, and proteins into amino acids. Digestion can be subdivided into mechanical and chemical processes. Mechanical digestion is the physical breakdown of large food particles into smaller food particles, but does not involve breaking chemical bonds. Chemical digestion is the enzymatic cleavage of chemical bonds, such as the peptide bonds of proteins or the glycosidic bonds of starches.

how do calluses form and what is their point?

excess keratin deposition in areas of repeated strain due to friction; they provide protection to avoid damage in the future.

What are exoskeletons and endoskeletons? Explain who has what and what the differences are.

exoskeletons encase whole organisms and are usually found in arthropods such as crustaceans and insects. Vertebrates, including humans, have endoskeletons. Endoskeletons are internal, but are not able to protect the soft tissue structures as well as exoskeletons. However, exoskeletons must be shed and regrown to accomodate growth. Endoskeletons are better able to accommodate the growth of a larger organism.

Explain Hill's coefficient

explains enzyme cooperatively. The value of Hill's coefficient indicates the nature of binding by the molecule: - If Hill's coefficient > 1, positively cooperative binding is occurring, such that after one ligand is bound the affinity of the enzyme for further ligand(s) increases. -If Hill's coefficient < 1, negatively cooperative binding is occurring, such that after one ligand is bound the affinity of the enzyme for further ligand(s) decreases -If Hill's coefficient = 1, the enzyme does not exhibit cooperative binding

What is the postabsorptive state? What molecules oppose the actions of insulin? What are they called and why? What happens in this state?

fasting state. glucagon, cortisol, epi, norepi, and GH oppose the actions of insulin. they are sometimes termed counter-regulatory hormones because of their effects on skeletal muscle, adipose tissue, and the liver, which are opposite to the actions of insulin. In the liver, glycogen degradation and the release of glucose into the blood are stimulated. Hepatic gluconeogenesis is also stimulated by glucagon, but the response is slower than that of glycogenolysis. Whereas glycogenolysis begins almost immediately at the beginning of the postabsorptive state, glyconeogenesis takes about 12 hours to hit maximum velosity. The release of amino acids from skeletal muscle and fatty acids from adipose tissue are both stimulated by the decrease in insulin and by an increase in levels of epinephrine. Once carried into the liver, amino acids and fatty acids can provide the necessary carbon skeletons and energy required for gluconeogenesis.

What is Vitamin K? Explain its importance.

fat-soluble vitamin actually a group of compounds, including phylloquinone (K1) and the menaquinone (K2). - vital to the post translational modifications required to form prothrombin, an important clotting factor in the blood. - The aromatic ring of vitamin K undergoes a cycle of oxidation and reduction during the formation of prothrombin. Vitamin K is also required to introduce calcium-binding sites on several calcium-dependent proteins

What are fatty acids and triacylglycerols? What are the two types of fatty acid chains?

fatty acids are carboxylic acids that contain a hydrocarbon chain and terminal carboxyl group triacylglycerols (triglycerides) are storage lipids involved in human metabolic processes. They contain three fatty acid chains esterified to a glycerol molecule. Fatty acid chains can be either saturated or unsaturated.

what is disruptive selection

favors individuals at both extremes of the phenotypic range

Explain the role of fructose-1,6-bisphosphatase in gluconeogenesis.

fructose-1,6-bisphosphatase in the cytoplasm is a key control point of gluconeogenesis and represents the rate-limiting step of the process. It reverses the action of phosphofructokinase-1, the rate limiting step of glycolysis, by removing phosphate from fructose 1,6-bisphosphate to produce fructose 6-phosphate. A common pattern to note is that phosphates oppose kinases. Fructose-1,6-bisphosphatase is activated by AMP and fructose-2,6-bisphosphate, This should make sense: high levels of ATP imply that a cell is energetically satisfied enough to produce glucose for the rest of the body, whereas high levels of AMP imply that a cell needs energy and cannot afford to produce energy for the rest of the body before satisfying its own requirements. Fructose-2,6-bisphosphate (F2,6-BP) is sometimes though tof as a marker for satisfactory energy levels in liver cells. It helps these cells override the inhibition of phosphofructokinase-1 that occurs when high levels of acetyl-CoA are formed, signaling to the liver cell that it should shift its function from burning to storing fuel. F2,6-BP produced by PFK2 controls both gluconeogenesis and glycolysis (in the liver). Recall from the earlier discussion of this enzyme that PFK-2 is activated by insulin and inhibited by glucagon. Thus, glucagon will lower F2,6-BP and stimulate gluconeogenesis, whereas insulin will increase F2,6-BP and inhibit gluconeogenesis.

Ionizable groups tend to ____ protons under acidic conditions and ___ them under basic conditions. So, in general, at low pH, ionizable groups tend to be _______, at high pH they tend to be _____.

gain- acidic lose- basic low pH- protonated high pH- deprotonated

meiosis occurs in ____ and results in what?

gametocytes (germ cells) and results in up to four nonidentical sex cells (gametes)

what is genetic leakage?

genetic leakage is a flow of genes between species. in some cases individuals from different but closely related species can mate to produce hybrid offspring.

high levels of what molecules inhibit and promote the citric acid cycle?

high levels of ATP and NADH inhibit the citric acid cycle, high levels of DAP and NAD+ promote it

Explain immovable joints vs movable joints.

immovable joints consist of bones that are fused together to form sutures or similar fibrous joints. These joints are found primarily in the head, where they anchor bones of the skull together. Movable joints include hinge joints (like elbow or knee), ball-and-socket joints (like the shoulder or hip), and others. They permit bones to shift relative to one another. Movable joints are strengthened by ligaments, which are pieces of fibrous tissue that connect bones to one another, and consist of a synovial capsule, which encloses the actual joint cavity (articular cavity). A layer of soft tissue called the synovium secretes synovial fluid, which lubricates the movement of structures in the joint space. The articular cartilage contributes to the joint by coating the articular surfaces of the bones so that impact is restricted to the lubricated joint cartilage, rather than to the bones.

How does insulin impact the metabolism of carbohydrates?

insulin increases the uptake of glucose and increases carbohydrate metabolism in muscle and fat. Increased glucose in muscle can be used as additional fuel to burn during exercise, or can be stored as glycogen. Insulin also increases glycogen synthesis in the liver by increasing the activity of glucokinase and glycogen synthase, while decreasing the activity of enzymes that promote glycogen breakdown (glycogen phosphorylase and glucose-6-phosphatase).

Insulin is a _______ hormone secreted by what? What it is a key player in? Glucose is absorbed by what via what type of transport? The tissues that require insulin for effective uptake of glucose are what two tissue types? What are the 5 tissues in which glucose uptake is not affected by insulin?

insulin is a peptide hormone secreted by the B-cells fo the pancreatic islets of Langerhans. It is a key player in the uptake and storage of glucose. Glucose is absorbed by peripheral tissues via facilitated transport mechanisms that utilize glucose transporters located in the cell membrane. The tissues that require insulin for effective uptake of glucose are adipose tissue and resting skeletal muscle. Tissues in which glucose uptake is not affected by insulin include: nervous tissue, kidney tubules, intestinal mucosa, RBCs(erythrocytes), B-cells of the pancreas

What is the endoplasmic reticulum (ER)? What are the two types? What are their differences?

it is a series of interconnected membranes that are contiguous with the nuclear envelope. smooth and rough ER. rough ER (RER)- studded with ribosomes, which permit the translation of proteins destined for secretion into the lumen smooth ER (SER)- lacks ribosomes, used primarily for lipid synthesis (such as the phospholipids In the cell membrane) and the detoxification of certain drugs and poisons. The SER also transports proteins from the RER to the Golgi apparatus

explain alcohols effects on the brain

it's a depressant increases activity of the GABA receptor, a chloride channel that causes hyperpolarization of the membrane. This causes generalized brain inhibition, resulting in diminished arousal at moderate doses. Behavior may seen less inhibited because the centers of the brain that prevent inappropriate behavior are also depressed. Alcohol also increases dopamine levels, causing a sense of mild euphoria. At higher doses, brain activity becomes more disrupted. Logical reasoning and motor skills are affected, and fatigure may result. One of the main effects on logical reasoning is the inability to recognize the consequences of actions, creating a short-sighted view of the world called alcohol myopia.

describe a protein's tertiary structure.

its 3-dimensional shape. mostly determined by hydrophilic and hydrophobic interactions between R groups.

What is the locus of a gene?

its location of a gene on a chromosome

What is hybridization (DNA)?

joining complementary base pair sequences. Can be DNA-DNA recognition or DNA-RNA recognition. This technique uses two single-stranded sequences and is a vital part of PCR and southern blotting.

What are the two broad functions of the mitochondria?

keeping the cell alive by providing energy killing the cell by releasing enzymes from the electron transport chain, kick-starting apoptosis.

explain narcolepsy

lack of voluntary control over the onset of sleep. The symptoms include cataplexy (a loss of muscle control and sudden intrusion of REM sleep during waking hours, usually caused by an emotional trigger), sleep paralysis (a sensation of being unable to move despite being awake), and hypnagogic and hypnopompic hallucnations, which are hallucinations when going to sleep or awakening.

Explain the function of B cells in the immune system and where they are located.

located in the spleen. they turn into plasma cells to produce antibodies as a part of adaptive immunity. Note that when B-cells leave the bone marrow, they are considered mature but naïve (because they have not yet been exposed to an antigen). Because these antibodies dissolve and act in the blood (rather than within cells), this division of adaptive immunity is called humoral immunity.

What are flagella? What does it do? What is it composed of?

long, whiplike structures that can be used for propulsion (bacteria may have 1, 2, or many flagella, depending on the species). Flagella can be used to move towards food or away from toxins or immune cells. (The ability of a cell to detect chemical stimuli and move toward or away from them is called chemotaxis). composed of a filament, a basal body, and a hook. The filament is a hollow, helical structure composed of flagellin. The basal body is a complex structure that anchors the flagellum, which rotates. The hook connects the filament and the basal body so that as the basal body rotates, it exerts torque on the filament, which spins and propels the bacterium forward.

cofactors and coenzymes are usually kept at what concentrations in cells? why? cofactors are generally what types of things? give examples. how are they ingested? coenzymes are generally what types of things?

low concentrations so they can be recruited only when needed cofactors- generally inorganic molecules or metal ions, and are ingested often as dietary minerals. coenzymes-small organic groups, the vast majority of which are vitamins or derivatives of vitamins such as NAD+, FAD, and coenzyme A. The water-soluble vitamins include the B complex vitamins and ascorbic acid (vitamin C) and are important coenzymes that must be replinished regularly b/c they are excreted easily. the fat-soluble vitamins- A, D, E, and K-are better regulated by partition coefficients, which quantify the ability of a molecule to dissolve in a polar vs non polar environment.

What is a surfactant? Explain how this is relevant to soaps.

lowers the surface tension at the surface of a liquid, serving as a detergent or emulsifier This is important to how soap works. If we try to combine oil and an aqueous solution (vinegar and olive oil in salad dressing), these solutions will remain in separate places. If you add a soap, the two phases would appear to combine in to a single phase, forming a colloid. -This occurs because of the formation of micelles: tiny aggregates of soap with the hydrophobic tails turned inward and they hydrophobic heads turned outward, thereby shielding the hydrophobic lipid tails and allowing for overall salvation.

Which of the following organelles is surrounded by a single membrane? A. lysosomes C. mitochondria C. nuclei D. ribosomes

lysosomes

What can a spirometer do?

measure many lung volumes (except residual volume)

explain lipid absorption

micelles diffuse to the brush border of the intestinal mucosal cells where they are absorbed. the digested lipids pass through the brush border, where they are absorbed into the mucosa and re-esterified to form triactylglycerols and cholesteryl esters and packaged, along with certain apoprotins, fat-soluble vitamins, and other lipids, into chylomicrons. Chylomicrons leave the intestine via lacteals, the vessels of the lymphatic system, and re-enter the bloodstream via the thoracic duct, a long lymphatic vessel that empties into the left subclavian vein at the base of the neck. The more water-soluble short-chain fatty acids can be absorbed by simple diffusion directly into the bloodstream.

how is water absorbed into the small intestine?

much of the water in chyme is actually the result of secretions. The avg person may consume up to 2 liters of fluid per day, but secretions into the upper GI tract may total up to seven liters of fluid per day. In order to maintain proper fluid levels within the body, much of this fluid must be reabsorbed by osmosis. As solutes are absorbed into the bloodstream, water is drawn with them, eventually reaching the capillaries. Water passes not only transcellularly (across the cell membrane) but also paracellularly (squeezing between cells) to reach the blood.

What is myogenic activity? What types of muscle exhibit myogenic activity?

muscle can actually contract without nervous system input. smooth muscle and cardiac muscle both exhibit this.

What is mutarotation?

mutarotation is the spontaneous change of configuration about C1 that occurs when monosaccharides are exposed to water (and occurs v. rapidly when catalyzed by acid or base) Because the substituents on the single bond between C1 and C2 can rotate freely, either the alpha or B anomer can be formed.

What are the different types of granulocytes?

neutrophils, eosinophils, basophils. the names of the cells refer to the way the cells appear after staining with certain chemicals.

Would you be expected to have antibodies to the Ebola virus if you were never exposed to it?

no. antibodies are created in response to an antigen, and they specifically target that antigen.

Does glucose produced by hepatic (liver-based) gluconeogenesis represent an energy source for the liver? Why?

no. gluconeogenesis requires expenditure of ATP that is provided by B-oxidation of fatty acids. Therefore hepatic gluconeogenesis is always dependent on B-oxidation of fatty acids in the liver.

What are pyranose and furanose rings?

pyranose are 6 membered rings furanose are 5 membered rings

For amino acids with non-ionizable side chains, the pI is usually around ___. Amino acids with acidic side chains have pI values around what; basic side chains have pI values around what?

non-ionizable: 6 acidic: well below 6 basic: well above 6

explain noncompetitive inhibition does it alter Vmax? does it alter Km?

noncompetitive inhibitors bind to an allosteric site instead of the active site, which induces a change in enzyme conformation. Allosteric sites are non-catalytic regions fo the enzyme that bind regulators B/c the 2 molecules don't compete for the same site, inhibition is considered noncompetitive and cannot be overcome by adding more substrate. Noncompetitive inhibitors bind equally well to the enzyme and the enzyme-substrate complex, unlike mixed inhibitors. Once the enzyme's conformation is altered, no amount of extra substrate will be conductive to forming an enzyme-substrate complex. Adding a noncompetitive inhibitor dec the value of Vmax b/c there is less enzyme available to react doesn't alter Km b/c any copies of the enzyme that are still active maintain the same affinity for their substrate

translocation mutation

occur when a segment of DNA is swapped with a segment of DNA from another chromosome

explain cytokinesis

occurs at the end of telophase (4th phase of mitosis). the separation of the cytoplasm and organelles, giving each daughter cell enough material to survive on its own each cell undergoes a finite # of divisions before programmed death. for human somatic cells, this is usually between 20-50. after than, the cell can no longer divide continuously

Explain opiates and opioids

opiates are naturally occurring forms, including morphine and codeine opioids are semisynthetic derivates, including oxycodone, hydrocodone, and heroin. These compounds bind to opioid receptors in the PNS and CNS, causing a dec reaction to pain and a sense of euphoria. OD can cause death by respiratory suppression, in which the brain stops sending signal to breathe

What is the induced fit model for enzyme catalysis?

opposed to the lock and key model. this is the more scientifically accepted theory of the 2. the induced fit model starts with a substrate and an enzyme active site that don't seem to fit together. However, once the substrate is present and ready to interact with the active site, the molecules find that the induced form, or transition state, is more comfortable for both of them. Thus, the shape of the active site becomes truly complementary only after the substrate begins binding to the enzyme. Similarly, a substrate of the wrong type will not cause the appropriate conformational shift in the enzyme. Thus, the active site will not be adequately exposed, the transition state is not preferred, and no reaction occurs.

what is overconfidence?

overconfidence is a tendency to erroneously interpret one's decisions, knowledge, and beliefs as infallible

Explain Tumor suppressor Genes famous example

p53 or Rb -encode proteins that inhibit the cell cycle or participate in DNA repair processes. -stop tumor progression Control apoptosis unless they are under-active or mutated and results in a cancer promoting environment through unregulated cell growth

Explain how phospholipids and lipid rafts move within the plasma membrane.

phospholipids move rapidly in the plane of the membrane through simple diffusion. Lipid rafts are collections of similar lipids with or without associated proteins that serve as attachment points for other biomolecules; these rafts often serve roles in signaling. Both lipid rafts and proteins also travel within the plane of the membrane, but more slowly. Lipids can also move between the membrane layers, but this is energetically unfavorable because the polar head group of the phospholipid must be forced through the nonpolar tail region in the interior of the membrane. Specialized enzymes called flippases assist in the transition or flip between layers.

enzymes can be activated or deactivated by what processes?

phosphorylation, dephosphorylation also Glycosylation-the covalent attachment of sugar moieties

The cell membrane is composed predominantly of what with some associated what?

predominantly of lipids with some associated proteins and carbohydrates

Explain the structure and function of the large intestine. What is the role of the large intestine in digestion?

primarily involved in water absorption. large diameter but shorter length than the small intestine. divided into three sections: the cecum, the colon, and the rectum. The cecum is an outpocketing that accepts fluid exiting the small intestine through the ileocecal valve and is the site of attachment of the appendix. The appendix is a small fingerlike projection that was once thought to be vestigial. Recent evidence suggests it may have a role in warding off certain bacterial infections and repopulating the large intestine with normal flora after episodes of diarrhea. The colon itself is divided into the ascending, transverse, descending, and sigmoid colons. Its main function is to absorb water and salts from the undigested material left over from the small intestine. The small intestine actually absorbs much more water than the colon, so the colon primarily concentrates the remaining material to form feces. Finally, the rectum serves as a storage site for feces, which consists of indigestible material, water, bacteria, and certain digestive secretions that are not reabsorbed. The anus is the opening through which wastes are eliminated and consists of two sphincters: the internal and external anal sphincters. The external sphincter is under voluntary control but the internal sphincter is under involuntary control.

Proteins have 4 levels of structure:

primary, secondary, tertiary, quaternary

What does the bone marrow do for the immune system?

produces all of the leukocytes (WBCs) that participate in the immune system through a process of hematopoiesis.

What is the most important physiological promoter of glucagon secretion? What is the most important inhibitor? How do amino acids affect glucagon secretino?

promoter is low plasma glucose (hypoglycemia). inhibitor is elevated plasma glucose (hyperglycemia). amino acids, especially basic amino acids (arg, lys, his) also promote the secretion of glucagon.

conjugated proteins derive part of their function from covalently attached molecules called what?

prosthetic groups

Explain the erythrocyte.

red blood cell. designed for oxygen transport. each erythrocyte contains about 250 million molecules of hemoglobin, each of which can bind four molecules of oxygen. Each RBC can carry approximately 1 billion molecules of oxygen. RBCs are biconcave, or indented on both sides, which serves a dual purpose. First, this shape assists them in traveling through tiny capillaries. Second, it increases the cells surface area, which inc gas exchange. When RBCs mature, the nuclei, mitochondria, and other membrane-bound organelles are lost. The loss of organelles makes space for the molecules of hemoglobin. Also, the loss of mitochondria means that the RBC doesn't consume the oxygen it is carrying before it is delivered to the peripheral tissues. Because RBCs lack nuclei, they can't divide.

What are enhancers? Explain how they work. How are the enhancer and promoter brought together.

response elements outside the normal promoter regions can be recognized by specific transcription factors to enhance transcription levels. Several response elements may be grouped together to form an enhancer, which allows for the control of one gene's expression by multiple signals. Signal molecules, such as cAMP, cortisol, and estrogen, bind to specific receptors. For the examples given, these receptors are CREB, the flucocorticoid (cortisol) receptor, and the estrogen receptor; all the transcription factors that bind to their respective response elements within the enhancer. The large distance between the enhancer and promoter regions for a given gene means that DNA often must bend into a hairpin loop to bring these elements together spatially. Enhancers in the DNA can be up to 1000 base pairs away from the gene they regulate and can even be located within an intron, or noncoding region, of the gene. They differ from upstream promoter elements in their locations because upstream promoter elements must be within 25 bases of the start of a gene. by utilizing enhancer regions, genes have an increased likelihood to be amplified because of the variety of signals that can increase transcription levels.

What does oxytocin do?

secreted by the post pit during childbirth and allows for coordinated contraction of uterine smooth muscle. It's secretion may also be stimulated by suckling, as it promotes milk ejection through contraction of smooth muscle in the breast. Finally, oxytocin may be involved in bonding behavior. -oxytocin is unusual in that it has a positive feedback loop: the release of oxytocin promotes uterine contraction, which promotes more oxytocin release, which promotes stronger uterine contractions and so on.

What does ADH do?

secreted by the post pit in response to low blood volume (as sensed by baroreceptors) or inc blood osmolarity (as sensed by osmoreceptors) it's action is at the level of the collecting duct, where it inc permeability of the duct to water. This inc the reabsorption of water from the filtrate in the nephron. This results in greater retention of water, which results in inc blood volume and higher blood pressure

Ghrelin

secreted by the stomach in response to signals of an impending meal (sight, sound, taste, smell). Increases appetite and also stimulates secretion of orexin.

What are transposons?

segments of DNA that can move from one region of DNA to another

What are Jean Piaget's stages of cognitive development?

sensorimotor preoperational concrete operational formal operational

What does the somatic nervous system consist of? What about the autonomic? What is the main difference?

sensory and motor neurons distributed throughout the skin, joints, and muscles. Sensory neurons transmit info through afferent fibers. Motor impulses travel along efferent fibers. ANS regulates heartbeat, respiration, digestion, and glandlual secretions (involuntary muscles). These functions are automatic (independent of conscious control) One difference is that the peripheral component of the ANS contains 2 neurons. By contrast, a motor neuron in the SNS goes directly from the spinal cord to the muscle without synapsing. In the ANS, 2 neurons work in series to transmit messages from the spinal cord. The first neuron Is the preganglionic neuron, the second is the postganglionic neuron. The soma of the preganglionic neuron is in the CNS, and its axon travels to a ganglion in the PNS. Here, it synapse on the cell body of the postganglionic neuron, which then stimulates the target tissue.

how can the primary structure of a protein be determined in a lab?

sequencing

what are isoforms?

slightly different versions of the same protein

how would you analyze the composition of amino acids in a small protein vs a large protein?

small-Edman degradation large- digestion with chymotrypsin, trypsin and cyanogen bromide, a synthetic reagent may be used. This digestion selectively cleaves proteins at specific amino acid residues, creating smaller fragments that can then be analyzed by electrophoresis or the Edman degradation

Explain the reabsorption function of the kidneys.

some compounds that are filtered or secreted may be taken back up for use via reabsorption. Certain substances are almost always reabsorbed, such as glucose, amino acids, and vitamins. In addition, hormones such as ADH (vasopressin) and aldosterone can alter the quantity of water reabsorbed within the kidney in order to maintain blood pressure.

What are Langerhans cells?

special macrophages that reside within the stratum spinosum. They are capable of presenting antigens to T-cells in order to activate the immune system

What are neurons?

specialized cells capable of transmitting electrical impulses and then translating those electrical impulses into chemical signals.

what are teratogens?

substances that interfere with development, causing defects or even death of the developing embryo. common teratogens-alcohol, prescription drugs, viruses, bacteria, environmental chemicals

Explain the Whorfian hypothesis (linguistic relativity hypothesis)

suggests that our perception of reality is determined by the content of language. Language effects the way we think not the other way around.

What happens if BP is too high?

sympathetic impulses could decrease, permitting relaxation of the vasculature with a concurrent drop in BP. Within the heart, specialized atrial cells are able to secrete a hormone called atrial natriuretic peptide (ANP). This hormone aids in the loss of salt within the nephron, acting as a natural diuretic with loss of fluid. Interestingly, ANP is a fairly weak diuretic. Some fluid is lost, but it is not often enough to counter the effects of a high-salt diet on BP. Indeed, the human body has a lot of ways to raise BP, but few ways to lower it.

Messenger RNA (mRNA) is synthesized in what direction? What DNA strand is it complementary and antiparallel to? What DNA strand is it almost identical to? How is it different from this strand?

synthesized in the 5' --> 3' direction complementary and antiparallel to the DNA template strand identical to the DNA coding strand (except for Ts in DNA= Us in RNA)

messenger RNA (mRNA) is synthesized in what direction and is complementary and antiparallel to which DNA strand (coding or template)?

synthesized in the 5'-->3' direction and is complementary and antiparallel to the DNA template strand. It is identical to the DNA coding strand.

explain phonology in terms of language.

the actual sound of language. There are about 40 speech sounds or phonemes in English. Children must learn to produce and recognize the sounds of language, separating them from environmental noises and other human-created sounds, like coughing. They must also learn when subtle differences between speech sounds represent a change in meaning or not; the pronunciation of a word varies between people. The ability to make this categorical distinction is called categorical perception, and is an auditory example of constancy.

explain semantics in terms of language.

the association of meaning with a word a child must learn that certain combinations of phonemes represent certain physical objects or events, and that words may refer to entire categories, such as women, which others refer to specific members of categories, such as mommy. one can see this skill developing in young children as they may refer to all women as mommy

the two hormones released from the posterior pit are synthesized where?

the hypothalamus. The post pit itself does not synthesize any hormones

The space between the inner and outer membranes of mitochondrion is called what? The space inside the inner membrane is called the what?

the intermembrane space. the mitochondrial matrix

What does the appendicular skeleton consist of?

the remaining bones that aren't listed as part of the axial skeleton, in particular those of the upper and lower limbs and their respective girdles

What role does the GI tract play in nonspecific immunity?

the stomach secretes acid, resulting in the elimination of most pathogens. In addition, the gut is colonized by bacteria. Most of these bacteria lack the necessary characteristics to cause infection. Because there is already such a large bacterial population in the gut, many potential invaders aren't able to compete and are thus kept at bay. Many antibiotics reduce the population of gut flora, providing an opportunity for the growth of pathogens resistant to that antibiotic.

explain morphology in terms of language

the structure of words. many words are composed of multiple building blocks called morphemes, each of which connotes a particular meaning.

What is Km (Michaelis constant)?

the substrate [ ] at which half of the enzyme's active sites are full under certain conditions, it Is a measure of the affinity of the enzyme for its substrate

what are inborn errors of metabolism?

this is a class of deleterious mutation. these are defects in genes required for metabolism. children born with these defects often require very early intervention in order to prevent permanent damage from the buildup of metabolites in various pathways.

What are VLDLs? where are they produced and assembled? what is their main function? what do they also do?

very low density lipoprotein. The second lowest density form of lipoprotein, after chylomicrons and before IDLs. they are produced and assembled in liver cells. Like chylomicrons, the main function of VLDLs is the transport of triacylglycerols to other itssues. VLDLs also contain fatty acids that are synthesized from excess glucose or retrieved from chylomicron remnants.

What are the 4 tenants of cell theory?

1. All living things are composed of cells 2. The cell is the basic functional unit of life 3. Cells arise only from preexisting cells 4. Cells carry genetic info in the form of DNA that is passed on from parent to daughter cell

what are some of the roles that forming quaternary structures can serve?

1. they can be more stable, by reducing the surface area of the protein complex 2. they can reduce the amount of DNA needed to encode the protein complex 3. they can bring catalytic sites close together, allowing intermediates form one reaction to be directly shuttled to a second reaction 4. **they can induce cooperatively or allosteric effects**

Which of these is most likely to be preserved when a protein is denatured? A. primary structure B. secondary structure C. tertiary structure D. quaternary structure

A. Denaturing a protein results in the loss of 3-D space and function . Because the denaturation process does not normally result in breaking the peptide chain, the primary structure should be conserved.

when a peptide bond forms, the free amino end is known as the ____ terminus. the free carboxyl end is the ____ terminus. How are peptides drawn conventionally?

Amino end: N-terminus Carboxyl end: C-terminus N on left and C on right

In a neutral solution, most amino acids exist as: A. positively charged compounds B. zwitterions C. negatively charged compounds D. hydrophobic molecules

B

Sodium dodecyl sulfate (SDS) contains a 12-carbon tail attached to a sulfate group and is used in denuaturing gel electrophoresis of proteins. Numerous SDS molecules will bind to the exposed hydrophobic regions of denatured proteins. The use of SDS in this experiment allows for the separation of proteins by: A. charge B. molecular weight C. shape D. solubility

B

Which is true about the oxygen binding of hemoglobin at different pH values? A. Low pH favors the high affinity oxygen binding state B. Low pH favors the low affinity oxygen binding state C. Oxygen affinity is independent of pH D. Oxygen binding is non-cooperative at low pH

B

In the ZW sex-determination system for birds, males (ZZ) are the homogametic sex while females (ZW) are the heterogametic sex. What is the most likely regulatory mechanism for the potential imbalance in gene expression between male and female birds? A. post-transcriptional modification of half the male sex chromosomes gene products B. Inactivation of one Z chromosome in males C. Inactivation of the W chromosome in females D. Destruction of one Z chromosome in males

B In mammalian XY systems, females randomly inactivate one of their X chromosomes to prevent a redundancy in gene products (mRNA and proteins). These are known as Barr bodies. In the case of bird species, the homogametic males (ZZ0 must inactivate one of their Z chromosomes to achieve the appropriate reduction in gene products.

Collagen consists of three helices with carbon backbones that are tightly wrapped around one another in a "triple helix." Which of these amino acids is most likely to be found in the highest concentration in collagen? A. Proline B. Glycine C. Threonine D. Cysteine

B because collagen has a triple helix, the carbon backbones are very close together. Thus, steric hindrance is a potential problem,. To reduce that hindrance, we need small side chains.

Adding concentrated strong base to a solution containing an enzyme often reduces enzyme activity to zero. In addition to causing protein denaturation which of the following is another plausible reason of the loss of enzyme activity? A. Enzyme activity, once lost, cannot be recovered B. The base can cleave peptide residues C. Adding a base catalyzes protein polymerization D. Adding a base tends to deprotonate amino acids on the surface of proteins

B. Bases can catalyze peptide bond hydrolysis. A is incorrect because enzyme activity can be recovered in some cases. D is true but fails to explain the loss of enzyme activity

An infectious agent is probably a virus instead of a bacterium if it: A. lacks membrane-bound organelles B. cannot reproduce in acellular culture C. has a length of 1um, about the size of a mitochondrion D. contains DNA

B. viruses require host cells to reproduce

In order to send pain signals to the brain, a nerve in the spinal cord would have to synapse with an: a) afferent dorsal motor neuron b) efferent ventral sensory neuron c) afferent dorsal sensory neuron d) efferent ventral motor neuron

C

Prior to vaccinations for poliomyelitis, widespread polio infection created wards of patients at risk of death due to loss of control of respiratory muscles. In 1928, Drinker and Shaw invented the first "iron lung," a device that sealed the patient's body from the neck down in a chamber with controllable pressure. The pressure change in the "iron lung" aided respiration by: a) increasing the positive pressure in the lungs, driving diffusion of O2. b) increasing the positive pressure in the lungs, thereby increasing lung ATP utilization efficiency. c) increasing negative pressure within the iron lung caused the patient's lungs to expand. d) delivering positive abdominal pressure driving the diaphragm.

C

Which of these amino acids has a chiral carbon in its side chain? I. Serine II. Threonine III. Isoleucine A. I only B. II only C. II and III only D. all

C

All of the following are true regarding the function of neurons EXCEPT: A. hyper polarization at the end of an action potential is one mechanism by which neurons limit the rate at which action potentials may fire B. The flow of sodium into the neuron depolarizes the membrane in the first phase of an action potential C. The transmitting neuron secretes neurotransmitters, such as serotonin or dopamine, into the synaptic cleft from its dendrites D. An action potential is initiated when the axon hillock reaches the threshold potential

C. A neuron generally secretes neurotransmitter, like serotonin or dopamine, form its axon (specifically from its axon terminal or synaptic bouton). Retrograde neurotransmission (the secretion of neurotransmitters from dendrites) is rare, occurring mostly with nitric oxide and cannabinoid neurotransmitters

Which of the following statements is most likely to be true of non polar R groups in aqueous solution? A. they are hydrophilic and found buried within proteins B. they are hydrophilic and found on protein surfaces C. they are hydrophobic and found buried within proteins D. they are hydrophobic and found on protein surfaces

C. Non polar groups are not capable of forming dipoles or H bonds, this makes them hydrophobic.

How are the basal layer of the epidermis and the innermost lining of the small intestine similar? A. both are nondividing tissues B. both are derived from ectoderm C. both are composed of squamous cells D. the cells of both are connected by tight junctions

D

Aldosterone acts on the kidney by A. Increasing water permeability of the collecting duct B. Increasing water permeability in the ascending limb of the loop of Henle C. Increasing water permeability in the proximal tubule D. Increasing sodium absorption and potassium secretion in the distal tubule

D Filtrate pressure in the distal tubule is monitored by the juxtaglomerular apparatus. Special cells in the juxtaglomerular apparatus secrete an enzyme called renin either when stretch receptors in vessel walls detect a blood pressure drop (decreased Renal perfusion) or when macula dense cells in the distal convoluted tubule detect a drop of sodium concentration in the tubular filtrate. Renin release triggers a cascade of angiotensin I, II, and III which in turn causes the adrenal cortex to secrete aldosterone. Aldosterone acts on the collecting tubule of the distal tubule (not the collecting duct) to increase membrane proteins absorbing sodium and secreting potassium. For the MCAT: decreased [Na+] filtrate or decreased blood pressure --> juxtaglomerular apparatus releases renin-->angiotensin I,II,III cascade --> aldosterone-->increased distal tubule Na+ absorption from and K+ secretion into the filtrate. Increased blood pressure is a secondary effect of the aldosterone.

An individual is born with a mutation causing her to partially retain a form of fetal hemoglobin into adulthood. Compared to a normal individual, this person would exhibit: A. no differences from a normal adult B. significantly reduced oxygen binding in the lungs C. no symptoms, since retention of fetal hemoglobin would be fatal D. increased oxygen binding to hemoglobin in the tissues

D. Fetal hemoglobin has a higher affinity for oxygen than does adult hemoglobin. Thus, it binds and holds oxygen more tightly. An individual with fetal hemoglobin would show increased oxygen binding to hemoglobin as a result. c- false. in fact, reactivating fetal hemoglobin can be used as a treatment for sickle-cell

In lysine, the pKa of the side chain is about 10.5. Assuming that the pKa of the carboxyl and amino groups are 2 and 9 respectively, the pI of lysine is closest to: A) 5.5 B) 6.2 C) 7.4 D) 9.8

D. because lysine has a basic side chain, we ignore the pKa of a carboxyl group and avg the pKa of the side chain and amino group, the avg of 9 and 10.5 is 9.75 which is closest to D

Which of the following is a reason for conjugating proteins? I. To direct their delivery to a particular organelle. II. To direct their delivery to the cell membrane III. To add a cofactor needed for their activity A. I only B. II only C. II and III only D. all

D. conjugated proteins can have lipid or carbohydrate "tags" added to them. These tags can indicate that these proteins should be directed to the cell membrane or to specific organelles. They can also provide the activity of the protein

When titrating glutamic acid, where do the 1st and 2nd protons losses come from?

Glutamic acid has 2 carboxyl groups and 1 amino group. The 1st comes from the main carboxyl group, and the 2nd comes from the 2nd carboxyl group NOT the amino group!

When titrating lysine, where do the 1st and 2nd proton losses come from?

Lysine has 2 amino groups and 1 carboxyl group. The 1st comes from the carboxyl and the 2nd comes from the amino.

Methionine: 3 letter abbreviation, 1 letter abbreviation, polar or non polar

Met. M. Non-polar

Phenylalanine: 3 letter abbreviation, 1 letter abbreviation, polar or non polar

Phe. F. Non-polar

Tyrosine: 3 letter abbreviation, 1 letter abbreviation, polar or non polar

Tyr. Y. Polar

What is the nucleus surrounded by? What is the purpose of this?

a nuclear membrane or envelope it is a double membrane that maintains a nuclear environment separate and distinct from the cytoplasm

what are the 2 most common secondary structures of proteins? explain both

a-helices and B-pleated sheets a- helices: rodlike structure in which the peptide chain coils clockwise around a central axis. The helix is stabilized by intramolecular H-bonds between a carbonyl oxygen atom and an amide H atom 4 residues down the chain. The side chains of the amino acids in the a-helical conformation point away from the helix core. B-pleated sheets: can be parallel or antiparallel. the peptide chains lie alongside one another, forming rows or strands held together by intramolecular H bonds between carbonyl oxygen atoms on one chain and amide H atoms in an adjacent chain. To accommodate as many H bonds as possible, the B-pleated sheets assume a pleated or rippled shape. The R groups of the amino residues point above and below the plane of the sheet.

explain quaternary structure of proteins

arrangement of two or more polypeptide chains to form a protein. Only proteins with more than one polypeptide chain have QS for these proteins, the QS is an aggregate of smaller globular peptides, or subunits, and represents the functional form of a protein. Ex: hemoglobin (4 subunits)

what is the deal with Proline and secondary structures?

b/c of its rigid cyclic structure, proline will introduce a kink in the peptide chain when it is around in the middle of an a-helix. Proline residues are thus rarely found in a-helices, except in helices that cross the cell membrane. It is also not found in the middle of pleated sheets. It is often found in the turns between the chains of a B pleated sheet and as the residue at the start of an a-helix.

peptide bond formation is an example of what type of reaction? why?

condensation and dehydration because it results in the removal of a water molecule. it can also be viewed as an acyl substitution reaction, which can occur with all carboxylic derivatives

What is the major difference between eukaryotic and prokaryotic cells?

eukaryotic calls contain a true nucleus enclosed in a membrane. prokaryotic cells do not contain a nucleus

proteins can be broadly divided into what 2 categories?

fibrous proteins (structures resemble sheets or long strands) globular proteins (tend to be spherical - like.a globe)

what are the 2 main causes of protein denaturation?

heat and solutes

hydrophobic residues tend to be where in proteins? why? What happens to the hydrophilic residues?

hydrophobic- interior of proteins (reduces their proximity to water). The hydrophilic residues get pulled in by the hydrophobic residues. they form electrostatic interactions and H bonds that further stabilize the protein from the inside. As a result, most of the amino acids on the surface of proteins have hydrophilic R groups.

When pH of an amino acid equals the isoelectric point (pI) of the amino acid, what does this tell us?

it exists as electrically neural molecules

which amino acids are strongly hydrophobic? where are they most likely to be in the body?

the amino acids with long alkyl side chains: alanine, isoleucine, leucine, valine, and phenylalanine. found in the interior of proteins, away from water on the surface of the protein

explain the secondary structure of proteins

the local structure of neighboring amino acids. Primarily the result of H-bonding between nearby amino acids. 2 most common: a-helices and B-pleated sheets

What happens when disulfide bonds are present in a tertiary structure of an amino acid chain?how do they form>

they form when 2 cysteine molecules become oxidized to form cystine. they create loops in the protein chain

Explain the follicular phase of the menstrual cycle

1st phase of the menstrual cycle, followed by ovulation. The follicular phase begins when the menstrual flow, which sheds the uterine lining of the previous cycle begins. GnRH secretion from the hypothalamus increases in response to the decreased concentrations of estrogen and progesterone, which fall off toward the end of each cycle. The inc [ ]s of GnRH cause inc secretions of FSH and LH. These two hormones work together to develop several ovarian follicles. The follicles begin to produce estrogen, which has neg feedback and caused dec of GnRH, FSH, and LH. Estrogen stimulates regrowth of the endometrial lining, stimulating vascularization and glandularization of the decidua.

Where are the nucleus, endoplasmic reticulum, and ribosomes found in a neuron?

all found in the cell body (soma)

On a very cold day, a man waits for over an hour at the bus stop. Which of the following structures helps his body set and maintain a normal temperature? A. Hypothalamus B. Kidneys C. Posterior pituitary D. Brainstem

A

From which of the following layers does the notochord form? A. Ectoderm B. Mesoderm C. Endoderm D. Archenteron

B

what are astrocytes?

cells that nourish neurons and form the blood-brain barrier, which controls the transmission of solutes from the bloodstream into nervous tissue

What are the four regions of the spinal cord? What is the spinal cord protected by? Does the spinal cord have white and/or grey matter? Explain. How is info transmitted to the brain?

cervical, thoracic, lumbar, sacral the vertebral body yes. The white matter lies on the outside of the cord, and the grey matter is deep within it. The axons of motor and sensory neurons are in the spinal cord. The sensory neurons bring info in from the periphery and enter on the dorsal (back) side of the spinal cord. The cell bodies of these sensory neurons are found in the dorsal root ganglia. Motor neurons exit the spinal cord ventrally (on side closest to front of body). - Sensory neurons transmit info about pain, temp, and vibration up to the brain and have cell bodies in the dorsal root ganglia toward the dorsal part of the spinal cord; the motor neurons run from the brain along the opposite side of the spinal cord and in the ventral root to control movements of skeletal muscle and glandular secretions

Explain conformational coupling.

chemiosmotic coupling describes a dierect relationship between the proton gradient and ATP synthesis. it is the predominant mechanism accepted in the scientific community when describing oxidative phosphorylation. However, another mechanism called conformation coupling suggests that the relationship between the proton gradient and ATP synthesis is indirect. Instead, ATP is released by the synthase a a result of conformational change caused by the gradient. In this mechanism, the F1 portion of ATP synthase is reminiscent of a turbine, spinning with a stationary compartment to facilitate the harnessing of gradient energy for chemical bonding.

steroid hormones are derived from _______ and are produced primarily by what two things? can steroid hormones cross the cell membrane? why?

cholesterol the gonads and adrenal cortex -yes because they are derived from nonpolar molecules

Do muscle cells exhibit an all or nothing response or a gradient response?

an all or nothing response. the strength of a response from one muscle cell cannot be changed.

What are the 5 basic components of language?

phonology, morphology, semantics, syntax, and pragmatics

membranes of eukaryotic cells consist of a __________. the membrane is unique in that its surfaces are ______, while its inner portion is _______.

phospholipid bilayer. its surfaces are hydrophilic, inner portion is hydrophobic

What is the stratum lucidum and what is its importance?

it is one of the layers of the epidermis. only present in thick, hairless skin, such as the skin on the sole of the foot or the palms, and is nearly transparent.

explain the role of glycogen phosphorylase in glycogenolysis. what is it activated and inhibited by?

it is the rate limiting enzyme. in contrast to a hydrolast, a phosphorylase breaks bonds using an inorganic phosphate instead of water. It breaks alpha-1,4 glycosidic bonds, releasing glucose 1-phosphate from the periphery of the granule. It cannot break alpha-1,6 bonds and therefore stops when it nears the outermost branch points. Glycogen phosphorylase is activated by glucagon in the liver, so that glucose can be provided for the rest of the body. In skeletal muscle, it is activated by AMP and epi, which signals that the muscle is active and requires more glucose. it is inhibited by ATP.

Starting with the translation initiation codon, the following sequence encodes a polypeptide of how many amino acids? 5'-CUGCCAAUGUGCUAAUCGCGGGGG-3' A. 2 B. 3 C. 6 D. 8

A

What are some key differences between fetal and adult circulation?

-The lungs and liver both don't serve significant functions before birth -Gas exchange doesn't occur at the lungs, but rather the placenta - Detoxification and metabolism are primarily controlled by mom's liver and nutrient and waste exchange occur at the placenta also

How does growth hormone affect bone growth?

-growth hormone prevents glucose uptake in certain tissues (those that aren't growing) and stimulates the breakdown of fatty acids. This increase the availability of glucose overall, allowing muscle and bone to use it. - Bone growth originates in special regions of the bone known as epiphyseal plates, which seal shut during puberty. - In adults, because the long bones are sealed, GH still has an effect, but it is primarily in the smaller bones.

What are apoenzymes and holoenzymes? what are prosthetic groups?

apoenzymes- enzymes without their cofactors holoenzymes- those containing their cofactors prosthetic groups- tightly bound cofactors or coenzymes that are necessary for enzyme function

FIGURE 3 BIOLOGY 1. During muscle contraction, which of the following regions decrease(s) in length? A. I only B. 1 and 2 only C. 3 and 4 only D. 2, 3, and 4 only 2. Region 1 refers to: A. the thick filaments only B. the thin filaments only C. the A band D. the I band 3. Which region represents 1 sarcomere? A. 1 B. 2 C. 3 D. 4

1. D We are given a diagram of a sarcomere and asked to determine which regions shorten during muscle contraction. All bands and zones of the sarcomere shorted during contraction except the A band, which is the full length of the thick filaments. In this diagram, that is region 1. Thus, the remaining regions all shorted, making D the correct answer. Region 2 represents the I-band, region 3 represents the H-zone, and region 4 is the length of the sarcomere between Z-line 2. C Region 1 contains both thick and thin filaments overlapping each other. This region refers to the A-band and is measured form one end of the thick filaments to the other. This is also the only portion of the sarcomere that does not change length during muscle contraction. 3. D The sarcomere is the contractile unit in striated muscle cells. One sarcomere is represented by the area between the two vertical lines, referred to as the Z-lines. In addition, the Z-lines anchor the thin filaments.

What are the two main methods of energy storage in the body? What are some advantages/disadvantages of the methods?

1. as triacylglycerols in adipose tissue 2. as carbohydrates in glycogen Glycogen offers access to metabolic energy in a faster water-soluble form; however, because of it's low energy density, glycogen can only provide energy for a bit less than one day. In contrast, a moderately obese individual with 15-20kg of stored triacylglycerols in adipose tissue could draw upon fat stores for months, but it takes more time to mobilize this energy.

What is the net ATP yield per glucose. Why is there a range?

30-32 because efficiency of aerobic respiration varies between cells. This variable efficiency is caused by the fact that cytosolic NADH formed through glycolysis cannot directly cross into the mitochondrial matrix. Because it cant contribute its electrons to the transport chain directly, it must find alternate means of transportation referred to as shuttle mechanisms. A shuttle mechanism transfers the high-energy electrons of NADH to a carrier that can cross the inner mitochondrial membrane. Depending on which of the two shuttle mechanisms NADH participates in, either a 1.5 or 2.5 ATP will end up being produced

Explain the luteal phase of the menstrual cycle.

3rd phase. After the ovulation phase before menstruation. After ovulation, LH causes the ruptured follicle to form the corpus lutem, which secretes progesterone. Estrogen helps regenerate the uterine lining, but progesterone maintains it for implantation. Progesterone levels begin to rise, while estrogen levels remain high. The high levels of progesterone cause negative feedback on GnRH, FSH, and LH, preventing the ovulation of multiple eggs.

Telophase II

8th and last phase of meiosis. 4th phase of meiosis II a nuclear membrane forms around each new nucleus. cytokinesis follows, two daughter cells are formed by completion of meiosis II, up to 4 haploid daughter cells are produced per gametocyte.

Which of the following is false with regard to adult stem cells? A. They retain inherent pluripotent if harvested from selected organs B. they are less controversial than embryonic stem cells C. They require treatment with various transcription factors D. There is a reduced risk of rejection if the patient's own stem cells are used.

A

Which of the following statements about enzyme kinetics is FALSE? A. An increase in the substrate [ ] (at constant enzyme conc.) leads to proportional inc in the rate of the reaction B. most enzymes operating in the human body work best at a temp of 37 degrees Celsius C. an enzyme-substrate complex can either form a product or dissociate back into the enzyme and substrate D. Max activity of many human enzymes offers around pH 7.4

A

Which of the following statements conflicts with the fluid mosaic model? A. The cell membrane is static in structure B. Membrane components can be derived from multiple biomolecules C. Hydrophobic interactions stabilize the lipid bilayer D. Proteins are asymmetrically distributed within the cell membrane

A

Which of the following types of intelligence is NOT described by Gardner's theory of multiple intelligences? A. Fluid intelligence B. Bodily-kinesthetic intelligence C. Visual-spatial intelligence D. Linguistic intelligence

A

Which of the following correctly pairs the molecule with its primary site of absorption? A. Chylomicrons--lacteals B. Amino acids--large intestine C. Vitamin A and E--stomach D. Cholesterol--ascending colon

A Chylomicrons contain triacylglycerols, cholesterol esters, and fat-soluble vitamins and are secreted by intestinal cells into lacteals. Amino acids, fat-soluble vitamins (like vitamins A and E), and cholesterol are all absorbed in the small intestine.

Topoisomerases are enzymes involved in: A. DNA replication and transcription B. posttranscriptional processing C. RNA synthesis and translation D. posttranslational processing

A DNA gyrase is a type of topoisomerase that enhances the action of helicase enzymes by the introduction of negative supercoils into the DNA molecule. These negative supercoils facilitate DNA replication by keeping the strands separated and untangled

explain Conjugation in bacteria

A form of bacterial recombination. It is the bacterial form of mating. It involves 2 cells forming a conjugation bridge between them that facilitates the transfer of genetic material. The transfer is unidirectional, from the donor male (+) to the recipient female (-). The bridge is made from appendages called sex pili that are found on the donor male. To form the pills, bacteria must contain plasmids known as sex factors that contain the necessary genes. The best studied sex factor is the fertility (F) factor in E. coli. Bacteria possessing this plasmid are termed F+ cells, those with our are called F- cells. During conjugation the F+ cell replicates its F factor and donates the copy to the F- cell, converting it to an F+ cell. This enables the cell obtaining the new plasmid to then transfer copies to other cells. This method of genetic recombination allows for rapid acquisition of antibiotic resistance or virulence factors throughout a colony because ofter plasmids can also be passed through the conjugation bridge. The sex factor is a plasmid, but through processes such as transformation, ti can become integrated into the host genome. In this case, when conjugation occurs, the entire genome replicates because it now contains the sex factor. The donor cell will then attempt to transfer an. entire copy of its genome into the recipient; however, the bridge usually breaks before the full DNA sequence can be moved. Cells that have undergone this change are referred to by the abbreviate Her for high frequency of recombinaition

Leptin

A hormone produced by adipose (fat) cells that decreases appetite by suppressing orexin production. knockouts of leptin have been linked to obseity

what is the hypodermis?

A layer of connective tissue containing adipose (fat) tissue and fibrous tissues that connects the skin to the rest of the body.

What are the categories of fat-soluble vitamins?

A, D, E, and K

Myoclonic epilepsy and ragged-red fiber (MERRF) is an extremely rare disorder that affects neuromuscular systems. MERRF results from a mutation in mitochondrial DNA (mtDNA) that impairs protein synthesis, oxygen consumption, and energy production. When an affected male and a normal female reproduce, which of the following best predicts the expected phenotypic ratios of the offspring? A. None of the offspring will be affected B. All males and no females will be affected C. Half of males and half of females will be affected D. One-fourth of the offspring will be affected

A. This question is asking the examinee to determine offspring phenotypes for a gene that is located on mitochondrial DNA. Since mtDNA follows maternal inheritance, all the offspring receive their mitochondrial DNA from the mother. In this case, the MERRF-causing mutation is located on the father's mtDNA. Therefore, the mutations are not passed on to the offspring. Since all of the offspring will receive functioning mtDNA from the mother, none of the offspring will inherit mutated genes causing MERRF.

What is ADH? steroid or peptide hormone? how its it released? what is its mechanism of action and what does it do?

ADH (vasopressin) anidiuretic hormone is a peptide hormone synthesized by the hypothaamus and released by the posterior pituitary in response to high blood osmolarity. It directly alters the permeability of the collecting duct, allowing more water to be reabsorbed by making the cell junctions of the duct leaky. Increased [ ] in the interstitium (hypertonic to the filtrate) will then cause the reabsorption of water from the tubule. Alcohol and caffeine both inhibit ADH release and lead to the frequent excretion of dilute urine. In addition to the kidneys, the cardiovascular system also regulates BP, specifically by vasoconstricting or vasodilating in order to maintain blood pressure. Constriction of the afferent arteriole will lead to a lower pressure of blood reaching the glomeruli, which are adjacent to the juxtaglomerular cells. Therefore, this vasoconstriction will secondarily lead to renin release, which will also help raise BP.

Explain the process of inhalation.

Active process. We use our diaphragm as well as the external intercostal muscles to expand the thoracic cavity. As the diaphragm flattens and the chest wall expands outward, the intrathoracic volume (volume of the chest cavity) increases. Specifically, b/c the intrapleural space most closely abuts the chest wall, its volume increases first. By understanding Boyle's law, an increase in intrapleural volume leads to a decrease in intrapleural pressure. Now we have low pressure in the intrapleural space. The gas in the lungs is initially at atmospheric pressure, which is now higher than the pressure in the intrapleural space. The lungs will therefore expand into the intrapleural space and the pressure in the lungs will drop. Air will then be sucked in from a higher-pressure environment-the outside world. The mechanism is referred to as negative-pressure breathing b/c the driving force is the lower (relatively negative) pressure in the intrapleural space compared with the lungs.

Explain cleavage in the embryonic development process. What are the two types of cleavage?

After fertilization in the fallopian tubes, the zygote must travel to the uterus for implantation. As it moves to the uterus for implantation, the zygote undergoes rapid mitotic cell divisions in a process called cleavage. The first cleavage officially creates an embryo. By dividing into progressively smaller cells, the cells inc two rations: the nuclear-to-cytoplasmic (N:C) ratio and the surface area-to-volume ratio. Thus, the cells achieve inc area for gas and nutrient exchange relative to overall volume. Indeterminate cleavage- results in cells that can still develop into complete organisms. In fact, monozygotic twins have identical genomes b/c they both originate from indeterminately cleaved cells of the same embryo. Determinate cleavage- results in cells with fates that are already determined. In other words, these cells are committed to differentiating into a certain type of cell.

Explain how amino acid catabolism, ketones, and alcohol, individually, can be used to form acetyl-CoA.

Amino-acid catabolism: certain amino acids can be used to form acetyl-CoA. These amino acids must lose their amino group via transamination; their carbon skeletons can then form ketone bodies. These amino acids are termed ketogenic for that reason. Ketones: Although acetyl-CoA is typically used to produce ketones when the pyruvate dehydrogenase complex is inhibited, the reverse reaction can occur as well. Alcohols: When alcohol is consumed in moderate amounts, the enzymes alcohol dehydrogenase and acetaldehyde dehydrogenase convert it to acetyl-CoA. However, this reaction is accompanied by NADH buildup, which inhibits the Krebs cycle. Therefore, the acetyl-CoA formed through this process is used primarily to synthesize fatty acids.

Following a myocardial infarction, the heart often heals by the creation of a scar by fibroblasts. This is an example of: A. complete regeneration B. incomplete regeneration C. competency D. multipotency

B

Explain sleep as a state of consciousness in terms of EEGs.

As soon as you doze off, you enter stage 1, which is detected on the EEG by the appearance of theta waves. At this point, EEG activity is characterized by irregular waveforms with slower frequencies and higher voltages. As you fall more deeply asleep, you enter stage 2. The EEG shows theta waves along with sleep spindles and K complexes. As you fall even more deeply asleep, you enter stages 3 and 4, aka slow-wave sleep (SWS). EEG activity grows progressively slower until only a few sleep waves are seen per second. These low-frequency, high-voltage sleep waves are called delta waves. During these stages, it becomes especially difficult to rouse someone from sleep. SWS has been associated with cognitive recovery and memory consolidation, as well as increased growth hormone release. Stages 1 through 4 are all part of non-rapid eye movement (NREM) sleep; rapid eye movement (REM) sleep is interspersed between cycles of the NREM sleep stages. In this stage, arousal levels reach that of wakefulness, but the muscles are paralyzed. It is also called paradoxical sleep because one's HR, breathing patters, and EEG mimic wakefulness, but the individual is still asleep. This is the stage in which dreaming is most likely to occur and is also associated with memory consolidation.

Which of the following is NOT part of the small intestine? A. Ileum B. Cecum C. Jejunum D. Duodenum

B The small intestine is divided into three sections: the duodenum, the jejunum, and the ileum. The cecum is a part of the large intestine

Which of the following INCORRECTLY pairs a structure of the male reproductive system with a feature of this structure? A. seminal vesicles- produce alkaline fructose-containing secretions B. epididymis- surrounded by muscle to raise and lower the testes C. vas deferens- tube connecting the epididymis to the ejaculatory duct D. Cowper's glands- produce a fluid to clear traces of urine in the urethra

B. The epididymis is the site of sperm maturation at the posterior side of of the testes. It is the gas deferens that is surrounded by muscle that raises and lowers the testis to maintain a constant temp suitable for sperm production

An infectious agent is probably a virus instead of a bacterium if it: A. lacks membrane bound organelles B. cannot reproduce in acellular culture C. has a length of 1um, about the size of a mitochondrion D. contains DNA

B. viruses require host cells in order to reproduce

A man is given antibiotics to treat a UTI and develops an eipsode of RBC lysis. Further studies show weakness fo the plasma membrane and Heinz bodies (collections of oxidized hemoglobin). Which of the following enzymes is most likely defective in this patient? A. Fructose-1,6-bisphosphatase B. Glucose-6-phosphate dehydrogenase C. Hexokinase D. Pyruvate kinase

Based on this question stem, we can infer that the antibiotics must have been an oxidative stress on the patient (indeed, antibiotics, antimalarial medications, infections, certain foods like fava beans, and other common exposures can induce an oxidative stress). The pentose phosphate pathway is responsible for generating NADPH, which is used to reduce glutathione, one of the natural antioxidants present in the body. In individuals with glucose-6-phosphate dehydrogenase (G6PD) deficiency, NADPH cannot be produced at sufficient levels, and oxidative stresses lead to cell membrane and protein (hemoglobin) damage. Note that you do not need to actually know the disease to answer this question; merely knowing that the enzyme must be from the pentose phosphate pathway, which is involved in mitigating oxidative stress, is sufficient.

Explain how protein obtained from the diet or body during prolonged fasting or starvation may be used as an energy source.

Body protein is catabolized primarily in muscle and liver. Amino acids released from proteins usually lose their amino group through transamination or demanination. The remaining carbon skeleton can be used for energy. The amino groups removed by transamination or deamination constitute a potential toxin to the body in the form of ammonia, and must be excreted safely. The urea cycle occurs in the liver and is the body's primary way of removing excess nitrogen from the body.

Explain Broca's area and Wernicke's area. How are they connected?

Broca- located in the inferior frontal gyrus of the frontal lobe, controls the motor function of speech via connections with the motor cortex Wernicke-located in the superior temporal gyrus of the temporal lobe, is responsible for language comprehension. Broca's and Wernicke's area are connected by the arcuate fasciculus, a bundle of axons that allows appropriate association between language comprehension and speech production.

What are phospholipids? How do they behave in cell membranes?

By substituting one of the fatty acid chains of triacylglycerol with a phosphate group, a polar head group joins the nonpolar tails, forming a glycerophospholipid, commonly called a phospholipid. Phospholipids spontaneously assemble into micelles (small monolayer vesicles) or liposomes (bilayered vesicles) due to hydrophobic interactions. Phospholipids are used for membrane synthesis and can produce a hydrophilic surface layer on lipoproteins such as very-low-density lipoprotein (VLDL), a lipid transporter. In addition, phospholipids are the primary component of cell membranes. Phospholipids serve not only structural roles, but can also serve as second messengers in signal transduction. The phosphate group also provides an attachment point for water-soluble groups, such as choline or inositol.

Destruction of all beta cells in the pancreas would cause A. glucagon secretion to stop and an increase in blood glucose [ ] B. glucagon secretion to stop and a decrease in blood glucose [ ] C. insulin secretion to stop and an increase in blood glucose [ ] D. insulin secretion to stop and a decrease in blood glucose [ ]

C

Which of the following is least likely to cause denaturation of proteins? A. heating the protein to 100C B. adding 8M urea C. moving it to a more hypotonic environment D. adding a detergent such as sodium dodecyl sulfate

C

Which protein properties allow UV spectroscopy to be used as a method of determining concentration? A. Proteins have partially planar characteristics in peptide bonds. B. Globular proteins cause scattering of light C. Proteins contain aromatic groups in certain amino acids D. All organic macromolecules can be assessed with UV spectroscopy

C

An investigator is measuring the activity of various enzymes involved in reactinos of intermediary metabolism. one of the enzymes has greatly decreased activity compared to reference values. The buffer of the assay contains citrate. Which of the following enzymes will most likely be directly affected by the use of citrate? A. Fructose-2,6-bisphosphatase B. Isocitrate dehydrogenase C. Phosphofructokinase-1 D. Pyruvate carboxylase

C Citrate is produced by citrate synthase from acetyl-CoA and oxaloacetate. This reaction takes place in the mitochondria. When the citric acid cycle slows down, citrate accumulates. In the cytosol, it acts as a negative allosteric regulator of phosphofructokinasae-1, the enzyme that catalyzes the rate-limiting step of glycolysis.

How many distinct tripeptides can be formed form one valine molecule, one alanine molecule, and one leucine molecule? A. 1 B. 3 C. 6 D. 27

C. There are 3 coaches for the first amino acid, leaving 2 choices for the 2nd, and 1 choice for the 3rd. 3 x 2 x 1 = 6.

Explain the first step (initiation) of muscle contraction?

Contraction starts at the neuromuscular junction, where the nervous system communicates with muscles via motor (efferent) neurons. This signal travels down the neuron until it reaches the nerve terminal *synaptic bouton), where Ach is released into the synase. In the case of the neuromuscular junciton, the nerve terminal can also be called the motor end plate. Ach binds to receptors on the sarcolemma, causing depolarization. Each nerve terminal controls a group of myocytes; together, the nerve terminal and its myocytes constitute a motor unit. Depolarization triggers an action potential, which spread down the sarcolemma to the T-tubules. The action potential travels down the T-tubules into the muscle tissues to the sarcoplasmic reticulum. When the action potential reaches the sarcoplasmic reticulum, Ca2+ is ultimately released. The calcium ions bind to a regulatory subunit in troponin, triggering a change in the confirmation of tropomyosin, to which troponin is bound. This change exposes the myosin-binding sites on the actin thin filament.

Which of the following is NOT a method by which enzymes decrease the activation energy for biological reactions? A. modifying the local charge environment B. forming transient covalent bonds C. acting as electron donors or receptors D. breaking bonds in the enzyme irreversibly to provide energy

D

Which of the following is true with regard to the action potential? A. All hyper polarized stimuli will be carried to the axon terminal without a decrease in size B. the size of the action potential is proportional to the size of the stimulus that produced it C. Increasing the intensity of the depolarization increases the size of the impulse D. Once an action potential is triggered, an impulse of a give magnitude and speed is produced

D Action potentials are often described as all or nothing. The magnitude of the potential difference of an action potential is fixed, regardless of the intensity of the stimulus. Neuronal info is coded by the frequency and # of action potentials, no the size of an action potential.

Which process is expected to begin earliest in a prolonged fast A. Ketone bodies are used by the brain B. Glycogen storage is halted C. Proteins are broken down D. Enzyme phosphorylation and dephosphorylation

D A prolonged fast is characterized by an increase in glucagon, which accomplishes its cellular activity by phosphorylating and dephosphorylating metabolic enzymes. Glycogen storage is then halted but this requires enzyme regulation by glucagon to occur. Later in the postabsorptive state protein breakdown begins. Eventually in starvation ketone bodies are used by the brain for its main energy source.

Hyperbaric oxygen may be used as a treatment for certain types of bacterial infections. In this therapy, the patient is placed in a chamber in which the partial pressure of oxygen is significantly increased, increasing the partial pressure of oxygen in the patient's tissues. This treatment is most likely used for infections with: A. obligate aerobic bacteria B. facultative anaerobic bacteria C. aerotolerant anaerobic bacteria D. obligate anaerobic bacteria

D. onbligate anaerobes cannot survive in the presence of oxygen and would likely be killed

What is the goal of DNA cloning? How does it work?

DNA cloning is a technique that can produce large amounts of a desired sequence. Often, the DNA to be cloned is present in a small quantity and is part of a heterogenous mixture containing other DNA sequences. The goal is to produce a large quantity of homogenous DNA for other applications. Cloning requires that the investigator ligate the DNA of interest into a piece of nucleic acid referred to as a vector, forming a recombinant vector (this can be accomplished via restriction enzymes creating sticky ends on both the vector and the DNA of interest). Vectors are usually bacterial or viral plasmids that can be transferred to a host bacterium after insertion of the DNA of interest. The bacteria are then grown in colonies, and a colony containing the recombinant vector is isolated. This can be accomplished by ensuring that the recombinant vector also includes a gene for antibiotic resistance; antibiotics can then kill off all of the colonies that do not contain the recombinant vector. The resulting colony can then be grown in large quantities.

Explain telomeres and the role of telomerase. What is the sequence of a telomere?

DNA replication can't extend all the way to the end of a chromosome b/c this would result in losing sequences and info with each round of replication. The solution is a repeating sequence of (TTAGGG) at the end of DNA, forming a telomere. Some of the sequence is lost in each round of replication and can be replaced by the enzyme telomerase. Telomerase is more highly expressed in rapidly dividing cells. Animal studies indicate that there are a set number of replications possible, and that the progressive shortening of telomeres contributes to aging. Telomeres also serve a second function: their high GC content creates exceptionally strong strand attractions at the end of chromosomes to prevent unraveling; "knotting off" the end of chromosome.

What begins the process of DNA replication?

DNA unwinds at points called origins of replication. The generation of new DNA proceeds in both directions, creating replication forks on both sides of the origin

DNA--> DNA is called what? in what direction is new DNA synthesized? RNA--> RNA is called what? in what direction new RNA it synthesized? what direction is the template read? RNA--> protein is called what? in what direction is mRNA read?

DNA--> DNA (replication): new DNA synthesized in 5'-->3' direction DNA-->RNA (transcription): new RNA synthesized in 5'-->3' direction (template read in 3'-->5' direction) RNA-->protein (translation): mRNA read in 5'-->3' direction

Explain deductive vs inductive reasoning

Deductive (Top-down) reasoning starts from a set of general rules and draws conclusions from the info given. Inductive (Bottom-up) reasoning seeks to create a theory via generalizations. This type of reasoning starts with specific instances, and then draws a conclusion from them.

What is the second stage of cell specialization? Explain it. What are the ways that it can occur?

Determination. the commitment of a cell to a particular function in the future. Prior to determination the cell can become any cell type, even if it has already gone through specification. After determination the cell is irreversibly committed to a specific lineage. There are multiple pathways by which determination may occur. - During cleavage, where the existing mRNA and protein in the parent cell has been asymmetrically distributed between the daughter cells, the presence of specific mRNA and protein molecules may result in determination. - Can also occur due to secretion of morphogens from nearby cells. Morphogens may cause neighboring cells to follow a particular developmental pathway. *Note: Determination is a commitment to a particular cell type, but the cell has not yet actually produced what it needs to carry out the functions of that cell type (that's the goal of differentiation)

Explain how DNA proofreading works. For excision, how does the enzyme discriminate which is the template strand and which is the incorrectly paired daughter strand? Explain how mismatch repair works.

During synthesis, the two double-strand of DNA molecules will pass though a part of the DNA polymerase enzyme for proofreading -when the complementary strands have incorrectly paired bases, the H bonds between the strands can be unstable. This lack of stability is detected by the DNA polymerase. It looks at the level of methylation: the template strand has existed in the cell for a longer period of time, and is therefore heavily methylated. Cells have machinery in the G2 phase of the cell cycle for mismatch repair; these enzymes are encoded by genes MSH2 and MLH1, which detect and remove errors introduced in replication that were missed during the S phase of the cell cycle. These enzymes are homologous of MutS and MutL in prokaryotes.

Explain the G2 stage of cell division.

Postsynthetic gap. the cell passes through another quality control checkpoint. DNA has already been duplicated during the D stage, the cell checks to ensure that there are enough organelles and cytoplasm for the 2 daughter cells. The cell also checks to make sure that DNA replication happened right.

How does base excision repair work? What's an example?

Ex: thermal energy absorbed by DNA and leads to cytosine deamination. results in cytosine--> uracil. uracil shouldn't be found in a DNA molecule and is easily detected. This is repaired by base-excision repair. First, the affected base is recognized and removed by a glycosylase enzyme, leaving behind an apurinic/apyrimidinic (AP) site, this is also called an abasic site. The AP site is recognized by an AP endonuclease that removes the damaged sequence from the DNA. DNA poly and DNA ligase can then fill in the gap and seal the strand.

What are the two types of digestion that occur?

First, intracellular digestion, as part of metabolism, involves the oxidation of glucose and fatty acids for energy. However, our diets do not consist of pure glucose and fatty acids for energy, rather these substances must be extracted from our food. The process by which these nutrients are obtained form food occurs within the lumen of the alimentary canal and is known as extracellular digestion. This is technically "outside" of the body because the lumen of the GI tract communicates directly with the outside world.

How would our respiratory systems adjust if we moved to higher altitudes where less oxygen is available?

First, we would breath more rapidly to try to avoid hypoxia. Second, the binding dynamics of hemoglobin to oxygen would be altered to facilitate the unloading of oxygen at the tissues. In the long term, the body could develop more blood vessels (vascularization), which would facilitate the distribution of oxygen to tissues

How does impulse propagation in a neuron work?

For a signal to be conveyed to another neuron, the action potential must travel down the axon and initiate neurotransmitter release. This movement (impulse propagation) happens as sodium rushes into one segment of the neuron, causing a depolarization in the surrounding region of the axon, which will bring subsequent segments of the axon to threshold, opening the Na+ channels in these segments. Each of these segments then continues through the rest of the action potential in a wavelike fashion until the action potential reaches the nerve terminal. After the action potential has fired in one segment of the axon, that segment becomes momentarily refractory. The functional consequence is that this info can only flow in one direction.

What is the anomeric carbon? What are anomers? What are the two types of anomers in glucose? Explain them.

For hemiacetal or hemiketals. The carbonyl carbon becomes chiral in the process of cylization, and is referred to as the anomeric carbon. If two molecules differ at the anomeric carbon, they are anomers. In glucose, the alpha anomer has the -OH group of C-1 trans to the -CH2OH substituent (axial and down). The B-anomer has the OH group of C-1 cis to the -CH2OH substituent (equatorial and up)

Explain the difference between gap junctions, tight junctions, and desmosomes.

Gap junctions allow for direct cell-cell communication and are often found in small bunches together. Gap junctions are also called connexons and are formed by the alignment and interaction of pores composed of six molecules of connexin. They permit movement of water and some solutes directly between cells. Proteins are generally not transferred through gap junctions. Tight junctions form a watertight seal, preventing paracellular transport of water and solutes. They prevent solutes from leaking into the space between cells via a paracellular route. They are found in epithelial cells and function as a physical link between the cells as they form a single layer of tissue. Tight junctions can limit permeability enough to create a trans-epithelial voltage difference based on different [ ]s of ions on either side of the epithelium. To be effective, tight junctions must form a continuous band around the cell; otherwise, fluid could leak through spaces between tight junctions. Desmosomes bind adjacent cells by anchoring to their cytoskeletons. Desmosomes are formed by interactions between transmembrane proteins associated with intermediate filaments inside adjacent cells. Desmosomes are primarily found at the interface between two layers of epithelial tissue. Hemidesmosomes have a similar function, but their main function is to attach epithelial cells to underlying structures, especially the basement membrane.

Glutamine. 3 letter abbreviation, 1 letter abbreviation, polar or non polar

Gln. Q. Polar

Glutamic Acid: 3 letter abbreviation, 1 letter abbreviation

Glu. E.

Which of the following types of nucleic acid could form the genome of a virus? I. Single-stranded RNA II. Double-stranded DNA III. Single-stranded DNA

I, II, and III

During which of the following stages does dreaming occur? I. Stage 3 II. Stage 4 III. REM

I, II, and III About 75% of dreaming occurs during REM, but dreams occur in all other stages of sleep also .

Resting membrane potential depends on: I. The differential distribution of ions across the membrane II. Active transport process III. Selective permeability of the phospholipid bilayer

I, II, and III

How does prokaryotic DNA differ from eukaryotic DNA? I. Prokaryotic DNA lacks nucleosomes II. Eukaryotic DNA has telomeres III. Prokaryotic DNA is replicated by a different DNA polymerase IV. Eukaryotic DNA is circular when not restricted by centromeres

I, II, and III Prokaryotic DNA is circular and lacks histone proteins, and thus does not form nucleosomes. Both prokaryotic and eukaryotic DNA are replicated by DNA polymerases, although these polymerases differ in identity. Eukaryotic DNA is organized into chromatin, which can condense to form linear chromosomes; only prokaryotes have circular chromosomes. Only eukaryotic DNA has telomeres.

Which of the following could results from an absence of apolipoproteins? I. An inability to secrete lipid transport lipoptoreins II. An inability to endocytose lipoproteins III. A decreased ability to remove excess cholesterol from blood vessels

I, II, and III While the transport and lipid binding functions of most lipoproteins are independent of the apolipoprotein component, the interaction of these lipoproteins with the environment is controlled almost exclusively by apolipoproteins. Lipoproteins cannot exit or enter cells without apolipoproteins, and are unable to transfer lipids without specialized apolipoproteins or cholesterol-specific enzymes.

Resistance to antibiotics is a well-recognized medical issue. Which mechanism(s) can account for a bacterium's ability to increase its genetic variability and thus adapt itself to resist different antibiotics? I. Binary fission II. Conjugation III. Transduction

II and III Bacterial cells reproduce by binary fission, an asexual process in which the progeny is identical to the parent. So it doesn't increase genetic variability. Conjugation is basically mating in bacteria; it is the transfer of genetic material between two bacteria that are temporarily joined Transduction occurs when fragments of the bacterial chromosome accidentally become packaged into viral progeny produced during a viral infection and are introduced into another bacterium by the viral vector

Which type(s) of muscle is/are always multinucleated? I. Cardiac muscle II. Skeletal muscle III. Smooth muscle

II only Cardiac muscle may contain one or two centrally located nuclei. Smooth muscle always has only one cntrally located nucleus

Explain glycolysis in erythrocytes.

In erythrocytes (RBCs), anaerobic glycolysis represents the only path for ATP production, yielding a net 2 ATP per glucose. RBCss have bisphosphoglycerate mutase, which produces 2,3-bisphosphoglycerate (2,3-BPG) from 1,3-BPG in glycolysis. 2,3-BPG binds allosterically to the B-chains of hemoglobin A (HbA) and decreases its affinity for oxygen. This effect causes a right shift in the oxygen dissociation curve, which allows unloading of oxygen in tissues with still a 100% saturation in the lungs.

What are the three steps of muscle contraction?

Initiation, shortening of the sarcomere, and relaxation

According to Jean Piaget, how do infants learn? How do infants process new information? How does this come about?

Infants learn through instinctual interaction with the environment Ex: grasping reflex-->grasp objects Infants process new info via adaptation, which comes about by two complementary processes: assimilation and accommodation. Assimilation is the process of classifying new info into existing schemata. If the new info doesn't fit neatly into existing schemata, then accommodation occurs. Accommodation is the process by which existing schemata are modified to encompass this new info.

How can integrating a phage into a host genome benefit the bacterium?

Infection with one strain of the phage generally makes the bacterium less susceptible to superinfection (simultaneous infection) with other phages

What is the total lung capacity (TLC)?

Maxiumum volume of air that lungs can contain when one inhales completely. (TLC = IRV + Vt + ERV + RV) - 6.0/7 L.

How does gene therapy work?

Intended for diseases in which a given gene is mutated or inactive. By transferring a normal copy of the gene into the affected tissues, the pathology should be fixed, essentially curing the individual. Because viruses naturally infect cells to insert their own genetic material, most gene delivery vectors in use are modified viruses. A portion of the viral genome is replaced with the cloned gene such that the virus can infect by not complete its replication cycle. *Randomly integrated DNA poses a risk of integrating near and activating a host oncogene. Among the kids treated for SCID, a small number have developed leukemias.

How are T3 and T4 made? What do they do? What would an increase in the amounts of T3 and T4 do in the body? What are the symptoms of hypothyroidism?

Iodination of tyrosine in the follicular cells of the thyroid. The numbers refer to the number of iodine atoms attached to the tyrosine Thyroid hormones are capable of resetting the basal metabolic rate of the body by making energy production more or less efficient, as well as altering the utilization of glucose and fatty acids. Inc amounts of T3 and T4 will lead to inc cellular respiration. This leads to increased protein and fatty acid turnover by speeding up both synthesis and degradation of these compounds. High plasma levels of thyroid hormones will lead to decreased TSH and TRH synthesis; negative feedback prevents excessive secretion of T3 and T4 hypothyroidism is characterized by lethargy, decreased body temp, slowed respiratory and HR, cold intolerance,, and weight gain

Explain Mendel's Second Law. What part of mitosis/meoisis does this refer to?

Law of independent assortment. inheritance of one gene does not affect the inheritance of another gene. spermatogonia and oogonia undergo genome replication before meiosis I. The daughter DNA strand is held to the parent strand at the centromere. Together, these DNA strands are known as sister chromatids. During prophase I of meiosis, homologous chromosomes pair up to form tetrads, which derive their name from the four chromatids involved (two chromatids in each of two homologous chromosomes). Small segments of genetic material are swapped between chromatids in homologous chromosomes, resulting in novel combinations of alleles that were not present in the OG chromosomes (recombination). This allows the inheritance of one gene to be independent of the inheritance of all others. Mendel's second law has been complicated by the discovery of linked genes.

Explain the roles of LCAT and CETP in cholesterol transport.

Lecithin-cholesterol acytransferase (LCAT) is an enzyme found in the bloodstream that is activated by HDL apoproteins. LCAT adds a fatty acid to cholesterol, which produces soluble cholesteryl esters such as those in HDL. HDL cholesteryl esters can be distributed to other lipoproteins like IDL, which becomes LDL by acquiring these cholesteryl esters. The cholesteryl ester transfer protein (CETP) facilitates this transferprocess.

Explain lipid digestion

Lipid digestion is minimal in the mouth and stomach; lipids are transported to the small intestine essentially intact. Upon entry into the duodenum, emulsification (the mixing of two normally immiscible liquids) occurs, in this case, fat and water. Formation of an emulsion increases the surface area of the lipid, which permits greater enzymatic interaction and processing. Emulsification is aided by bile, which contains bile salts, pigments, and cholesterol. Bile is secreted by the liver and stored in the gallbladder. Finally, the pancreas secreates pancreatic lipase, colipase, and cholesterol esterase into the small intestine; together, these enzymes hydrolyze the lipid components to 2-monoacylglycerol, free fatty acids, and cholesterol.

Michaelis-Menten graph is a_______ graph whereas a lineweaver-burk plot is a _____ graph what do cooperativity graphs look like?

M-M: hyperbola L-B: double reciprocal graph of the Michaelis mention equation Cooperativity: Sigmoidal (S-shaped)

Explain the response to bacterial (extracellular pathogen) infections.

Macrophages are always on the lookout for potential invaders. Let's say a person suffers a laceration and bacteria are introduced into the body via the laceration. First, macrophages (and other antigen presenting cells) engulf the bacteria and subsequently release inflammatory mediators. These cells also digest the bacteria and present antigens from the pathogen on their surfaces in conjunction with MHC-II. The cytokines attract inflammatory cells, including neutrophils and additional macrophages. Mast cells are activated by the inflammation and degranulate, resulting in histamine release and increased leakiness of the capillaries. This augments the ability of the immune cells to leave the bloodstream to travel to the affected tissue. The dendritic cell then leaves the affected tissue and travels to the nearest lymph node, where it presents the antigen to B-cells. B-cells that produce the correct antibody proliferate through clonal selection to create plasma cells and memory cells. Antibodies then travel through the bloodstream to the affected tissue, where they tag the bacteria for destruction. At the same time, dendritic cells are also presenting the antigen to T-cells, activating a T-cell response. In particular, CD4+ T-cells are activated. These cells come in two types, Th1 and Th2. Th1 cells release interferon gamma (INF-gamma) which activates macrophages and increase their ability to kill bacteria. Th2 cells help activate B-cells and are more common in parasitic infections. After the pathogen has been eliminated, plasma cells die, but memory B and T cells remain. These memory cells allow for a much faster secondary response upon exposure to the pathogen at a later time.

How can I remember the names of peptide vs amino acid derivative vs steroid hormones?

Most peptide and amino acid derivative hormones have names that end in -in or -ine (insulin, vasopressin, thyroxine, etc). Most steroid hormones have names that end in -one, -ol, or -oid (testosterone, aldosterone, cortisol, glucocorticoids)

Explain the nativist (biological theory) of language development

Noam Chomsky advocates for the existence of some innate capacity for language. Chomsky is known for his studyu of transformational grammar. He focused on syntactic transformations, or changes in word order that retain the same meaning. Ex: I took the MCAT vs The MCAT was taken by me. Chomsky notes that children learn to make such transformations effortlessly at an early age. He therefore concluded that this ability must be innate. In this theory, the innate ability is called the language acquisition devise (LAD), a theoretical pathway in the brain that allows infants to provess and absorb language rules. Nativists believe in a critical period for language acquisition between two years and puberty. If no language exposure occurs during this time, later training is largely ineffective. However, when this actually happened with a girl. She was able to learn some aspects of syntax, though she couldn't master many rules of language. This is suggestive of a sensitive, rather than a critical period. A sensitive period is a time when environmental input has maximal effect on the development of an ability. Most psychologists consider the sensitive period for language development to be before the onset of puberty

explain the process of neurulation.

Once the three germ layers are formed, neurulation, or development of the nervous system, can begin. The nervous system is derived from the ectoderm. First, a rod of mesodermal cells known as the notochord forms along the long axis of the organism like a primitive spine. The notochord includes a group of overlying ectodermal cells to slide inward to form neural folds, which surround a neural groove. The neural folds grow toward one another until they fuse into a neural tube, which gives rise to the CNS. At the tip of each neural fold are neural crest cells. These cells migrate outward to form the PNS as well as specific cell types in other tissues. Finally, ectodermal cells will migrate over the neural tube and crest to cover the rudimentary nervous system.

Explain the function and purpose of growth hormone (GH).

One of 7 hormones released by the anterior pituitary. Direct hormone. - promotes the growth of bone and muscle -this sort of growth is energetically expensive and requires large quantities of glucose. -GH release is stimulated by GHRH (growth hormone-releasing hormone) from the hypothalamus

Explain the function of endorphins.

One of 7 hormones released by the anterior pituitary. Direct hormone. -decrease the perception of pain -can even induce a sense of euphoria

Explain the role of phosphoenolpyruvate carboykinase (PEPCK) in gluconeogenesis.

PEPCK in the cytoplasm is induced by glucagon and cortisol, which generally act to raise blood sugar levels. It converts OAA to phosphoenolpyruvate (PEP) in a reaction that requires GTP. PEP continues in the apthway to fructose 1,6-bisphosphate. Thus, the combination of pyruvate carboxylase and PEPCK are used to circumvent the action of pyruvate kinase by converting pyruvate back into PEP.

What is penetrance? Explain full penetrance, high penetrance, reduced penetrance, low penetrance, and nonpenetrance.

Penetrance is a population measure defined as the proportion of individuals in the population carrying the allele who actually express the phenotype. It is the probability that, given a particular genotype, a person will express the phenotype. Alleles can be classified by their degree of penetrance. Ex, in Huntington's disease, indivifuals with more than 40 sequence repeats have Full penetrance, where 100% of the individuals with this allele show symptoms of Huntington's disease. Individuals with fewer sequence repeats show high penetrance, wherein most (but not all) of those with the allele show symptoms of the disease. With fewer sequence repeats the gene comes to have reduced penetrance, low penetrance, or even nonppenetrance.

What are the substrates of the citric acid cycle, in order?

Please Can I Keep Selling Seashells For Money, Officer? Pyruvate Citrate Isocitrate alpha-ketoglutarate Succinyl-CoA Succinate Fumarate Malate Oxaloacetate

Explain the G1 stage of cell division. How does the cell enter into the next stage?

Presynthetic Gap Stage: cells create organelles for energy and protein production (mitochondria, ribosomes, etc) while also increasing in size. Passage into the S (synthesis) stage is governed by a restriction point(G1/S checkpoint). Certain criteria, such as containing the proper complement of DNA must be met for the cell to pass the restriction point

Proline: 3 letter abbreviation, 1 letter abbreviation, polar or non polar

Pro.P. Non-polar

A cellular protein, Protein X, is translated during interphase and undergoes proteolysis during mitosis. Which formula describes the intracellular Protein X concentration? (x is the independent variable, and a, b, and c are constants, and e is Euler's constant) a) [Protein X] = a(x+b) b) [Protein X] = asin(bx+c) c) [Protein X] = a(x+b)^2 d) [Protein X] = ae^b(x+c)

Protein X is described as undergoing synthesis and destruction along with the cell-cycle. The correct formula describing the concentration must oscillate. Choice a) is linear, choice b) oscillates, choice c) climbs or descends parabolically, and choice d) exhibits exponential growth or decay. Only choice b) provides the correct qualitative behavior. a) [Protein X] = a(x+b), incorrect, This choice is linear. b) [Protein X] = asin(bx+c), correct. c) [Protein X] = a(x+b)^2, incorrect, This choice is parabolic. d) [Protein X] = ae^b(x+c), incorrect, This choice is exponential.

Nucleic acids are classified according to the pentose they contain. If the pentose is ribose, the nucleic acid is _______. If the pentose is deoxyribose, then it is______.

RNA DNA

Explain the gross structure of myocytes.

Sarcomeres are attached end to end to form myofibrils. Myofibrils are surrounding by a covering known as the sarcoplasmic reticulum (SR), a modified endoplasmic reticulum that contains a high [Ca2+] ions. The sarcoplasm is a modified cytoplasm located just outside the sarcoplasmic reticulum. The cell membrane of a myocyte is known as the sarcolemma. The sarcolemma is capable of propogating an action potential and can distribute the action potential to all sarcomeres in a muscle using a system of transverse tubules (T-tubles) that are oriented perpendicularly to the myofibrils. Each myocyte, or muscle cell, contains many myofibrils arranged in parallel and cal also be called a muscle fiber. The nuclei, of which there are many, are usually found at the periphery of the cell. Finally, many myocytes in parallel form a muscle.

Explain the equilibrium potential of sodium across a cell. What is it?

Sodium's [ ] gradient is the reverse of K+s, with a [ ] of about 12mM inside and 145 mM outside of the cell, meaning there is a driving force pushing sodium into the cell. This movement is facilitated by sodium leak channels. The slow leak of sodium into the cell causes a build-up of electric potential. The equilibrium potential of sodium is around 60mV and is positive because sodium is moving into the cell.

Explain the equilibrium potential of potassium across a cell. What is it?

The K+ [ ] inside the cell averages about 140 mM, as compared to 4mM outside of the cell. This [ ] difference makes it favorable for K+ to move outside of the cell. K+ moves outside via K+ leak channels, which allow the slow leak of K+ outside the cell. As K+ leaks out of the cell, the cell loses a small amount of pos charge, leaving behind a small amount of neg charge. As neg charge builds up inside the cell, some potassium will be drawn back into the cell due to the attraction between pos K+ ions and the neg potential building inside the cell. As the potential difference continued to grow, K+ will also be more strongly drawn back into the cell. At a certain potential, each K+ cation that is pushed out due to the [ ] gradient will be matched by another K+ cation pulled back in due to the electric potential. At this point, there is no more net movement of the ion, as the cell is in equilibrium with respect to K+. The potential difference that represents this K+ equilibrium is called the equilibrium potential of K+. Potassiums equilibrium potential is around -90mV. The neg sign is assigned due to convention, and because a pos ion (K+) is leaving the cell

Explain marijuana

The active chemica is tetrahydrocannabinol (THC). THC exerts its effects by acting at cannabinoid receptors, glycine receptors, and opioid receptors. How these receptors interact to create the "high" achieved from MJ use is unknown. It is known that THC inhibits GABA activity and indirectly inc dopamine activity (causing pleasure). Physiological effects are mixed, including eye redness, dry mouth, fatigue, impairment of short-term memory, inc HR, inc appetite, and dec BP. Psychologically, effects seem to fall into the categories of stimulant, depressant, and hallucinogen.

What is the role of the cytoskeleton? What are the 3 components of the cytoskeleton?

The cytoskeleton helps the cell maintain its shape and provides structure to the cell. It also provides a conduit for the transport of materials around the cell. microfilaments, microtubules, and intermediate filaments

What is the proximal convoluted tubule? Explain its funciton.

The filtrate first enters the proximal convoluted tubule (PCT) in the nephron. Here, amino acids, glucose, water-soluble vitamins, and the majority of salts are reabsorbed along with water. Almost 70% of filtered sodium will be reabsorbed here, but the filtrate remains isotonic to the interstitium, as other solutes and a large volume of water are also reabsorbed. Solutes that inter the interstituim- the connective tissue surrounding the nephron- are picked up by the vasa recta to be returned to the bloodstream for reuse within the body. The PCT is also the site of secretion for a number of waste products, including hydrogen ions, potassium ions, ammonia, and urea.

Explain posttranslational processing.

The nascent polypeptide chain is subject to posttranslational modifications before it will become a functioning protein, similar to how hnRNA is modified prior to being released from the nucleus. One essential step for the final synthesis of the protein is proper folding. There is a specialized class of proteins called chaperones, the main function of which is to assist in the protein-folding process. Many proteins are also modified by cleavage events. A common example of this is insulin, which needs to be cleaved from a larger, inactive peptide to achieve its active form. In peptides with signal sequences, the signal sequence must be cleaved if the protein is to enter the organelle and accomplish its function. In peptides with quaternary structure, subunits come together to form the functional protein. A classic example is hemoglobin, which is composed of two alpha chains and two beta chains. Other biomolecules may be added to the peptide via the following processes - Phosphorylation. Addition of a phosphate group by protein kinases to activate or deactivate proteins; phosphorylation in eukaryotes is most commonly seed with serine, threonine, and tyrosine - Carboxylation. Addition of carboxylic acid groups, usually to serve as calcium-binding sites - Glycosylation. Addition of oligosaccharides as proteins pass through the ER and Golgi apparatus to determine cellular destination -Prenylation. Addition of lipid groups to certain membrane-bound enzymes.

Explain female sexual development.

The ovaries are under the control of FSH and LH secreted by the ant pit. The ovaries produce estrogens and progesterones Estrogens are secreted in response to FSH and result in the development and maintenance of the female reproductive system and female secondary sex characteristics (breast growth, widening of hips, changes in fat distribution). in the embryo, estrogens stimulate development of the reproductive tract. As adults, estrogens lead to the thickening of the lining of the uterus (endometrium) each moth in prep for the implantation of a zygote. Progesterone is secreted by the corpus lute-the remains of the follicle following ovulation-in response to LH. Progesterone is involved in the development and maintenance of the endometrium, but not in the initial thickening of the endometrium (the role of estrogen). This means that both estrogen and progesterone are required for the generation, development, and maintenance of an endometrium capable of supporting a zygote. By the end of the first trimester of a pregnancy, progesterone is supplied by the placenta, while the corpus lute atrophies

What is the role of the pancreas in digestion?

The pancreas serves two quite different roles in the body, reflecting its exocrine and endocrine functions. The endocrine functions of the pancreas include the release of insulin, glucagon, and somatostatin-peptide hormones necessary for the maintenance of proper blood sugar levels. The hormonal function of the pancreas is limited to cells residing in the islets of Langerhands scattered throughout the organ. The bulk of the pancreas however is made up of exocrine cells called acinar cells that produce pancreatic juices. Pancreatic juices are bicarbonate-rich alkaline secretions containing many digestive enzymes that work on all three classes of biomolecules. pancreatic amylase breaks down large polysaccharides into small disaccharides and is therefore responsible for carbohydrate digestion. The pancreatic peptidases (trypsinogen, chymotrypsinogen, and carboxypeptidases A and B) are released in their zymogen form, but once activated are responsible for protein digestion. Enteropeptidase, produced by the duodenum, is the master switch. it converts trypsinogen to trypsin, which can then activate the other zymogens, and also activates procarboxypeptidases A and B. Finally, the pancreas secretes pancreativ lipase, which is capable of breaking down fats into free fatty acids and glycerol.

Explain the structure and function of the ribosome.

The ribosome is composed of proteins and rRNA. In both prokaryotes and eukaryotes, there are large and small subunits; the subunits only bind together during protein synthesis. The structure of the ribosome dictates its main function, which is to bring the mRNA message together with the charged aminoacyl0tRNA complex to generate the protein. There are three binding sites in the ribosome for tRNA: the A site (aminoacyl), the P site (peptidyl), and E site (exit). Eukaryotic ribosomes contain four strands of rRNA, designated the 28S, 18S, 5.8S, and 5S rRNAs, the "S" values indicate the size of the strand. These genes for some of the rRNAs (28S, 18S, and 5.8S rRNAs) used to construct the ribosome are found in the nucleolus. RNA polymerase I transcribes the 28S, 18S, and 5.8S rRNAs as a single unit within the nucleolus, which results in a 45S ribosomal precursor RNA. This 45S pre-rRNA is processed to become the 18S rRNA of the 40S (small) ribosomal subunit and the 38S and 5.8S of the 60S (large) ribosomal subunit. RNA polymerase III trasnscribes the 5S rRNA, which is also found in the 60S ribosomal subunit; this process takes place outside the nucleolus. The ribosomal subunits created are the 60S and the 40S subunits; these subunits join during protein synthesis to form the whole 80S ribosome.

Explain the sarcomere. What is it? What is it made of? How is it organized? What are the different parts? What happens to the functional units of the sarcomere during contraction?

The sarcomere is the basic contractile unit of skeletal muscle. Sarcomeres are made of thick and thin filaments. The thick filaments are organized bundles of myosin, the thin filaments are made of actin along with two other proteins: troponin and tropomyosin. These proteins help to regulate the interaction between the actin and myosin filaments. Another protein, titin, acts as a spring and anchors the actin and myosin filaments together, preventing excessive stretching of the muscle. Each sarcomere is divided into different lines, zones, and bands. Z-lines define the boundaries of each sarcomere. The M-line runs down the center of the sarcomere, through the middle of the myosin filaments. The I-band is the region containing exclusively thin filaments, whereas the H-zone contains only thick filaments. The A-band contains thick filaments in their entirety, including any overlap with thin filaments. During contraction, the H-zone, I-band, the distance between the Z-lines, and the distance between the M-lines all becomes smaller, whereas the A-bands size remains constant. Mnemonic Parts of the sarcomere: Z-Z is the end of the alphabet, and the end of the sarcomere M-Middle of the Myosin filaments I-I is a THIN letter (THIN filaments only) H-H is a THICK letter (THICK filaments only) A-All of the thick filament, whether or not it is overlapping

Many bacteria contain plasmids. What are they? What do they do?

They are extrachromosomal material that often carry genes that impart some benefit to the bacterium, such as antibiotic resistance. Plasmids may also carry additional virulence factors, or traits that increase pathogenicity such as toxin production, projections that allow attachment to certain kinds of cells, or features that allow evasion of the host's immune system. A subset of plasmids called episomes are capable of integrating into the genome of the bacterium.

Explain microtubules. What 3 important things are composed of microtubules?

They are one of the 3 elements of the cytoskeleton. They are hollow polymers of tubule proteins. Microtubules radiate throughout the cell, providing the primary pathways along which motor proteins like kinesin and dynein carry vessels. cilia, flagella, and centrioles

explain microfilaments. What are they made of? What types of things are they involved in?

They are one of the 3 elements of the cytoskeleton. They are made up of solid polymerized roots of actin. the actin filaments are organized into bundles and networks are resistant to both compression and fracture, providing structure for the cell. Actin filaments can also use ATP to generate force for movement by interacting with myosin, such as in muscle contraction. Microfilaments play a role in cytokinesis, or the division of materials between daughter cells. During mitosis, the cleavage furrow is formed from microfilaments, which organize as a ring at the site of division between the two new daughter cells. As the actin filaments within this ring contract, the ring becomes smaller, eventually pinching off the connection between the two daughter cells.

What is special about the mitochondria? This makes them examples of what?

They are semi-autonomous. They contain some of their own genes and replicate independently of the nucleus via binary fission. They are examples of cytoplasmic or extranuclear inheritance (the transmission of genetic material independent of the nucleus)

Where are epithelial tissues found? What does this do? What 3 things are epithelial cells involved in (in certain organs)? How are they joined together?

They cover the body and line its cavities, providing a means for protection against pathogen invasion and desiccation In certain organs, epithelial cells are involved in absorption, secretion, and sensation. Joined together and to an underlying layer of connective tissue known as the basement membrane

How do prokaryotes work?

They do not contain any membrane-bound organelles, and their genetic material is organized into a single circular molecule of DNA concentrated in an area of the cell called the nucleoid region. Despite the simplicity of prokaryotes, they are incredibly diverse, and knowledge of this diversity is essential for the study of medicine b/c many prokaryotes can cause infection. choosing the appropriate antibiotic to fight an infection requires knowledge about the basic structure of the bacteria causing the infection.

How does the direction of rotation (+) (-) relate to D and L naming in a molecule?

They do not relate. Direction of rotation is determined experimentally and CANNOT be determined from D vs L rotation.

What are catalysts?

They help a reaction proceed at a much faster rate. They do not impact the thermodynamics of a biological reaction, that is the deltaH and equilibrium position do not change. As a catalyst, the enzyme is not changed during the course of a reaction.

What are the three types of membrane proteins accounted for in the fluid mosaic model? Explain their differences and functions.

Transmembrane proteins pass completely through the lipid bilayer Embedded proteins are associated with only the interior (cytoplasmic) or exterior (extracellular) surface of the cell membrane. Together, transmembrane and embedded proteins are considered integral proteins because of their association with the interior of the plasma membrane, which is usually assisted by one or more membrane-associated domains that are partially hydrophobic. Membrane-associated (peripheral) proteins may be bound through electrostatic interactions with the lipid bilayer, especially at lipid rafts, or to other transmembrane or embedded proteins, like the G proteins found in G protein-coupled receptors. Transporters, channels, and receptors are generally transmembrane proteins.

What is the recognition-primed decision model?

Used for intuitive decision making. Explains how people can make effective decisions under time pressure and uncertainty. It is considered part of naturalistic decision making. It *attempts to understand how humans make decisions in complex, real-world settings such as firefighting and critical care nursing without having to compare options* the doctor's brain is actually sorting through a wide variety of info to match a pattern.

what are polymorphisms

Variations in the DNA sequence that occur in at least 1% of the population. ex light and dark coloration in the same species of butterfly

How does virus infection work?

Viruses can only infect a specific set of cells because they must bind to a specific receptor. Once it binds, enveloped viruses fuse with the plasma membrane of a cell, allowing entry of the virion into the host cell. Bacteriophages use tail fibers to anchor themselves to the cell membrane and inject their genome into the host bacterium through the tail sheath. Depending on the virus, different portions of the virion will be inserted into host cells. Enveloped viruses such as HIV fuse with the membrane and enter the cell intact, whereas bacteriophages only insert their genetic material, leaving their capsids outside the host cell.

A patient presents to the ER with an asthma attack. The patient has been hyperventilating for the past hour and has a blood pH of 7.52. The patient is given treatment and does not appear to respond, but a subsequent blood pH reading is 7.41. Why might this normal blood pH NOT be a reassuring sign? A. The patient's kidneys may have compensated for the alkaleimia. B. the normal blood pH reading is likely inaccurate C. the patient may be descending into respiratory failure. D. the patient's blood should ideally become academic for some time to compensate for the alkalemia

When a patient with an asthma attack does not respond to treatment and has been hyperventilating for over an hour, he or she may become fatigued and may not be able to maintain hyperventilation. In this case, the patient begins to decrease his or her breathing rate and is not receiving adequate O2. By extension, CO2 is trapped in the blood and may start demonstrating academia in the near future. While the kidneys should compensate for alkaleimia, this is a slow process and would not normalize the blood pH within an hour. Adequate compensation by the kidneys would be a reassuring sign, eliminating A. There is no evidence that the reading is inaccurate, eliminating B. After treatment, the patient should return to a normal blood pH with adequate ventilation and would not be expected to overcompensate by becoming academic, eliminating D.

When the pH of a solution is approximately equal to the pKa of the solute, the solution acts as what?

a buffer

What is Bowman's capsule?

a capsule-shaped membranous structure surrounding the glomerulus of each nephron in the kidneys of mammals that extracts wastes, excess salts, and water from the blood.

All bacteria contain what? Some have what?

a cell membrane and cytoplasm. some have flagella or fimbria (similar to cilia)

How does column chromatography work?

a column is filled with silica or alumina beads as an adsorbent, and gravity moves the solvent and compounds down the column. As the solution flows through the column, both size and polarity have a role in determining how quickly a compound moves through the polar silica or alumina beads: the less polar the compound, the faster it can elute through the column. In column chromatography, the solvent polarity, pH, or salinity can easily be changed to help elute the protein of interest.

what are pancreatic juices?

a complex mixture of several enzymes in a bicarbonate-rich alkaline solution. This bicarbonate helps to neutralize acidic chime, as well as provide an ideal working environment for the digestive enzymes, which are most active around pH 8.5. Pancreatic juices contain enzymes that can digest all three types of nutrients: carbohydrates, fats, and proteins.

What is a schema?

a concept or framework that organizes and interprets information

What is glycolysis?

a cytoplasmic pathway that converts glucose into two pyruvate molecules, releasing a modest amount of energy captured in two substrate level phosphorylations and one oxidation reaction. If a cell has mitochondria and oxygen, the energy carriers produced in glycolysis (NADH) can feed into the aerobic respiration pathway to generate energy for the cell. Glycolysis also provides intermediates for other pathways. In the liver, glycolysis is part of the process by which excess glucose is converted to fatty acids for storage.

What are centromeres? What are their functions?

a region of DNA in the center of chromosomes. -sites of constriction because they form noticeable indentations. -composed of heterochromatin, which is composed of tandem repeat sequences that also contain high GC content.

what is facilitated diffusion?

a type of passive transport, the diffusion of molecules down a concentration gradient through a port in the membrane created by a transmembrane protein. it is used for molecules that are impermeable to the membrane (large, polar, or charged)

Upon ovulation, the oocyte is released into the

abdominal cavity, where it is close to the fallopian tube , drawn in by beating cilia.

Which filament is associated with troponin and tropomyosin? Are these thick or thin?

acTin (has a t in it). Acthin: actin filaments are THIN. Myosin filaments are thick.

How are different epithelia classified? What are the 6 classifications?

according the the number of layers they have and the shape of their cells Cell Classifications: Simple epithelia (one layer of cells), Stratified epithelia (multiple layers of cells), pseudo stratified epithelia (appear to have multiple layers due to differences in cell height but are in reality only one layer) Shape Classifications: Cuboidal (cube shaped), Columnar (long and thin), Squamous (flat and scalelike)

What is active transport? What are the two types? Explain them. What's the difference between symport and antiport channels.

active transport results in the net movement of a solute against its [ ] gradient. Active transport always requires energy, but this source of this energy can vary. Primary active transport uses ATP or another energy molecule to directly power the transport of molecules across a membrane. Generally, primary active transport uses a transmembrane ATPase. Secondary active transport, (aka coupled transport) also uses energy to transport particles across the membrane; however, in contrast to primary active transport, there is no direct coupling to ATP hydrolysis. Instead, secondary active transport harnesses the energy released by one particle going down its electrochemical gradient to drive a different particle up its gradient. When both particles flow the same direction across the membrane, it is termed symport. When the particles flow in opposite directions, it is called antiport. Active transport is important in many tissues. For instance, primary active transport maintains the membrane potential of neurons in the nervous system. The kidneys use secondary active transport, usually driven by sodium, to reabsorb and secrete various solutes into and out of the filtrate.

what is neo-Darwinism (the modern synthesis model)

adds knowledge of genetic inheritance and changes int he gene pool to Darwin's original theory. when mutation or recombination results in a change that is favorable to the organism's reproductive success, that changes is more likely to pass on to the next generation. The opposite is true. This is called differential reproduction.

In embryonic development, the adrenal cortex is derived from the _______ and the adrenal medulla is derived from the __________. Why?

adrenal cortex-mesoderm adrenal medulla-ectoderm because the adrenal medulla contains from nervous tissue

Explain the role of adipose tissue after a meal and during the fasting state.

after a meal, elevated insulin levels stimulate glucose uptake by adipose tissue. insulin also triggers fatty acid release from VLDL and chylomicrons (which carry triacylglycerols absorbed from the gut). Lipoprotein lipase, an enzyme found in the capillary bed of adipose tissue, is also induced by insulin. The fatty acids that are released from lipoproteins are taken up by adipose tissue and re-esterified to triacylglycerols for storage. The glycerol phosphate required for triacylglycerol synthesis comes from glucose that is metabolized in adipocytes as an alternative product of glycolysis. Insul can also effectively suppress the release of fatty acids from adipose tissue. During the fasting state, decreased levels of insulin and increased epi activate hormone-sensitive lipase in fat cells, allowing fatty acids to be released into circulation.

What are the functions of glucagon, insulin, and somatostatin? What are they secreted by?

all three secreted by the islets of Langerhans cells in the pancreas. glucagon is secreted by the alpha cell subtype, insulin is secreted by the beta cell subtype, somatostatin is secreted by the delta cell subtype. Glucagon is secreted during times of fasting. When glucose levels are low, glucagon increases glucose production by triggering glycogenolysis, gluconeogenesis, and the degradation of protein and fat. In addition to low blood glose [ ]s, certain GI hormones (such as cholecystokinin and gastin) inc glucagon release from alpha cells. When blood glucose [ ]s are high, glucagon release is inhibited. Insulin is antagonistic to glucagon and is therefore secreted when blood glucose levels are high. Insulin induces muscle and liver cells to take up glucose and store it as glycogen for later use. In addition, b/c it is active when glucose levels are high, insulin stimulates anabolic processes such as fat and protein synthesis. Somatostatin is an inhibitor of both insulin and glucagon secretion. High blood glucose and amino acid [ ]s stimulate its secretion. Somatostatin is also produced by the hypothalamus, where it decreases growth hormone secretion in addition to its effects on insulin and glucagon.

How can you determine the [ ] of a protein?

almost exclusively through spectroscopy. The Bradford Protein Assay method is most common. There is also the BCA assay and Lowry reagent assay

Explain suppressor or regulatory T-cells

also express CD4, but can be differentiated from helperer T-cells because they also express a protein called Foxp3. These cells help to tone down the immune response once infection has been adequately contained. These cells also turn off self-reactive lymphocytes to prevent autoimmune diseases: this is termed self-tolerance.

What are the two major function of the pentose phosphate pathway (PPP)? What is it also called and where does it occur?

also known as the hexose monophosphate (HMP) shunt. occurs in the cytoplasm of all cells. Functions: production of NADPH and serving as a source of ribose 5-phosphate for nucleotide synthesis.

What is a silent mutation?

alters a base but does not change the amino acid

what are antibodies produced by? What are they? What is their method of action?

antibodies, also called immunoglobulins [Ig]. they are produced by B cells. each B cell can make only one type of antibody, but we have many B cells, so our immune system can recognize many antigens. Not all B-cells that are generated actively or constantly produce antibodies b/c antibody production is an energetically expensive process, and there isnt' reason to expend energy to produce antibodies that aren't needed. Antibodies can be present on the surface of a cell or secreted into body fluids. When an antibody binds to an antigen, the response will depend on the location. For antibodies secreted into body fluids, there are 3 main possibilities. First, once bound to a specific antigen, antibodies may attract other leukocytes to phagocytize those antigens immediately. This is called opsonization. Second, antibodies may cause pathogens to clump together (agglutinate) forming large insoluble complexes that can be phagocytized. Third, antibodies can block the ability of a pathogen to invade tissues, essentially neutralizing it. For cell-surface antibodies, the binding of antigen to a B-cell causes activation of that cell, resulting in its proliferation and formation of plasma and memory cells. In contrast, when antigen binds to antibodies on the surface of a mast cell, it causes degranulation (exocytosis of granule contents), releasing histamine and causing inflammatory allergic reaction.

Explain the process of antibody production. Explain primary vs secondary response and how this relates to vaccines.

antibodies, also called immunoglobulins [Ig]. they are produced by B cells. each B cell can make only one type of antibody, but we have many B cells, so our immune system can recognize many antigens. Not all B-cells that are generated actively or constantly produce antibodies b/c antibody production is an energetically expensive process, and there isnt' reason to expend energy to produce antibodies that aren't needed. Instead, naïve B cells (those that haven't been exposed to an antigen) wait in the lymph nodes for their particular antigen to come along. Upon exposure to the correct antigen, a B cell will proliferate and produce two types of daughter cells. Plasma cells produce large amounts of antibodies, whereas memory B-cells stay in the lymph node, awaiting re-exposure to the same antigen. This initial activation takes approx. 7-10 days and is known as the primary response. The plasma cells will eventually die, but the memory cells may last the lifetime fo the organism. If the same microbe is every encountered again, the memory cells jump into action and produce the antibodies specific to that pathogen. This immune response, called the secondary response, will be more rapid and robust. The development of these lasting memory cells is the basis of the efficacy of vaccination.

What is ANP and what secretes it?

atrial natriuretic peptide, secreted by the heart. helps regulate salt and water balance. when cells in the atria are stretched from excess blood volume, they release ANP. This hormone promotes excretion of sodium and therefore increases urine volume. This effect is functionally antagonistic to aldosterone because it lowers blood volume and pressure, and has no effect on blood osmolarity

How is Rh factor inherited?

autosomal dominant inheritance; one positive allele is enough for that protein to be expressed.

what is the mechanism of action of a steroid hormone? What are the effects like compared to peptide hormones? How do steroid hormones get to places in the body?

b/c they are derived from non-polar molecules, they can easily cross the cell membrane, hence their receptors are usually intracellular (in the cytosol) or intranuclear (in the nucleus). upon binding to the receptor, steroid-hormone receptor complexes undergo conformational changes. The receptor can then bind directly to DNA, resulting in either inc or dec transcription of particular genes. - One common form of conformational change is dimerization, or pairing of two hormone-receptor complexes. the effects of steroid hormones are slower but longer-lived than peptide hormone because steroid hormones participate in gene regulation, causing alterations in the amount of mRNA and protein present ina cell by direct action on DNA. steroid hormones are not water-soluble, so they need a protein carrier to travel in the bloodstream. -some of these proteins are very specific and carry only one hormone while other proteins are nonspecific. -note that hormones are generally inactive while attached to a carrier protein and must dissociate from the carrier to function. Therefore, levels of carrier proteins can change the levels of active hormone. Ex: some conditions inc the quantity of a protein that carries thyroid hormones (TBG), which causes the body to perceive a lower level of thyroid hormone b/c of the inc quantity of TBG binds a larger proportion of the hormone, meaning that there is less free hormone available

What is the mechanism of action of a peptide hormone?

because they are charged and can't pass through the plasma membrane, they must bind to an extracellular receptor. The peptide hormone is considered the first messenger, it binds to the receptor and triggers the transmission of a second signal, the second messenger. The connection between the hormone at the surface and the effect brought about by second messengers within the cell is known as a signaling cascade. At each step, there is the possibility of amplification. Some common second messengers are cAMP, calcium, and IP3.

Why are fats preferred for long-term energy storage compared to carbohydrates, proteins, or ketones

because they are so much more energy gense. complete combustion of fat results in 9kcal/g of energy compared with only 4kcal/g derived from carbs, proteins, or ketones.

Run through the digestive tract. In addition to the digestive tract itself, what organs help to provide the enzymes and lubrication necessary to aid in the digestion of food?

begins with the oral cavity (mouth) followed by the pharynx, a shared pathway for both food entering the digestive system and air entering the respiratory system. From the pharynx, food enters the esophagus, which transports it to the stomach. From the stomach, food travels to the small intestine and then to the large intestine. Finally, waste products of digestion enter the rectum, where feces are stored until an appropriate time of release. The salivary glands, pancreas, liver, and gallbladder help to provide the enzymes and lubrication necessary to aid in the digestion of food

explain uncompetitive inhibition

bind only to the enzyme-substrate complex and essentially lock the substrate in the enzyme, preventing its release. can be interpreted as inc affinity between the enzyme and substrate. b/c the ES complex has already formed upon binding, uncompetitive inhibitors must bind at an allosteric site (it is the formation of the ES complex that creates a conformational change that allows the uncompetitive inhibitor to bind) lowers Km and Vmax

What is senescence? Explain cellular level senescence in depth. Touch on organismal level senescence.

biological aging. At the cellular level, senescence results in the failure of cells to divide, normally after approx 50 divisions in vitro. Research has demonstrated that this may be due to shortened telomeres, or the ends of chromosomes. Telomeres reduce the loss of genetic info from the ends of chromosomes and prevent DNA from unraveling-their high [ ] of guanine and cytosine enables telomeres to "knot off" the end of the chromosome. Telomeres are difficult to replicated, however, so they shorten during each round of DNA synthesis. Eventually, the telomeres become too short, and the cell is no longer able to replicate. Some cells, including germ cells, fetal cells, and tumor cells, express an enzyme called telomerase. This enzyme is a reverse transcriptase that is able to synthesize the ends of chromosomes, preventing senescence. Telomerase allows cells to divide indefinitely and may play a role in the survival of cancer cells. At the organismal level, senescence represents changes in the body's ability to respond to a changing environment. Aging is complex and often involves not only cellular senescence but also the accumulation of chemical and environmental insults over time.

Explain Paivio's dual-coding theory

both verbal association and visual images are used to process and store info. Ex: the word "dad" can recall some info, and the picture of "dad" can recall the same info. The fact that we can code this info two different ways builds redundancy and increases the chance that the info can be retrieved and used effectively when cued, much like search engine optimization within a computer program.

how can you determine protein activity?

by monitoring a known reaction with a given [ ] of substrate and comparing it to a standard. Activity is correlated with [ ] but is also affected by the purification methods used and the conditions of the assay

Explain sleep deprivation

can result from as little as one night without sleep, or from multiple nights with poor-quality, short-duration sleep. While one can't make up lost sleep, people who are permitted to sleep normally after sleep deprivation often exhibit REM rebound, an earlier onset and greater duration of REM sleep compared to normal.

When cell cycle control becomes deranged, and the damaged cells are allowed to undergo mitosis, _____ may result.

cancer

Explain capillaries structure and role in the cardiovascular system.

capillaries are vessels with a single endothelial cell layer and are so small that RBCs must pass through the capillaries in a single file line. The thin wall of the capillary allows easy diffusion of gases (O2 and CO2), nutrients (most notably, glucose), and wastes (ammonia and urea, among others). Capillaries are therefore the interface for communication of the circulatory system with the tissues. Remember too that blood carries hormones also, so capillaries allow endocrine signals to arrive at their target tissues.

When describing a fatty acid as 8:4, what does this mean?

carbons:double bonds 8 carbons 4 double bonds

what is cartilage? what is it made from? how does it differ from bone and why is it important? Explain the process of hardening cartilage into bone and what it is called. What is the other process of bone formation? Where do these respective processes occur?

cartilage is softer and more flexible than bone. It consists of a firm but elastic matrix called chondrin that is secreted by chondrocytes. Fetal skeletons are mostly made up of cartilage. This is good because fetuses must grow and develop in a confined environment and then must traverse the birth canal. Adults have cartilage only in body parts that need a little extra flexibility or cushioning (external ear, nose, walls of larynx and traches, intervertebral discs, and joints). Cartilage also differs from bones in that it is avascular and is not innervated. This process is endochondral ossificaiton and is responseible for the formation of most of the long bones of the body. Bones may also be formed through intramembranous ossification, in which undifferentiated embryonic connective tissue (mesenchymal tissue) is transformed into, and replaced by, bone. This occurs in the bones of the skull.

What is Wernike-Korsakoff syndrome?

caused by a deficiency of thiamine (vitamin B1) and characterized by severe memory impairment with changes in mental status and loss of motor skills

Explain the complement system. Is it a specific or nonspecific defense?

consists of a number of proteins in the blood that act as a nonspecific defense against bacteria. Complement can be activated through a classical pathway (which requires the binding of an antibody to a pathogen) or an alternative pathway (which does not require antibodies). The complement proteins punch holes in the cell walls of bacteria, making them osmotically unstable. Despite the association with antibodies, complement is considered a nonspecific defense because it cannot be modified to target a specific organism over others.

What is the Golgi apparatus? How do they work?

consists of stacked membrane bound space. Materials from the ER are transferred to the GA in vesicles. Once inside the Golgi apparatus, these cellular products may be modified by addition of groups like carbohydrates, phosphates, and sulfates. The Golgi apparatus may also modify cellular products through the induction of signal sequences, which direct the delivery of the product to a specific cellular location. After modification and sorting in the Golgi apparatus, cellular products are repackaged in vesicles, which are directed to the correct cellular location. If the product is destined for secretion, the secretory vesicle merges with the cell membrane and its contents are released via exocytosis.

how does sweating work

controlled by autonomic nervous system. cools body down when body temp rises above set point determined by the hypothalamus. Postganglionic sympathetic neurons that utilize acetylcholine innervate sweat glands and promote the secretion of water with certain ions on to the skin. heat is then absorbed from the body as the water molecules undergo a phase change to evaporate. The production of sweat itself is not the main mechanism of cooling; it is the evaporation of water from the skin which absorbs body heat. At the same time, arteriolar vasodilation occurs to maximize heat loss. This brings a large quantity of blood to the skin, which accelerates the evaporation of sweat by maximizing the heat energy available for the liquid-gas phase change.

explain cooperativity in enzymes.

cooperative enzyme have multiple subunits and multiple active sites. Subunits and enzymes may exist in one of 2 states: - low affinity tense state (T) - high affinity relaxed state (R) binding of the substrate encourages the transition of other subunits from the T state to the R state, which inc the likelihood of substrate finding by these other subunits conversely, loss of substrate can encourage the transition from the R state to the T state and promote dissociation of substrate from the remaining subunits. think of cooperative enzyme kinetics like a party. as more people start arriving, the atmosphere becomes more relaxed and the party seems more appealing, but as people start going home the party dies down and more people are encouraged to leave so the tense hosts can clean up

Explain direct vs tropic hormones.

direct hormones are secreted and then act directly on a target tissue. tropic hormones require an intermediary to act. Tropic hormones usually originate in the brain and anterior pituitary gland

How are disaccharides digested? Explain what happens when there is a break down in this process.

disaccharidases digest disaccharides. Maltase digests maltose, isomaltase digests isomaltose, lactase digests lactose, and surcrase digests sucrose. Lack of a particular disaccharide causes an inability to break down the corresponding disaccharide. Then bacteria in the intestines are able to hydrolyze that disaccharide, producing methane gas as a byproduct. In addition, undigested disaccharides can have an osmotic effect, pulling water into the stool and causing diarrhea. This is why people who are lactose intolerant have symptoms of bloating, flatulence, and possibly diarrhea after ingesting dairy products.

All blood vessels are lined with ________ cells. Why? Do arteries or veins have more smooth muscle?

endothelial. This type of cell helps to maintain the vessel by releasing chemical that aid in vasodilation and vasoconstriction. In addition, endothelial cells can allow WBCs to pass through the vessel wall and into the tissues during an inflammatory response. Finally ,endothelial cells release certain chemicals when damaged that are involved in the formation of blood clots to repair the vessel and stop bleeding. arteries have much more smooth muscle than veins

What are the 4 types of tissue?

epithelial, connective, muscle, nervous

What is facilitated diffusion? Which molecules use this? What are the two classic types of facilitated diffusion? Explain them.

facilitated diffusion is simple diffusion for molecules that are impermeable to the membrane (large, polar, or charged); the energy barrier is too high for these molecules to cross freely. Facilitated diffusion requires integral membrane proteins to serve as transporters or channels for these substrates. Classic examples are carriers or channel proteins. Carriers are only open to one side of the cell membrane at any given point. This model is similar to a revolving door because the substrate binds to the transport protein (walks in), remains in the transporter during a conformational change (spins), and then finally dissociates from the substrate-binding site of the transporter (walks out). Binding of the substrate molecule to the transporter protein induces a conformational change; for a brief time, the carrier is in the occluded state, in which the carrier is not open to either side of the phospholipid bilayer. In addition to carrier, channels are also viable transporters. Channels may be in an open or closed conformation. In their open conformation, channels are exposed to both sides of the cell membrane and act like a tunnel for the particles to diffuse through, thereby permitting much more rapid transport kinetics.

what is the function of the collecting duct?

final part of the nephron. the final [ ] of urine will depend largely on the permeabillity of the collecting duct, which is responsible to both aldosterone and ADH (vasopressin). As permeability of the collecting duct increases, so does water reabsorption, resulting in further [ ] of the urine. The reabsorbed water enters the interstitium and makes its way to the vasa recta, where it reenters the bloodstream to once again become part of the plasma. The collecting duct almost always reabsorbs water, but the amount is variable. When the body is very well hydrated, the collecting duct will be fairly impermeable to salt and water. When in conservation mode, ADH and aldosterone will each act to increase reabsorption of water in the collecting duct , allowing for greater water retention and more [ ]ed urine output. Ultimately, anything that is not reabsorbed from the tubule by the end of the collecting duct will be excreted; the collecting duct is the point of no return. After that, there are no further opportunities for reabsorption. As the filtrate leaves the tubule, it collects in the renal pelvis. The fluid, which mostly carries urea, uric acid, and excess ions, flows through the ureter to the bladder where it is stored until voiding.

What is the liver's role in digestion?

first, bile ducts connect the liver with the gallbladder and small intestine. Bile is produced in the liver and travels down these bile ducts where it may be stored in the gallbladder or secreted into the duodenum. The liver also receives all blood draining form the abdominal portion of the digestive tract through the hepatic portal vein. This nutrient rich blood can be processed by the liver before draining into the inferior vena cava on its way to the right side of the heart. Ex: the liver takes up excess sugar to create glycogen, the storage form of glucose, and stores fats as triacylglycerols. The liver can also reverse these processes, producing glucose for the rest of the body through glycogenolysis and gluconeogenesis and mobilizing fats in lipoproteins. The liver detoxifies both endogenous and exogenous compounds. The liver also detoxifies and metabolizes alcohol and medications. Some drugs actually require activation by the enzymes of the liver. Also some drugs cant be taken orally because modification of these drugs by the liver renders them inactive. The liver also synthesizes albumin, a protein that maintains plasma oncotic pressure and also serves as a carries for many drugs and hormones, and clotting factors used during blood coagulation.

Explain the social interactionist theory of language development

focuses on the interplay between biological and social processes. Language acquisition is driven by the child's desire to communicated and behave in a social manner, such as interacting with caretakers and other kids. Interactionist theory allows for the role of brain development in the acquisition of language. As the biological foundation for language develops and the kids are exposed to langugae, the brain groups sounds and meanings together. Then, as the kid interacts with others, certain brain circuits are reinforced, while others are de-emphasized, resulting in atrophy of those circuits.

Explain selective attention

focusing on one part of the sensorium (sensory environment) while ignoring other stimuli. selective attention is probably more of a filter that allows us to focus on one thing wile allowing other stimuli to be processed in the background, as evidenced by the cocktail-party phenomenon (you're actively engaging in a convo with a friend but you hear your name being spoken in the background).

What are cristae? What is their purpose?

folds in the inner membrane of mitochondria. they increase the surface area available for electron transport chain enzymes

Archea are notable for what? Explain. What is the hypothesis for the origin of Archea? Why?

for their ability to use alternative sources of energy. While some are photosynthetic, many are chemosynthetic and can generate energy from inorganic compounds, including sulfur and nitrogen based compounds such as ammonia. Due to the similarities of this domain to eukaryotes, it is hypothesized that eukaryotes and the Archaea share a common origin. Both eukaryotes and Archaea start translation with methionine, contain similar RNA polymerases, and associate their DNA with histones. However, Archea contain a single circular chromosome, divide by binary fission ir budding, and share a similar overall structure to bacteria. Interestingly, Archaea are resistant to many antibiotics.

Glucocorticoids from where are responsible for what? Explain this response.

from the adrenal cortex. responsible for the stress response. In order to make a getaway in the fight or flight response glucose must be rapidly mobilized from the liver in order to fuel activity contracting muscle cells while fatty acids are released from adipocytes. Glucocorticoids, esp cortisol, are secreted with many forms of stress, including exercise, cold, and emotional stress.

Glucagon is a ______ hormone secreted by what? What is the primary target for glucagon action? How does glucagon act? What are the effects?

glucagon is a peptide hormone secreted by the alpha cells of the pancreatic islets of Langerhans. The primary target for glucagon action is the hepatocyte. Glucagon acts through second messengers to cause the following effects: - Increased liver glycogenolysis. Glucagon activates glycogen phosphorylase and inactivates glycogen synthase. - Increased liver gluconeogenesis. Glucagon promotes the conversion of pyruvate to phorphoenolpyruvate by pyruvate carboxylase ad phosphoenolpyruvate carboxykinase (PEPCK). glucagon increase the conversion of fructose 1,6-bisphosphate to fructose-6-phosphate by frusctose-1,6-bisphosphatase. - Increased liver ketogenesis and decreased lipogenesis - Increased lipolysis in the liver. glucagon activates hormone-sensitive lipase in the liver. Because the action is on the liver and not the adipocyte, glucagon is not considered a major fat-mobilizing hormone.

What are the classes of corticosteroids? Where are corticosteroids secreted from?

glucocorticoids, mineralocorticoids, cortical sex hormones from the adrenal cortex

What are glucogenic amino acids? What is their importance? How do they compare to ketogenic amino acids?

glucogenic amino acids (all except leucine and lysine) can be converted into intermediates that feed into gluconeoogenesis, while ketogenic amino acids can be converted into ketone bodies, which can be used as an alternative fuel, particularly during periods of prolonged starvation.

what is starch?

glucose in long alpha linked chains in plants

what is the one exception to the rule that amino acids are chiral?

glycine which has a H side chain

What bonds between what groups form disaccharides?

glycosidic bonds formed between hydroxyl groups of two monosaccharides

One of the main methods of cell signaling occurs via the use of what? Explain this using morphogens.

gradients. Morphogens, or molecules that cause determination of cells, diffuse throughout the organism. Locations closer to the origin of the morphogen will be exposed to higher [ ] s, while areas further away will have less exposure. Multiple morphogens are secreted simultaneously, resulting in unique combinations of morphogen exposure throughout the organism, which can thereby induce the differentiation of specific cell types.

In bacteria, what are the two type of cell wall? How is the type determined?

gram positive and gram negative. Determined by the Gram staining process: a crystal violet stain, followed by a counterstain with a substance called safranin. If the envelope absorbs the crystal violet stain, it will appear deep purple, and the cell is gram-positive. If the envelope does not absorb the stain, but absorbs the safranin counterstain, the cell will appear pink-red, and the cell is said to be negative.

What are eosinophils?

granulocytes. a type of WBC. contain bright red-orange granules and are primarily involved in allergic reactions and invasive parasitic infections. Upon activation, eosinophils release large amounts of histamine, an inflammatory mediator. This results in vasodilation and increased leakiness of the blood vessels, allowing additional immune cells (especially macrophages and neutrophils) to move out of the bloodstream and into the tissue. Inflammation is particularly useful against extracellular pathogens including bacteria, fungi, and parasites.

Explain step 5 of the citric acid cycle, succinate formation

hydrolysis of the thioester bond on succinyl-CoA yields succinate and CoA-SH, and is coupled ot the phosphorylation of GDP to GTP. This reaction is catalyzed by succinyl-CoA synthetase. Synthetases, unlike synthases, create new covalent bonds WITH energy input. Thioester bonds are unique in that their hydrolysis is accompanied by a significant release of energy. Therefore, phosphorylation of GDP to GTP is driven by the energy released by thioester hydrolysis. Once GTP is formed, an enzyme called nucleosidediphosphate kinase catalyzes phosphate transfer from GTP to ADP, thus producing ATP. Note that this is the only time in the citric acid cycle that ATP is produced directly; ATP production occurs predominantly within the electron transport chain.

What are hydrostatic and osmotic pressure and what do they do?

hydrostatic pressure is the force per unit area that the blood exerts against the vessel walls. This is generated by the contraction of the heart and the elasticity of the arteries, and can be measured upstream in the large arteries as blood pressure. Hydrostatic pressure pushes fluid out of the bloodstream and into the interstitum through the capillary walls, which are somewhat leaky by design. Osmotic pressure is the sucking pressure generated by solutes as they attempt to draw water into the bloodstream. B/c most of this osmotic pressure is attributable to plasma proteins, it is usually called oncotic pressure. At the arteriole end of a capillary bed, hydrostatic pressure (pushing fluid out) is much larger than oncotic pressure (drawing fluid in), and there is a net efflux of water from the circulation. As fluid moves out of the vessels, the hydrostatic pressure drops significantly, but the osmotic pressure stays about the same. Therefore, at the venule end of the capillary bed, hydrostatic pressure has dropped below oncotic pressure, and there is a net influx of water back into circulation. The balance of these opposing pressures, also called Starling forces, is essential for maintaining the proper fluid volumes and solute [ ]s inside and outside the vasculature.

Explain the path of electrical conduction in the heart in detail.

impulse initiation occurs at the SA node, which generates 60-100 signals/minute w/o neurological input. This small collection of cells is located in the wall of the right atrium. As the depolarization wave spreads from the SA node, it causes the two atria to contract simultaneously. While most ventricular filling is passive, atrial systole (contraction) results in an increase in at2q3!q`rial pressure that forces a little more blood into the ventricles. This additional volume of blood is called the atrial kick and accounts for about 5-30% of cardiac output. Next, the signal reaches the AV node, which sits at the junction of the atria and ventricles. The signal is delayed here to allow the ventricles to fill completely before they contract. The signal then travels down the bundle of His and its branches, embedded in the interventricular septum (wall), and to the Purkinje fibers, which distribute the electrical signal through the ventricular muscle. The muscle cells are connected by intercalated discs, which contain many gap junctions directly connecting the cytoplasm of adjacent cells. This allows for coordinated ventricular contraction. v

What is the G2/M checkpoint?

in the cell cycle. ensures that the cell has achieved adequate size and the organelles have been properly replicated to support 2 daughter cells. p53 plays a role

The maturation of T cells happens where and is facilitated by what? What happens upon exposure to an antigen?

in the thymus by thymosin, a peptide hormone secreted by thymic cells. Once the T cell has left the thymus, it is mature by naïve. Upon exposure to antigen, T-cells will undergo clonal selection so that only those with the highest affinity for a given antigen proliferate (B cells also do this).

How does B-oxidation of unsaturated fatty acids differ from saturated acids?

in unsaturated fatty acids, two additional enzymes are necessary because double bonds can disturb the stereochemistry needed for oxidative enzymes to act on the fatty acid. To function, these enzymes can have at most one double bond in their active site; this bond must be located between carbond 2 and 3. Enoyl-CoA isomerase rearranges cis double bonds at the 3,4 position to trans double bonds at the 2,3 position once enough acetyl-CoA has been liberated to isolate the double bond within the first three carbons. In monounsaturated fatty acids this single step permits B-oxidation to proceed. In polyunsaturated fatty acids, a further reduction is required using 2,4 dienoyl-CoA reductase to convert two conjugated double bonds to just one double bond at the 3,4 position, where it will then undergo the same rearrangement as monounsaturated fatty acids to form a trans 2,3 double bond.

What is belief perseverance?

inability to reject a particular belief despite clear evidence to the contrary

Explain hallucinogens

include drugs like LSD. The exact mechanisms of hallucinogens is unknown, but it is thought to be a complex interaction between various neurotransmitters, especially serotonin. These drugs typically cause distortions of reality and fantasy, enhancement of sensory experiences, and introspection. Physiological effects include increased HR and inc BP, dilation of pupils, sweating, and inc body temp.

What are the two divisions of the immune system? Explain the difference.

innate and adaptive immunity. Innate immunity is composed of defenses that are always active against infection, but lack the ability to target specific invaders; it is also called nonspecific immunity. Adaptive immunity (specific immunity) refers to the defenses that target a specific pathogen. This system is slower to act, but can maintain immunological memory of an infection to mount a faster attack in subsequent infections.

what are the two ways that reproductive isolation can occur? what is it?

isolation means that the progeny of these populations could no longer freely interbreed. reproductive isolation may occur prezygotically or postzygotically. prezygotic mechanisms prevent formation of the zygote completely. postzygotic mechanisms allow for gamute fusion but yield either nonviable or sterile offspring

What does the presence of fetal hemoglobin do?

it has a higher affinity for oxygen than adult hemoglobin. Fetal RBCs must literally pull oxygen off maternal hemoglobin and onto fetal hemoglobin. Therefore it causes a left shift in the oxyhemoglobin dissociation curve, reducing the oxygen that is released to tissues.

What is the purpose of the spleen in the immune system.

it is a location of blood storage and activation of B-cells, which turn into plasma cells to produce antibodies as part of adaptive immunity.

The speed at which action potentials move depends on what? Explain how. What do most mammals have to maximize the speed of transmission of action potentials? What type of conduction is used as a consequence?

length and cross-sectional area of the axon. Increased length of the axon results in higher resistance and slower conduction. Greater cross sectional areas allow for faster propagation due to decreased resistance. The effect of cross-sectional area is more significant than the effect of length. In order to maximize the speed of transmission, mammals have myelin. Which is an extraordinarily good insulator, preventing the dissipation of the electric signal. The insulation is so effective that the membrane is only permeable to ion movement at the nodes of Ranvier. Thus, the signal "hops" from node to node- what is called saltatory conduction.

proteins with lipid, carbohydrate, and nucleic acid prosthetic groups are referred to as...... respectively

lipoproteins, glycoproteins, and nucleoproteins, respectively

What are LDLs? What is their funciton?

low density lipoproteins. They are the second highest density form of lipoprotein. After chylomicrons, VLDLs, and IDLs and before HDLs (the highest density lipoprotein). Although LDL and HDL are primarily cholesterol particles, the majority of the cholesterol measured in blood is associated with LDL. The normal role of LDL is to deliver cholesterol to tissues for biosynthesis. However, cholesterol also plays na important role in cell membranes. In addition, bile acids and salts are made from cholesterol in the liver, and many other tissues require cholesterol for steroid hormone synthesis (steroidogenesis).

what RNA contains information that is translated into protein? How is this accomplished?

mRNA. it is read in tree-nucleotide segments termed codons

How do Na+ - K+ ATPase pumps work?

maintain a low [ ] of Na+ ions and high [ ] of K+ ions intracellularly by pumping 3 Na+ out for every 2K+ pumped in. This maintains the negative resting potential of the cell.

In eukaryotic cells, most organelles are ________, allowing for what?

membrane bound allowing for compartmentalization of functions.

Drug addiction is highly related to what pathway? Explain the pathway.

mesolimbic reward pathway, one of the four dopaminergic pathways in the brain. This pathway includes the nucleus accumbens (NAc), the ventral tegmental area (VTA), and th econnections between them called the medial forebrain bumdle (MFB). This pathway is normally involved in motivation and emotional response, and its activation accounts for the positive reinforcement of substance use. This addiction pathway is activated by all substances that produce psychological dependence. Gambling and falling in love also activate this pathway.

define amino acids

molecules that contain two functional groups: an amino group (-NH2) and a carboxyl group (-COOH)

Explain the brains metabolic needs.

normal brain function depends on a continuous glucose supply from the bloodstream. in hypoglycemic conditions, hypothalamic centers in the brain sense a fall in blood glucose level and the release of glucagon and epi is triggered. fatty acids cannot cross the blood-brain barrier and therefore are not used at all as an energy source. between meals, the brain relies on blood glucose supplied by either hepatic glycogenolysis or gluconeogenesis. only during prolonged fasting does the brain gain the capacity to use ketone bodies for energy and even then, the ketone bodies only supply approx 2/3rds the fuel, the remainder is glucose

Once the mRNA transcript is created and processed it can exit the nucleus through ______________. Where does it go? What happens once it gets there?

nuclear pores. it goes to the cytoplasm. once it gets to the cytoplasm, mRNA finds a ribosome to begin the process of translation

what is the difference between obligate aerobes and anaerobes? What are obligate anaerobes? What are facultative anaerobes? What are aerotolerant anaerobes?

obligate aerobes require oxygen for metabolism anaerobes use fermentation or some other form of cellular metabolism that does not require oxygen anaerobes that cannot survive in an oxygen-containing environment. The presence of o2 leads to the production of reactive oxygen-containing radicals in these species, which leads to cell death. bacteria that can toggle between metabolic processes, using oxygen for aerobic metabolism if it is present, and switching to anaerobic metabolism if it is not unable to use oxygen for metabolism, but are not harmed by its presence in the environment.

Explain lyases

one of the 6 classes of enzymes. catalyze the cleavage of a single molecule into 2 products. They don't require water as a substrate or act as oxidoreductases. Because most enzymes can also catalyze the reverse of their specific reactions, the synthesis of two molecules into a single molecule may also be catalyzed by a lyase. When fulfilling. this function, it is common for them to be referred to as syntheses.

where does fatty acid biosynthesis occur? what are the products used for? What is the end point of fatty acid synthesis?

occurs in the liver and its products are subsequently transported to adipose tissue for storage. Adipose tissue can also synthesize smaller quantities of fatty acids. Both of the major enzymes of fatty acid synthesis, acetyl-CoA carboxylase and fatty acid synthease, are also stimulated by insulin. Palmitic acid (palmitate) is the primary end point of fatty acid synthesis.

What is ketogenesis. where does it occur?

occurs in the mitochondria of liver cells when excess acetyl-CoA accumulates in the fasting state. HMG-CoA synthase forms HMG-CoA, and HMG-CoA lyase breaks down HMG-CoA into acetoacetate, which can subsequently be reduced to 3-hydroxybutyrate. Acetone is a minor side product that is formed by will not be used as energy for tissues.

Deletion mutation

occurs when a large segment of DNA is lost from a chromosome. Small deletion mutations are considered frameshift mutations

Explain dementia

often begins with impaired memory, but later progresses to impaired judgement and confusion. Personality changes are also very common as dementia progresses. The most common cause of dementia is Alzheimer's disease.

What are inducers?

often growth factors, which are peptides that promote differentiation and mitosis in certain tissues. Most growth factors only function on specific cell types or in certain areas, as determined by the competence of these cells. In this way, certain growth factors can code for particular tissues.

Explain transferases

one of the 6 classes of enzymes. catalyze the movement of a functional group from one molecule to another Ex: in protein metabolism, an aminotransferase can convert aspartate and a-ketoglutarate, as a pair, to glutamate and oxaloacetate by moving the amino group from aspartate to a-ketoglutarate Most transferases will be straightforwardly named, but kinases are also a member of this class Kinases catalyze the transfer of a phosphate group, generally from ATP, to another molecule.

Explain isomerases.

one of the 6 classes of enzymes. catalyze the rearrangement of bonds within a molecule. some isomerases can also be classified as oxidoreductases, transferases, or lyases, depending on the mechanism of the enzyme isomerases catalyze reactions between stereoisomers as well as constitutional isomers

what is the stratum spinosum and what is its importance?

one of the layers of the epidermis. keratinocytes become connected to each other; this layer is also the site of Langerhans cells.

Epithelial cells are often polarized, meaning what?

one side faces the lumen (hollow inside of an organ or tube) (outside world) while the other side interacts with underlying blood vessels and structural cells

what is osmotic pressure and what is oncotic pressure?

osmotic pressure is the "sucking" pressure that draws water into the vasculature caused by all dissolved particles. Oncotic pressure is the osmotic pressure that is attributable to dissolved proteins specifically.

What are the 2 layers of the mitochondrion? Explain them.

outer and inner membrane outer- serves as a barrier between the cytosol and the inner environment of the mitochondrion inner- arranged into numerous infoldings called crystal, contains the molecules and enzymes of the electron transport chain

Some bacteria are mutualist symbioses. Others are ______ or (_____), meaning what?

pathogens (parasites), meaning that they provide no advantage or benefit to the host, but rather cause disease.

What is the difference between expressivity and penetrance?

penetrance is a population parameter (what percentage of individuals with a given genotype express the phenotype), expressivity reflects the gray area in expression and is more commonly considered at the individual level

Hormones can be divided into different groups based on their chemistry. These categories include

peptides, steroids, or amino acid derivatives

What are sphingolipids? How do the classes differ? What's the most well known example?

phospholipids built on a sphingosine backbone. these molecules, like glycerophospholipids, have long-chain, non polar fatty acids and polar head groups. they differ by their head groups ABO cell-surface antigens on RBCs

What are portal systems? What are the three portal systems in the body?

portal systems are when the blood passes through two capillary beds in series instead of one (as all the rest do) before returning to the heart In the hepatic portal system, blood leaving capillary beds in the walls of the gut passes through the hepatic portal vein before reaching the capillary beds in the liver. In the hypophyseal portal system, blood leaving the capillary beds in the hypothalamus travels to a capillary bed in the ant pit to allow for paracrine secretion of releasing hormones. In the renal portal system, blood leaving the glomerulus travels through an efferent arteriole before surrounding the nephron in a capillary network called the vasa recta.

Explain positive and negative selection for T cells.

positive selection refers to allowing only the maturation of cells that can respond to the presentation of antigen on MCH (cells that cannot respond to MHC undergo apoptosis because they will not be able to respond in the periphery). Negative selection refers to causing apoptosis in cells that are self-reactive (activated by proteins produced by the organism itself).

Single-stranded RNA viruses may be what 2 things? What is the difference between the two?

positive sense or negative sense positive sense implies that the genome may be directly translated to functional proteins by the ribosomes of the host cell, just like mRNA. Negative sense RNA viruses are a bit more complicated: the negative-sense RNA strand acts as a template for synthesis of a complementary strand, which can then be used as a template for protein synthesis. Negative sense RNA viruses must carry an RNA replicase in the virion to ensure that the complementary strand is synthesized

Explain cardiac muscle. Innervated by what? How many nuclei does cardiac muscle have? How does it appear when viewed microscopically? How does each cardiac myocyte communicate? How do the nervous and endocrine systems play a role in the regulation of cardiac muscle contraction?

primarily uninucleated, but can contain two nuclei. Like smooth muscle, cardiac muscle contraction is involuntary and innervated by the ANS. Unlike smooth muscle, cardiac muscle appears striated like skeletal. Cardiac muscle cells are connected by intercalated discs, which contain many gap junctions. These gap junctions are connections between the cytoplasm of adjacent cells, allowing for the flow of ions directly between cells. This allows for rapid and coordinated depolarization of muscle cells and efficient contraction of cardiac muscle. Cardiac muscle cells are able to define and maintain their own rhythm, termed myogenic activity. The vagus nerve provides parasypathetic outflow to the heart and slows the heart rate. Norepi from sympathetic neurons or epi from the adrenal medulla binds to adrenergic receptors in the heart, causing an increased HR and greater contractility. One of the ways epi does this is by increasing intracellular calcium levels within cardiac myocytes. Ultimately, cardiac contraction-like that of all types of muscle- relies on calcium.

What does the adrenal medulla secrete? What is it? What are the effects of it's secretions?

produces epi and norepi. both are amino-acid derivative hormones that belong to a larger class of molecules known as catecholamines it is a derivative of the nervous system. the specialized nerve cells int he medulla are capable of secreting these compounds directly in to the blood stream. Epi can increase the breakdown of glycogen to glycose (glycogenolysis) in the liver and muscle, as well as increase the basal metabolic rate. Both epi and norepi will increase the HR, dilate the bronchi, and shunt blood flow to the systems that would be used in a sympathetic response. That is, there is vasodilation of blood vessels leading to inc bloodflow to the skeletal muscle, heart, lungs, and brain. Concurrently, vasoconstriction decreases bloodflow to the gut, kidneys, and skin. Not that the stress response involves both cortisol and epi. Classically, cortisol is understood to mediate long-term (slow) stress responses, while catecholaimes are understood to control short-term (fast) stress responses. In fact, cortisol actually increases the synthesis of catecholamines as well, resulting in an inc of catecholamine release.

One of the most common mutations found in cancer is mutation of the gene that produces _____, called ____. When this gene is mutated, what happens?

produces p53 called TP53 the cell cycle is not stopped to repair damaged DNA, allowing mutations to accumulate and resulting in a cancerous cell that divides continuously and without regard to the quality or quantity of the new cells produced. Often, cancer cells undergo rapid cell division, creating tumors. Eventually, if the cell begins to produce the right factors (such as proteases that can digest basement membranes or factors that encourage blood vessel formation) the damaged cells are then able to reach other tissues. This may include both local invasion as well as distant spread of cancerous cells through the bloodstream or lymphatic streams. The latter result is known as metastatis

The DNA that makes up a chromosome is would around a group of small basic proteins called ___________, forming ____________.

proteins called histones, forming chromatin

What does the cell wall do in prokaryotes?

provides structure and controls the movements of solutes into and out of the bacterium. Maintains [ ] gradient with the environment.

Explain the Hershey and Chase experiment

radioactively labeled P and S (DNA was protein) in bacteriophages, and let bacteriophages infect bacteria and observe radioactivity

when might proteins be used as energy?

rarely. under conditions of extreme energy depirivation.

describe a repressible system. given an example.

repressible systems allow constant production of a protein product. In contrast to the inducible systems, the repressor made by the regulator gene is inactive until it binds to a corepressor. This complex can then bind the operator site to prevent further transcription. Repressible systems tend to serve as negative feedback; often, the final structural product can serve as a corepressor. Thus, as its levels increase, it can bind the repressor, and the complex will attach to the operator region to prevent further transcription of the same gene. The trp operon, described above, operates in this way as a negative repressible system. When tryptophan is high in the local environment, it acts as a corepressor. The binding of two molecules of tryptophan to the repressor causes the repressor to bind to the operator site. Thus, the cell turns off its machinery to synthesize its own tryptophan, which is an energetically expensive process because fo tis easy availability in the environment.

Explain the difference between saturated and unsaturated fatty acids.

saturated fatty acid tails will have only single bonds; the carbon atom is considered saturated when it is bonded to 4 other atoms, with no pi bonds. Saturated fatty acids have greater van de Waals forces and a more stable overall structure. Therefore, they only form solids at room temp. Ex: butter unsaturated fatty acids have one or more double bonds. Double bonds introduce kinks into the fatty acid chain, which makes it difficulty for them to stack and solidify. Therefore , they are usually liquid at room temp. Ex: olive oil

what is the difference between saturated and unsaturated fatty acids?

saturated fatty acids have no double bonds while unsaturated fatty acids have one or more double bonds.

How are simple sugars, amino acids, simple carbohydrates, and fats absorbed into the small intestine?

simple sugars and amino acids are absorbed by secondary active transport and facilitated diffusion into epithelial cells lining the small intestine. then, these substance move across the epithelial cell membrane into the intestinal capillaries. Blood is constantly passing by the epithelial cells, carrying the carbohydrate and amino acid molecules away. This creates a [ ] gradient such that blood always has a lower [ ] of monosaccharides and amino acids than inside the epithelial cells. Thus, simple carbohydrates and amino acids diffuse from the epithelial cells into the capillaries. The absorbed molecules then go to the liver via the hepatic portal circulation. Short-chain fatty acids will follow the same process as carbs and amino acids by diffusing directly into the intestinal capillaries. These fatty acids do not require transporters because they are nonpolar, so they can easily transverse the cellular membrane. Larger fats, glycerol, and cholesterol move separately into the intestinal cell sbut then reform into triglycerides. The triglycerides and esterified cholesterol molecules are packaged into chylomicrons. Rather than entering the bloodstream, chylomicrons enter the lymphatic circulation through lacteals, small vessels that form the beginning of the lymphatic system. These lacteals converge and enter the venous circulation at the thoracic duct in the base of the neck, which empties into the left subclavian vein.

Archaea are ____-celled organisms. How are they similar and different from bacteria? Historically, archaea were considered _______. Why?

single celled. They are visually similar to bacteria but contain genes and several metabolic pathways that are more similar to eukaryotes than to bacteria. Historically, they were considered extremophiles, in that they were most commonly isolated from harsh environments with extremely high temperatures, high salinity, or no light. More recent research has demonstrated a greater variety of habitats for these organisms, including the human body.

What are the three subtypes of muscle? All muscle is capable of ___________, which relies on what ions? All muscle is ___________.

skeletal muscle, smooth muscle, and cardiac muscle. All muscle is capable of contraction, which relies on calcium ions. All muscle is innervated.

what does the axial skeleton consist of?

skull, vertebral column, rib cage, and hyoid bone (used for swallowing)

Explain our circadian cycle in humans.

sleepiness can partially be attributed to blood levels of melatonin, a serotonin-derived hormone from the pineal gland. The retina has direct connections to the hypothalamus, which controls the pineal gland; thus, decreasing light can cause the release of melatonin. cortisol, a steroid hormone produced in the adrenal cortex, is also related to the sleep-wake cycle. Its levels slowly increase during early morning b/c inc light causes the release of corticotropin releasing factor(CRF) fromthe hypothalamus. CRF causes the release of ACTH from the ant pit, which stimulates cortisol release. Cortisol contributes to wakefulness.

How do slow twitch and fast twitch muscle cells generate the large amounts of ATP they need?

slow twitch (red) muscle fibers have a lot of mitochondria and thus use oxidative phosphorylation to make ATP. This means that high [O2] are required to generate ATP. Creatine phosphate is created by transferring a phosphate group from ATP to creatine during times of rest. This reaction can then be reversed during muscle use to quickly generate ATP from ADP creatine + ATP <--> creatine phosphate + ADP Muscle also contains myoglobin, which binds oxygen with high affinity. As exercising muscles run out of oxygen, they use myoglobin reserves to keep aerobic metabolism going. Fast-twitch (white) muscle fibers have fewer mitochondria and must rely on glycolysis and fermentation to make ATP under most circumstances. WHen a person exercises, heart rate and respiratory rate increase in order to move more oxygen to actively respiring muscles. The oxyhemoglobin dissociation curve undergoes a right shift in the presence of increased carbon dioxide [ ], increased hydrogen ion [ ] (decreased pH), and increased temp. However, even with these adaptations, muscle use can quickly overwhelm the ability of the metabolism and produce lactic acid, at which point the muscle begins to fatigue. The amount present is called the oxygen debt. After the cessation of strenuous exercise, the body must metabolize all of the lactic acid it has produced. Most lactic acid is converted back into pyruvate, which can enter the citric acid cycle. This process requires oxygen, and the amount of oxygen required to recover from strenuous exercise is equal to the oxygen debt.

What are adipocytes?

specialized fat cells that house triglycerides store large amounts of fat and are found primarily under the skin, around mammary glands, and in the abdominal cavity.

What is the fluid mosaic model?

states that a membrane is a fluid structure with a "mosaic" of various proteins embedded in it it is the theory that underlies the structure and function of the cell membrane

What is aldosterone? steroid or peptide hormone? secreted by what in response to what? Explain how it is stimulated. What is its mechanism of action and what does it do?

steroid hormone secreted by adrenal cortex in response to decreased blood pressure. Decreased blood pressure stimulates the release of renin from juxtaglomerular cells in the kidney. Renin then cleaves angiotensinogen, a liver protein, to form angiotensin I. This peptide is then metabolized by angiotensin-converting enzyme in the lungs to form angiotensin II, which promotes the release of aldosterone from the adrenal cortex. Aldosterone works by altering the ability o the distal convoluted tubule and collecting duct to reabsorb sodium. Remember that water does not move on its own, but rather travels down its osmolarity gradient. Thus, if we reabsorb more sodium, water will flow with it. This reabsorption of isotonic fluid has the net effect of increasing blood volume and therefore blood pressure. Aldosterone will also increase potassium and hydrogen ion excretion.

cortisol is a _____ hormone that promotes the mobilization of energy through what processes? How does it effect blood glucose levels? What are the mechanisms by which it does this?

steroid hormone that promotes the mobilization of energy stores through the degradation and increased deliver of amino acids and increased lipolysis. Cortisol also elevates blood glucose levels, increasing glucose availability for nervous tissue through two mechanisms. First, cortisol inhibits glucose uptake in most tissues and increase hepatic output of glucose via gluconeogenesis, particularly from amnio acids. Second, cortisol has a permissive function that enhances the activity of glucagon, epi, and other catecholamines. Long-term exposure to glucocorticoids may be required clinically but causes persistent hyperglycemia, which stimulates insulin. This actually promotes fat storage in the adipose tissue, rather than lipolysis.

Explain ecstasy (MDMA)

stimulant acts as a hallucinogen combined with an amphetamine. It's mechanism and effects are similar to amphetamines. Physiologically, ecstasy causes increased HR, inc BP, blurry vision, sweating, nausea, and hyperthermia. Psychologically, ecstasy causes feelings of euphoria, inc alertness, and an overwhelming sense of well-being and connectedness. It's a rave/club drug, packaged in colorful pills

Explain cocaine

stimulant decreases reuptake of dopanine, norepi, and serotonin, similar to amphetamines. The effects are similar to amphetamines. Cocaine also have anesthetic and vasoconstrictive properties, and is therefore sometimes used in surgeries in highly vascularized areas, such as the nose and throat. These vasoconstrictive properties can also lead to heart attacks and strokes when used recreationally. Crack is a form of cocaine that can be smoked.

what does actin do? What is it?

structural protein. makes up microfilaments and the thin filaments in myofibrils. It is the most abundant protein in eukaryotic cells. Actin proteins have a positive side and a negative side; this polarity allows motor proteins to travel unidirectionally along an actin filament, like a one-way street

what does tubulin do? What is it?

structural protein. makes up microtubules. Microtubules are important for providing structure, chromosome separation in mitosis and meiosis, and intracellular transport with kinesis and dynein. Like actin, tubulin has polarity: the negative end of a microtubule is usually located adjacent to the nucleus, whereas the positive end is usually in the periphery of a cell.

The molecule upon which an enzyme acts is known as its ______. What is the active site?

substrate active site- location within the enzyme where the substrate is held during the chemical reaction

What is summation? What are the two types? Explain them

summation is the additive effect of multiple signals temporal summation- multiple signals are integrated during a relatively short period of time. A number of small excitatory signals firing at nearly the same moment could bring a postsynaptic cell to threshold, enabling an action potential. spatial summation- the additive effects are based on the number and location of the incoming signals. A large number of inhibitory signals firing directly on the soma will cause a more profound hyper polarization of the axon hillock that the depolarization caused by a few excitatory signals firing on the dendrite of a neuron.

One thing that lipids have in common is a tail composed of what? These hydrocarbon chains vary by what?

tail composed of long-chain fatty acids vary by their degree of saturation and length

molecular clock model

the degree of difference in the genome between two species is related to the amount of time since the two species broke off from a common ancestor

How does ion-exchange chromatography work?

the beads in the column are coated with charged substances, so they attract or bind compounds that have an opposite charge.

What is proteolysis? Explain this process.

the breakdown of proteins. begins in the stomach with pepsin and continues with the pancreatic proteases trypsin, chymotrypsin, and carboxypeptidases A and B, all of which are secreted as zymogens. Protein digestion is completed by the small intestinal brush-border enzymes dipeptiase and animopeptidase. The main end products of protein digestion are amino acids, dipeptides, and tripeptides. Absorption of amino acids and small peptides through the luminal membrane is accomplished by secondary active transport linked to sodium. At the basal membrane, simple and facilitated diffusion transports amino acids into the blodostream.

explain the glycerol 3-phosphate shuttle for NADH

the cytosol contains one isoform of glycerol-3-phosphate dehydrogenase, which oxidizes cytosolic NADH to NAD+ while forming glycerol 3-phosphate from dihydroxyacetone phosphate (DHAP). On the outer face of the inner mitochondrial membrane, there exists another isoform of glycerol-3-phosphate dehydrogenase that is FD-dependent. This mitochondrial FAD is the oxidizing agent, and ends up being reduced to FADH2. Once reduced, FADH2 proceeds to transfer its electrons to the ETC via Complex II, thus generating 1.5 ATP for every molecule of cytosolic NADH that participates in this pathway.

Explain pragmatics in terms of language

the dependence of language on context and pre-existing knowledge. In other words, the manner in which we speak may differ depending on the audience and our relationship to that audience. Pragmatics are also affected by prosody-the rhythm, cadence, and inflection of our voices.

explain the mechanism of peptide bond formation

the electrophilic carbonyl carbon on the first amino acid is attacked by the nucleophilic amino group on the second amino acid. after the attack, the hydroxyl group of the carboxylic acid is kicked off.

What is the esophagus? How is it divided? Explain the esophagus' role in digestion. What is peristalsis?

the esophagus is a muscular tube that connects the pharynx to the stomach. The top third of the esophagus is composed of skeletal muscle and the bottom third is composed of smooth muscle. The middle third is a mix of both. While the top of the esophagus is under somatic (voluntary) motor control, the bottom- and most of the rest of the GI tract, is under autonomic (involuntary) nervous control. The rhythmic contraction of smooth muscle that propels food toward the stomach is called peristalsis. Under normal circumstances, peristalsis proceeds down the digestive tract. However, certain factors such as exposure to chemicals, infectious agents, physical stimulation in the posterior pharynx, and even cognitive stimulation, can lead to a reversal of peristalsis in the process of emesis (vomiting). No mechanical or chemical digestion takes place in the esophagus, except for the continued enzymatic activity initiated in the mouth by salivary enzymes. Swallowing is initiated in the muscles of the oropharynx, which constitute the upper esophageal sphincter. Peristalsis squeezes, pushes, and propels the bolus toward the stomach. As the bolus approaches the stomach, a muscular ring known as the lower esophageal sphincter (cardiac sphincter) relaxes and opens to allow the passage of food.

what is the function of the distal convoluted tubule?

the filtrate enters the DCT from the loop of Henle. The DCT responds to aldosterone, which promotes sodium reabsorption. because sodium ions are osmotically active particles, water will follow the sodium, concentrating the urine and decreasing its volume. The DCT is also a site of wast product secretion, like the PCT.

What are the different types of agranulocytes? What are they responsible for? What happens to one of them in the tissues?

the lymphocytes, which are responsible for antibody production, immune system modulation, and targeted killing of infected cells. Monocytes, which are phagocytic cells in the bloodstream, are also agranulocytes. They become macrophages in tissues; many tissues have resident populations of macrophages with specific names (such as microglia in the CNS, Langerhans cells in the skin, and osteoclasts in the bone).

what are the organic and inorganic components of the bone matrix?

the organic components include collagen, glycoproteins, and other peptides. The inorganic components include calcium, phosphate, and hydroxide ions, which harden together to form hydroxyapatite crystals. Minerals such as sodium, magnesium, and potassium are also stored in bone.

The pKa of a group is what? If the pH is less than the pKa, a majority of the species will be ______. If the pH is more than the pKa, a majority of the species will be ______.

the pH at which, on average, half o f the molecules of that species are deprotonated. [protonated version of the ionizable protein]=[deprotonated version] [HA]=[A-] pH < pKa. protonated pH > pKa. deprotonated

what is chromatography? How does it work?

the separation of a mixture by passing it in solution or suspension or as a vapor (as in gas chromatography) through a medium in which the components move at different rates. The process begins by placing the sample onto.a solid medium called the stationary phase or adsorbent. The next step is to run the mobile phase through the stationary phase. This will allow the sample to run through the stationary phase, or elute. Depending on the relative affinity of the sample for the stationary and mobile phases, different substances will migrate though at different speeds. That is, components that have a high affinity for the stationary phase will barely migrate at all; components with a high affinity for the mobile phase will migrate quickly. The amount of time a compound spends in the stationary phase is referred to as the retention time. Varying retention times of each compound in the solution results in separation of the components within the stationary phase, or partitioning.

Explain the stomach and its role in digestion. What are its divisions? That are the different cell types of the gastric glands? What enzymes are secreted?

the stomach is a highly muscular organ with a capacity of approximately two liters. In humans, the stomach is located in the upper left quadrant of the abdominal cavity, underneath the diaphragm. This organ uses HCl and enzymes to digest food, creating a fairly harsh environment. Therefore, its mucosa is quite thick to prevent autodigestion. The stomach can be divided into four main anatomical divisions: the fundus and body, which contain mostly gastric glands, and the antrum and pylorus, which contain mostly pyloric glands. The internal curvature of the stomach is called the lesser curvature; the external curvature is called the greater curvature. The lining of the stomach is thrown into folds called rugae. The mucosa of the stomach contain gastric glands and pyloric glands. The gastric glands respond to signals from the vagus nerve of the parasympathetic NS, which is activated by the brain in response to the sight, taste, and smell of food. Gastric glands have three different cell types: mucous cells, chief cells, and parietal cells. Mucous cells produce the bicarbonate-rich mucus that protects the muscular wall from the harshly acidic (pH=2) and proteolytic environment of the stomach. Gastric juice is a combination of secretions from the other two cell types in the gastric glands: chief cells and parietal cells. the chief cells secrete pepsinogen. This is the inactive, zymogen form of pepsin, a proteolytic enzyme. Hydrogen ions in the stomach, secreted by parietal cells as HCl, cleave pepsinogen to pepsin. Pepsin digests proteins by cleaving peptide bonds near aromatic amino acids, resulting in short peptide fragments. Because pepsin is activated by the acidic environment, it follows that pepsin is most active at a low pH. This is a unique characteristic among human enzymes, as most human enzymes are most active at physiological pH. Stomach acid also kills most harmful bacteria (w/ the exception of Heliobacter pylori, infection with which is usually asymptomatic but can cause inflammation, ulcers, and even certain gastric cancers). The acid environment also helps to denature proteins and can break down some intramolecular bonds that hold food together. In addition to HCl, parietal cells secrete intrinsic factor, a glycoprotein involved in the proper absorption of vitamin B12.

what are the layers of the dermis? what is located in the dermis?

the upper layer (right below the epidermis) is the papillary layer, which consists of loose connective tissue. Below that is the denser reticular layer. Sweat glands, blood vessels, and hair follicles originate in the dermis. Most sensory receptors are also located in the dermis.

What is expiratory reserve volume (ERV)?

the volume of additional air that can be forcibly exhaled after a normal exhalation

What is inspiratory reserve volume (IRV)?

the volume of additional air that can be forcibly inhaled after a normal inhalation

Oocytes are surrounded by 2 layers. What are they?

the zona pellucida and the corona radiata zona pellucida- surrounds the oocyte itself and is an acellular mix of glycoproteins that protect the oocyte and contain compounds necessary for sperm cell binding. corona radiata- lies outside the zona pellucida and is a layer of cells that adhered to the oocyte during ovulation.

Explain monosynaptic reflex arcs. What is the best example? Explain it.

there is a single synapse between the sensory neuron that receives the stimulus and the motor neuron that responds to it. Classic example: knee-jerk reflex. When the patellar tendon is stretched, info travels up the sensory (afferent, presynaptic) neuron to the spinal cord, where it interfaces with the motor (efferent, postsynaptic) neuron that causes contraction of the quads muscles. The net result is extension of the leg, which lessons the tension on the patellar tendon.

Explain a polysynaptic reflex. What is the example? Explain it.

there is at least one interneuron between the sensory and motor neurons. Classic example: stepping on a nail (withdrawal reflex). The extremity with which you step on the nail will be stimulated to flex, using the hip muscles and hamstring muscles, pulling the foot away from the nail. this is a monosynaptic reflex. However, if the person is to maintain balance, the other foot must be planted firmly on the ground. For this to occur, the motor neuron that controls the quads muscles in the opposite limb must be stimulated, extending it. Interneurons in the spinal cord provide the connections from the incoming sensory info to the motor neurons in the supporting limb.

what are melanocytes? what do they do? what are they derived from? where are they found?

they are a cell type derived from neural crest cells and found in the stratum basale. They produce melanin, a pigment that serves to protect the skin from DNA damage caused by ultraviolet radiation. Once produced, the pigment is transferred to the keratinocytes. ALl humans actually have comparable numbers of melanocytes; skin color is caused by varying levels of activity of the melanocytes. More active = darker skin.

What do the interstitial cells of Leydig do? What are they?

they one of two functional components of the testis. they secrete testosterone and other male sex hormones (androgens)

what do the lymph nodes do?

they provide a place for immune cells to communicate and mount an attack; B-cells can be activated here as well.

What do DNA polymerases do?

they read the DNA template, or parental strand, and synthesize the new daughter strand. It reads the template strand in the 3' to 5' direction while synthesizing the complementary strand in the 5' t o 3' direction. This will result in a new double helix of DNA that has the required antiparallel orientation. **With the exception of DNA polymerases READING direction, everything in molecular biology is 5' to 3' - DNA synthesis - DNA repair - RNA transcription - RNA translation

What are some functions of lipids other than their passive structural roles?

they serve as coenzymes in the electron transport chain and in glycosylation reactions. they also function as hormones that transmit signals over long distances they function as intracellular messengers responding to extracellular signals.

explain anaphase

third phase of mitosis. the centromeres split so that each chromatid has its own distinct centromere, thus allowing the sister chromatids to separate. The sister chromatids are pulled toward the opposite poles of the cell by the shortening of the kinetochore fibers.

Explain step 6 of the citric acid cycle, fumarate formation

this is the only step of the citric acid cycle that doesn't take place in the mitochondrial matrix; it instead occurs on the inner membrane. Why? Succinate undergoes oxidation to yield fumarate. This reaction is catalyzed by succinate dehydrogenase. Succinate dehydrogenase is considered a flavoprotein because it is covalently bonded to FAD, the electron acceptor in this reaction. This enzyme is an integral protein on the inner mitochondrial membrane. As succinate is oxidized to fumarate, FAD is reduced to FADH2. Each molecule of FADH2 then passes the electrons it carries to the electron transport chain, which eventually leads to the production of 1.5 ATP (unlike NADH, which will give rise to 2.5 ATP). FAD is the electron acceptor in this reaction because the reducing power of succinate is not great enough to reduce NAD+.

How does control of pH work in the respiratory system?

through the bicarbonate buffer system in the blood ** big on MCAT** CO2(g) + H2O(l) <-> H2CO3(aq) <-> H+(aq) + HCO3-(aq) The body attempts to maintain a pH between 7.35 and 7.45. When the pH is lower, and hydrogen ion [ ] is higher (academia), acid-sensing chemoreceptors just outside the blood-brain barrier send signals to the brain to increase the respiratory rate. Further, an increasing [H+] will cause a shift in the bicarb buffer system, generating additional CO2. The respiratory centers in the brain are sensitive to this increasing partial pressure of CO2 and will also promote an inc in respiratory rate. As the respiratory rate inc, more CO2 is blown off. This will push the buffer equation to the left, but notice the difference: a shift to the left in the previous paragraph was cause by an inc in the [H+], which elevated the [CO2]. Here, the removal of CO2 causes a shift to the left that allows the [H+] to drop back to normal. If the blood is too basic (alkaleimia), then the body will seek to inc acidity. If the respiratory rate is slowed, more CO2 will be retained, shifting the buffer equation to the right and producing more hydrogen ions and bicarb ions. This results in a lower pH. ***The kidneys also play a role in this by modulating secretion and reabsorption of acid and base within the nephron.

Explain thyroid hormones and their activity in the body. How does T3 differ from T4?

thyroid hormone activity is largely permissive. in other words, the thyroid hormone levels are kept more or less constant, rather than undulating with changes in metabolic state. Thyroid hormones increase the BMR, as evidenced by increased O2 consumption and heat production when they are secreted. The increase in metabolic rate produced by a dose of thyroxine (T4) occurs after a latency of several hours but may last for several days, while triiodothyronine (T3) produces a more rapid increase in metabolic rate and has a shorter duration of activity. The subscript numbers refer to the number of iodine atoms in the hormone. T4 can be thought of as a precursor to T3; deiodonases (enzymes that remove iodine form a molecule) are located in target tissues and convert T4 to T3. Thyroid hormones have their primary effects in lipid and carbohydrate metabolism. They accelerate cholesterol clearance from the plasma and increase the rate of glucose absorption from the small intestine. Epi requires thyroid hormones to have a significant metabolic effect.

Why do viruses need a host cell? What do they do after the hijack the host cell?

to express and replicate genetic info because they lack ribosomes to carry out protein synthesis. they replicate and produce viral progeny called virions, which can be released to infect additional cells.

What are the two major roles of the liver in fuel metabolism?

to maintain a constant level of blood glucose under a wide range of conditions and to synthesize ketones when excess fatty acids are being oxidized.

What is histone acetylation and how does it work? What about histone deacetylases?

transcription factors that bind to the DNA can recruit other coactivators such as histone acetylases. These proteins are involved in chromatin remodeling, because they acetylate lysine residues found in the amino terminal tail regions of histone proteins. Acetylation of histone proteins decreases the positive charge on lysine residues and weakens the interaction of the histone with DNA, resulting in an open chromatin conformation that allows for easier access of the transcriptional machinery to the DNA. Specific patterns of histone acetylation can lead to increased gene expression levels. On the other hand, gene silencing can occur just as easily with chromatin remodeling. Histone deacetylases are proteins that function to remove acetyl groups from histones, which results in a closed chromatin conformation and overall decrease in gene expression levels in the cell.

If you examine thyroid tissue under a light microscope, what would you see?

two distinct cell populations within the gland. -Follicular cells produce thyroid hormones -C-cells (parafollicular cells) produce calcitonin

Explain the difference between the umbilical arteries and umbilical veins.

umbilical arteries carry blood away from the fetus toward the placenta (still away from heart b/c away from fetus heart) umbilical veins carry blood toward the fetus from the placenta (still toward heart b/c toward fetus heart) it's just weird b/c umbilical arteries carry deoxygenated blood (b/c oxygenation occurs at placenta)

What are transgenic mice and knockout mice? How can you create them?

transgenic mice are altered at their germ line by introducing a cloned gene into fertilized ova or into embryonic stem cells. The cloned gene that is introduced is referred to as a transgene. If the transgene is a disease-producing allele, the transgenic mice can be used to study the disease process from early embryonic development through adulthood. A similar approach can be used to produce knockout mice, in which a gene has been intentionally deleted (knocked out). There are different approaches to developing transgenic mice. A cloned gene may be microinjected into the nucleus of a newly fertilized ovum. Rarely, the gene may subsequently incorporate into the nuclear DNA of the zygote. The ovum is implanted into a surrogate mother, and, if successful, the resulting offspring will contain the transgene in all of their cells, including their germ line cells (gametes). Consequently, the transgene will be passed to their offspring. The transgene coexists in the animals with their own copies of the gene, which have not been deleted. This approach is useful for studying dominant gene effects but is less useful as a model for recessive disease because the # of copies of the gene that insert into the genome cannot be controlled; the transgenic mice may each contain a different number of copies of the transgene. Embryonic stem cell lines can also be used for developing transgenic mice. Advantages of using stem cell lines are that the cloned genes can be introduced in cultures, and that one can select for cells with the transgene successfully inserted. The altered stem cells are injected into developing blastocysts and implanted into surrogate mothers. The blastocyst itself is thus composed of two types of stem cells: the ones containing the transgene and the original blastocyst cells that lack the transgene. The resulting offspring is a chimera, meaning that it has patches of cells, including germ cells, derived from each of the two lineages. This is evident if the two cell lineages (transgenic cells and host blastocyst) come from mice with different coat colors. The chimeras will have patchy coats of two colors, allowing them to be easily identified. These chimeras can then be bred to produce mice that are heterozygous for the transgene and mice that are homozygous from the transgene.

explain the metabolic function of cardiac muscle.

unlike other tissues of the body, cardiac myocytes prefer fatty acids as their major fuel, even in the well-fed state. when ketones are present during prolonged fasting, they can also be used. thus cardiac myocytes most closely parallel skeletal muscle during extended periods of exercise. in patients with cardiac hypertrophy (thickening of the heart muscle), this situation reverses to some extent. in a failing heart, glucose oxidation increases and B-oxidation falls.

What is the role of the gallbladder in digestion?

upon the release of CCK, the gallbladder contracts and pushes bile out into the biliary tree. The bile duct system merges with the pancreatic duct before emptying into the duodenum. The gallbladder is a common site of cholesterol or bilirubin stone formation. This painful condition causes inflammation of the gallbladder. The stones may also travel into the bile ducts and get stuck in the biliary tree.

What is affinity chromatography?

uses a bound receptor or ligand and an eluent with free ligand or a receptor for the protein of interest accomplished by coating beads with a receptor that binds the protein or a specific antibody to the protein.

Explain veins structure and role in the cardiovascular system.

veins are thin-walled, inelastic vessels that transport blood to the heart. Venules are smaller venous structures that connect capillaries to the larger veins of the body. The smaller amount of smooth muscle in the walls of veins gives them less recoil than arteries. Veins are able to stretch to accommodate larger quantities of blood. Indeed, 3/4ths of our total blood volume may be in venous circulation at any one time. Even though the volume of arterial blood is normally much less than the volume of venous blood, the total volume passing through either side of the heart per unit time (cardiac output) is the same.

What are the two types of vitamins? what vitamins are in each type? how are vitamins absorbed into the small intestine?

vitamins can be either fat soluble or water soluble. Fat soluble vitamins (A, D, E, and K) and water soluble vitamins are all the other ones. Fat-soluble vitamins dissolve directly into chylomicrons to enter the lymphatic circulation. Failure to digest and absorb fat properly may lead to deficiencies of fat-soluble vitamins. The water-soluble vitamins are taken up, along with water, amino acids, and carbohydrates, across the endothelial cell so the small intestine, passing directly into the plasma.

Explain waxes structure and function in the cell membrane.

waxes are a class of lipids that are extremely hydrophobic and are rarely found in the cell membrane of animals, but are sometimes found in the cell membranes of plants. A wax is composed of a long-chain fatty acid and a long-chain alcohol, which contribute to the high melting point of these substances. When present within the cell membrane, waxes can provide both stability and rigidity within the nonpolar tail region only. Most waxes serve an extracellular function in protection or waterproofing.

Explain alertness as a state of consciousness

we are awake and able to think. we are able to perceive, process, access info, and express that info verbally. In the alert state, we experience a certain level of physiological arousal. Cortisol levels tend to be higher. Alertness is maintained by neurological circuits in the prefrontal cortex at the very front of the brain. Fibers from the prefontal cortex communicate with the reticular formation, a neural structure located in the brainstem, to keep the cortex awake and alert. A brain injury that results in disruption of these connects results in a coma.

Missence mutation.

when a point mutation results in substituting one amino acid for another

Divergent evolution

when two or more species sharing a common ancestor become more different over time

Explain electrophoresis.

works by subjecting compounds to an electric field, which moves them according to their net charge and size. Negatively charged compounds migrate toward the positively charged anode, positively charged compounds migrate toward the negatively charged cathode. Polyacrylamide gel is the standard medium for protein electrophoresis. The gel acts like a sieve, allowing smaller particles to pass through easily while retaining large particles.

What happens during the third trimester of pregnancy?

- The 7th and 8th months have rapid growth and further brain development - Antibodies are transported by highly selective active transport form the mother to fetus for protection against foreign agents in preparation for life outside the womb; this transfer beings earlier in pregnancy but is highest in the 9th month just before birth. - The growth rate slows and the fetus becomes less active, as it has less room to move about

track an incoming signal through a neuron, explaining the structure of a neuron as well.

- The dendrites receive incoming messages from other cells. The info received from the dendrites is transmitted through the cell body before it reaches the axon hillock, which integrates the incoming signals. - The axon hillock plays an important role in action potentials (transmission of electrical impulses down the axon) - Signals arriving from the dendrites can be either excitatory or inhibitory. The axon hillock sums up these signals and if the result is excitatory enough (reaches threshold), it will initiate an action potential. - The axon is a long appendage than terminates in close proximity to a target structure (a muscle, gland, or another neuron). - Most mammalian nerve fibers are insulated by myelin, a fatty membrane to prevent signal loss or crossing of signals. Just like insulation prevents wires next to each other from accidentally discharging each other, the myelin sheath maintains the electrical signal within one neuron. Also, myelin inc the speed of conduction in the axon. Myelin is produced by oligodendrocytes in the CNS and Schwann cells in the PNS. - At certain intervals along the axon, there are small breaks in the myelin sheath with exposed areas of axon membrane called nodes of Ranvier. These nodes are critical for rapid signal conduction - Finally, at the end of the axon is the nerve terminal or synaptic bouton (knob). This structure is enlarged and flattened to maximize transmission of the signal to the next neuron and ensure proper release of neurotransmitters.

explain telophase

- fourth and final stage of mitosis - SHORTEST PHASE -essentially the reversal of prophase - spindle apparatus disappears -a nuclear membrane reforms around each set of chromosomes and the nucleoli reappear. -the chromosomes uncoil, resuming their interphase form. - each of the two new nuclei receives a complete copy of the genome identical to the original genome and to each other - new cells enter G1 phase and cycle starts again

What happens in the first trimester of pregnancy?

- The major organs begin to develop in the first few weeks. - The heart begins to beat at approx 22 days, and soon after the eyes, gonads, limbs, and liver start to form. - By 5 weeks the embryo is 10 mm in length - By week 6 the embryo is 15 mm - The cartilaginous skeleton begins to harden into bone by week 7 - By the end of 8 weeks most of the organs have formed, the brain is fairly developed and the embryo becomes known as a fetus. - At the end of the 3rd month the fetus is about 9 cm long.

What are the requirements of a DNA cloning vector?

- at least one DNA sequence, if not many, recognized by restriction enzymes - an origin of replication - at least one gene for antibiotic resistance

Explain prophase.

- first phase of mitosis - the replicated chromatin condenses to form chromosomes - centriole pairs separate and move towards opposite poles of the cell and begin to form spindle fibers (made of microtubules) - this establishes the centrosome as one of the two microtubule organizing centers of the cell (the other being the basal body of a flagellum or cilium) - some microtubules form asters that anchor the centrioles to the cell membrane, others extend toward the middle of the cell - the nuclear membrane dissolves during prophase, allowing these spindle fibers to contact the chromosomes. - the nucleoli become less distinct and may disappear completely - kinetochores, which appear at the centrosome, are protein structures located on the centromeres that serve as attachment points for specific fibers of the spindle apparatus (called kinetochore fibers)

What is a cells resting membrane potential? (Numerical value and definition) what are the two most important ions involved in generating and maintaining the resting potential?

-70mV the net electric potential difference that exists across the cell membrane, created by movement of charged molecules across that membrane K+ and Na+

What are the control points of the citric acid cycle? Explain each

1. Citrate synthase: ATP and NADH function as allosteric inhibitors of citrate synthase, which makes sense because both are products (indirect and direct, respectively) of the enzyme. Citrate also allosterically inhibits citrate synthase directly, as does succinyl-CoA 2. Isocitrate dehydrogenase: As we discussed in the beginning of this section, the enzyme that catalyzes the citric acid cycle is likely to be inhibited by energy products: ATP and NADH. Conversely, ADP and NAD+ function as allosteric activators for the enzyme and enhance its affinity for substrates. 3. alpha=Ketoglutarate dehydrogenase complex: Once again, the reaction products of succinyl-CoA and NADH function as inhibitors of this enzyme complex. ATP is also inhibitory and slows the rate of the cycle when the cells has high levels of ATP. The complex is stimulated by ADP and calcium ions.

Enzyme Facts 1. How do enzymes affect the activation energy of a reaction? 2. How do enzymes affect the rate of a reaction? 3. How do enzymes affect the equilibrium constant of a reaction? 4. How are enzymes changed during the course of a reaction? Are they consumed? 5. How do enzymes react to pH and temperature? 6. How do enzymes affect the overall deltaG (free energy) of a reaction? 7. Are enzymes specific for a particular reaction or classes of reactions?

1. Enzymes lower the activation energy 2. They inc. the rate 3. Do not alter the equilibrium constant 4. They are not changed. They are not consumed. 5. they are pH and temp sensitive, with optimal activity at specific pH ranges and temps 6. Do not affect it**very important 7. Yes they are.

what 6 products does the stomach secrete? what are their functions?

1. HCl (kills microbes, denatures proteins, and converts pepsinogen into pepsin) 2. Pepsinogen (cleaved in the stomach to pepsin; an enzyme that partially digests proteins) 3. Mucus (protects mucosa) 4. Bicarbonate (protects mucosa) 5. Water (dissolves and dilutes ingested material) 6. Intrinsic factor (required for normal absorption of vitamin B12)

What are the 4 steps of B-oxidation?

1. Oxidation of the fatty acid to form a double bond 2. Hydration of the double bond to form a hydroxyl group 3. Oxidation of the hydroxyl group to form a carbonyl (B-ketoacid) 4. Splitting of the B-ketoacid into a shorter acyl-CoA and acetyl-CoA This process then continues until the chain has been shortened to two carbons, creating a final acetyl-CoA

What are the 4 key features of the Watson-Crick model

1. The two stands of DNA are antiparallel (the strands are oriented in opposite directions. One strand has polarity 3' to 5', the other 5' to 3') 2. The sugar-phosphate backbone is one the outside of the helix with the nitrogenous bases on the inside 3. There are specific base-pairing rules, often referred to as complementary base-pairing. An adenine (A) is always paired with a thymine (T) via 2 hydrogen bonds. A guanine (G) is always paired with cytosine (C) via 3 hydrogen bonds. The 3 H bonds make the G-C base pair interaction stronger. 4. Because of the special base-pairing, the amount of A equals the amount of T and the amount of G equals the amount of C. Thus, total purines will be equal to total pyrimidines overall (Chargaff's rule)

What are the three segments of the small intestine? Which one is responsible for the majority of chemical digestion and has some minor involvement in absorption? Which ones are where most of the absorption in the small intestine takes place?

1. duodenum (digestion) jejunum and ileum (absorption)

1. The same sugars, in different optical families are ________________ (such as D-glucose and L glucose) 2. Two sugars that are in the same family )both are either ketoses or aldoses, and have the same number of carbons) but are not identical and are not mirror images of each other are _________________. 3. A special subtype of diastereomers are those that differ in configuration at exactly one chiral center. These are defined as ________________ (such as D-ribose and D-arabinose, which only differ at C-2)

1. enantiomers 2. diastereomers 3. epimers

The human genome is composed of _____ pairs of homologous chromosomes (_______). Whats the difference between homologous chromosomes and sister chromatids? After the S phase, there are ____ chromatids organized into _____ chromosomes, organized into _____ homologous pairs

23 pairs-homologs homologs are considered separate chromosomes, sister chromatids are identical strands of DNA connected at the centromere After the S phase, there are 92 chromatids organized into 46 chromosomes, which are organized into 23 homologous pairs

What are the three basic tenants of the natural selection theory

1. organisms produce offspring, few of which survive to reproduce maturity 2. Chance variations within individuals in a population may be heritable. If these variations give an organism even a slight survival advantage, the variation is termed favorable. 3. Individuals with a greater preponderance of these favorable variations are more likely to survive to reproductive age and produce offspring; the overall result will be an increase in these traits in future generations. this level of reproductive success is termed fitness, and an organisms fitness is directly related to the relative genetic contribution of these individual to the next generation

what happens during prophase I?

1st phase of Meiosis. - chromatin condenses into chromosomes, the spindle apparatus forms, and the nucleoli and nuclear membrane disappear. - The first major difference between meiosis and mitosis occurs: homologous chromosomes come together and intertwine in a process called synapsis. At this point each chromosome consists of 2 sister chromatids, so each synaptic pair contains four chromatids and is referred to as a tetrad. The homologous chromosomes are held together by a group of proteins called the synaptonemal complex. - chromatids of homologous chromosomes may break at the point of contact called the chiasma (plural: chiasmata) and exchange equivalent pieces of DNA. This process is called crossing over (can be either single or double crossovers- depending on the # of crossover events that occur in one strand of DNA) - crossing over occurs between homologous chromosomes not sister chromatids

Explain the ovulation phase of the menstrual cycle.

2nd phase of menstrual cycle. After the follicular phase, before the luteal phase. Estrogen is interesting bc it can have neg and pos feedback effects. Late in the follicular phase, the developing follicles secrete higher [ ]s of estrogen. Eventually, estrogen [ ]s reach a threshold that paradoxically results in pos feedback, and GnRH, LH, and FSH levels spike. (weird b/c estrogen did the exact opposite in the follicular phase) The surge in LH is important because it induces ovulation, the release of the ovum from the ovary into the abdominal (peritoneal) cavity

explain anaphase I.

3rd phase of Meiosis. homologous pairs separate and are pulled to opposite poles of the cell. This process is called disjunction. During disjunction, each chromosome of paternal origin separates from its homologue of maternal origin, and either chromosome can end up in either daughter cell. Thus, the distribution of homologous chromosomes to the two intermediate daughter cells is random with respect to parental origin. This separating of the two homologous chromosomes is referred to as segregation.

Explain the information processing model

4 Components: 1. Thinking requires sensation, encoding, and storage of stimuli 2. Stimuli must be analyzed by the brain (rather than responded to automatically) to be useful in decision-making 3. Decisions made in one situation can be extrapolated and adjusted to help solve new problems (also called situational modification) 4. Problem solving is dependent not only on the person's cognitive level, but also on the context and complexity of the problem.

Explain the menstruation phase of the menstrual cycle.

4th and last phase. After luteal phase. Assuming that implantation doesn't occur, the corpus lutem loses its stimulation from LH, progesterone levels decline, and the uterine lining is sloughed off. The loss of high levels of estrogen and progesterone removes the block on GnRH so that the next cycle can begin

Explain Telophase I

4th phase of Meiosis. a nuclear membrane forms around each new nucleus. At this point, each chromosome still consists of 2 sister chromatids joined at the centromere. The cells are now haploid; once homologous chromosomes separate, only n chromosomes are found in each daughter cell (23 in humans). The cell divides into two daughter cells by cytokinesis. Between cell divisions, there may be a rest period, interkinesis, during which the chromosomes partially uncoil

Explain Prophase II

5th phase of Meiosis. 1st phase of Meiosis II. the nuclear envelope dissolves, nucleoli disappear, the centrioles migrate to opposite poles, and the spindle apparatus begins to form

Explain Metaphase II

6th phase of Meiosis. 2nd phase of Meiosis II chromosomes line up at metaphase plate

Anaphase II

7th phase of meiosis. 3rd phase of Meiosis II the centromeres divide, separating the chromosomes into sister chromatids. these chromatids are pulled to opposite poles by spindle fibers

Explain the timeline of language acquisition

9-12 months: babbling 12-18 months: about 1 word per month 18-20 months: "explosion of language" and combining words 2-3 years: longer sentences (3 words or more) 5 years: language rules largely mastered

What property of protein-digesting enzymes allows for a sequence to be determined without fully degrading the protein? A. Selectivity B. Sensitivity C. Turnover D. Inhibition

A

Which of the following accurately describes sensory neurons? A. Sensory neurons are afferent and enter the spinal cord on the dorsal side B. Sensory neurons are efferent and enter the spinal cord on the dorsal side C. Sensory neurons are afferent and enter the spinal cord on the ventral side D. Sensory neurons are efferent and enter the spinal cord on the ventral side

A

Which of the following characteristics is NOT attributed to antibodies? A. antibodies bind to more than one distinct antigen B. Antibodies label antigens for targeting by other immune cells C. Antibodies can cause agglutination by interaction with antigen D. Antibodies have two heavy chains and two light chains

A

Which of the following correctly traces the circulatory pathway? A. superior vena cava --> right atrium --> right ventricle --> pulmonary artery --> lungs --> pulmonary veins --> left atrium --> left ventricle --> aorta B. superior vena cava --> left atrium -->left ventricle --> pulmonary artery --> lungs --> pulmonary veins --> right atrium --> right ventricle --> aorta C. aorta --> right atrium --> right ventricle --> pulmonary artery --> lungs --> pulmonary veins --> left atrium --> left ventricle --> superior vena cava D. superior vena cava --> right atrium --> right ventricle --> pulmonary veins --> lungs --> pulmonary artery --> left atrium --> left ventricle --> aorta

A

Which of the following neurotransmitters is used in the ganglia of both the sympathetic and parasympathetic nervous systems? A. Acetylcholine B. Dopamine C. Norepi D. Serotonin

A

Which of the following cell types is NOT a granulocyte? A. Natural killer cell B. Mast cell C. Eosinophil D. Basophil

A Granulocytes are cells with granules in their cytoplasm. These granules are released in immune responses to attack a pathogen. Granulocytes include neutrophils, basophils, eosinophils, and mast cells. Natural killer cells are agranulocytes, which include lymphocytes (B-, T-, and NK cells) and monocytes/macrophages.

A bacterial cell is noted to be resistant to penicillin. The bacterium is transferred to a colony that lacks the fertility factor, and the rest of the colony does not become resistant to penicillin. However, the penicillin-resistant cell has also started to exhibit other phenotypic characteristics including secretion of a novel protein. Which of the following methods of bacterial recombination is NOT likely to account for this change? A. Conjugation B. transformation C. transduction D. Infection with a bacteriophage

A A bacterial cell that does not rapidly cause a phenotypic change in the rest of the colony is likely not F+, meaning that this cell is not able to form a sex pilus for conjugation, making A correct

Which of the following is true regarding passive and active immunity? A. Active immunity requires weeks to build, whereas passive immunity is acquired immediately B. Active immunity is short lived whereas passive immunity is long lived C. Active immunity may be acquired during pregnancy through the placenta. D. Passive immunity may be acquired through vaccination

A Adaptive immunity refers to the production of antibodies during an immune response. Active immunity may be conferred on an individual by vaccination, such as when an individual is injected with a weakened, inactive, or modified form of a particular antigen that stimulates the immune system to produce antibodies. Active immunity may require weeks to build. Passive immunity involves the transfer of antibodies through, for example, breast milk or injection. An example would be during pregnancy, when some maternal antibodies cross the placenta and enter fetal circulation, conferring passive immunity to the fetus. Although passive immunity is acquired immediately, it is very short-lived, lasting only as long as the antibodies circulate in the blood.

Which of the following are involved in antibody production? A. Plasma cells B. Memory cells C. Helper T-cells D. Cytotoxic cells

A Antibodies are produced by plasma cells derived from B-lymphocytes. The other cells are all types of T-lymphocytes, although memory B-cells can also exist. Still, memory B-cells do not produce antibodies; plasma cells do.

Which of the following sequences correctly shows the passage of blood through the vessels of the kidney? A. Renal artery --> afferent arterioles --> glomerulus --> efferent arterioles --> vasa recta --> renal vein B. Afferent arterioles --> renal artery --> glomerulus --> vasa recta --> --> renal vein --> efferent arterioles C. Glomerulus --> renal artery --> afferent arterioles --> efferent arterioles --> renal vein --> vasa recta D. Rental vein --> efferent arterioles --> glomerulus --> afferent arterioles --> vasa recta --> renal artery

A Blood enters the kidney through the renal artery, which divides into may afferent arterioles that run through the medulla and into the cortex. Each afferent arteriole branches into a convoluted network of capillaries called a glomerulus. Rather than convergin directly into a vein, the capillaries converge into an efferent arteriole, which divides into a fine capillary network known as the vasa recta. The vasa recta capillaries envelop the nephron tubule, where they reabsorb various ions, and then converge into the renal vein. The arrangement of tandem capillary beds is known as a portal system.

Consider a reaction catalyzed by enzyme A with a Km value of 5x10^-6M and Vmax of 20 mol/min At a concentration of 5x10^-6 m substrate the rate of the reaction will be A. 10 B. 15 C. 20 D. 30 ----mmol/min At a concentration of 5 x 10^-4m substrate the rate of the reaction will be A. 10 B. 15 C. 20 D. 30 ----mmol/min

A C

The reduction half-reaction in the last step of the electron transport chain is: A. O2 + 4e- + 4H+ --> 2H2O B. NADPH --> NADP+ + e- + H+ C. NADP+ + e- + H+ ---> NADPH D. Ubiquinone (Q) --> Ubiquinol (QH2)

A Reduction is a gain of electrons, which eliminates B because it is an oxidation reaction. NADPH, C, is a product of the pentose phosphate pathways. Ubiquinone, D, transfers electrons during the course of the ETC, but is not the final electron acceptor.

Which of the following correctly indicates the response of CD8+ T-cells when activated? A. Secretion of cytotoxic chemicals B. Causing isotope switching C. Presentation of antigens D. Activation of B cells

A CD8+ T-cells are largely responsible for the cytotoxic immune response. By releasing toxic chemicals into virally infected cells ,CD8+ T-cells are able to kill these cells in an effort to contain viral infections. Isotype switching refers to changes in the isotype of antibody produced, which isn't caused by CD8+ T-cells, eliminating B. Antigens are presented by macrophages, dendritic cells, certain epithelial cells, and some B-cells, eliminating C. B-cells are not activated by cytotoxic T-lymphocytes, eliminating D.

A researcher places several colonies of Clostridium, an obligate anaerobe, in a nutrient-rich broth but accidentally leaves the caps off the test tubes. Returning the next day, she notices that none of the tubes have shown bacterial growth. Which of the following is the most likely explanation for the lack of growth? A. Oxygen from the air diffused into the broth and the lack of enzymes capable of breaking down the superoxide anion lead to death by oxygen toxicity. B. Fungal spores in the air landed in the broth and the subsequent fungal growth consumed the available nutrients. C. The ambient temperature of the air cooled the broth below the optimal growing temperature of 37ºC, causing the Clostridium to die off. D. The failure to cap the test tubes was irrelevant and the Clostridium must have died due to some unknown factor.

A Clostridium is an obligate anaerobe, as the question states. In the presence of atmospheric oxygen, the bacteria will die off. There are number of various factors that create oxygen toxicity, but one of the major ones is the absence of enzymes necessary to deactivate toxic oxygen species. Thus (A) is the correct answer. B: The question makes no mention of fungal growth, and we have no reason to suspect that fungal spores in the air would be of a species capable of out-competing the bacteria for nutrients. C: The ambient temperature in the room is unlikely to kill off bacteria. Cooler than optimal growing conditions may slow bacterial growth, but room temperature is unlikely to be fatal. D: See explanation above for (A).

Each of the following catalyzes a rate-limiting step of a carbohydrate metabolism pathway EXCEPT: A. hexokinase B. glycogen synthase C. glucose-6-phosphate dehydrogenase D. fructose-1,6-bisphosphatase

A Hexokinase catalyzes an important irreversible step of glycolysis, but it is not the rate-limiting step. Phosphofructokinase-1 catalyzes the rate-limiting step of glycolysis. Glycogen synthase, B, catalyzes the rate-limiting step of glycogenesis; glucose-6-phosphate dehydrogenase, C, catalyzes the rate-limiting step of the pentose phosphate pathway; and fructose-1,6 bisphosphatase, D, catalyzes the rate-limiting step of gluconeogenesis.

How do hormonal controls of glycogen metabolism differ from allosteric controls? A. Hormonal control is systemic and covalent B. Hormonal control is local and covalent C. Hormonal control is systemic and noncovalent D. Hormonal control is local and noncovalent

A Hormonal controls are coordinated to regulate the metabolic activity of the entire organism, while allosteric controls can be local or systemic. The modification of the enzymes of glycogen metabolism by insulin and glucagon is either through phosphorylation or dephosphorylation, both of which modify covalent bonds

A four year old toddler with cystic fibrosis (CF) is seen by his physician for an upper respiratory infection. Prior genetic testing has shown that there has been a deletion of three base pairs in exon 10 of the CFTR gene that affects codons 507 and 508. The nucleotide sequence in this region for normal and mutant alleles is shown below (X denotes this missing nucleotide): Codon Number 506 507 508 509 510 511 Normal gene (coding strand) ATC ATC TTT GGT GTT TCC Mutant gene (coding strand) ATC ATX XXT GGT GTT TCC What effect will this mutation have on the amino acid sequence of the protein encoded by the CFTR gene? A. Deletion of a phenylalanine residue with no change in the C-terminus sequence B. Deletion of a leucine residue with no change in the C-terminus sequence C. Deletion of a phenylalanine residue with a change in the C-terminus sequence D. Deletion of a leucine residue with a change in the C-terminus sequence

A In this table, we are given the sequence of the sense (coding) DNA strand. This will be identical to the mRNA transcript, except all thymine nucleotides will be replaced with uracil. With the deletion of these three bases, codon 507 changes from AUC to AUU in the transcript; these both code for isoleucine due to wobble. However, codon 508 (UUU) has been lose. UUU codes for phenylalanine. The C-terminus sequence will remain unchanged because the deletion of three bases (exactly 1 codon) will not throw off the reading frame.

What is the function of LCAT? A. LCAT catalyzes the production of cholesteryl esters B. LCAT catalyzes the production of cholesterol C. LCAT catalyzes the transfer of cholesteryl esters D. LCAT catalyzes the transfer of cholesterol

A LCAT adds a fatty acid to cholesterol, producing cholesteryl esters, which dissolve in the core of HDL, allowing HDL to transport cholesterol form the periphery to the liver

Which of the following side effects would be anticipated in someone taking leptin to promote weight loss? A. drowsiness B. increased appetite C. irritabilty D. fever

A Leptin acts to decrease appetite by inhibiting the production of orexin. orexin is also associated with alertnes,s so decreasing the level of orexin in the body is expected to cause drowziness.

Tumor suppressor genes are most likely to result in cancer through: A. loss of function mutations B. gain of function mutations C. overexpression D. proto-oncogene formation

A Oncogenes are most likely to result in cancer through activation, (B), while tumor suppressor genes are most likely to result in cancer through inactivation

Which of the following cells is correctly coupled with its definition? A. Osteoblasts- bone cells involved in the secretion of bone matrix B. Osteoclasts- immature bone cells C. Osteocytes- polynucleated cells actively involved in bone resorption D. Chondrocytes- undifferentiated bone marrow cells

A Osteoblasts are bone cells involved in teh secretion of bone matrix. Osteoclasts are large, polynucleated cells involved in bone resorption. Osteocytes are mature bone cells that eventually become surrounded by their matrix; their primary role is bone maintenance. Finally, chondrocytes are cells that secrete chondrin ,an elastic matrix that makes up cartilage.

A eukaryotic cell has been found to exhibit a truncation mutation that creates an inactive RNA polymerase I enzyme. Which type of RNA will be affected by this inactivation? A. rRNA B. tRNA C. snRNA D. hnRNA

A RNA polymerase I in eukaryotes is found in the nucleolus and is in charge of transcribing most of the rRNA for use during ribosomal creation. RNA polymerase II is responsible for hnRNA and snRNA. RNA polymerase III si responsible for tRNA and 5S rRNA.

What is the fate of long-chain fatty acids that are contained within micelles? A. Transport into chylomicrons released into the lymphatic system B. transport into chylomicrons released into the circulatory system C. Direct diffusion across the intestine into the lymphatic system D. Direct diffusion across the intestine into the circulatory system

A Short-chain fatty acids are soluble in the intestinal lumen, and thus do not interact with micelles as longer fatty acid chains do. The long-chain fatty acids are taken up by the intestinal cells and packaged into triacylglycerols for transport as chylomicrons. Chylomicrons exit the intestine through lacteals that feed into the lymphatic system, which joins with the bloodstream in the base of the neck through the thoracic duct.

Statin drugs inhibit HMG-CoA reductase. As such, they are likely prescribed for: A. hypercholesterolemia (high cholesterol) B. hypertriglyceridemia (high triacylglycerol) C. hypocholesterolemia (low cholesterol) D. visceral adiposity (obesity)

A Statins are drugs that are prescribed to treat high cholesterol and act as competitive inhibitors of HMG-CoA reductase. HMG-CoA reductase is the rate limiting enzyme of de novo cholesterol synthesis; inhibition of this enzyme lowers production of cholesterol, thus lowering overall levels of cholesterol

Which of the following choices INCORRECTLY pairs a digestive enzyme with its site of secretion? A. Sucrase--salivary glands B. Carboxypeptidase--pancreas C. Trypsin--pancreas D. Lactase--duodenum

A Sucrase is a brush-border enzyme found on duodenal cells and is not secreted by the salivary glands. This enzyme hydrolyzes sucrose (a disaccharide) to form glucose and fructose (monosaccharides). the other associations are all correct.

In DiGeorge syndrome, the thymus can be completely absent. The absence of the thymus would leave an individual unable to mount specific defenses against which of the following types of pathogens? A. Viruses B. Bacteria C. Parasites D. Fungi

A T-lymphocytes, which mature in the thymus, are the ony specific defense against intracellular pathogens. While some bacteria, fungi, and parasites can live intracellularly, viruses-by definition-must replicate within cells. The absence of T-cells would leave an individual unable to fight viral infections with specific defenses.

When the knee moves back and forth during walking, what prevents the surfaces of the leg bones from rubbing against each other? A. Articular cartilage B. Epiphyses C. Synovial fluid D. Smooth muscle

A The articular surfaces of the bones are covered with a layer of smooth articular cartilage. The epiphyses is a portion of the bone itself. Synovial fluid lubricates the movement in the joint space, but does not stop the bones from contacting one another. There is no appreciable function for smooth muscle in the joint space.

If a test cross on a species of plant reveals the appearance of a recessive phenotype in the offspring, what must be true of the phenotypically dominant parent? A. It must be genotypically heterozygous B. It must be genotypically homozygous C. It could be either genotypically heterozygous or homozygous. D. It must have the same genotype as the test cross control parent.

A The control parent in a test cross is always recessive. Therefore, if the test parent is phenotypically dominant, yet can provide a recessive allele (as evidenced by the presence of recessive children), then the parent must have both a dominant and recessive allele. Therefore, this test parent must be heterozygous.

Diabetic nephropathy is commonly detected by finding protein in the urine of a patient. In such a disease, where is the likely defect in the nephron? A. Glomerulus B. Proximal convoluted tubule C. Loop of Henle D> Collecting duct

A The glomerulus is the most likely location of pathology if large proteins are detected in the urine. This is because large proteins should not be able to pass through the filter of the glomerulus in the first place. Once large proteins are in the filtrate, no other nephron structure can reabsorb them.

Water does not easily penetrate the skin because: A. dead keratinocytes and oil create a barrier to water penetration. B. false, water easily crosses the skin by passive diffusion through sweat glands. C. skin has a low osmolarity, thereby preventing influx of external water. D. skin cells have temperature-gated membrane channels that allow sweat to leave only at increased body temperatures.

A The human body is primarily made of water, and this water must be retained. The skin is constantly shedding dead keratinocytes (skin cells) from its surface while deeper living keratinocytes divide to replenish them. This layer of dead cells also has interspersed natural oils which combine with the dead cells to create a barrier to water. Keratinocytes also secrete keratin, a structural protein, to assist in the formation of this impenetrable barrier. At times, the body must quickly shed heat, and sweat glands act as pores through the skin surface allowing internal, higher temperature, water and salt to leave in the form of sweat. A. dead keratinocytes and oil create a barrier to water penetration. Correct. B. false, water easily crosses the skin by passive diffusion through sweat glands. Incorrect, sweat glands act as small pores through the skin with open channels. Sweat does not pass through the skin by passive diffusion, it is secreted by cells in sweat glands. C. skin has a low osmolarity, thereby preventing influx of external water. Incorrect, Water does not pass through skin because of the barrier created by hydrophobic oils and dead cells. The barrier has nothing to do with low osmotic pressure. D. skin cells have temperature-gated membrane channels that allow sweat to leave only at increased body temperatures. Incorrect, Skin cells do not have temperature-gated channels, and the cells in the outermost skin layer are dead.

In emphysema, the alveolar walls are destroyed, decreasing the recoil of the lung tissue. Which of the following changes may be seen in a patient with emphysema? A. Increased residual volume B. Decreased total lung capacity C. Increased blood [O2] D. Decreased blood [CO2]

A The intrinsic elastic properties of the lung are important during exhalation as the passive recoil of lung tissue helps decrease lung volume. With decreased recoil, the patient will have difficulty exhaling completely, increasing the residual volume. The total lung capacity would be expected to increase in this case because there would be less recoil opposing inhalation, eliminating B. With decreased alveolar surface area, one would expect decreased gas exchange, which would decrease blood [O2] while increasing blood [CO2], eliminating C and D.

The unique enzymes of gluconeogenesis are used to circumvent specific irreversible steps of glycolysis. Which of the following correctly paris an enzyme from glycolysis with its corresponding enzymes used in gluconeogenesis? A. Phosphofructokinase-1/fructose-1.6-bisphosphatase B. Pyruvate dehydrogenase / pyruvate carboxylase and phosphoenolpyruvate carboxykinase C. Hexokinase/glucokinase D. Pyruvate kinase/ glucose-6-phosphatase

A The irreversible enzymes in glycolysis are hexokinase (or glucokinase in liver and pancreatic B-cells), phosphofructokinase-1, and pyruvate kinase. Pyruvate dehydrogenase is not considered a glycolytic enzyme because it requires the mitochondria to function. The list below shows the correct pairing of glycolytic enzymes with gluconeogenic enzymes: -Hexokinase or glucokinase/glucose-6-phosphatase -Phosphofructokinase-1/fructose-1,6-bisphosphatase -Pyruvate kinase/pyruvate carboxylase and phosphoenolpyruvate carboxykinase (PEPCK)

Under normal physiological circumstances, the primary function of the nephron is to create urine that is A. hypertonic to the blood B. hypotonic to the blood C. isotonic to the filtrate D. hypotonic to the vasa recta

A The kidneys function to eliminate wastes such as urea, while reabsorbing various important substances such as glucose and amino acids for reuse by the body. Generation of a solute [ ] gradient form the cortex to the medulla allows a considerable amount of water to be reabsorbed. Excretion of [ ]ed urine serves to limit water losses from the body and helps to preserve blood volume. Water should be reabsorbed from the filtrate so urine should be hypertonic to the filtrate.

Which of the following DNA sequences would have the highest melting temp? A. CGCAACCATCCG B. CGCAATAATACA C. CGTAATAATACA D. CATAACAAATCA

A The melting temperature of DNA is the temperature at which a DNA double helix separates into two single strands (denatures). To do this, the hydrogen bonds linking the base pairs must be broken. Cytosine binds to guanine with three hydrogen bonds, whereas adenine binds to thymine with two hydrogen bonds. The amount of heat needed to disrupt the bonding is proportional to the number of bonds. Thus, the higher the GC content in a DNA segment, the higher the melting point.

The diagram shown in FIGURE 1 BIOCHEMISTRY shows the effects of arsenic on the metabolism of glyceraldehyde-3-phosphate. As a result, in the presence of arsenic, how many molecules of ATP would be created directly from the conversion of two glucose molecules to four pyruvate molecules? A. 0 B. 1 C. 2 D. 4

A The net ATP yield from glycolysis is 2 ATP per glucose. According to the question, arsenic bypasses glyceraldehyde-3-phosphate dehydrogenase and 3-phosphoglycerate kinase, directly forming 3-phosphoglycerate. 3-phosphoglycerate kinase is one of the two substrate-level phosphorylation steps and normally produces 2 ATP (one for each of the two molecules of glyceraldehyde 3-phosphate formed from glucose). If these two ATP molecules are lost, the net yield of glycolysis is now 0 ATP.

Which cell type is a phagocyte that attacks bacterial pathogens in the bloodstream? A. Neutrophils B. Eosinophils C. Basophils D. Dendritic cells

A The only phagocytes that attack bacteria on this list are neutrophils and dendritic cells. Dendritic cells are able to sample and present any type of material, and reside in the skin. Neutrophils are present in the bloodstream and can attack bacteria present there or in tissues. Eosinophils and basophils are involved in the development of allergies; eosinophils also defend against parasites.

Which of the following statements regarding polymerase chain reaction is FALSE? A. Human DNA polymerase is used because it is the most accurate B. A primer must be prepared with a complementary sequence to part of the DNA of interest C. repeated heating and cooling cycles allow the enzymes to act specifically and replaces helicase D. each cycle of the polymerase chain reaction doubles the amount of DNA of interest

A The polymerase chain reaction is used to clone a sequence of DNA using a DNA sample, a primer, free nucleotides, and enzymes. The polymerase from Thermus aquatics is used because the reaction is regulated by thermal cycling, which would denature Hyman enzymes.

Which region of the kidney has the lowest solute concentration under normal physiological circumstances? A. Cortex B. Outer medulla C. Inner medulla D. Renal pelvis

A The region of the kidney that has the lowest solute [ ] is the cortex, where the proximal convoluted tubule and a part of the distal convoluted tubule are found. The solute [ ] increases as one descends into the medulla, and [ ]ed urine can be found in the renal pelvis.

The generation of an action potential within a cell is shown in the graph below. Which of the following statements is true of the membrane during repolarization at the indicated by the arrow in FIGURE 2 BIOLOGY? A. The voltage-gated sodium channels close stopping the flow of sodium into the cell and the voltage-gated potassium channels open allowing the flow of potassium out of the cell. B. The voltage-gated sodium channels close stopping the flow of sodium out of the cell and the voltage-gated potassium channels open allowing the flow of potassium into the cell. C. The voltage-gated potassium channels close stopping the flow of potassium into the cell and the voltage-gated sodium channels open allowing the flow of sodium out of the cell. D. The voltage-gated potassium channels close stopping the flow of potassium out of the cell and the voltage-gated sodium channels open allowing the flow of sodium into the cell.

A This question asks the examinee to identify the cellular response during an action potential at the time of repolarization. The time preceding that indicated by the arrow corresponds to the depolarization of the cell. During depolarization, the voltage-gated sodium channels are open and sodium flows down its concentration gradient into the cell. After the cell depolarizes, the cell reestablishes the negative resting membrane potential by closing the voltage-gated sodium channels preventing the flow of sodium into the cell, and opening the voltage-gated potassium channels allowing the flow of potassium down its concentration gradient and out of the cell. The flow of positively charged potassium ions during repolarization results in the net loss of positive ions within the cell, which in effect creates a negatively charged interior. Thus, choice A is the correct answer.

In glucose degradation under aerobic conditions: A. oxygen is the final electron acceptor B. oxygen is necessary for all ATP synthesis C. net water is consumed D. the proton-motive force is necessary for all ATP synthesis

A This question is testing our general knowledge of cellular respiration. Notice that all types of cellular respiration (aerobic and anaerobic) start with the degradation of glucose by glycolysis. In aerobic respiration, oxygen is the final electron acceptor, and water is therefore produced at the end of the electron transport chain. While oxygen is needed for aerobic respiration in order to produce the optimal 32 molecules of ATP per glucose, it is not the only method by which ATP is produced. Glycolysis still provides 2 ATP per glucose without the need to oxygen, thus making B and D incorrect. Water, mentioned in C, is produced in aerobic metabolism, not consumed.

Premature infants with respiratory distress are often placed on ventilators. Often, the ventilators are set to provide positive end-expiratory pressure. Why might this setting be useful for a premature infant? A. Premature infants lack surfactant B. Premature infants lack lysozyme C. Premature infants cannot thermoregulate D. Premature infants are unable to control pH

A This question stem states that premature infants often require ventilation using positive end-expiratory pressures, meaning, at the end of expiration, the ventilator will provide a higher pressure than normal, which forces extra air into the alveoli. This pressure must be used to prevent alveolar collapse, which should remind you that surfactant serves the same purpose by reducing surface tension.

Which of the following is LEAST likely to be required for a series of metabolic reactions? A. triacylglycerol acting as a coenzyme B. oxidoreductase enzymes C. magnesium acting as a cofactor D. transferase enzymes

A Triglycerides are unlikely to act as coenzymes for a few reasons, inc their large size, neutral charge, and ubiquity in cells. Cofactors and coenzymes tend to be small in size, such as metal ions like C or small organic molecules. They can usually carry a charge by ionization, protonation, or deprotonation. Finally, they are usually in low, tightly regulated [ ]s within cells. Metabolic pathways would be expected to inc both oxidation-reduction reactions and movement of functional groups, eliminating B and D

In high doses, aspirin functions as a mitochondrial uncoupler. How would this affect glycogen stores? a. It causes depletion of glycogen stores. b. It has no effect on glycogen stores. c. It promotes additional storage of glucose as glycogen. d. Its effect on glycogen stores varies from cell to cell.

A Uncouplers inhibit ATP synthesis without affecting the electron transport chain. Because the body must burn more fuel to maintain the proton-motive force, glycogen stores will be mobilized to feed into glycolysis, then the TCA, and finally oxidative phosphorylation.

A man collapses while running a marathon and is taken to the emergency room. His blood is found to be somewhat acidic. and further test show increased lactate dehydrogenese activity. This enzyme is involved in which of the following pathways? A. Anaerobic glycolysis B. B-Oxidation of fatty acids C. Citric acid cycle D. Pentose phosphate pathway

A Under normal conditions, when oxygen is readily available, the pyruvate generated in glycolysis enters the mitochondrion and is converted into acetyl-CoA by the action of pyruvate dehydrogenase. During strenuous exercise, particularly by individuals in poor physical condition, the oxygen demands of the skeletal muscle may exceed the ability of the heart and lungs to provide oxygen. In this setting, the muscle switch to anaerobic glycolysis, and the pyruvate that is produced is fermented to lactate by the actin of lactate dehydrogenase.

Which of the following biomolecules does NOT drain into the liver before arriving at the right side of the heart? A. Chloecalciferol (Vitamin D) B. threonine (an amino acid) C. Fructose (a monosaccharide) D. Pantothenic acid (vitamin B5)

A While the capillaries from the intestine come together to form the portal vein, which drains to the liver, the lacteals come together to form the thoracic duct, which drains directly into the left subclavian vein. Therefore, fat-soluble compounds do not pass through the liver before reaching the right heart. Only A, vitamin D, is fat-soluble.

Sarin is a potent organophosphate that can be used in chemical warfare. As an inhibitor of acetylcholinesterase, sarin causes excessive buildup of acetylcholine in all synapses where it is the neurotransmitter. Which of the following symptoms would be most likely seen in an individual with sarin poisoning? A. Increased urination and increased sweating B. Increased urination and decreased sweating C. Decreased urination and increased sweating D. Decreased urination and decreased sweating

A an excess of acetylcholine will lead to activation of all parasympathetic neurons, preganglionic sympathetic neurons, and the postganglionic sympathetic neurons that innervate sweat glands. Because the parasympathetic nervous system causes contraction of the bladder, one would expect increased urination. The increased activation of sweat glands would lead to increased sweating as well.

The dynamic properties of molecules in the cell membrane are most rapid in: A. phospholipids moving within the plane of the membrane B. phospholipids moving between the layers of the membrane C. proteins moving within the plane of the membrane D. proteins exiting the cell through exocytosis

A movement of individual molecules in the cell membrane will be affected by size and polarity, just as with diffusion. Lipids are much smaller than proteins in the plasma membrane and will move more quickly. Lipids will move fastest within the plane of the cell membrane because the polar head group does not need to pass through the hydrophobic tail region in the same way that it would if it were moving between the membrane layers

Enhancers are transcriptional regulatory sequences that function by enhancing the activity of: A. RNA polymerase at a single promoter site B. RNA polymerase at multiple promoter sites C. Spliceosomes and a lariate formation of the ribosome D. Transcription factors that bind to the promoter but not to RNA polymerase

A specific transcription factors bind to a specific DNA sequence, such as an enhancer, and to RNA polymerase at a single promoter sequence. They enable the RNA polymerase to transcribe the specific gene for that enhancer more efficiently.

Which of the following is LEAST likely to result from protein degradation and processing by the liver? A. Fatty acids B. Glucose C. Acetoacetate D. 3-Hydroxybutyrate

A the degredation of protein and processing by the liver implies a prolonged starvation state; protein will not be used for energy unless absolutely necessary. thus, gluconeogenesis is the most likely process. When gluconeogenesis is not possible, easily metabolized molecules such as ketone bodies are synntheized. Fatty acid production occurs when energy is being stored proteins would not be broken down to store energy in fatty acids.

At the venous end of a capillary bed, the osmotic pressure A. is greater than the hydrostatic pressure B. results in a net outflow of fluid C. is significantly higher than the osmotic pressure at the arterial end D. causes proteins to enter the interstitium

A the exchange of fluid is greatly influenced by the differences in hydrostatic and osmotic pressures of blood and tissues. The osmotic (oncotic) pressure remains relatively constant; however, the hydrostatic pressure at the arterial end is greater than the hydrostatic pressure at the venous end. As a result, fluid moves out of the capillaries at the arterial end and back in at the venous end. Fluid is reabsorbed at the venous end because the osmotic pressure exceeds the hydrostatic pressure. Proteins should not cross the capillary wall under normal circumstances.

Which of the following is true of diffusion and osmosis? A. Diffusion and osmosis rely on the electrochemical gradient of only the compound of interest B. Diffusion and osmosis rely on the electrochemical gradient of all compounds in a cell C. Diffusion and osmosis will proceed in the same direction if there is only one solute D. Diffusion and osmosis cannot occur simultaneously

A the movement of any solute or water by diffusion or osmosis is dependent only on the [ ] gradient of that molecule and on membrane permeability

At any given time, there is more blood in the venous system than the arterial system. Which of the following features of veins allows for this? A. Relative lack of smooth muscle in the wall B. presence of valves C. proximity of veins to lymphatic vessels D. thin endothelial lining

A the relative lack of smooth muscle in venous walls allows stretching to store most of the blood in the body. Valves in the veins allow for one-way flow of blood toward the heart, not stretching. Both arteries and veins are close to lymphatic vessels, which has no bearing on their relative difference in volume. Both arteries and veins have a single-cell endothelial lining

explain Transformation in bacteria

A form of bacterial recombination that results from the integration of foreign genetic material into the host genome. This foreign genetic material most frequently comes from other bacteria that, upon lysing, spill their contents into the vicinity of a bacterium capable of transformation. Many gram-negative rods are able to carry out this process.

explain transduction in bacteria

A form of bacterial recombination. Transduction is the only genetic recombination process that requires a vector-a virus that carries genetic material from one bacterium to another. Viruses are obligate intracellular pathogens, which means they cannot reproduce outside a host cell. Because of this, bacteriophages can accidentally incorporate a segment of host DNA during assembly. When the bacteriophage infects another bacterium, it can release this trapped DNA into the new host cell. This transferred DNA can then integrate into the genome, giving the new host additional genes

Increased synthetic activity of the parathyroid glands would lead to: a. an increase in renal calcium and reabsorption b. a decrease in the rate of bone resorption c. a decrease in basal metabolic rate d. a decrease in blood glucose concentration

A. The parathyroid glands secrete parathyroid hormone (PTH), a hormone that increases blood calcium levels.

After infection of a cell, a viral particle must transport itself to the nucleus in order to produce viral proteins. What is the likely genomic content of the virus? A. Double-stranded DNA B. Double-stranded RNA C. Positive-sense RNA D. Negative-sense RNA

A. A virus that requires transport to the nucleus in order to produce viral proteins likely requires use of nuclear RNA polymerase in order to create mRNA that can be translated to protein. Only DNA viruses need to be transported to the nucleus to produce viral proteins, eliminating everything but A v

Which of the following is the most plausible explanation for a patient experiencing hypoglycemia? A. accidental self-injection of excess insulin B. Increased gluconeogenesis C. A 4-hour fast following a carb-rich meal D. Increased rate of glycogenolysis

A. Insulin removes glucose from the blood and takes it into the cell.

Which of the following does NOT describe connective tissue cells? A. They account for most cells in muscles, bones, and tendons B. They secrete substances to form the extracellular matrix C. In organs, they tend to form the stroma D. In organs, they provide support for epithelial cells

A. While bones and tendons are composed predominantly of connective tissue cells, muscle tissue is a different tissue type. Other examples of connective tissue include cartilage, ligaments, adipose tissue and blood.

Some studies suggest that in patients with Alzheimer's disease, there is a defect in the way the spindle apparatus attaches to kinetochore fibers. At which stage of mitotic division would one first expect to be able to visualize this problem? A. prophase B. metaphase C. anaphase D. telophase

A. the spindle apparatus first interacts with the kinetochore fibers near the end of prophase. while the spindle apparatus aligns the chromosomes at the equatorial plate during metaphase (B), the initial connection of the microtubule to the kinetochore occurs in prophase

The formation of ATP is _____ and electron transport is an ________ pathway. Why is this important?

ATP is endergonic. ETC is exergonic pathway. By coupling these reactions, the energy yielded by one reaction can fuel the other.

How is ATP formed? (two basic processes) Most of the ATP in a cell is produced by what? Some ATP is produced how?

ATP is formed via substrate-level phosphorylation and oxidative phosphorylation. Most ATP is produced by mitochondrial ATP synthase, but some is produced during glycolysis and (indirectly from GTP) in the citric acid cycle.

Explain blastulation in the embryonic development process.

After cleavage, the cell is dividing and several divisions later, the embryo becomes a solid mass of cells known as a morula. Once the morula is formed, it undergoes blastulation, which forms the blastula, a hollow ball of cells with a fluid-filled inner cavity known as a blastocoel. The mammalian blastula is known as a blastocyst and consists of two noteworthy cell groups, the trophoblast and inner cell mass. The trophoblast cells surround the blastocoel and give rise to the chorion an dlater the placenta, whereas the inner cell mass protrudes into the blastocoel and gives rise to the organism itself.

How does virus translation and progeny assembly work?

After infection, translation of viral genetic material must occur in order for the virus to reproduce. The requires translocation of the genetic material to the correct location in the cell. Most DNA viruses must enter the nucleus to be transcribed into mRNA. mRNA goes to the cytoplasm where it's translated into proteins. Genetic material from the positive-sense RNA viruses stays in the cytoplasm, where it is directly translated into protein by host cell ribosomes. Negative-sense RNA viruses require synthesis of a complementary RNA strand via RNA replicase, which can then be translated to form proteins. DNA formed through reverse transcription in retroviruses also travels to the nucleus, where it can be integrated into the host genome. using the ribosomes, tRNA, amino acids, and enzymes of the host cell, viral RNA is translated into protein. Many of these proteins allow for the creation of new virions. Now the viral genome must return to its original form (ex retroviruses must transcribe new copies of their single stranded RNA from the DNA that entered the host genome) before they can be packaged back within the capsid.

Explain the pathway of air through the respiratory tract

Air enters the respiratory tract through the external nares of the nose and then passes through the nasal cavity, where it is filtered by mucous membranes and nasal hairs (vibrissae). Next, air passes into the pharynx and the larynx. (The pharynx resides behind the nasal cavity and at the back of the mouth; it is a common pathway for both air destined for the lungs and food destined for the esophagus). (The larynx lies below the pharynx and is only a pathway for air). - To keep food out of the respiratory tract, the opening of the larynx (glottis) is covered by the epiglottis during swallowing. The larynx contains two vocal cords that are maneuvered using skeletal muscle and cartilage. From the larynx, air passes into the cartilaginous trachea and then into one of the two mainstream bronchi. The bronchi and trachea contain ciliated epithelial cells to catch material that has made it past the mucous membranes in the nose and mouth. In the lungs, the bronchi continue to divide into smaller structures known as bronchioles, which divide further until they end in the tiny balloon-like structures in which gas exchange occurs (alveoli). Each alveolus is coated with surfactant, a detergent that lowers the surface tension and prevents the alveolus from collapsing on itself. A network of capillaries surrounds each alveolus to carry oxygen and CO2. The branching and minute size of the alveoli allow for an exceptionally large surface area for gas exchange-approx 100m^2 in total

Explain the structure of an antibody.

Antibodies are Y-shaped molecules that are made up of 2 identical heavy chains and 2 identical light chains. Disulfide linkages and noncovalent interactions hold the heavy and light chains together. Each antibody has an antigen-binding region at the end of which is called the variable region (domain), at the tips of the Y. Within this region, there are specific polypeptide sequences that will bind one, and only one, specific antigenic sequence. The remaining part of the antibody molecule is known as the constant region (domain). It is this region that cells such as natural killer cells, macrophages, monocytes, and eosinophils have receptors for, and that can initiate the complement cascade

Explain the filtration function of the kidney.

Approx 20% of the blood that passes through the glomerulus is filtered as fluid into Bowman's space. The collected fluid is known as filtrate. The movement of fluid into Bowman's space is governed by Starling forces, which account for the pressure differentials in both hydrostatic and oncotic pressures between the blood and Bowman's space. The hydrostatic pressure in the glomerulus is significantly higher than that in Bowman's space, which causes fluid to move into the nephron. On the other hand, the osmolarity of blood is higher than that of Bowman's space, resulting in pressure opposing the movement of fluid into the nephron. However, the hydrostatic pressure is much larger than oncotic pressure, so the net flow is still from blood into the nephron. The filtrate is similar in composition to blood but does not contain cells or proteins due to the filter's ability to select based on size. The blood remaining in the glomerulus travels into the efferent arterioles, which empty into the vasa recta.

What are the 3 overarching domains into which all live is classified? Which 2 contain prokaryotes?

Archaea, Bacteria, and Eukarya Archaea and bacteria.

How are sperm stored? What happens to them during ejaculation? What happens to them as the pass through the reproductive tract?

As sperm are formed they are passed to the epididymis, where their flagella gain motility, and they are then stored until ejaculation. During ejaculation, sperm travel through the vas deferent and enter the ejaculatory duct at the posterior edge of the prostate gland. The two ejaculatory ducts then fuse to form the urethra, which carries sperm through the penis as they exit the body. As they pass through the reproductive tract, sperm are mixed with seminal fluid which is produced through a combined effort by the seminal vesicles, prostate gland, and bulbourethral gland.

Soap bubbles form because fatty acid salts organize into: A. Lysosomes B. Micelles C. Phospholipid bilayers D. Hydrogen bonds

B

Some enzymes require the presence of a nonprotein molecule to behave catalytically. An enzyme devoid of this molecule is called a(n) A. holoenzyme B. apoenzyme C. coenzyme D. zymoenzyme

B

When the potential across the axon membrane is more negative that the normal resting potential, the neuron is in a state of A. depolarization B. hyperpolarization C. repolarization D. polarization

B

Which language theory states that language development occurs due to preferential reinforcement of certain phonemes by parents and caregivers? A. Nativist theory B. Learning theory C. Social interactionist theory D. Neurocognitive theory

B

Which of the following is the correct sequence of the development of a mature sperm cell? A. spermatid --> spermatocyte --> spermatogonium --> 2 spermatocyte --> spermatozoan B. spermatogonium --> 1 spermatocyte --> spermatid --> spermatozoan C. spermatozoan --> 1 spermatocyte --> 2 spermatocyte --> spermatogonium --> spermatid D. spermatogonium -->1 spermatocyte--> 2 spermatocyte--> spermatozoan --> spermatid

B

After an overnight fast which of the following processes would be expected to occur at an elevated rate with the well-fed state? A. Glycolysis B. Glycogenolysis C. Glycogenesis D. Glycerol synthesis

B After a fast, the liver must contribute glucose into the bloodstream through two main processes: glycogenolysis (early to intermediate fasting) and gluconeogenesis (intermediate to late fasting). The other processes would continue at normal basal levels or have decreased activity after a fast.

After an overnight fast, which of the following enzymes would be expected to have little if any physiological activity? A. Malate dehydrogenase B. Glucokinase C. a-ketoglutarate dehydrogenase D. Phosphofructokinase-1

B After an overnight fast, the liver is producing glucose and glucokinase activity would be insignificant. Glucokinase is used to trap extra glucose in liver cells as part of a storage mechanism; with low blood glucose, liver cells would be generating new glucose, not storing it. It is also in the pancreas, where ti serves as a glucose sensor; fif glucose levels are low, it has little activity in this tissue as well. Malate dehydrogenase and alpha-ketoglutarate dehydrogenase, are citric acid cycle enzymes. Phosphofructokinase-1 is a glycolytic enzyme. Other enzymes used in glycolysis, the citric acid cycle, or gluconeogenesis, such as phosphofructokinase-1, would be expected to maintain normal activity after an overnight fast, using glucose derived from glycogen or gluconeogenesis, rather than orally ingested glucose.

To facilitate the process of birth, an infant's head is somewhat flexible. This flexibility is due in part to the two fontanelles, which are soft spots of connective tissue in the infant's skull. With time, the fontanelles will close through a process known as: A. endochondral ossification B. intramembranous ossification C. bone resorption D. longitudinal growth

B Bones form in one of two ways: endochondral ossification and intramembranous ossification. Endochondral ossification is the replacement of cartilage with bone and occurs mostly in long bones. Intramembranous ossification is the formation of bone from undifferentiated connective tissue cells and occurs mostly in the skull.

Embryos generated from In Vitro Fertilization should have roughly 8 cells after 72 hours. What is the time required for the number of cells in the blastomere to double? A) 9 hours B) 24 hours C) 36 hours D) 72 hours

B Cells in blastomeres double on a relatively constant time period. Starting with 1 cell, the number of cells increases to 2, 4, and 8 with three doubling times. With 8 cells at 72 hours, a time of 24 hours is required for the number of cells in the blastomere to double. A) 9 hours, incorrect, This answer is 72 hours divided by 8 and does not reflect a constant doubling time. B) 24 hours, correct. C) 36 hours, incorrect, This answer choice is 72 hours divided by 2. D) 72 hours, incorrect, The number of cells in the blastomere has doubled 3 times by 72 hours.

Which of the following is true about cholesterol? A. Cholesterol always increases membrane fluidity in cells B. Cholesterol is a steroid hormone precursor C. Cholesterol is a precursor for Vitamin A, which is produced in the skin D. Cholesterol interacts only with the hydrophobic tails of phospholipids

B Cholesterol has variable effects on membrane fluidity, depending on the temp. It interacts with both the hydrophobic tails and hydrophilic heads of membrane lipids It is also a precursor for vitamin D which can be produced in the skin in a UV-driven reaction

How do chylomicrons and VLDL differ? A. Chylomicrons contain apoproteins, VLDL do not B. Chylomicrons are synthesized in the intestine, VLDL in the liver C. Chylomicrons transport triacylglycerol, VLDL transport cholesterol D. VLDL are another term for chylomicron remnants; they differ in age

B Chylomicrons and VLDL are very similar. Both contain apolipoproteins and primarily transport triacylglycerols. the only major difference is the tissue of origin. Chylomicrons transport dietary triacylglycerol and originate in the small intestine, while VLDL transport newly synthesized triacylglycerols and originate in the liver

Which of the following statements is incorrect regarding inheritance of traits according to the modern synthesis model? A. A mutation due to excessive amounts of UV light occurs in an unfertilized egg; this will affect the child who is born from that egg. B. The muscular strength gained by a weight lifter during his lifetime is inherited by his children. C. A green-feathered bird that survived all of the predators in the forest will pass on the green feather genes to its offspring D. A flower with a tasty nectar eaten by a butterfly is more likely to pass on its genes through the pollen spread by the butterfly than a flower with less desirable nectar.

B Duh

The intrapleural pressure is necessarily lower than the atmospheric pressure during: A. inhalation, because the expansion of the chest cavity causes compression of the intrapleural space, decreasing its pressure B. inhalation, because the expansion of the chest cavity causes expansion of the intrapleural space, decreasing its pressure C. exhalation, because the compression of the chest cavity causes compression of the intrapleural space, decreasing its pressure D. exhalation, because the compression of the chest cavity causes expansion of the intrapleural space, decreasing its pressure

B During inhalation, the chest cavity expands, causing expansion of the intrapleural space. According to Boyle's law, an increase in volume at a constant temp is accompanied by a decrease in pressure. When the intrapleural pressure (and by extension, the alveolar pressure) is less than atmospheric pressure, air enters the lungs. During exhalation, these pressure gradients reverse; thus, during exhalation, intrapleural pressure is higher than atmospheric pressure, not lower.

Which of the following correctly shows the amount of ATP produced from the given high-energy carriers? A. FADH2--> 1 ATP B. FADH2 --> 1.5 ATP C. NADH --> 3ATP D. NADH --> 3.5 ATP

B During oxidative phosphorylation, energy is harvested from the energy carriers FADH2 and NADH in order to form ATP. One molecule of mitochondrial FADH2 is oxidized to produce 1.5 molecules of ATP. Similarly, one molecule of mitochondrial NADH is oxidized to produce 2.5 molecules of ATP in the electron transport chain

Which of the following indicates the pattern of sleep stages during a complete sleep cycle early in the night? A. 1-2-3-4-1-2-REM B. 1-2-3-4-3-2-REM C. 4-3-2-1-2-3-REM D. 4-3-2-4-3-1-REM

B Early in the evening, sleep cycles include deepening of sleep (Stages 1-2-3-4), followed either by lightening of sleep (Stages 4-3-2), and then REM, or just directly moving from Stage 4 into REM. Later in the evening, the cycle may be shortened as slow-wave sleep becomes less common.

An autoimmune disease attacks the voltage-gated calcium channels in the synaptic terminal of an excitatory neuron. What is a likely symptom of this condition? A. spastic paralysis (inability to relax muscles) B. flaccid paralysis (inability to contract muscles) C. inability to reuptake neurotransmitters once released D. retrograde flow of action potentials

B First, consider the function of voltage-gated Ca+ channels. When the nerve terminal depolarized, voltage-gated Ca+ channels open, allowing for influx of Ca+. This influx triggers fusion of the synaptic vesicles containing neurotransmitters with the membrane of the neuron at the nerve terminal. This allows for exocytosis of the neurotransmitters into the synapse. If a disease blocked the influx of Ca+, there would be no release of neurotransmitters. A lack of neurotransmitters meal that the neuron cannot send excitatory signals. Thus, any symptom resulting from this disease would be due to an inability of neurons to transmit excitatory signals to the muscle. If neurons cannot communicate, flaccid paralysis may be one of the results.

Which of the following enzymes is NOT used for lipid digestion in the digestive tract? A. Pancreatic lipase B. Hormone-sensitive lipase C. Cholesterol esterase D. Colipase

B Hormone sensitive lipase response to low insulin levels as well as cortisol and epi to mobilize fatty acids from adipocytes. It is not involved in digestion, but rather mobilizaiton of fatty acids

Which fatty acid can be synthesized by humans? A. 12:0 B. 16:0 C. 16:1 D. 18:3

B Humans can only synthesize one fatty acid, palmitic acid. Palmitic acid is fully saturated and therefore does not contain any double bonds. It has 16 carbbons and is synthesized from eight molecules of acetyl-CoA

The ability to exist in both an oxidized and a reduced state is characteristic of: A. adenosine triphosphate (ATP) B. electron carriers C. regulatory enzymes D. peptide hormones

B In order to transport electrons, electron carriers like flavoproteins must be able to exist in a stable oxidized state and a stable reduced form. ATP can be dephosphorylated but is generally not oxidized or reduced. Regulatory enzymes may also be phosphorylated or dephosphorylated but are not generally oxidized or reduced.

Studies have indicated that premature babies are often deficient in lysozyme. What is a possible consequence of this deficiency? A. Respiratory distress and alveolar collapse shortly after birth B. Increased susceptibility to certain infections C. Inability to humidify air as it passes through the nasal cavity D. Slowing of the respiratory rate in response to acidemia

B Lysozyme is an enzyme present in the nasal cavity, saliva, and tears that degrades peptidoglycan, preventing infection by gram-positive bacteria.

A drug is used that prevents the conversion of angiotensin I to angiotensin II. What is a likely effect of this drug? A. Increased sodium reabsorption B. Increased potassium reabsorption C. Increased blood pressure D. Increased blood pH

B Normally angiotensin II causes secretion of aldosterone from the adrenal cortex. Aldosterone serves to increase reabsorption of sodium, while promoting excretion of potassium and hydrogen ions. Thus, blocking the release of aldosterone should result in decreased reabsorption of sodium, while decreasing excretion of potassium and hydrogen ions.

In a single strand of a nucleic acid, nucleotides are linked by: A. hydrogen bonds B. phosphodiester bonds C. ionic bonds D. van der Waals forces

B Nucleotides bond together to form polynucleotides. The 3' hydroxyl group of one nucleotide's sugar joins the 5' hydroxyl group of the adjacent nucleotide's sugar by a phosphodiester bond. Hydrogen bonding, (A), is important for holding complementary strands together, but does not play a role in the bonds formed between adjacent nucleotides on a single strand

Which of the following is an ethical concern of gene sequencing? A. gene sequencing is invasive, thus the potential health risks must be thoroughly explained B. Gene sequencing impacts relatives, thus privacy concerns may be raised C. gene sequencing is very inaccurate, which increases anxiety related to findings D. gene sequencing can provide false-negative results, giving a false sense of security

B One of the primary ethical concerns related to gene sequencing is the issue of consent and privacy. Because genetic screening provides information on direct relatives, there are potential violations of privacy in communicating this info to family members who may be at risk. There are not significant physical risks, eliminating (A), and gene sequencing is fairly accurate, eliminating C and D.

Peptidyl transferase connects the carboxylate group of one amino acid to the amino group of an incoming amino acid. What type of linkage is created in this peptide bond? A. Ester B. Amine C. Anhydride D. Ether

B Peptidyl transferase connects the incoming amino terminal to the previous carboxyl terminal; the only functional group listed here with a carbonyl and amino group is the amide.

During a myocardial infarction, the oxygen supply to an area of the heart is dramatically reduced, forcing the cardiac myocytes to switch to anaerobic metabolism. Under these conditions, which of the following enzymes would be activated by increased level of intracellular AMP? A. Succinate dehydrogenase B. Phosphofructokinase-1 C. Isocitrate dehydrogenase D. Pyruvate dehydrogenase

B Phosphofructokinase-1 (PFK-1), which catalyzes the rate-limiting step of glycolysis, is the only enzyme listed here that functions under anaerobic conditions. The other enzymes are all involved in the oxygen-requiring processes discussed in this chapter. Succinate dehydrogenase (A), appears in both the citric acid cycle and as part of Complex iI of the electron transport chain. Isocitrate dehydrogenase (C), catalyzes the rate-limiting step of the citric acid cycle. Pyruvate dehydrogenase (D), is one of the five enzymes that make up the pyruvate dehydrogenase complex.

You have just sequenced a piece of DNA that reads as follows: 5'--TCTTTGAGACATCC--3' What would the base sequence of the mRNA transcribed from this DNA be? A. 5'--AGAAACUCUGUAGG--3' B. 5'--GGAUGUCUCAAAGA--3' C. 5--AGAAACTCTGTAGG--3' D. 5'--GGATCTCTCAAAGA--3'

B Remember that mRNA is antiparallel to DNA. Our answer should be 5' to 3' mRNA, with the 5' end complementary to the 3' end of the DNA that is being transcribed.

In which of the following segments of the nephron is sodium NOT actively transported out of the nephron? A. Proximal convoluted tubule B. Thin portion of the ascending limb of the loop of Henle C. Distal convoluted tubule D. Thick portion of the ascending limb of the loop of Henle

B Sodium is actively transported out of the nephron in the proximal and distal convoluted tubules, where the [ ] of sodium outside of the nephron is higher than inside; this, energy is required to transport the sodium molecules against their [ ] gradient. In the inner medulla, however, sodium and other ions diffuse passively down their [ ] gradients at the thin ascending limb of the loop of Henle. The thick ascending limb of the loop of Henle is thick because its cells contain may mitochondria--which produce the ATP needed for active transport of sodium and chloride out of the filtrate.

In a species of plant, a homozygous red flower (RR) is crossed with a homozygous yellow flower (rr). If the F1 generation is self-crossed and the F3 generation has a phenotypic ratio of red:orange:yellow of 1:2:1, which characteristic accounts for these results? A. Codominance B. Incomplete dominance C. Penetrance D. Expressivity

B Some progeny in the second generation are apparently blends of the parental phenotypes.

A respiratory quotient approaching 0.7 indicates metabolism primarily of which macromolecule? A. carbohydrates B. lipids C. nucleic acids D. amino acids

B The RQ gives and indication of the primary fuel being utilized. 0.7 indicates lipid metabolism, 0.8-0.9 indicates amino acid metabolism, and 1 indicates carbohydrate metabolism. Nucleic acids don't contribute significantly to the RQ

In the absence of oxygen, which tissue will experience damage most rapidly? A. skin B. brain C. RBCs D. liver

B The brain uses aerobic metabolism of glucose exclusively and therefore is very sensitive to oxygen levels. The extremely high oxygen requirement of the brain (20% of the body's oxygen content) relative to its size 92% of total body weight) implies that brain is the most sensitive organ to oxygen deprivation

Which enzyme converts GDP to GTP? A. Nucleosidediphosphate phosphatease B. Nucleosidediphosphate kinase C. Isocitrate dehydrogenase D. Pyruvate dehydrogenase

B The conversion of GDP to GTP is a phosphorylation reaction, in which a phosphate group is added to a molecule. Such reactions are catalyzed by kinases. Nomenclature is helpful here, as nucleosidediphosphate kinase is the only enzyme that contains kinase in its name.

Which of the following definitions is FALSE? A. Penetrance-the percentage of individuals in the population carrying the allele who actually express the phenotype associated with it B. Expressivity- the percentage of individuals in the population carrying the allele who do not express the phenotype associated with it C. Incomplete dominance- occurs when the phenotype of the heterozygote is an intermediate of the phenotypes of the homozygotes D. Codominance- occurs when multiple alleles exist for a given gene and more than one of them is dominant

B The definition given here for expressivity is a much better match for 1- the pentrance. Expressivity refers to the variable manifestations of a given genotype as different phenotypes; the degree to which various phenotypes are expressed.

A surgeon accidentally clips a dorsal root ganglion during a spinal surgery. What is a likely consequence of this error? I. loss of motor function at that level II. loss of sensation at that level III. loss of cognitive function a) I only b) II only c) I and II only d) I, II, and III

B The dorsal root ganglion contains cell bodies of sensory neurons only. If it is cut, there will be disruption of sensation

When trypsin converts chymotrypsinogen to chymotrypsin, some molecules of chymotrypsin bind to a repressor, which in turn binds to an operator region and prevents further transcription of trypsin. This is most similar to which of the following operons? A. trp operon during lack of tryptophan B. trp operon during abundance of tryptophan C. lac operon during lack of lactose D. lac operon during abundance of lactose

B The example given is a sample of repression due to the abundance of a corepressor. In other words, this is a repressible system that is currently blocking transcription. trp operon is a repressible system lac operon is an inducible system

The two graphs below show the relative activities of two enzymes in solutions of varying pH. Which of the following choices correctly identifies the two enzymes? BIOLOGY FIGURE 1 A. 1--chymotrypsin; 2--pepsin B. 1--pepsin; 2--carboxypeptidase B C. 1--lactase; 2--aminopeptidase D. 1--eteropeptidase; 2--amylase

B The first graph shows maximal activity at a very acidic pH, implying that this is an enzyme acting in the stomach. The second graph shows maximal activity around pH 8.5, implying that this is an enzyme acting in the duodenum. The only choice that matches the first graph with a stomach enzyme (pepsin) and the second with a duodenal enzyme (carboxyepptidase B) is B.

Which of the following would most likely be filtered through the glomerulus into Bowman's space? A. Erythrocytes B. Monosaccharides C. Platelets D. Proteins

B The glomerulus functions like a sieve; small molecules dissolved in the fluid will pass through the glomerulus, including glucose, which is later reabsorbed. Large molecules, such as proteins, and cells, such as erythroytes and platelets, will not be able to pass through the glomerular filter.

Which of the following are not involved in cell-mediated immunity? A. Memory cells B. Plasma cells C. Cytotoxic cells D. Suppressor cells

B The lymphocytes involved in cell-mediated immunity are the T-lymphocytes, or T-cells. There are four types of T-cells, each playing a different role in cell-mediated immunity: cytotoxic T-cells, helper T-cells, memory T-cells, and suppressor T-cells. Plasma cells are differentiated immunoglublin-secreting B-lymphocutes involved in humoral immunity

A scientist discovers a new hormone that is relatively large in size and triggers the conversion of ATP to cAMP. Which of the following best describes the type of hormone that was discovered? A. Amino acid-derivative hormone B. Peptide hormone C. Steroid hormone D. Tropic hormone

B hormones that act via second messengers and are relatively large in size are peptides.

Double-stranded RNA cannot be translated by the ribosome and is marked for degradation in the cell. Which of the following strands of RNA would prevent mature mRNA in the cytoplasm from being translated? A. Identical mRNA to the one produced B. Antisense mRNA to the one produced C. mRNA with thymine substituted for uracil D. Sense mRNA to the one produced

B The mRNA produced has the same structure as the sense strand of DNA (with uracils instead of thymines). Because bonding of nucleic acids is always complementary but antiparallel, the antisense strand of mRNA would be the one that binds to the produced mRNA, creating double-stranded RNA that is then degraded once found in the cytoplasm.

The lymphatic system: A. transports hormones throughout the body B. transports chylomicrons to the circulatory system C. causes extravasation of fluid into tissues D. is the site of mast cell activation

B The main function of the lymphatic system is to collect excess interstitial fluid and return to to the circulatory system, maintaining the balance of body fluids. However, this is not one of the answer choices. In addition, the lymphatic system accepts chylomicrons from the small intestine and delivers the to the cardiovascular circulation. Transport of hormones is a function of the cardiovascular system, eliminating A. The lymphatic system absorbs fluid that has been pushed into tissues, but does not cause the extravasation of the fluid, eliminating C. Mast cells reside and are activated in the skin and mucous membranes, eliminating D.

A double stranded RNA genome isolated from a virus was found to contain 15% uracil. What percentage of guanine should exist in this virus' genome? A. 15% B. 35% C. 70% D. 85%

B The percentage of uracil must equal that of adenine due to base-pairing because the genome is double-stranded. This accounts for 30% of the genome. The remaining 70% must be split evenly between guanine and cytosine, so they each account for 355 of the genome.

A two-week old male infant is brought to the ER. His mother reports that he has been unable to keep any milk down; shortly after he nurses, he has sudden projectile vomiting. During exam, an olive-shaped mass can be felt in his upper abdomen. It is determined that there is a constriction in the digestive system that prevents food from reaching the small intestine from the stomach. Which structure is most likely the site of the problem? A. Cardiac sphincter B. Pyloric sphincter C. Ileocecal valve D. Internal and sphincter

B The question is asking us to identify the structure that lies between the stomach and the small intestine. This is the pyloric sphincter; the presentation given in the question is a classic example of what is called pyloric stenosis, in which the pyloric sphincter is thickened and cannot relax to permit chyme through. The cardiac sphicter, A, lies between the esophagus and the stomach. The ileocecal valve, C, lies between the ileum of the small intestine and the cecum of the large intestine. The internal and sphincter, D, lies at the end of the rectum.

How does the ideal temp for a reaction change with and without an enzyme catalyst? A. The ideal temp is generally higher with a catalyst than without B. The ideal temp is generally lower with a catalyst than without C. The ideal temp is characteristic of the reaction, not the enzyme D. No conclusion can be made without knowing the enzyme type

B The rate of reaction inc with temp because of the inc kinetic energy of the reactants, but reaches a peak temp bc the enzyme denatures within the disruption of hydrogen bonds at excessively high temps. In the absence of enzyme, this peak temp is generally much hotter. Heating a reaction provides molecules with an inc chance of achieving the activation energy, but the enzyme catalyst would typically reduce activation energy. Keep in mind that thermodynamics and kinetics are not interchangeable, so we are not considering the impact of heat on the equilibrium position.

As the climate became colder during the Ice Age, a particular species of mammal evolved a thicker layer of fur. What kind of selection occurred in this population? A. Stabilizing selection B. Directional selection C. Disruptive selection D. Speciation selection

B The situation described in the question stem is an example of directional selection. In directional selection, the phenotypic norm of a particular species shifts toward an extreme to adapt to a selective pressure, such as an increasingly colder environment.

What type of immunity is likely to be affected by removal of the spleen? A. Cytotoxic immunity B. Humoral immunity C. Innate immunity D. Passive immunity

B The spleen is a location where B-cells mature and proliferate. Therefore, removal of the spleen is likely to result in a reduction of humoral immunity. In fact, many people receive vaccinations prior to removal of the spleen in order to bolster their immunity.

Why is it preferable to cleave thioester links rather than typical ester links in aerobic metabolism? A. Oxygen must be conserved for the electron transport chain B. Thioester hydrolysis has a higher energy yield C. Typical ester hydrolysis cannot occur in vivo D. Thioester cleavage requires more energy

B Thioester links release a great deal of energy when hydrolyzed, making them well-suited as respiration reaction drivers. They are particularly useful because they release more energy than typical ester cleavage. It is thioester formation, not hydrolysis, that requires a great deal of energy, making D incorrect.

What is the gene order of linked genes M, N, O, and P, given the following recombination frequencies? MN: 6% MO: 12% MP: 5% NO: 18% NP: 1% OP: 17% A. MOPN B. NPMO C. ONPM D. PNMO

B This is a gene mapping problem. Because there is a correlation between the frequency of recombination and the distance between genes on a chromosome, if we are given the frequencies, we can determine gene order. 1 map unit equals 1 percent recombination frequency. N and O recombine with a frequency of 18%, so they are 18 map units apart on the chromosome.

Which of the following is NOT a function of the cell membrane? A. Cytoskeletal attachment B. Protein synthesis C. Transport regulation D. Second messenger reservoir

B ribosomes are the site of protein synthesis within a cell and are not coupled to the cell membrane.

The bulk movement of liquid into a cell through vesicular infolding is knowns as: A. Phagocytosis B. Pinocytosis C. Exocytosis D. Drinking

B the endocytosis of fluid is pinocytosis

Mono-nucleated muscle cells are a predominant muscle fiber found in all of the following EXCEPT: A. the stomach B. the pharynx C. the heart D. the lungs

B This question is testing your familiarity with the predominant muscle cell types found in various structures or organs within the body. To answer this question, you must know the characteristics of the three different muscle types: 1) skeletal, 2) cardiac, and 3) smooth. Skeletal muscle is striated, multi-nucleated, and under voluntary control and is typically attached to bones and cartilages. Cardiac muscle is involuntary, striated, and mono-nucleated and is found only in the heart. Smooth muscle is involuntary, mono-nucleated and non-striated is generally associated with viscera. Since skeletal muscles contain only multi-nucleated cells, the correct answer to this question must be a structure that is under voluntary control. The stomach and the lungs are internal organs found in the thoracic and abdominal cavities. These visceral organs are involuntary and consist primarily of smooth muscle fibers eliminating choices A and D. The heart is also involuntary and contains mostly cardiac muscle eliminating answer choice C. The pharynx is part of the digestive tract and is located above the esophagus. Both the pharynx and the upper third of the esophagus consist mostly of skeletal muscle, while the rest of the digestive system is controlled by smooth muscle. Since the pharynx contains primarily skeletal muscle, the cells are multi-nucleated. Thus the correct answer is B.

In the digestive system, triglycerides interact with amphipathic: A. pepsin B. bile salts C. glycogen D. sphingomyelin

B Triglycerides are hydrophobic dietary fats. In order to be absorbed by the digestive system, bile salts create small emulsions containing triglycerides. Bile salts themselves are created in the liver and stored in the gall bladder prior to release. Since bile salts are amphipathic, they can interact with the hydrophobic triglycerides and the aqueous environment of the digestive tract lumen. A, pepsin, incorrect, Pepsin is the active catabolic enzyme of the zymogen pepsinogen, secreted by chief cells in the stomach. B. bile salts, correct. C. glycogen, incorrect, Glycogen is a branched polymer of glucose found primarily in muscle and liver cells. D. sphingomyelin, incorrect, Sphingomyelin is a membrane sphingolipid found in animal cell membranes, especially those of myelinated nerve axons. Sphingomyelin is unlikely to be tested on the MCAT unless described in a passage.

Which of the statements regarding waxes is FALSE? A. Waxes generally have melting points above room temp B. Waxes are produced only in plants and insects and therefore must be consumed by humans C. Waxes protect against dehydration and parasites D. Waxes are esters of long-chain fatty acids and long-chain alcohols

B Waxes are also produced in animals for similar protective function (hello, earwax)

Some people with anxiety disorders respond to stress by hyperventilating. It is recommended that these people breathe into a paper bag and then rebreather this air. Why is this treatment appropriate? A. Hyperventilation causes an increase in blood carbon dioxide, and breathing the air in the bag helps to readjust blood levels of carbon dioxide. B. Hyperventilation causes a decrease in blood carbon dioxide, and breathing the air in the bag helps to readjust blood levels of carbon dioxide. C. Hyperventilation causes an increase in blood oxygen, and breathing the air in the bag helps to readjust blood levels of oxygen. D. Hyperventilation causes a decrease in blood oxygen and breathing the air in the bag helps to readjust blood levels of oxygen.

B When people hyperventilate, their respiratory rate increases. When the resp rate increases, more CO2 is blown off. this causes a shift to the left in the bicarbonate buffer equation, and the blood becomes more alkaline. Breathing into the bag allows some of this CO2 to be returned to the bloodstream to maintain proper pH

Which of the following directly provides the energy needed to form ATP in the mitochondrion? A. Electron transfer in the electron transfer chain B. An electrochemical proton gradient C. Oxidation of acetyl-CoA D. B-oxidation of fatty acids

B While all of the other answers contribute to energy production, it is the electrochemical gradient (proton-motive force) that directly drives the phosphoylation of ATP by the F1 portion of ATP synthase.

Which vascular structure creates the most resistance to blood flow? A. aorta B. arterioles C. capillaries D. veins

B arterioles are highly muscular and have the ability to contract and dilate in order to regulate blood pressure

In a particular Hardy Weinberg population, there are only two eye colors: Brown (B) and blue (b). 36% of the population has blue eyes, the recessive trait. What percentage of the population is heterozygous for brown eyes? A. 24% B. 48% C. 60% D. 64%

B bb=37%=0.36=q^2. q=0.6 so p= 1-0.6 = 0.4 2pq= 2 x 0.4 x 0.6 = 0.48 = 48%

How is cDNA best characterized? A. cDNA results from a DNA transcript with noncoding regions removed. B. cDNA results from the reverse transcription of processesd mRNA C. cDNA is the abbreviation for deoxycytosine D. cDNA is the circular DNA molecule that forms the bacterial genome

B cDNA (complementary DNA) is formed from a processed mRNA strand by reverse transcription. cDNA is used in DNA libraries and contains only the axons of genes that are transcriptionally active in the sample tissue

A patient presents with muscle weakness, slow movement, and calcium deposits in his tissues. A blood test revealed very low calcium levels in the blood. Administration of which of the following would be an appropriate treatment for the blood test findings? A. Calcitonin B. Parathyroid hormone D. Aldosteron D. Thymosin

B calcitonin would be a poor choice as it would lower blood calcium [ ]s. parathyroid hormone would raise blood calcium [ ]

Which lipid type is LEAST likely to contribute to membrane fluidity? A. Unstaturated glycerphospholipids B. Trans glycerophospholipids C. Cholesterol D. Unsaturated sphingolipids

B compounds that contribute to membrane fluidity will lower the melting point or disrupt the crystal structure. cholesterol and unsaturated lipids are known for these functions. trans glycerophospholipids tend to increase the melting point of the membrane and therefore decrease membrane fluidity

At 25C the deltaG^not for a certain reaction A <--> B + 2C is 0. If the [ ] of A, B, and C in the cell at 25C are all 10mM, how does the deltaG compare to the measurement taken with 1 M [ ]s? A. delta G is greater than deltaG^not, thus the reaction is spontaneous B. delta G is less than deltaG^not, thus the reaction is spontaneous C. delta G is greater than deltaG^not, thus the reaction is nonspontaneous D. delta G is less than deltaG^not, thus the reaction is nonspontaneous

B deltaG=deltaG^not + RTlnQ Q is [B][C]^2/[A]. geltaG= 0 + RTln [10E-3][10E-3]^2/[10E-3] = RTln10E-4 = -4RT Because both R and T are positive, deltaG must be negative and therefore lower than the original value.

A gene encodes a protein with 150 amino acids. There is one intron of 1000 base pairs (bp), a 5'-untranslated region of 100bp, and a 3'-untranslated region of 200 bp. In the final mRNA, about how many bases lie between the start AUG codon and final termination codon? A. 150 B. 450 C. 650 D. 1750

B the intron will not be a part of the final, processed mRNA, and the untranslated regions of the mRNA will not be turned into amino acids. Translation will begin with codon 1 (AUG). Because there are 150 amino acids, we can surmise that there will be 151 codons. Each codon will use 3 nucleotides, so 150 x 3 = 450 because codon 151 will be the stop codon

How does the inner mitochondrial membrane differ from the outer mitochondrial membrane? A. The inner mitochondrial membrane is more permeable and lacks cholesterol B. The inner mitochondrial membrane is less permeable and lacks cholesterol C. The inner mitochondrial membrane is more permeable and has cholesterol D. The inner mitochondrial membrane is less permeable and has cholesterol

B the outer mitochondrial membrane is very permeable while the inner membrane is highly impermeable. The inner mitochondrial membrane is unique within the cell because it lacks cholesterol

Which of the following statements is false based on Darwin's theory of evolution? A. Natural selection is the driving force for evolution B. Favorable genetic variations become more and more common in individuals throughout their lives C. Natural selection can drive organisms living in groups to ultimately become distinct species. D. Fitness is measured by reproductive success

B the theory of natural selection applies to a population of organisms, not to a particular individual. as such, favorable genetic variations become more and more common from generation to generation, not during the lifetime of an individual

What are the two types of lymphocytes of the adaptive immune system? What do they both govern? Where are they created and where do they mature?

B cells and T cells. B- cells govern the humoral response, T cells mount the cell mediated response. All cells of the immune system are created in the bone marrow, but B and T cells mature in different locations. B cells mature in the bone marrow and T cells mature in the Thymus.

Which of the following correctly lists a pair of analogous structures and a pair of homologous structures, respectively? A. The wing of a bee and the wing of a bird; the wing of a bird and the leg of a bird B. The wing of a bee and the wing of a bird; the arm of a human and the flipper of a walrus C. The arm of a human and the wing of a bat; the wing of a bird and the wing of a bat D. The wing of a bird and the wing of a bat; the wing of a bee and the wing of a bat

B is correct. Analogous structures are those structures that evolved independently to carry out the same function. Thus, the wing of a bee and the wing of a bird are analogous structures. Homologous structures are those that have a similar evolutionary history, arising from the same source, even if they now have different functions. The forelimbs of mammals (human arm, walrus flipper, bat wing) would all be homologous despite their different functions. Thus, this choice correctly indicates a pair of analogous structures and then a pair of homologous structures. A: The first set of structures listed here are analogous, but the second set (which come from the same type of organism) are neither analogous nor homologous. C: These are homologous and analogous structures, respectively. D: Both of these sets of structures are analogous. Content Foundations: Evolution In the 1973 words of the evolutionary biologist (and self-described creationist) Theodosius Dobzhansky, "nothing in biology makes sense except in the light of evolution." Natural selection is the mechanism through which evolution takes place. The concept of natural selection refers to the tendency of certain phenotypes to be favored in terms of reproduction. Natural selection is closely linked to the term fitness, which in the evolutionary context only refers to the chance of reproduction associated with a certain phenotype compared to baseline. Fitness must be defined in terms of specific environmental constraints. In the 20th century, the concept of group selection was proposed, arguing that natural selection could act on the level of the group, not the individual. The related concept of inclusive fitness expands the rigorous evolutionary definition of fitness (defined in terms of the differential reproduction of alleles) to account not just for individuals but also their relatives, who can be expected to share many of the same alleles. This idea helps to explain altruistic behavior. Additionally, multiple "types" of selection have been identified to describe the outcomes associated with different types of selective pressures on phenotypes that vary along a spectrum. Stabilizing selection occurs if both extremes are selected against, directional selection occurs if only one extreme phenotype is selected against and the other extreme is favored, and disruptive selection occurs when the median phenotype is selected against. The concept of Hardy-Weinberg equilibrium is often used to model stable gene pools that satisfy the following assumptions: (1) organisms must be diploid and reproduce sexually; (2) mating is random; (3) the population size is very large; (4) alleles are randomly distributed by sex; (5) no mutations occur; and (6) there is no migration into or out of the population. The Hardy-Weinberg equations allow us to use allele frequencies to predict the distribution of phenotypes in the population and vice versa. If p and q are the only two alleles of a gene present in the population, then p + q = 1. Squaring the equation yields: (p + q)2 = 12 → p2 + 2pq + q2 = 1. This second equation allows us to connect genotypes and phenotypes, because the p2 and q2 terms correspond to individuals homozygous for p and q, respectively, while the 2pq term gives the frequency of heterozygotes. Other mechanisms are associated with changes in the gene pool of a species, such as genetic drift and bottlenecks. Genetic drift refers to the role of chance, in the absence of strong selective pressures, in determining the reproductive fitness of various alleles. When no strong pressure exists for a certain allele, it may randomly happen to be reproduced more or less often. These random effects can add up over the course of evolution. A related, but more specific, concept is that of an evolutionary bottleneck. Bottlenecks occur when some external event dramatically reduces the size of a population in a way that is essentially random with regard to most, if not all, alleles. This dramatically reduces diversity in the gene pool.

Explain the process of B-oxidation in mitochondria

B-oxidation reverses the process of fatty acid synthesis by oxidizing and releasing (rather than reducing and linking) molecules of acetyl-CoA. The pathway is a repitition of four steps; each four-step cycle releases one acetyl-CoA and reduces NAD+ and FAD (producing NADH and FADH2). The FADH2 and NADH are oxidized in the electron transport chain, producing ATP. In muscle and adipose tissue, acetyl-CoA enters the citric acid cycle. In the liver, acetyl-CoA, which cannot be converted to glucose, stimulates glyconeogenesis by activating pyruvate carboxylase. In a fasting state, the liver produces more acetyl-CoA from B-oxidation than is used in the citric acid cycle. Much of the acetyl-CoA is used to synthesize ketone bodies (essentiall two acetyl-CoA molecules linked together) that are released into the bloodstream and transported to other tissues. Four steps of B-oxidation 1. Oxidation of the fatty acid to form a double bond 2. Hydration of the double bond to form a hydroxyl group 3. Oxidation of the hydroxyl group to form a carbonyl (B-ketoacid) 4. Splitting of the B-ketoacid into a shorter acyl-CoA and acetyl-CoA This process then continues until the chain has been shortened to two carbons, creating a final acetyl-CoA

What is the function of sodium dodecyl sulfate (SDS) in SDS-PAGE? A. SDS stabilizes the gel matrix, improving resolution during electrophoresis B. SDS solubilizes proteins to give them uniformly negative charges, so the separation is based purely on size C. SDS raises the pH of the gel, separating multiunit proteins into individual subunits. D. SDS solubilizes proteins to give them uniformly positive charges, so separation is based purely on pH.

B.

Many medications have anticholinergic side effects, which block the activity of parasympathetic neurons throughout the body. Older individuals may be on many such medications simultaneously, exacerbating the side effects. Which of the following would NOT be expected in an individual taking medications with anticholinergic activity? A. Dry mouth B. Diarrhea C. Slow gastric emptying D. Decreased gastric acid production

B. The parasympathetic nervous system has many roles in the digestive system. It promotes motility of the gut tube and secretion from glands. Therefore, blocking the parasympathetic nervous system would likely result in dry mouth (from reduced secretion of saliva), slow gastric emptying (from decreased peristalsis), and decreased gastric acid production (from reduced HCl secretio from the parietal cells in the gastric glands), eliminating A, C, and D. B is the correct answer because we would expect constipation in such an individual, not diarrhea; slowed motility through the colon would lead to increased water re-absorption, making the feces too firm and causing constipation.

Which of the following activities occurs in the Golgi apparatus? A. synthesis of proteins B. modification and distribution of proteins C. breakdown of lipids and carbs D. production of ADP

B. A takes place in the ribosomes and rough ER C takes place in the peroxisomes and cytoplasm D. takes place in the mitochondria

A patient presents to the ER with a stab wound to the left side of the chest. On a chest x-ray, blood is noted to be collecting in the chest cavity, causing collapse of both lobes of the left lung. The blood is most likely located between: A. the parietal pleura and the chest wall B. the parietal pleural and the visceral pleura C. the visceral pleura and the lung D. the alveolar walls and the lung surface

B. The intrapleural space, bounded by the parietal and visceral pleurae, is a potential space. As such , it is normally collapsed and contains a small amount of fluid. However, introduction of fluid or air into the intrapleural space can fill the space, causing collapse of the lung. The other options listed are too firmly apposed to permit blood to collect in these spaces

B1-? B2-? B3-? B5-? B6-? B7-? B9-? B12-?

B1- thiamine B2- riboflavin B3- niacin B5- pantothenic acid B6- pyridoxal phosphate B7-biotin B9- folic acid B12- cyanocobalamin

Explain the learning (behaviorist) theory of language development

BF Skinner explained language acquisition via operant conditioning. Very young babies are capable of distinguishing between phonemes of all human languages, but by 6 months show a strong preference for phonemes in the language spoken by their parents. Skinner explained language acquisition by reinforcement. Parents repeat and reinforce sounds that sound most like the language spoken by the parents. Thus, over time, the infant perceives that certain sounds have little value and are not reinforces, while other sounds are valuable. Many psycho-linguists point out that this theory cannot fully explain the explosion in vocab that occurs during early childhood.

A child goes to the pediatrician with a sore throat. The pediatrician swabs the throat and places the swab in an acellular culture medium. No pathogens are seen in the culture, even under a light microscope. The pathogen may be a: A. bacterium B. fungus C. virus D. parasitic worm

Bacteria, fungus, and parasitic worms could all be seen under the light microscope. Viruses cannot be seen under light microscopes, and require tissue-culture or cellular growth medium in order to reproduce. A) bacterium, incorrect, Bacteria can be seen with light microscope with a size on the order of microns. B) fungus, incorrect, Fungus and yeast can be seen with the light microscope. C) virus, correct, Virus particles have size on the order of nanometers making them undetectable with the light microscope. D) parasitic worm, incorrect, Parasitic worms are eukaryotes with sizes of many microns up to centimeters in size.

What are cilia and flagella?

Both are extensions of the cell membrane. Cilia are hair-like and flagella resemble a tail. They are composed of microtubules. Cilia are projections from a cell that are involved in the movement of materials along the surface of the cell (ex: cilia line the respiratory tract and are involved in the movement of mucus) Flagella are involved in the movement of the cell itself, such as the movement of sperm cells. Cilia and flagella share the same structure, composed of 9 pairs of microtubules forming an outer ring, with 2 microtubules in the center, known as a 9+2 structure, which is seen only in eukaryotic organelles of motility.

How is breathing regulated?

Breathing requires input from our nervous control center. Ventilation is primarily regulated by a collection of neurons in the medulla oblongata called the ventilation center that fire rhythmically to cause regular contraction of respiratory muscles. These neurons contain chemoreceptors that are primarily sensitive to carbon dioxide [ ]. As the partial pressure of carbon dioxide in the blood rises (hypercarbia or hypercapnia), the respiratory rate will increase so that more carbon dioxide is exhaled, causing carbon dioxide levels in the blood to fall. These cells also respond to changes in oxygen [ ], although this tends to have significance only during periods of significant hypoxemia (low oxygen [ ] in the blood).

A cell releases a substance that diffuses through the environment, resulting in differentiation of a nearby cell. This is an example of what type of cell-cell communication? A. Autocrine B. Juxtacrine C. Paracrine D. Endocrine

C

A child is born with a number of unusual phenotypic features and genetic testing is performed. The child is determined to have partial trisomy 21, with three copies of some segments of DNA from chromosome 21 and partial monosomy 4, with only one copy of some segments of DNA from chromosome 4. Which of the following mutations could have occurred in one of the parental gametes during development to explain both findings? A. Deletion B. Insertion C. Translocation D. Inversion

C

A child speaks in sentences of at least 3 words, but makes grammatical errors including misuse of the past tense. How old is this child likely to be? A. 14 months B. 22 months C. 30 months D. 5 years

C

A nine-year old girl is brought to the pediatrician. Her parents describe that any time she is startled, she appears to collapse and fall asleep. She also complains of waking up in the morning unable to move. Which sleep disorder should be suspected? A. Insomnia B. Sleep deprivation C. Narcolepsy D. Sleep apnea

C

A patient's chart reveals that he has a cardiac output of 7500mL per minute and a stroke volume of 50 mL. What is his pulse, in beats per minute? A. 50 B. 100 C. 150 D. 400

C

Allergic reactions occur due to an overactive immune response to a substance. Which cells within the respiratory tract play the largest role in the generation of allergic reactions? A. Alveolar epithelial cells B. Macrophages C. Mast cells D. Ciliated epithelial cells

C

Herpes simplex virus (HSV) enters the human body and remains dormant in the nervous system until it produces an outbreak after exposure to heat, radiation, or other stimuli. Which of the following statements correctly describes HSV? A. While it remains dormant in the nervous system, the virus is in its lytic cycle B. During an outbreak, the virus is in the lysogenic cycle C. Herpes simplex virus adds its genetic info to the genetic info of the cell D. the herpes simplex virus contains a tail sheath and tail fibers

C

In which layer of the skin can the stem cells of keratinocytes be found? A. Stratum lucidum B. Stratum granulosum C. Stratum basale D. Stratum corneum

C

Increases in which of the following hormones cause sleepiness? A. Cortisol B. Growth hormone C. Melatonin D. Oxytocin

C

Which fo the following shows the correct order of early developmental milestones during embryogenesis? A. blastula --> gastrula --> morula B. morula --> gastrula --> blastula C. morula --> blastula --> gastrula D. gastrula --> blastula --> morula

C

Which of the following best describes the elbow joint in a human? A. Gliding joint B. Ball and socket C. Hinge joint D. Saddle joint

C

Which of the following statements regarding fetal circulation is false? A. In the umbilical cord, there are more arteries than veins B. The foramen ovale is the only shunt that connects two chambers of the heart C. Blood flow in the ductus arterioles is from the aorta to the pulmonary artery D. the ductus venous is the only shunt that bypasses the liver

C

All of the following are associated with the myelin sheath EXCEPT: A. faster conduction of nerve impulses B. nodes of Ranvier forming gaps along the axon C. increased magnitude of the potential difference during an action potential D. saltatory conduction of action potentials

C Action potentials are often described as all or nothing. The magnitude of the potential difference of an action potential is fixed, regardless of the intensity of the stimulus.

Which of the following statements about terpenes is FALSE? A. Terpenes are strongly scented molecules that sometimes serve protective functions. B. Terpenes are steroid precursors C. A triterpene is made of three isoprene moieties and therefore has 15 carbons D. terpenes are made by plants and insects

C A triterpene is made of 6 isoprene moieties(remember 1 terpene unit contains 2 isoprene units) and therefore has a 30-C backbone

Which of the following statements is FALSE? A. ADH increases water reabsorption in the kidney B. Aldosterone indirectly increases water reabsorption in the kidney C. ADH directly acts on the proximal convoluted tubule D. Aldosterone stimulates reabsorption of sodium from the collecting duct

C ADH does not act on the proximal convoluted tubule, but rather on the collecting duct.

Which of the following associations between a hormone and its category is INCORRECT? A. Aldosterone-mineralocorticoid B. Testosterone-cortical sex hormone C. ADH-mineralocorticoid D. Cortisone-glucocorticoid

C ADH is not secreted by the adrenal cortex and is therefore not a -corticoid. Rather, ADH is a peptide hormone produced by the hypothalamus and released by the post pit that promotes water reabsorption.

Which of the following statements is false? A. growth hormone participates in glucose counterregulation B. T4 acts more slowly than T3 C. ATP stores are turned over more than 10,000 times daily D. catecholamines stimulate the sympathetic nervous system

C ATP stores are turned over 1000 times per day

Which of the following is an example of adaptive immunity? A. PRRs recognize that a pathogen is an invasive parasite and eosinophils are recruited to the area. B. Complement is activated, causing osmotic instability in a bacterium C. Memory B-cells generated through vaccination are activated when their antigen is encountered D. Dendritic cells sample bacteria within a laceration and travel to the lymph nodes to present the antigen

C Adaptive immunity involves the activation of B-cells and T-cells specific to the encountered antigen. Any choice that conforms to this paradigm will be correct. C indicates that B cells are activated. Pattern recognition receptors, or PRRs, in A recognize patterns common to certain pathogens, but do not identify the specific pathogen. Complement is an example of a blood-borne nonspecific defense against bacteria, eliminating B. Dendritic cells traveling to the lymph nodes in D are a part of the interaction between the innate and adaptive immune systems, but the dendritic cells themselves are nonspecific.

Which of the following biomolecules is LEAST likely to contain an aromatic ring? A. Proteins B. Purines C. Carbohydrates D. Pyrimidines

C Aromatic rings must contain conjugated pi electrons, which require alternating single and multiple bonds, or lone pairs. In carbohydrate ring structure, only single bonds are present, thus preventing aromaticity. Nucleic acids contain aromatic heterocyclic, while proteins will generally contain at least one aromatic amino acid (tryptophan, phenylalanine, or tyrosine).

While viruses are incredibly simple structures composed of little more than a protein coat and some DNA, some viruses require enzymes to be packaged in with their genetic material. Human immunodeficiency virus (HIV) is an example of a retrovirus. In HIV, which of the following proteins must be packaged in with its nucleic acids in order to reproduce successfully? A. DNA polymerase B. Helicase C. Reverse transcriptase D. None of the above; HIV can utilize its host cell for all of the proteins or enzymes it needs.

C C is correct. A retrovirus is a single-stranded RNA virus that is able to convert its RNA to DNA with the help of the enzyme reverse transcriptase. A. DNA polymerase, incorrect, DNA polymerase is the enzyme that builds the new DNA strand during replication in the nucleus. HIV does not have any particular need for DNA polymerase, and in any case, the host cell contains this enzyme. B. helicase, incorrect, helicase is the enzyme that unwinds the DNA double helix for replication. This choice is incorrect for a similar reason as choice A. C. reverse transcriptase, correct. D. incorrect. While this choice is tempting, note that HIV infects human cells. Human cells typically do not create DNA from RNA, so they would not be expected to contain reverse transcriptase. This enzyme, then, must be provided by HIV itself.

In which part of the cell is cytochrome c located? A. mitochondrial matrix B. outer mitochondrial membrane C. inner mitochondrial membrane D. cytosol

C Cytochrome c carries electrons from CoQH2-cytochrome c oxidoreductase (Complex III0 to cytochrome c oxidase (Complex IV) as part of the electron transport chain. The ETC takes place on the inner mitochondrial membrane

Which of the following was NOT a belief of Darwin's? A. Evolution of species occurs gradually and evenly over time B. There is a struggle for survival among organisms C. Genetic recombination and mutation are the driving forces of evolution. D. Those individuals with fitter variations will survive and reproduce.

C Darwin believed that natural selection is the driving force of evolution

Which of the following enzymes is NOT involved in DNA replication? A. primase B. DNA ligase C. RNA polymerase I D. telomerase

C During DNA replication, the strands are separated by DNA helices. At the replication fork, primate, (A), creates a primer for the initiation of replication, which is filled by DNA polymerase. On the lagging strand, Okazaki fragments form and are joined by DNA ligase, (B). After the chromosome has been processed, the ends, called telomeres, are replicated with the assistance of the enzyme telomerase, (D). RNA polymerase I is located in the nucleolus and synthesizes rRNA.

Which of the following developmental stages has the great nuclear-to-cytoplasmic ratio? A. Eight-cell embryo B. Morula C. Blastula D. Zygote

C During the series of rapid mitotic division known as cleavage, the number of cells increases dramatically but the overall volume of the embryo does not change. In fact, the volume of the embryo does not significantly increase until after the blastula has already implanted. Therefore, a high ratio of nuclear to cytoplasmic material will be found at the stage with the greatest number of cells during early development. From the given choices, the stage with the greatest number of cells is the blastula.

Which of the following would NOT likely lead to elevated levels of bilirubin in the blood? A. Cholangiocarcinoma, a cancer of the bile ducts that can ultimately lead to full occlusion of the duct lumen B. Autoimmune hemolytic anemia, a disease in which the RBCs are attacked by antibodies and are lysed C. Menetrier's disease, in which rugae thicken and overlying glyands lose secretory ability D. Acetominophen (Tylenol) overdose, in which the accumulation of toxic metabolites can cause rapid liver failure

C Elevated bilirubin implies a blockage to bile flow, increased production of bilirubin (from massive hemoglobin release), or an inability of the liver to produce bile. If the bile duct were occluded, as in A, then bile would not be able to flow into the digestive tract and would build up, increasing bilirubin levels in the blood. If many RBCs were lysed as in B then bilirubin levels would rise in accordance with the increased hemoglobin release. If liver failure occurred as in D, then the liver would be unable to produce bile and bilirubin would again build up. C refers to a pathology in the stomach-the key word given here is rugae, which are the folds in the stomach wall. Lack of gastric function would have no effect on bilirubin levels, making this the correct choice.

Which term describes exchange of viral and/or bacterial genetic information from one cell to another via a viral bacteriophage vector? A. transformation B. translocation C. transduction D. transcription

C Exchange of viral and/or bacterial genetic information from one cell to another via a viral bacteriophage vector defines transduction. A. transformation, incorrect, Transformation describes a bacterium acquiring new genetic material from the surrounding environment through the cell membrane. B. translocation, incorrect, Translocation refers to movement of genetic information to a different chromosomal location on a nonhomologous chromosome. C. transduction, correct. D. transcription, incorrect, Transcription describes creation of mRNA from a DNA template.

Fatty acids enter the catabolic pathway in the form of: A. glycerol B. adipose tissue C. acetyl-CoA D. ketone bodies

C Fat molecules stored in adipose tissue can be hydrolyzed by lipases to fatty acids and glycerol. While glycerol cal be converted into glyceraldehydr 3-phosphate, a glycolytic intermediate, a fatty acid must first be activated in the cytoplasm by coupling the fatty acid to CoA-SH, forming fatty acyl-CoA. The fatty acid is then transferred to a molecule of carnitine, which can carry it across the inner mitochondrial membrane. Once inside, the fatty acid is transferred to a mitochondrial CoA-S, re-forming fatty acyl-CoA. Through fatty acid oxidation, this fatty acyl-CoA can become acetyl-CoA, which enters the citric acid cycle.

For a compound to be aromatic, all of the following must be true EXCEPT: A. the molecule is cyclic B. the molecule contains 4n + 2pi electrons C. the molecule contains alternating single and double bonds D. the molecule is planar

C For a compound to be aromatic, it must be cyclic, planar, conjugated, and contain 4n + 2 pi electrons, where n is any integer. Conjugation requires that every atom in the ring have at least one unhybridized p-orbital. While most examples of aromatic compounds have alternating single and double bonds, compounds can be aromatic if they contain triple bonds as well; this would still permit at least one unhybridized p-orbital.

A student is trying to determine the type of membrane transport occurring in a cell. She finds that the molecule to be transported is very large and polar, and when transported across the membrane, no energy is required. Which type of the following is the most likely mechanism of transport? A. Active transport B. Simple diffusion C. Facilitated diffusion D. Exocytosis

C large polar molecule and does not require energy= facilitated diffusion

A young woman presents to the ER with a broken hip. She denies any recent history of trauma to the joint. Blood tests reveal a calcium [ ] 11.5mg/dL (normal 8.4-10.2). Which tissue is likely responsible for these findings? A. Thyroid B. Cartilage C Parathyroid D. Smooth muscle

C Given that she has a high calcium level it is likely that she ha an increased level of bone resorption that is causing her bones to be more fragile. parathyroid hormone causes calcium release from bones. If this woman had an overactive parathyroid gland then it is likely that calcium could still be resorbed form her bones even though her blood calcium levels are already high

Due to kidney disease, a person is losing albumin into the urine. What effect is this likely to have within the capillaries? A. increased oncotic pressure B. increased hydrostatic pressure C. decreased oncotic pressure D. decreased hydrostatic pressure

C In circulation, plasma proteins play an important role in generating osmotic pressure. THis allows water that is displaced at the arterial end of a capillary bed by hydrostatic pressure to be reabsorbed at the venule end. Loss of these plasma proteins would cause a dec in the plasma osmotic pressure

Which of the following processes has the following net reaction? 2 acetyl-CoA + 6NAD+ + 2FAD + 2GDP + 2Pi + 6H2O --> 4CO2 + 6NADH + 2FADH2 + 2GTP + 6H+ + 2CoA-SH A. Pyruvate decarboxylation B. Fermentation C. Tricarboxylic acid cycle D. Electron transport chain

C It is not necessary to have all the net reactions memorized for each metaolic processs to answer this question; all we need is to identify a few ey reactants and products. In this case, we start with acetyl-CoA and end with CoA-SH. We also notice that in this reaction, NAD+ and FAD are reduced to NADH and FADH2, and that CO2 is formed. The only metabolic process in which all of the above reactions would occur is the citric acid cycle, also called the TCA or Krebs cycle.

Which of the following is true regarding ketolysis? A. Ketolysis occurs only in the brain B. Ketolysis occurs in the liver C. Ketolysis generates acetyl-CoA D. Ketolysis increases glucose metabolism

C Ketolysis is the breakdown of ketone bodies to acetyl-CoA for energy. this process occurs in the brain and muscle tissues, but cannot occur in the liver, which lacks an enzyme necessary for ketone body breakdown. Ketolysis is not associated with an increase in glucose metabolism because it most often occurs under conditions of starvation.

Lymphoma is cancer of the cells of lymphoid lineage. These cells often reside within lymph nodes. What type of cell is NOT likely to cause a lymphoma? A/ CD8+ T-cells B. B-cells C. Macrophages D. Th1 cells

C Lymphocytes arise from the lymphoid lineage, which includes B-cells and T-cells. Thus, all types of B- and T- cells are capable of causing lymphoma. Macrophages, however, are not lymphocytes and are not likely to cause lymphoma.

Which of the following statements about saturation is FALSE? A. It can describe the number of double or triple bonds in a fatty acid tail B. It determines at least one of the properties of membranes C. More saturated fatty acids make for a more fluid solution D. Fully saturated fatty acids have only single bonds

C More saturated fatty acids make for a less fluid solution. This is b/c they can pack more tightly and form more noncovalent bonds, resulting in more energy being needed to disrupt the overall structure.

Which of the following correctly describes a difference between nerves and tracts? A. Nerves are seen in the CNS; tracts are seen in the PNS B. Nerves have cell bodies in nuclei; tracts have cell bodies in ganglia C. Nerves may carry more than one type of info; tracts can only carry one type of info D. Nerves contain only one neuron; tracts contain many neurons

C Nerves are collections of neurons in the PNS and may contain multiple types of info (sensory or motor); the contain cell bodies in ganglia. Tracts are collections of neurons in the CNS and only contain one type of info; they contain cell bodies in nuclei

Why might uracil be excluded from DNA but not RNA? A. Uracil is much more difficult to synthesize than thymine B. Uracil binds adenine too strongly for replication C. Cytosine degradation results in uracil D. Uracil is used as a DNA synthesis activator

C One common DNA mutation is the transition from cytosine to uracil in the presence of heat. DNA repair enzymes recognize uracil and correct this error by excising the base and inserting cytosine. RNA exists only transiently in the cell, such that cytosine degradation is insignificant. Were uracil to be used in DNA under normal circumstances, it would be impossible to tell if a base should be uracil or if it is a damages cytosine nucleotide.

Which of the following is true of amphipathic molecules? A. They form protective spheres in any solvent, with hydrophobic molecules interior and hydrophilic molecules exterior B. They have 2 fatty acid chains and a polar head group C. They are important to the formation of the phospholipid bilayer and soap bubbles D. They have a glycerol base

C Phospholipids are amphipathic, as are fatty acid salts. Although amphipathic molecules take spherical forms with hydrophobic molecules interior in aqueous solution, as in (A), the opposite would be true in a nonpolar solvent. (B) describes phospholipids and sphingolipids (D) describes triacylglycerols and phospholipids; both groups do not include fatty acid salts

When fatty acid beta-oxidation predominates in the liver, mitochondrial pyruvate is most likely to be: A. carboxylated to phosphoenolpyruvate for entry into gluconeogenesis B. oxidatively decarboxulated to acetyl-CoA for oxidation in the citric acid cycle C. carboxylated to oxaloacetate for entry into gluconeogenesis D. reduced to lactate in the process of fermentation

C Pyruvate is converted primarily into three main intermediates: acetyl-CoA, B, for the citric acid cycle (via pyruvate dehydrogenase complex); or oxaloacetate, C, for gluconeogenesis (via pyruvate carboxylase). High levels of acetyl-CoA, which is produced during B-oxidation, will inhibit pyruvate dehydrogenase and shift the citric acid cycle to run in the reverse direction, producing oxaloacetate for gluconeogenesis. Acetyl-CoA also stimulates pyruvate carboxylase directly.

In which of the following reactions is the reactant oxidized? A. FAD --> FADH2 B. NAD+ --> NADH C. NADPH --> NADP+ D. ADP --> ATP

C Reduction is gain of electron. Oxidation is loss. In the case of the energy-storing molecules of cellular respiration, the high-potential electrons generally come from hydride ions (H-). Because the question is asking us to determine in which reaction the reactant gets oxidized, our task is to select the equation in which the reactant loses hydride ions. From the given choices, the only one that matches is C. Another way to look at this question is to notice that NADP+ has a +1 charge, which represents an increase from the zero charge of NADPH, implying that an electron was lost in the conversion from NADPH to NADP+

Glucocorticoids have been implicated in stress-related weight gain because: A. they increase appetite and decrease satiety signals B. they increase the activity of catabolic hormones C. the increase glucose levels, which causes insulin secretion D. they interfere with activity of the leptin receptor

C Short-term glucocorticoid exposure causes a release of glucose and the hydrolysis of fats from adipocytes. However, if this glucose is not used for metabolism, it causes an increase in glucose level which promotes fat storage. The net result is the release of glucose from the liver to be converted into lipids in the adipose tissue under insulin stimulation

Which of the following hormones is NOT derived from cholesterol? A. Aldosterone B. Estrogen C. Oxytocin D. Progesterone

C Steroid hormones are derived from cholesterol. Many of their names end with -one, -ol, or -oid and they include the hormones of the adrenal cortex and gonads

A student is volunteering in a hospital with a stroke center. When asked what he believes is the prevalence of stroke among those greater than 65 years old, the student states that it is probably about 40% even though data analysis indicates that it is significantly lower. What accounts for this error? A. Deductive reasoning B. Representativeness heuristic C. Base rate fallacy D. Confirmation bias

C The base rate fallacy occurs when prototypical or stereotypical factors are used for analysis rather than actual data.

Which of the following associations of a primary germ layer and an adult organ is correct? A. Endoderm-cardiac muscle B. Endoderm- lens of the eye C. Ectoderm- fingernails D. Mesoderm- lining of the digestive tract

C The ectoderm gives rise to the integument (the epidermis, hair, nails, and epithelia of the nose, mouth, and anal canal), the lens of the eye, and the nervous system (inc the adrenal medulla) The endoderm gives rise to the epithelial linings of the digestive and respiratory tracts and parts of the liver, pancreas, thyroid, and bladder. The mesoderm gives rise to the musculoskeletal system, the circulatory system, the excretory system, the gonads, and the adrenal cortex.

If a male with hemophilia (XhY) is crossed with a female carrier of both color blindness and hemophilia (XcXh), what is the probability that a female child will be phenotypically normal? A. 0% B. 25% C. 50% D. 100%

C The female in this example is a carrier of two sex-linked traits. If you draw out a punnet square XcXh x XhY you see that 25% of the offspring will be female hemophiliacs and 25% will be female carriers of both alleles. This question is asking what percentage of females will have a normal phenotype, which would be half of the females.

Glucose reabsorption in the nephron occurs in the A. loop of Henle B. distal convoluted tubule C. proximal convoluted tubule D. collecting duct

C The filtrate enters Bowman's capsule and then flows into the proximal convoluted tubule, where virtually all glucose, amino acids, and other important organic molecules are reabsorbed via active transport.

Which of the following statements is true about the hydrolysis of ATP? A. The free energy of ATP hydrolysis is independent of pH B. One mole of creatine phosphate can phosphorylate two moles of ADP C. The free energy of hydrolysis of ATP is nearly the same as for ADP D. ATP yields cyclic AMP after two hydrolysis reactions

C The hydrolysis of ATP is energetically favorable because there are repulsive negative charges that are relieved when hydrolyzed, and the new compounds are stabilized by resonance. This is true of both ATP and ADP. Some of the other answer choices are tempting though. In A, ATP hydrolysis relies on pH because a protonated ATP molecules contains less negative charge and therefore experiences less repulsive force. For B, the energy released by one mole of creatine phosphate upon hydrolysis is not sufficient to phosphorylate two moles of ADP; creatine phosphate donates one phosphate group to a molecules of ADP, so one mole of creatine phosphate will phosphorulate one mole of ADP. For D, the removal of two phoaphate groups from ATP yields AMP, not cyclic AMP.

Which amino acids contribute most significantly to the pI of a protein? I. Lysine II. Glycine III. Arginine

C The overall pI of a protein is determined by the relative number of acidic and basic amino acids. The basic amino acids are arginine, lysine, and histidine and the acidic amino acids are aspartic acid and glutamic acid. Glycine's side chain is a hydrogen atom, so it will have the least contribution of all the amino acids

Each of the following statements regarding the anatomy of the respiratory system is true EXCEPT: A. the epiglottis covers the glottis during swallowing to ensure that food does not enter the trachea B. the trachea and bronchi are lined by ciliated epithelial cells C. the pharynx contains two vocal cords, which are controlled by skeletal muscle and cartilage D. the nares are lined with vibrissae, which help filter out particulate matter from inhaled air

C The pharynx, located behind the nasal cavity and oral cavity, is a common pathway for food entering the digestive system and air entering the respiratory system. It is the larynx that contains the vocal cords, not the pharynx.

Which of the following best characterizes the process of fatty acid synthesis? A. Two reductions followed by a dehydration and bond formation B. Reduction followed by activation, bond formation, dehydration, and reduction C. Activation followed by bond formation, reduction, dehydration, and reduction D. Activation followed by bond formation, oxidation, dehydration, and reduction

C The steps in fatty acid synthesis are activation (attachment to acyl carrier protein), bond formation (between malonyl-CoA and the growing fatty acid chain), reduction (of a car-CoA) and the growing fatty acid chain, reduction (of a carboxyl group), dehydration, and reduction (of a double bond)

A laceration cuts down into a layer of loose connective tissue in the skin. Which layer of the skin is this? A. Stratum corneum B. Stratum lucidum C. Papillary layer D. Reticular layer

C The stratum corneum and stratum lucidum contain dead keratinocytes while the reticular layer consists of dense connective tissue

Which of the following is a FALSE statement regarding erythrocytes? A. Erythrocytes contain hemoglobin B. Erythrocytes are anaerobic C. The nuclei of erythrocytes are located in the middle of the biconcave disc D. Erythrocytes are phagocytized in the spleen and liver after a certain period of time

C erythrocytes, or RBCs are produced in the red bone marrow and circulate in the blood for about 120 days, after which they are phagocytized in the spleen and the liver. RBCs have a disk-like shape and lose their membranous organelles (like mitochondria and nuclei) during maturation. Erythrocytes are filled with hemoglobin; their lack of mitochondria makes their metabolism solely anaerobic

Which of the following best explains why cytosolic NADH can yield potentially less ATP than miochondrial NADH? A. Cytosolic NADH always loses energy when transferring electrons B. Once NADH enters the matrix from the cytosol, it becomes FADH2 C. Electron transfer from cytosol to matrix can take more than one pathway D. There is more energy cost for bringing cytosolic NADH into the matrix.

C The wording of these answer choices is critical. The electrons from cytosolic NADH can enter the mitochondrion through on e of two shuttle mechanisms: the glycerol 3-phosphate shuttle, which ultimately moves these electrons to mitochondrial FAD, and the malate-aspartate shuttle, which ultimately moves these electrons to mitochondrial NAD+. If the electrons are transferred using the malate-aspartate shuttle, then no energy is lost, making A and D incorrect. NADH cannot enter the matrix directly, making B incorrect. It is the fact that electrons can use more than one pathway- one of which loses energy that could be used for ATP synthesis- that accounts for the potentially decreased yield of ATP from cytosolic NADH.

Color blindness (B) is a recessive (b) sex-linked disorder that results from a mutation on the X chromosome. What would be the most likely phenotypes of the male and female progeny when a colorblind man and a woman with no family history of color blindness mate? A. All of the male and none of the female offspring will be colorblind B. Half of the male and half of the female offspring will be colorblind C. None of the male and none of the female offspring will be colorblind D. None of the male and all of the female offspring will be colorblind

C This question asks the examinee to determine the offspring phenotypes that result from cross between a woman with normal vision and a colorblind man. To answer this question, the easiest method is requires the use of a Punnett square. Since the woman has no family history of colorblindness, it is safe to assume that she is homozygous dominant (XBXB). The colorblind male on the other hand has the mutated allele (XbY). Refer to Punnet square question of the day August 1 2019 According to the Punnett square above, none of the males and none of the females will be affected by this recessive disorder. Therefore, none of the male and none of the female offspring will be colorblind making C the correct answer.

Why are triacylglycerols used in the human body for energy storage? A. They are highly hydrated and therefore can store lots of energy B. They always have short fatty acid chains for easy access by metabolic enzymes C. The carbon atoms of the fatty acid chains are highly reduced and therefore yield more energy upon oxidation D. Polysaccharides, which would actually be a better energy storage form, would dissolve in the body

C Triacylglycerols are highly hydrophobic and therefore not highly hydrated. The fatty acid chains produce 2x as much energy as polysaccharides during oxidation b/c they are highly reduced. The fatty acid chains vary in length and saturation

In the genetic code of human nuclear DNA, one of the codons specifying the amino acid tyrosine is UAC. If one nucleotide is changed, and the codon is mutated to UAG, what type of mutation will occur? A. Silent mutation B. Missense mutation C. Nonsense mutation D. Frameshift mutation

C UAG is a stop codon, so it would be a nonsense(truncation) mutation

Which of the following is correct about fat-soluble vitamins? I. Vitamin E is important for calcium regulation II. Vitamin D protects against cancer b/c it's a biological antioxidant III. Vitamin K is necessary for the post translational introduction of calcium-binding sites IV. Vitamin A is metabolized to retinal, which is important for sight. A. III only B. I and II only C. III and IV only D. II, III, and IV only

C Vitamin A is metabolized to retinal, which is important for sight. Vitamin D is metabolized to calcitriol, which is important for calcium regulation Vitamin E is made up of tocopherols, which are biological antioxidants Vitamin K is necessary for the introduction of calcium binding sites, such as during the posttranslational modification of prothrombin

Which of the following is the correct sequence of a cardiac impulse? A. SA node--> AV node --> Purkinje fibers --> bundle of HIS ---> ventricles B. AV node --> bundle of His --> Purkinje fibers --> ventricles --> atria C. SA node --> atria --> AV node --> bundle of His --> purkinje fibers --> ventricles D. SA node --> AV node --> atria --> bundle of His --> Purkinje fibers --> ventricles

C an ordinary cardiac contraction originates in, and is regulated by, the SA node. The impulse travels through both atria, stimulating them to contract simultaneously. The impulse then arrives at the AV node, which momentarily slows conduction, allowing for completion of atrial contraction and ventricular filling. The impulse is then carried by the bundle of His and its branches through the Purkinje fibers in the walls of both ventricles, generating a strong contraction

A child is born with an imperforate anus, in which the anal canal fails to form correctly and the rectum is not connected to the outside world. This pathology is most likely accounted for by a failure of: A. cell differentiation B. cell determination C. apoptosis D. neurulation

C apoptosis should occur in the digestive tract, where the central lumen is formed

Oxytocin and ADH are: A. peptide hormones produced and release by the pituitary B. steroid hormones produced and release by the pituitary C. peptide hormones produced by the hypothalamus and released by the pituitary D. steroid hormones produced by the hypothalamus and released by the pituitary

C both end with the suffix, -in, which should hint that they are peptide hormones

Which of the following is involved in the body's primary blood-buffering mechanism? A. fluid intake B. absorption of nutrients in the gastrointestinal system C. carbon dioxide produced from metabolism D. hormones released by the kidney

C carbon dioxide is a byproduct of metabolism in cells that later combines with water to form bicarbonate in a reation catalyzed by carbonic anhydrase. T While the kidney can be involved in acid-base balance, it carries out this function through its filtration, secretion, and reabsorption mechanisms, not through hormone release

Excessive levels of dopamine in the brain are associated with psychosis. Accordingly, many antipsychotic medications block dopamine receptors. Which of the following effects may be seen in an individual taking antipsychotics? A. Inc secretion of growth hormone B. Dec secretion of growth hormone C. Inc secretion of prolactin D. Dec secretion of prolactin

C dopamine inhibits prolactin. if the medication inhibits dopamine, then that could increase prolactin secretion

An individual who is phenotypically female is found to have only one copy of a disease carrying recessive allele on the X chromosome, yet she demonstrates all the classic symptoms of the disease. Geneticists determine she has a genotype that likely arose form nondisjunction in one of her parents. What is the likely genotype of this individual? A. 46, XX (46 chromosomes, with XX for sex chromosomes) B. 46 XY C. 45, X D. 47, XXY

C nondisjunction refers to the incorrect segregation of homologous chromosomes during anaphase I or of sister chromatids during anaphase II. In either case, one daughter cell ends up with 2 copies of related genetic material, while the other receives zero.

Which of the following is LEAST likely to be the resting membrane potential of a cell? A. -70 mV B. -55 mV C. 0 mV D. +35 mV

C signaling molecules and channels would not be as useful with a membrane potential of zero.

Language consists of multiple components. Which of the following involves the order in which words are put together? A. Phonology B. Semantics C. Syntax D. Pragmatics

C syntax is how words are put together

An x-ray of the right femur in a child shows that it is shorter than the opposite femur, and below the average length for a child of this age. Which region of the bone is most likely to have caused this abnormality? A. Diaphysis B. Metaphysis C. Epiphysis D. Periosteum

C the epiphyseal plate is the site of longitudinal growth and the growth plates.

Which of the following is true regarding arteries and veins? A. Arteries are thin-walled, muscular, and elastic, whereas veins are thick-walled and inelastic B. Arteries always conduct oxygenated blood, whereas veins always carry deoxygenated blood C. The BP in the aorta is always higher than the BP in the superior vena cava D. Arteries facilitate blood transport by using skeletal muscle contractions, whereas veins make use of the pumping of the heart to push blood

C the pressure in the aorta usually ranges between 120 and 80 mmHg, whereas the pressure in the superior vena cava is near zero. A is incorrect because arteries are thick walled and veins are thin walled B is incorrect because this relationship is reversed in pulmonary circulation D is reversed; arteries make use of pumping

Val-tRNA^val is the tRNA that carries valine to the ribosome during translation. Which of the following sequences gives an appropriate anticodon for this tRNA? A. CAU B. AUC C. UAC D. GUG

C there are 4 different codons for valine: GUU, GUC, GUA, and GUG. Through base-pairing, we can determine that the proper anticodon must end with AC. Remember that the codon and anticodon are antiparallel to each other and that nucleic acids are always written 5'-->3' on the MCAT. Therefore we are looking for an answer that ends with AC rather than starting with CA

A researcher treats a solution containing animal cells with ouabain, a poisonous substance that interferes with the sodium-potassium ATPase embedded in the cell membrane, and the cell lyses as a result. Which of the following statements best describe ouabians effect? A. Treatment with ouabain result in high levels of extracellular calcium B. Treatment with ouabain results in high levels of extracellular potassium and sodium C. Treatment with ouabain increases intracellular concentrations of sodium D. Treatment with ouabain decreases intracellular concentrations of sodium.

C this is an osmosis and sodium-potassium pump question. When a cell is placed in a hypertonic solution (a solution having a higher solute [ ] than the cell), fluid will diffuse out of the cell and result in cell shrinkage. When a cell is placed in hypotonic solution (a solution having a lower solute [ ] than the cell), fluid will diffuse from the solution into the cell, causing the cell to expand and possibly lyse. The sodium-potassium pump moves 3 sodium ions out of the cell for every two potassium ions it lets into the cell. Therefore, inhibition of the sodium-potassium pump by ouabain will cause a net increase in the sodium [ ] inside the cell and water will diffuse in, causing the cell to swell and lyse

Total lung capacity is equal to the vital capacity plus the: A. tidal volume B. expiratory reserve volume C. residual volume D. inspiratory reserve volume

C vital capacity (the max volume of air that can be forcibly inhaled and exhaled from the lungs) residual volume (the air that always remains in the lungs, preventing the alveoli from collapsing)

In Alzheimer's disease, a protein called the amyloid precursor protein (APP) is cleaved to form a protein called B-amyloid. This protein has a B-pleated sheet structure and precipitates to form plaques in the brain. This mechanism of disease is most similar to which of the following pathogens? A. Bacteria B. Viruses C. Prions D. Viroids

C. Prions are infectious proteins that cause misfolding of other proteins. Prions generally cause a shift towards B-pleated sheet conformations, causing decreased solubility and inc resistance to degradation.

Which of the following proteins is most likely to be found extracellularly? A. Tubulin B. Myosin C. Collagen D. Actin

C. tubulin and actin are primarily cytoskeletal proteins, while myosin is a motor protein

Certain ovarian tumors called granulosa cell tumors are known to produce excessive levels of estrogen. A physician who diagnoses a granulosa cell tumor should look for a secondary cancer in which of the following parts of the reproductive tract? A. Fallopian tube B. Cervix C. Endometrium D. Vagina

C. estrogen is known to cause growth of the endometrial lining during the follicular phase of the menstrual cycle, and its levels stay high during the luteal phase to promote vascularization and glandularization of this tissue. Excessive levels of estrogen may provide a strong enough signal for cell growth to promote tumor formation or even cancer.

Which of the following is not a function of the smooth ER? A. lipid synthesis B. poison detoxification C. protein synthesis D. transport of proteins

C. occurs in ribosomes and rough ER

What are CD4+ T cells better at doing as opposed to CD8+ T cells? What are the other names for both of these cells? What do they both respond to, respectively?

CD4+ T cells are better at fighting extracellular infections while CD8+ T cells are better at targeting intracellular infections. Mnemonic: CD4+ cells (helper T cells) respond to MHC-II (4x2=8) CD8+ cells (cytotoxic T cells) respond to MHC-1(8x1=8)

How does carbon dioxide move in the body? Explain this phenomenon.

CO2 is nonpolar and therefore has low solubility in the aqueous plasma. CO2 can be carried by hemoglobin, but hemoglobin has a much higher affinity for O2. The vast majority of CO2 exists in the blood as the bicarbonate ion (HCO3-). When CO2 enters a red blood cell, it encounters the enzyme carbonic anhydrase, which catalyzes the combination reaction between carbon dioxide and water to form carbonic acid (H2CO3). Carbonic acid, a weak acid, will dissociated into a proton and the bicarbonate anion. The hydrogen ion (proton) and bicarbonate ion both have high solubilities in water, making them a more effective method of transporting metabolic waste products to the lungs for excretion. Upon reaching the alveolar capillaries in the lungs, the same reactions that led to the formation of the proton and bicarbonate anion can be reversed, allowing us to breathe out CO2. This chemical reaction is important, not only because it provides an effective means of ridding the body's tissues of CO2 gas, but also because the [ ] of free proteons in the blood affects pH; the pH, in turn, can have allosteric effects on the oxygemoglobin dissociation curve. Inc CO2 production will cause a right shift in the bicarbonate buffer equation, resulting in inc [H+] (dec pH). These protons can bind to hemoglobin, reducing hemoglobins affinity for O2. This dec affinity can e seen in the oxyhemoglobin curge as a shift to the right; this is known as the Bohr effect. Note that triggers for this right shift (inc plasma CO2, inc H+, dec pH) are often associated with oxygen demand; high er rates of cellular metabolism result in inc carbon dioxide production and accumulation of lactic acid, both of which dec pH. This dec affinity allows more O2 to be unloaded at the tissues.

Explain the Bohr effect.

CO2(g) + H2O(l) <----------> H2CO3(aq) <-----> H+(aq) + HCO3-(aq) carbonic anhydrase This chemical reaction is important, not only because it provides an effective means of ridding the body's tissues of CO2 gas, but also because the [ ] of free proteons in the blood affects pH; the pH, in turn, can have allosteric effects on the oxygemoglobin dissociation curve. Inc CO2 production will cause a right shift in the bicarbonate buffer equation, resulting in inc [H+] (dec pH). These protons can bind to hemoglobin, reducing hemoglobins affinity for O2. This dec affinity can e seen in the oxyhemoglobin curge as a shift to the right; this is known as the Bohr effect. Note that triggers for this right shift (inc plasma CO2, inc H+, dec pH) are often associated with oxygen demand; high er rates of cellular metabolism result in inc carbon dioxide production and accumulation of lactic acid, both of which dec pH. This dec affinity allows more O2 to be unloaded at the tissues.

What are the 5 bases commonly found in nucleic acids? sort them into structures and pyrimidines vs purines.

CUT the PYe cytosine, uracil, and thymine are pyrimidines -a pie has one ring of crust and pyrimidines have only one ring in their structure PURe As Gold adenine and guanine are purines -think of gold wedding rings. it takes 2 gold rings at a wedding, just like purines have 2 rings in their structure

What are the four major subclasses of sphingolipids? Explain them.

Ceramide- has a single hydrogen atom as its head group Sphingomyelins- sphingolipids that are also phospholipids. Have either phosphatidylcholine or phosphatidylethanolamine as a head group, and thus contain a phosphodiester bond. - Sphingomyelin head groups have no net charge. -They are major components in the plasma membranes of cells producing myelin (oligodendrocytes and Schwann cells), the insulating sheath for axons Sphingolipids- head groups composed of sugars bonded by glycosidic linkages are considered glycolipids, more specifically glycosphingolipids. -These molecules are not phospholipids because they contain no phosphodiester linkage. -Glycosphingolipids are found mainly on the outer surface of the plasma membrane. Can be classified further as cerebrosides (have a single sugar) or globosides (have two or more sugars). -These molecules are also referred to as neutral glycolipids because they have no net charge at physiological pH Gangliosides-the most complex sphingolipids. Glycolipids that have polar head groups composed of oligosaccharides with one or more N-acetylneuraminic acid (NANA, aka sialic acid) molecules at the terminus and a negative charge. -These molecules are also considered glycolipids because they have a glycosidic linkage and no phosphate group. -Play a major role in cell interaction, recognition, and signal transduction.

what is genetic drift

Changes in allele frequencies that are due to chance, causes the loss of genetic diversity in a population

Explain heterochromatin and euchromatin?

Chromosomes have a diffuse configuration during interphase of the cell cycle. The cell will undergo DNA replication during the S phase of interphase and having the DNA uncondensed and accessible makes the process more efficient. A small percentage of the chromatin remains compacted during interphase and is referred to as heterochromatin. Heterochromatin appears dark under light microscopy and is transcriptionally silent. Heterochromatin often consists of DNA with highly repetitive sequences. In contrast, the dispersed chromatin is called euchromatin, which appears light under light microscopy. Euchromatin contains genetically active DNA.

Explain Complex III of the electron transport chain

CoQH2-cytochrome c oxidoreductase CoQH2 + 2 cytochrome c [with Fe3+] --> CoQ + 2 cytochrome c [with Fe2+] + 2H+ Also called cytochrome reductase, this complex facilitates the transfer of electrons from coenzyme Q to cytochrome c in a few steps. Though Complex III is drawn as two separate complexes in Figure 10.13 in book to illustrate the sequential reactions that occur within the complex, both of these steps are occurring within the same complex, using the same Coenzyme Q. The overall reaction is written above. . The following steps involve the oxidation and reduction of cytochromes: proteins with heme groups in which iron is reduced to Fe2+ and reoxidized to Fe3+. In the transfer of electrons from iron, only one electron is transferred per reaction, but because coenzyme Q has two electrons to transfer, two cytochrome c molecules will be needed. Complex III's main contribution to proton-motive force is via the Q cycle .In the Q cycle, two electrons are shuttled from a molecule of ubiquinol )CoQH2) near the intermembrane space to a molecule of ubiquinone (CoQ) near the mitochondrial matrix. Another two electrons are attached to heme moieties, reducing two molecules of cytochrome c. A carrier containing iron and sulfur assists this process. In shuttling these electrons, four protons are also displaced to the intermembrane space; therefore, the Q cycle continues to increase the gradient of the proton-motive force across the inner mitochondrial membrane.

What is the enteric nervous system? What does it do? How does it function in relation to the brain and spinal cord and how is it regulated?

Collection of one hundred million neurons that govern the function of the gastrointestinal system. These neurons are present in the walls of the digestive tract and trigger peristalsis, or rhythmic contractions of the gut tube, in order to move materials through the system. This system can function independently of the brain and spinal cord, although it is heavily regulated by the autonomic nervous system. The parasympathetic division is involved in stimulation of digestive activities, increasing secretions from exocrine glands and promoting peristalsis. The sympathetic division is involved in the inhibition of these activities. The fact that we feel sleep and lethargic after eating a big meal (food coma) is due in part to parasympathetic activity.

A cancer cell is removed from a patient and cultured. The cells in. this culture seem to be able to divide indefinitely with no cellular senescence. Which protein is likely activated in these cells that accounts for this characteristic? A. Epidermal growth factor B. Sonic hedgehog C. Transforming growth factor beta D. Telomerase

D

A certain cooperative enzyme has four subunits, two of which are bound to substrate. Which of the following statements can be made? A. The affinity of the enzyme for the substrate has increased B. The affinity of the enzyme for the substrate has just decreased C. The affinity of the enzyme for the substrate is at the average for this enzyme class D. The affinity of the enzyme for the substrate is greater than with one substrate bound

D

A pregnant woman is accidentally given a single does of a teratogenic drug late in the third trimester. The baby is born 3 days later. Which of the following is the most likely outcome? A. Compete failure of organ development and death of the fetus B. Partial failure of organ development with survival of the fetus C. Serious disfigurement of the fetus D. Respiratory distress at birth, but no long-term effects

D

An unconscious patient is rushed into the emergency room and needs an immediate blood transfusion. Because there is no time to check her medical history to determine her blood type, which type of blood should she receive? A. AB+ B. AB- C. O+ D. O-

D

Red bone marrow is involved in erythrocyte formation. In contrast, yellow bone marrow: A. is involved in leukocyte formation B. is responsible for drainage of lymph C. causes the formation of spicules D. contains predominantly adipose tissue

D

The influence of a specific group of cells on the differentiation of another group of cells is called: A. competence B. senescence C. determination D. induction

D

Which of the following does NOT likely contribute to genetic variability? A. random fertilization of an egg by a sperm B. random segregation of homologous chromosomes C. crossing over between homologous chromosomes during meiosis D. replication of the DNA during S stage

D

Which of the following organs does NOT require a constant supply of glucose from the blood for energy during a gast? A. RBCs B. Brain C. Pancreas D. Liver

D The liver, like all cells, needs a constant supply of glucose; however, it is able to produce its own glucose through gluconeogenesis (cells in the kidney can also complete low levels of gluconeogenesis). The other cells listed here are absolutely dependent on a glucose source from the blood for energy, although they may also use other fuels in addition to glucose. For example, the brain can utilize ketone bodies during lengthy periods of starvation; however, it still requires at least some glucose for proper function.

Which of the following is NOT a component of all trimeric G proteins? A. G (sub alpha) B. G(sub beta) C. G(sub gamma) D. G(sub i)

D all trimeric G proteins have alpha, beta, and gamma subunits. Gs, Gi, and Gq are subtypes of the Alpha subunit of the trimeric G protein.

A person suffering from severe dehydration and starvation would NOT be expected to have elevated plasma concentrations for which of the following hormones? A. ADH B. Cortisol C. Aldosterone D. Insulin

D This question asks you to determine the body's physiological response to the extreme conditions of dehydration and starvation. In such cases, the person would be expected to have low blood pressure and low nutrients (i.e. proteins, carbohydrates, and lipids). As a result, the body will respond by producing hormones that increase blood pressure and mobilize stored nutrients (glycogen, fat, etc.). ADH and aldosterone both increase blood pressure so they would be expected to appear in high plasma concentrations in response to the low blood pressure, eliminating choices A and C. Cortisol would be expected in high plasma to increase carbohydrate, protein, and fat availability, making B false. That leaves answer choice D. Insulin lowers blood glucose levels, which is expected to appear in the blood after a sugary meal. In the case of starvation, the body would respond by increasing blood glucose levels via the production of glucagon

In the nephron, amino acids enter the vasa recta via the process of: A. filtration B. secretion C. excretion D. reabsorption

D Essential substances, such as glucose, salts, amino acids, and water, are reabsorbed from the filtrate and returned to the blood in the vasa recta. In general, reabsorption refers to the movement of solutes from the filtrate back into the blood.

Which of the following hormones directly stimulates a target tissue that is NOT an endocrine organ? A. ACTH B. TSH C. LH D. GH

D A hormone that directly stimulates a non-endocrine target tissue is referred to as a direct hormone. The direct hormones secreted by the ant pit are prolactin, endorphins, and GH. All of the other choices are tropic hormones.

After a large, well-balanced meal, all of the following substances would be expected to be elevated EXCEPT: A. Fatty acid B. Insulin C. Glucose D. Glucagon

D After a large meal, one would expect blood to contain high levels of nutrients, such as glucose, C, and fatty acids, A, as well as regulators telling the body to utilize and store this fuel, like insulin, B. Glucagon is a peptide hormone used to raise blood sugar levels by promoting, among other processes, glycogenolysis and gluconeogenesis. Glucagon should be elevated during a fast.

Which of the following statements regarding differences between DNA and RNA is FALSE? A. In cells, DNA is double-stranded, whereas RNA is single-stranded B. DNA uses the nitrogenous base thymine; RNA uses uracil C. The sugar in DNA is deoxyribose; the sugar in RNA is ribose D. DNA strands replicate in a 5' to 3' direction, whereas RNA is synthesized in a 3' to 5' direction

D Because we are looking for the false statement, we have to read each choice to eliminate those that are true or find one that is overtly false. Let's quickly review the main differences between DNA and RNA. In cells, DNA is double-stranded, with a deoxyribose sugar and the nitrogenous bases, A, T, C, and G. RNA on the other hand, is usually single-stranded, with a ribose sugar and the bases A, U, C, and G. (D) is false because both DNA replication and RNA synthesis proceed in a 5' to 3' direction

Which of the following is FALSE regarding aldosterone regulation? A. Renin converts the plasma protein angiotensiongen to angiotensin I B. Angiontensin II stimulates the adrenal cortex to secrete aldosterone C. Angiotensin I is converted to angiotensin II by angiotensin-converting enzyme D. A decrease in blood glucose [ ]s stimulates renin production

D D describes the stimulus for erythropoietin secretion

Which of the following is NOT a difference between heterochromatin and euchromatin? A. Euchromatin has areas that can be transcribed, whereas heterochromatin is silent B. Heterochromatin is tightly packed, whereas euchromatin is less dense C. Heterochromatin stains darkly, whereas euchromatin stains lightly D. Heterochromatin is found in the. nucleus, whereas euchromatin is in the cytoplasm

D Euchromatin has a classic "beads on a string" appearance that stains lightly, while heterochromatin is tightly packed and stains darkly. Heterochromatin is primarily composed of inactive genes or untranslated regions, while euchromatin is able to be expressed. Al chromatin is found in the nucleus, not the cytoplasm.

Which of the following associations correctly pairs a stage of respiration with the muscle actions occurring during that stage? A. Inhalation-Diaphragm relaxes B. inhalation-internal intercostal muscles relax C. exhalation-diaphragm contracts D. exhalation-external intercostal muscles relax

D The muscles involved in ventilation are the diaphragm, which separates the thoracic cavity from the abdominal cavity, and the intercostal muscles. During inhalation, the diaphragm contracts and flattens, while the external intercostal muscles contract, pulling the rib cage up and out. These actions cause an overall increase in the volume of the thoracic cavity. During exhalation, both the diaphragm and the external intercostals relax, causing a decrease in the volume of the thoracic cavity because of the recoil of these tissues. In forced exhalation, the internal intercostals and abdominal muscles may contract to force out air.

Which of the following is a sphingolipid? A. Lecithin B. Phosphatidyllinositol C. Cholesterol D. Ganglioside

D gangliosides, eramide, sphingomyelin, and cerebrosides are sphingolipids

Aldosterone acts on the kidney by: A. increasing water permeability of the collecting duct B. increasing water permeability of the ascending limb of the loop of Henle C. increasing water permeability in the proximal tubule D. increasing sodium absorption and potassium secretion in the distal tubule

D Filtrate pressure in the distal tubule is monitored by the juxtaglomerular apparatus. Special cells in the juxtaglomerular apparatus secrete an enzyme called renin either when stretch receptors in vessel walls detect a blood pressure drop (decreased renal perfusion) or when macula densa cells in the distal convoluted tubule detect a drop of sodium concentration in the tubular filtrate. Renin release triggers a cascade of angiotensin I, II, and III which in turn causes the adrenal cortex to secrete aldosterone. Aldosterone acts on the collecting tubule of the distal tubule (not the collecting duct) to increase membrane proteins absorbing sodium and secreting potassium. For the MCAT: decreased [Na+]filtrate or decreased blood pressure -> juxtaglomerular apparatus releases renin -> angiotensin I,II,III cascade -> aldosterone -> increased distal tubule Na+ absorption from and K+ secretion into the filtrate. Increased blood pressure is a secondary effect of the aldosterone. A. Increasing water permeability of the collecting duct, incorrect, Antidiuretic Hormone (also known as ADH and vasopressin) causes increased water permeability in the collecting duct. B. Increasing water permeability of the ascending limb of the loop of Henle, incorrect, The ascending limb of the loop of Henle is nearly impermeable to water. C. Increasing water permeability in the proximal tubule, incorrect, Water reabsorption occurs in the descending limb of the loop of Henle, the collecting tubule, and the collecting duct. D. Increasing sodium absorption and potassium secretion in the distal tubule, correct.

In a nonevolving population, there are two alleles, R and r, which code for the same trait. The frequency of R is 30%. What are the frequencies of all the possible genotypes? A. 49% RR, 42% Rr, 9% rr B. 30% RR, 21% Rr, 49% rr C. 0.09% RR, 0.42% Rr, 0.49% rr D. 9% RR, 42% Rr, 49% rr

D Frequency of R=30%, as such, p=0.30. The frequency of the recessive gene is r = 100%-30%=70%; this q=0.7. The frequency of the genotypes, according to the Hardy-Weinberg equilibrium are given by p^2=RR, 2pq=Rr, and q^2=rr. Threfore, the frequency of the genotypes are (0.3)^2=0.09 = 9% RR, 2 x 0.3 x 0.7 = 0.42 = 42% Rr, and (0.7)^2 = 0.49=49%rr

When insulin is released, it acts to increase the absorption of glucose into skeletal muscle predominantly through which of the following transporters? A. GLUT 1 B. GLUT 2 C. GLUT 3 D. GLUT 4

D GLUT is an abbreviation for glucose transporter and describes a family of sugar transporters with varying distributions and activities. GLUT 4 is found in adipose tissue and muscle, and mediates insulin-stimulated glucose uptake; in fact, it is the only insulin-responsive glucose transporter. Insulin acts via its receptor to translocate GLUT 4 to the plasma membrane. GLUT 4 in skeletal muscle is also stimulated by exercise through an insulin-independent pathway.

All of the following facilitate gas exchange in the lungs EXCEPT: A. thin alveolar walls B. multiple subdivisions of the respiratory tree C. differences in the partial pressures of O2 and CO2 D. active transporters in alveolar cells

D Gas exchange in the lungs relies on passive diffusion of oxygen and carbon dioxide. This is accomplished easily b/c there is always a difference in the partial pressures of these two gases and because the subdivision of the respiratory tree creates a large surface area of interaction between the alveoli and the circulatory system. In addition, the thin alveolar walls allow for fast diffusion and gas exchange. Active transport is not used in the gas exchange process in the lungs

Some forms of pneumonia cause an excess of fluids such as mucus or pus to build up within an entire lobe of the lung. How will this affect the diffusion of gases within the affected area? A. CO2 can diffuse out but O2 can't enter the blood B. O2 can diffuse into the blood but CO2 can't diffuse out C. No change in diffusion will occur D. No diffusion will occur in the affected area

D If an area of the lung becomes filled with mucus and inflammatory cells, the area will not be able to participate in gas exchange.

Although linked at the same carbons as starch and glycogen, cellulose is indigestible to humans. This is a result of the: A. alpha-linkage from carbons 2 to 6 of D-glucose. B. beta-linkage at carbons 2 to 6 of D-glucose. C. alpha-linkage at carbons 1 to 4 of D-glucose. D. beta-linkage at carbons 1 to 4 of D-glucose.

D In humans, glycogen is broken down by debranching enzyme (alpha 1,6 linkages) and phosphorylases (alpha 1,4 linkages). Starch is broken down at alpha 1,4 linkages by alpha-amylase found in saliva and pancreatic secretions. Cellulose cannot be broken down by humans because it is beta-linked at carbons 1 to 4. For the MCAT: Alpha-1,4 linked glucose indicates digestible glycogen and starch, and beta-1,4 linkage indicates indigestible cellulose.

After a brief period of intense exercise, the activity of muscle pyruvate dehydrogenase is greatly increased. This increased activity is most likely due to: A. decreased ADP. B. increased acetyl-CoA. C. increased NADH/NAD+ ratio. D. increased pyruvate concentration.

D In most biochemical pathways, only a few enzymatic reactions are under regulatory control. These often occur either at the beginning of pathways or at pathway branch points. The pyruvate dehydrogenase (PDH) complex controls the link between glycolysis and the citric acid cycle, and decarboxylates pyruvate (the end product of glycolysis) with production of NADH and acetyl-CoA (the substrate for the citric acid cycle). After intense exercise, one would expect PDH to be highly active to generate ATP. ADP levels, A, should be high because ATP was just bured by the muscle. Acetyl-CoA, B, is an inhibitor of PDH, causing a shift of pyruvate into the gluconeogenic pathway. A high NADH/NAD+ ratio, C, would imply that the cell is already energetically satisfied and not in need of energy, which would not be expected in intensely exercising muscle.

Suppose that in a mammalian species, the allele for black hair (B) is dominant to the allele for brown hair (b), and the allele for curly hair (C) is dominant to the allele for straight hair (c). When an organism of unknown genotype is crossed with one straight, brown hair, the phenotypic ratio is as follows: I. 25% curly black hair II. 25% straight black hair III. 25% curly brown hair IV. 25% straight brown hair What is the genotype of the unknown parent? A. BbCC B. bbCc C. Bbcc D> BbCc

D In this dihybrid problem, a doubly recessive individual is crossed with an individual of unknown genotype; this is known as a test cross. The fact that both the dominant and recessive traits are present in the offspring means that the unknown parental genotype must contain both dominant and recessive alleles for each trait. The unknown parental genotypes must be BbCc

The snapdragon is a plant demonstrating incomplete dominance in the flower color phenotype. If homozygous snapdragons have either red or white flowers, what color are the heterozygotes? A. red, because the white coloring reflects all incoming light wavelengths in flowers expressing both red and white alleles B. some flowers red and some white, because only one gene can be expressed at a time C. white, because the white coloring will reflect all incoming light wavelengths in flowers expressing both red and white alleles D. pink, because it is in-between red and white

D Incomplete dominance, or partial dominance, refers to a heterozygous phenotype that is in-between the two homozygous phenotypes. One of the most common examples is snapdragon flowers, which may have one allele for white flowers and one allele for red flowers expressed as pink flowers in heterozygotes. If two heterozygotes are crossed, the offspring will have red:pink:white flowers in a ratio 1:2:1. A. red, because the white coloring reflects all incoming light wavelengths in flowers expressing both red and white alleles, incorrect, Red is not the dominant phenotype, and white objects reflecting light has nothing to do with gene expression. B. some flowers red and some white, because only one gene can be expressed at a time, incorrect, This is true for dominant and recessive genes, but incomplete dominant genes express an intermediate phenotype. C. white, because the white coloring will reflect all incoming light wavelengths in flowers expressing both red and white alleles, incorrect, White coloring reflecting light has nothing to do with phenotype expression of genes. D. pink, because it is in-between red and white, correct.

Which of the following INCORRECTLY pairs a digestive enzyme with its function? A. Trypsin-- hydrolyzes specific peptide bonds B. Lactase--hydrolyzes lactose to glucose and galactose C. Pacreatic amylase--hydrolyzes starch to maltose D. Lipase--emulsifies fats

D Lipase is involved in the digestion of fats, but its function is not to emulsify fats--this is the job of bile. Rather, lipase chemically digests fats in the duodenum, allowing them to be brought into duodenal cells and packaged into chylomicrons. The other associations given here are all correct.

During which phase of the cell cycle are DNA repair mechanisms least active? A. G1 B. S C. G2 D. M

D Mismatch repair mechanisms are active during S phase (proofreading) and G2 phase (MSH2 and MLH1), eliminating B and C. Nucleotide and base excision repair mechanisms are most active during the G1 and G2 phases, also eliminating A. These mechanisms exist during interphase because they are aimed at preventing propagation of the error into daughter cells during M phase (mitosis).

Which of the following statements regarding prostaglandins is FALSE? A. Prostaglandins regulate the synthesis of cAMP B. Prostaglandin synthesis is inhibited by NSAIDs C. Prostaglandins affect pain, inflammation, and smooth muscle function D. Prostaglandins are endocrine hormones, like steroid hormones

D Prostaglandins are paracrine or autocrine signaling molecules (they affect regions close to where they are produced, rather than the entire body)

Stable expression of harmful recessive traits across generations most likely occurs in populations with a high level of: A. emigration B. natural selection C. inbreeding D. random mating

D Stable expression of traits, even harmful recessive traits, most likely occurs in populations which satisfy the Hardy-Weinberg equilibrium conditions: 1) large population, 2) mutational equilibrium, 3) no immigration or emigration, 4) random mating, and 5) no selection for the fittest organism. The only answer choice satisfying one of these conditions is answer choice d) random mating. A. emigration, incorrect, Emigration from a population will artificially alter the gene pool frequencies. B. natural selection, incorrect, Selection for fittest organisms introduces bias towards certain genes in the pool, again taking away the random criterion for the statistical gene stability defined by Hardy-Weinberg. C. inbreeding, incorrect, Inbreeding tends to increase the frequency of harmful recessive traits because it artificially creates a smaller subpopulation within the larger gene pool. These smaller populations also frequently share deleterious genes between relatives, thereby amplifying recessive gene expression. Inbreeding contradicts the large population and the random mating criteria of Hardy-Weinberg. D. random mating, correct.

Where are most self-reactive T-cells eliminated? A. Spleen B. Lymph nodes C. Bone marrow D. thymus

D T cells mature in the thymus, where they are "educated" This education involves the elimination of T-cells with improper binding to MHC-antigen complexes (positive selection) and self reactive T-cells (negative selection). Thus, self-reactive T-cells are eliminated in the thymus.

Which of the following bones is NOT a part of the appendicular skeleton? A. The triquetrum, one of the carpal bones B. The calcaneus C. The ischium, one of the fused pelvis bones D. The sternum (breastbone)

D The axial skeleton inclues the skull, vertebral column, ribcage, and hyoid bone. The sternum is a point of attachment of the rib cage and is thus a part of the axial skeleton.

Which of the following is NOT an example of a nonspecific defense mechanism? A. Skin provides a physical barrier against invasion. B. Macrophages engulf and destroy foreign particles C. An inflammatory response is initiated in response to physical damage. D. Cytotoxic T-cells destroy cells displaying foreign antigens

D The body employs a number of nonspecific defense mechanisms against foreign invasion. The skin and mucous membranes provide a physical barrier against bacterial invasion. In addition, sweat contains enzymes that attack bacterial cells walls. Certain passages, such as the respiratory tract, are lined with lined with ciliated mucus-coated epithelia, which filter and trap foreign particles. The inflammatory response is initiated in response to physical damage. Cytotoxic T-cells are involved in (specific) cell mediated immunity

Which of the following is likely to be seen in a patient with liver failure. A. High [ ]s of urea in the blood B. High [ ]s of albumin in the blood C. Low [ ]s of ammonia in the blood D. Low [ ]s of clotting factors in the blood

D The liver serves many functions, including carrying out metabolic processes (glycogenesis and glycogenolysis, fat storage, and gluconeogenesis), detoxification or activation of medications, and synthesis of bile. Germane to this question are the roles of converting ammonia into urea through the urea cycle and synthesis of proteins, including albumin and clotting factors. A patient with liver failure would thus not be able to convert ammonia into urea and would have high [ ]s of ammonia and low [ ]s of urea in the blood, eliminating A and C. With decreased synthetic activity, both albumin and clotting factor [ ]s would be low, eliminating B and makign D the correct answer.

A biopsy is done on a child with an enlarged liver and shows accumulation of glycogen granules with single glucose residues remain at the branch points near the periphery of the granule. The most likely genetic defect is in the gene encoding: A. a-1,4 phosphorylase (glycogen phosphorylase) B. a-1,4:a-1,6 transferase (branching enzyme) C. a-1,4;a-1,4 transferase (part of the debranching enzyme complex) D. a-1,6 glucosidase (part of the debranching enzyme complex)

D The pattern described for this child's glycogen demonstrates appropriate production: there are long chains of glucose monomers, implying that glucogen synthase works. There are also branch points, implying that branching enzyme, B, works. During glycogenolysis, it seems that the child is able to remove individual glucose monomers and process glycogen down to the branch point itself, which requires glycogen phosphorylase, A, and alpha-1,4:alpha-1,4 transferase, C. The metabolic problem here is removing the final glucose at the branch point, which is an alpha-1,6 (not alpha-1,4) link, This requires, D, alpha-1,6 glucosidase.

Which of the following statements regarding the periosteium is incorrect? A. The periosteum serves as a site of attachment of bone to muscle B. Cells of the periosteum may differentiate into osteoblasts C. The periosteum is a fibrous sheath that surrounds long bones D. The periosteum secretes fluid into the joint cavity

D The periosteum, a fibrous sheath that surrounds long bones, is the site of attachment to muscle tissue. Some periosteum cells are capable fo differentiating into bone-forming cells called osteoblasts. It is the synovium that secretes fluid into the joint cavity (joint space), not the periosteum.

A 19-year old college student is picked up by campus police after shoplifting a large bag of corn chips and a dozen ice cream sandwiches. His eyes are bloodshot. During questioning, he repeatedly asks for water because his mouth dry, and he cannot stop giggling. What is the psychoactive substance in in the drug this student has most likely recently taken? A. Alprazolam B. 3,4-Methylenedioxy-N-methylamphetamine C. Diacetylmorphine D. Tetrahydrocannabinol

D This is definitely MJ. THC is the primary active substance in MJ

Which of the following phases are common to cells undergoing meiosis and mitosis? G1 phase G2 phase M phase I only I and III only II and III only I, II, and III

D This question asks the examinee to determine phases within the cell life cycle common to both mitotic-dividing cells and meiotic-dividing cells. The cell cycle is composed of interphase and M phase. In both meiotic- and mitotic-dividing cells, interphase includes the G1, S, and G2 phases, and in highly specialized cells, such as neurons, a G0 phase. Both cell types go through the M phase as either meiosis or mitosis. Thus, I, II and III are correct, making answer choice D the best answer.

Human skin serves which of the following functions? Innate immunity Production of the inactive form of vitamin D Thermoregulation I only I and III only II and III only I, II, and III

D This question asks the examinee to determine various functions performed by the human skin. Human skin has an acidic pH of approximately 5.5, which serves to protect the body from pathogens. The skin also synthesizes vitamin D, from cholesterol, in the presence of sunlight. This vitamin D synthesized in the skin is in its inactive form, cholecalciferol (the active form of vitamin D is calcitriol, which is produced from cholecalciferol in the kidneys). Thermoregulation is also a function of the skin. When the body overheats, the capillaries in the skin dilate to increase blood flow allowing for the dissipation of heat. When the body is cold, hair follicles in the skin contract, which serves to insulate the body and conserve heat. Therefore I, II, and III are all functions of skin in humans, making D the correct answer.

Antidiuretic hormone (ADH) is responsible for the retention of water in the body. In the kidney, ADH increases reabsorption at the collecting duct. Which of the following physiological responses will be most likely in a patient suffering from severe dehydration? A. Decreased secretion of ADH resulting in decreased blood volume B. Increased secretion of ADH resulting in decreased blood volume C. Decreased secretion of ADH resulting in increased blood volume D. Increased secretion of ADH resulting in increased blood volume

D This question is asking you to determine the body's hormonal response with respect to ADH in the case of a person suffering from dehydration. Dehydration is the result of a low blood volume, which leads to low blood pressure. To maintain homeostasis, the body will attempt to equilibrate by increasing blood volume, which in turn, increases blood pressure. Since ADH increases reabsorption of water from the filtrate to the bloodstream, the body is expected to secrete high levels of antidiuretic hormone, which will cause an increase in blood volume.

What proportion of offspring with the genotype AABBCC will result from a trihybrid cross for the unlinked genes A, B, and C? A. 1/8 B. 1/16 C. 1/32 D. 1/64

D This question tests your ability to calculate the probability of a possible genotype resulting from a trihybrid cross between two heterozygotes. One method to determine the probability is to make a Punnett square. An organism that is heterozygous for three genes can produce eight different gamete possibilities. Thus, resultant Punnett square would contain an 8×8 table with a total of 64 genotype combinations. A more efficient method is to calculate the probability of each event occurring separately and then use the rule of multiplication to determine the likelihood of all three events occurring at the same time. Probability of obtaining an AA genotype from a cross between Aa and Aa = ¼ Probability of obtaining a BB genotype from a cross between Bb and Bb = ¼ Probability of obtaining a CC genotype from a cross between Cc and Cc = ¼ Total probability of all three possibilities = ¼ x ¼ x ¼ = 1/64

Upon encountering an antigen, only T-cells with a specific T-cell receptor are activated. This is an example of: A. innate immunity B. a cytotoxic t-cell response C. humoral immunity D. clonal selection

D When the adaptive immune system encounters an antigen, only the cells with antibodies or T-cell receptors specific to that antigen are activated. This is known as clonal selection. While a T-cell response may be a cytotoxic response, it could also be the activation of helper T-cells; plus, this does not explain the specificity of the response, eliminating B.

When the pH of the blood is high, which substance is likely to be excreted in larger quantities in the urine? A. Urea B. Ammonia C. Hydrogen ions D. Bicarbonate ions

D When the pH of the blood is high, this indicates that the blood is alkalemic. To correct this, the kidney will incr excretion of a base, namely bicarb.

Which stage of protein synthesis does NOT require energy? A. Initiation B. Elongation C. Termination D. All stages of protein synthesis require energy

D all 3 stages require large amounts of energy

The world record for the longest-held breath is 22 minutes and 0 seconds. If a sample were taken from this individual during the last minute of breath bolding, which of the following might be observed? A. increased hemoglobin affinity for oxygen B. decreased PCO2 C. increased hematocrit D. decreased pH

D holding ones breath would result in a deop in oxygenation and inc in PCO2. The inc CO2 would associated with water to form carbonic acid, which would dissociate into a proton and a bicarbonate anion. Further, the low O2 sat would eventually lead to anaerobic metabolism in some tissues, causing an inc in lactic acid. These would all lead to a dec pH.

A genotypical female infant is born with ambiguous genitalia. Soon after birth, she suffers from hyponatremia, or low blood [ ]s of sodium. Which endocrine organ is most likely to be affected? A. Hypothalamus B. Pituitary C. Kidneys D. Adrenal cortex

D in a genotypic female, this indicates that she was exposed to androgens during the fetal period. So she has excess androgens. In addition, she is also losing sodium. So she is lacking in aldosterone.

Which of the following is most likely to be found bound to a protein in the body? A. Sodium B. Potassium C. Chloride D. Calcium

D ions that are not readily accessible in the cytoplasm or extracellular space are likely to be bound to a binding protein. Classically, calcium and magnesium are protein bound. Also, sodium and potassium must exist in their free states to participate in action potentials. Chloride is readily excreted by the kidney, which would not be true if it were protein-bound

A membrane receptor is most likely to be a(n) A. embedded protein with catalytic activity B. transmembrane protein with sequestration activity C. membrane-associated protein with sequestration activity D. transmembrane protein with catalytic activity

D membrane receptors must have both an extracellular and intracellular domain. in order to initiate a second messenger cascade, they typically display enzymatic activity, although some may act strictly as channels

What role does peptidyl transferase play in protein synthesis? A. It transports the initiator aminoacyl-tRNA complex. B. It helps the ribosome to advance three nucleotides along the mRNA in the 5' to 3' direction C. It holds the protein in its tertiary structure D. It catalyzes the formation of a peptide bond.

D peptidyl transferase is an enzyme that catalyzes the formation of a peptide bond between the incoming amino acid in the A site and the growing polypeptide chain in the P site. Initiation and elongation factors help transport charged tRNA molecules into the ribosome and advance the ribosome down the mRNA transcript. Chaperones maintain a protein's three-dimensional shape as it is formed

Which of the following is a function of the parasympathetic nervous system? A. inc blood sugar during periods of stress B. dilating the pupils to enhance vision C. inc oxygen delivery to muscles D. decreasing heart rate and blood pressure

D rest and digest

Which of the following statements concerning the somatic division of the PNS is INCORRECT? A. its pathways innervate skeletal muscle B. Its pathways are usually voluntary C. some of its pathways are referred to as reflex arcs D. its pathways always involve more than two neurons

D somatic=skeletal. The pathways of the somatic division can involve two, three, or more neurons, depending on the type of signal

Which of the following statements correctly identifies a key difference between mitosis and meiosis? A. in metaphase of mitosis, replicated chromosomes line up in single file, in metaphase II of meiosis, replicated chromosomes line up on opposite sides of the metaphase plate B. During anaphase of mitosis, homologous chromosomes separate, during anaphase I of meiosis, sister chromatids separate C. at the end of telophase of mitosis, the daughter cells are identical to each other; at the end of meiosis I, the daughter cells are identical to the parent cell D. during metaphase of mitosis, centromeres are present directly on the metaphase plate; during metaphase of Meiosis I, there are no centromeres on the metaphase plate

D synapsis and crossing over occur during prophase I and homologous chromosomes are separated during meiosis I

The tricuspid valve prevents backflow of blood from the: A. left ventricle into the left atrium B. aorta into the left ventricle C. pulmonary artery into the right ventricle D. right ventricle into the right atrium

D the atrioventricular valves are located between the atria and the ventricles on both sides of the heart. Their role is to prevent backflow of blood into the atria. The valve on the right side of the heart has three cusps and is called the tricuspid calve.

Which of the following incorrectly pairs a type of muscle fiber with a characteristic of that fiber? A. red fibers-rich in mitochondria B. red fibers- high levels of myoglobin C. white fibers- fast twitching D. white fibers- predominantly use aerobic respiration

D white fibers derive most of their energy anaerobically and fatigue more readily than red fibers

For most cells, the extracellular calcium concentration is around 10,000 times higher than the intracellular calcium concentration. What is the membrane potential established by this electrochemical gradient? A. -123 mV B. -61.5 mV C. +61.5 mV D. +123 mV

D use the Nernst equation pg 293 biochemistry E=(61.5/z) log([ion]outside/[ion]inside). For Calcium, z=+2, and the ratio of [ion]outside to [ion]inside is 10^4. =123mV

Which of the following is a correct sequence of passageways through which air travels during inhalation? A. Pharynx --> trachea --> bronchioles --> bronchi --> alveoli B. Pharynx --> trachea --> larynx --> bronchi --> alveoli C. Larynx --> pharynx --> trachea --> bronchi --> alveoli D. Pharynx --> larynx --> trachea --> bronchi --> alveoli

D. nostrils -> pharynx -> larynx -> trachea ->divides into two bronchi (one for each lung) -> bronchi branch into smaller bronchioles -> terminate in clusters of alveoli

A researcher wishes to incorporate a radio labeled deoxyadenine into the genome of one of the two daughter cells that would arise as a result of mitosis. What is the latest stage of cellular development during when the radio labeled deoxyadenine could be added to achieve this result? A. G1 B. G2 C. M D. S

D. To ensure that the labeled deoxyadenine will be incorporated into the DNA of one of the daughter cells, we have to insert the nucleotide before DNA replication has been completed. Because replication occurs during the S stage, we could introduce the deoxyadenine during G1 or S stage. G1 precedes S.

What are DNA libraries? How can you make a DNA library? What are the two types of DNA libraries and what is the difference between them? Which of the two libraries can be used to reliably sequence specific genes and identify disease-causing mutations, produce recombinant proteins (such as insulin, clotting factors, or vaccines) or produce transgenic animals? Why?

DNA libraries arise from DNA cloning which are large collection of known DNA sequences; in sum these sequences could equate to the genome of an organism. To make a DNA library, DNA fragments, often digested randomly, are cloned into vectors and can be utilized for further study. Libraries can consist of either genomic DNA or cDNA. Genomic libraries contain large fragments of DNA and include both coding (exon) and noncoding (intron) regions of the genome. cDNA (complementary DNA) libraries are constructed by reverse-transcribing processed mRNA. As such, cDNA lacks noncoding regions (introns), and only includes genes that are expressed in the tissue from which the mRNA was isolated. For that reason, these libraries are sometimes called expression libraries. While genomic libraries contain the entire genome of an organism, genes may by chance be split into multiple vectors. Therefore, only cDNA libraries can be used to reliably sequence specific genes and identify disease-causing mutations, produce recombinant proteins (such as insulin, clotting factors, or vaccines) or produce transgenic animals.

What is DNA methylation and how does it work?

DNA methylation is involved in chromatin remodeling and regulation of gene expression levels in the cell, like histone acetylation. DNA methylases add methyl groups to cytosine and adenine nucleotides; methylation of genes is often linked with the silencing of gene expression. During development, methylation plays an important role in silencing genes that no longer need to be activated. Heterochromatin regions of the DNA are much more heavily methylated, hindering access of the transcription machinery to the DNA.

Describe the mechanism of transcription.

During initiation of transcription, several enzymes, including helicase and topoisomerase, are involved in unwinding the double-stranded DNA and preventing formation of supercoils. -This step is important in allowing the transcriptional machinery access to the DNA and the particular gene of interest. Transcription results in a single strand of mRNA, synthesized from one of the two nucleotide strands of DNA called the template strand (or the antisense strand). The newly synthesized mRNA strand is both antiparallel and complementary to the DNA template strand. RNA is synthesized by a DNA-dependent RNA polymerase; RNA polymerase locates genes by searching for specialize DNA regions known as promoter regions. In eukaryotes, RNA polymerase II is the main player in transcribing mRNA, and its binding site in the promoter region is known as the TATA box, named for its high concentration of thymine and adenine bases. Transcription factors help the RNA polymerase locate and bind to this promoter region of the DNA, helping to establish where transcription will start. Unlike DNA polymerase III, RNA polymerase does not require a primer to start generating a transcript. In eukaryotes, there are three types of RNA polymerases, but only one is involved in the transcription of mRNA: -RNA polymerase I is located in the nucleolus and synthesizes rRNA - RNA polymerase II is located in the nucleus and synthesizes hnRNA (pre-processed mRNA) and some small nuclear RNA(snRNA) -RNA polymerase III is located in the nucleus and synthesizes tRNA and some rRNA RNA polymerase travels along the template strand in the 3'-->5' direction, which allows for the construction of transcribed mRNA in the 5'-->3' direction. Unlike DNA polymerase, RNA polymerase does not proofread its work, so the synthesized transcript will not be edited. The coding strand (or sense strand) of DNA is not used as a template during transcription. Because the coding strand is also complementary to the template strand, it is identical to the mRNA transcript except that all the thymine nucleotides in DNA have been replaced with uracil in the mRNA molecule. In the vicinity of a gene, a numbering system is used to identify the location of important bases in the DNA strand. The first base transcribed from DNA to RNA is defined as the +1 base of that gene region. Bases to the left of this start point (upstream, toward the 5' end) are given negative numbers. Bases to the right (downstream, toward the 3' end) are given positive numbers. Thus, no nucleotide is numbered 0. The TATA box, where RNA polymerase II binds, usually falls around -25. Transcription will continue along the DNA coding region until the RNA polymerase reaches a termination sequence or stop signal, which results in the termination of transcription. The DNA double helix then re-forms, and the primary transcript formed is termed heterogenous nuclear RNA (hnRNA). mRNA is derived from hnRNA via posttranscriptional modifications.

What are the two categories of sleep-wake disorders? Explain them.

Dyssomnias refer to disorders that make it difficult to fall asleep, stay asleep, or avoid sleep, and include insomnia, narcolepsy, and sleep apnea. Parasomnias are abnormal movements or behaviors during sleep, and include night terrors and sleep walking. Most sleep-wake disorders occur during NREM sleep

Explain how gas exchange occur in the lungs

Each alveolus is surrounded by a network of capillaries. The capillaries bring deoxygenated blood from the pulmonary arteries, which originate from the right ventricle of the heart. The walls of the alveoli are only one cell thick, which facilitates the diffusion of carbon dioxide from the blood into the lungs, and oxygen into the blood. The oxygenated blood returns to the left atrium of the heart via the pulmonary veins. The driving force for gas exchange is the pressure differential of the gasses. When it initially arrives at the alveoli, blood has a relatively low partial pressure of oxygen and a relatively high partial pressure of carbon dioxide, facilitating transfer of each down its respective [ ] gradient. Because the gradient between the blood and air in the lungs is already present as the blood enters the lungs, no energy is required for gas transfer?

What is enzyme specificity?

Enzymes are picky. The molecules upon which an enzyme acts are called substrates; a given enzyme will only catalyze a single reaction or class of reactions with these substrates

What are the three main functions of the lymphatic system? Explain them.

Equalization of Fluid Distribution. At the capillaries, fluid leaves the bloodstream and goes into the tissues. The quantity of fluid that leaves the tissues at the arterial end of the capillary bed depends on both hydrostatic and oncotic pressure (Starling forces). Remember that the oncotic pressure of the blood draws water back into the vessel and the venule end, once hydrostatic pressure has decreased. Because the net pressure drawing fluid in at the venule end is slightly less than the net pressure pushing fluid out at the arterial end, a small amount of fluid remains in the tissues. Lymphatic vessels drain these tissues and subsequently return the fluid to the bloodstream. The lymphatics offer some protection against pathology. Ex: if the blood has a low [ ] of albumin (plasma protein), the oncotic pressure of the blood is decreased and less water is driven back into the blood-stream at the venule end. Thus, the fluid will collect in the tissues. Provided that the lymphatic channels are not blocked, much of this fluid may eventually return to the bloodstream via the lymphatics. Only when the lymphatics are overwhelmed does edema occur. Transportation of Biomolecules. The lymphatic system transports fats from the digestive system to the bloodstream. Lacteals, small lymphatic vessels, are located at the center of each villus in the small intestine. Fats, packaged into chylomicrons by intestinal mucosal cells, enter the lacteal for transport. Lymphatic fluid carrying many cylomicrons takes on a milky white appearance and is called chyle. Immunity. Lymph nodes are a place for antigen-presenting cells and lymphocytes to interact. B-cells proliferate and mature in the lymph nodes in collections called germinal centers.

Neurons can receive both excitatory and inhibitory input. What does excitatory input cause and how? What about inhibitory input? How do these each affect the likelihood of having an action potential? How can an action potential be triggered? What is the additive effect of multiple signals called?

Excitatory input causes depolarization (raising the membrane potential, Vm, from its resting potential) and thus makes the neuron more likely to fire an action potential. Inhibitory input causes hyper polarization (lowering the membrane potential from its resting potential) and thus makes the neuron less likely to fire an action potential. If the axon hillock receives enough excitatory input to be depolarized to the threshold value (usually -55 to -40 V), an action potential will be triggered summation

What produces does the anterior pituitary secrete?

FLAT PEG Follicle-stimulating hormone (FSH) Lutenizing hormone (LH) Adrenocorticotropic hormone (ACTH) Thyroid stimulating hormone (TSH) Prolactin Endorphins Growth hormone (GH) FLAT are all tropic hormones PEG are all direct hormones

How does B-oxidation differ between fatty acids with an odd number of carbon atoms and an even number?

Fatty acids with an odd number of carbon atoms undergo B-oxidation in the same manner as even-numbered carbon fatty acids for the most part. The only difference is observed during the first cycle, where even-numbered fatty acids for the most part yield two acetyl-CoA molecules (from the four-carbon remaining fragment) and odd-numbered fatty acids yield one acetyl-CoA and one propionyl-CoA (from the five carbon remaining fragment). Propinonyl-CoA is converted to methylmalonyl-CoA by propionyl-CoA carboxylase, which requires biotin (VitaminB7). Methylmalonyl-CoA is then converted into succinyl-CoA by methylmalonyl-CoA mutase, which requires cobalamin (Vitamin B12). Succinyl-CoA is a citric acid cycle intermediate and can also be converted to malate to enter the gluconeogenic pathway in the cytosol. Odd-carbon fatty acids thus represent an exception to the rule that fatty acids cannot be converted to glucose in humans.

Explain GLUT 2 transporters.

GLUT 2 is a low-affinity transporter in hepatocytes and pancreatic cells. After a meal, blood traveling through the hepatic portal vein from the intestine is rich in glucose. GLUT 2 captures the excess glucose primarily for storage. When the glucose [ ] drops below the Km for the transporter, much of the remainder bypasses the liver and enters the peripheral circulation. The Km of GLUT 2 is quite high (around 15 mM). This means that the liver will pick up glucose in proportion to its [ ] in the blood (first order kinetics). In other words, the liver will pick up excess glucose and store it preferentially after a meal, when blood glucose levels are high. In the B-islet cells of the pancreas, GLUT 2 along with the glycolytic enzyme, glucokinase, serves as the glucose sensor for insulin release.

Explain GLUT 4 transporters.

GLUT 4 is in adipose tissue and response to the glucose [ ] in peripheral blood. The rate of glucose transport in these two tissues is increased by insulin, which stimulates the movement of additional GLUT 4 transporters to the membrane by a mechanism involving exocytosis. The Km of GLUT 4 is close to the normal glucose levels in blood (approx. 5 mM). This means that the transporter is saturated when blood glucose levels are just a bit higher than normal. When a person has high blood sugar [ ]s, these transporters will still permit only a constant rate of glucose influx because they will be saturated (zero order kinetics). Then how can cell with GLUT 4 transporters inc their intake of glucose? By increasing he number of GLUT 4 transporters on their surface.

Explain the issue with blood flow back toward the heart and how this issue is overcome.

Given that the heart is located in the chest, bloodflow in most veins is upward from the lower body back tot he heart, against gravity. In the inferior vena cava, this translates into a large amount of blood in a vertical column. The pressure at the bottom of this column in the large veins of the legs can be quite high. Thus, veins must have structures to push tot blood forward and prevent backflow. Larger veins contain valves; as blood flows forward in the veins, the valves open. When blood tries to move backward, the valves will slam shut. Failure of the venous valves can result in varicose veins, which are distended where blood has pooled. in addition to high pressure in the lower extremities, the small amount of smooth muscle also creates a challenge for propelling blood forward. Thus, the veins must rely on an external force to generate the pressure to push blood toward the heart. Most veins are surrounded by skeletal muscles, which squeeze the veins as the muscles contract , forcing the blood up against gravity in much the same way that squeezing the bottom of a tube of toothpaste causes toothpaste to shoot up to the top.

What are the two NADH shuttles? What are they and why do they exist?

Glycerol-3-phosphate shuttle and malate-aspartate shuttle. efficiency of aerobic respiration varies between cells. This variable efficiency is caused by the fact that cytosolic NADH formed through glycolysis cannot directly cross into the mitochondrial matrix. Because it cant contribute its electrons to the transport chain directly, it must find alternate means of transportation referred to as shuttle mechanisms. A shuttle mechanism transfers the high-energy electrons of NADH to a carrier that can cross the inner mitochondrial membrane. Depending on which of the two shuttle mechanisms NADH participates in, either a 1.5 or 2.5 ATP will end up being produced

What does branching enzyme do and how does it do it? Name it.

Glycosyl alpha-1,4:alpha-1,6 transferase responsible for introducing alpha-1,6-linked branches into the granule as it grows. It hydrolyzes one of the alpha-1,4 bonds to release a block of oligoglucose (a few glucose molecules bonded together in a chain), which is then moved and added in a slightly different location. Forms an alpha-1,6 bond to create a branch. alpha-1,4 keeps the same branch moving "4ward" alpha-1,6 puts a branch in the mix

Explain the Griffith Experiment

Griffith put living S strain in bacteria that kill the mouse and R strain which does not kill the mouse. He heat killed the S strain and the mouse lived. He added R strain to the heat killed S strain and the mouse died. The conclusion was that R strain was somehow transformed in living cells. (The question was ---is it protein or DNA) He theorized that the live, nonvirulent bacteria must have acquired the ability to form smooth capsules from the dead virulent bacteria. This was known as the transforming principle.

Which of the following of Piaget's stages of cognitive development occur before adolescence? I. Preoperational II. Sensorimotor III. Formal operational

I and II

Mitochondrial DNA is I circular II self-replication III single-stranded

I and II Both mDNA and bacterial DNA are organized into a single circular chromosome of double-stranded DNA that can replicate during binary fission

Which of the following is/are function(s) of NADPH in the cell? I. Antimicrobial resistance via bacterial destruction by bleach within lysosomes. II. Involvement in the production of the precursors to steroid hormones. III. Functional carriage of energy across organelle membranes for use within the mitochondria.

I and II NADPH has three primary functions: involvement in biosynthesis of lipids and cholesterol (the precursor to steroid hormones), production of bactericidal bleach in the lysosomes of certain WBCs, and maintenance of a supply of reduced glutathione for protection against free radical damage. Energy carriage is an important function of NADH, not NADPH.

The plasma membrane maintains the concentration gradients between the intracellular and extracellular spaces. In so doing, some molecules can diffuse directly through the membrane. Which of the following molecule(s) can do so? I. Water II. Testosterone III. Glucose IV. Na+

I and II This question asks the examinee to identify the types of molecules that are able to enter the cell through passive diffusion. To answer this question, you must understand the various transport mechanisms for molecules into and out of the cell. Passive diffusion, facilitated diffusion, active transport, and secondary active transport are the four types of transport processes. This question asks about passive diffusion, which involves the movement of molecules down a concentration gradient directly through the plasma membrane (without transmembrane proteins). The passive diffusion of molecules depends on two things: 1) size and 2) polarity. The size of a molecule is important because molecules that are too large to fit between the phospholipids that comprise the cell membrane must use integral proteins. Polarity is also an important consideration because the nonpolar interior of the plasma membrane. This hydrophobic environment prevents the passage of all charged ions. Of the choices above, water is small enough to pass through the membrane. Testosterone is a steroid hormone, which freely passes through the plasma membrane. Glucose is too large to pass through the membrane and sodium ions are charged and thus unable to pass through the membrane.

Which of the following is (are) true regarding genetic drift? I. Genetic drift has more pronounced effect on alleles with a low allele frequency than on alleles with a high allele frequency, regardless of population size. II. Genetic drift is due to random sampling during reproduction. III. The more isolated a population is, the less pronounced the effect of genetic drift is.

I and II When a population reproduces, the alleles in the offspring are a random sample of the alleles in the parental generation. Due to this, changes may occur in the genetic composition of a population due to chance. These changes are genetic drift. Thus (II) is correct, eliminating choice (A). Genetic drift is less pronounced in large populations that are connected with other populations. When a population is isolated, there is a higher likelihood that a particular allele can disappear entirely, or become entirely fixed. Thus, (III) is false, eliminating choice (D). Finally, the effect of genetic drift is larger when an allele is less common. The fewer copies of an allele there are, the more likely it is that the allele may disappear entirely due to genetic drift. While genetic drift has a bigger impact on smaller populations, within any given population (regardless of size), the effects of genetic drift will be strongest on alleles with a low allele frequency. Thus (I) is correct and choice (C) is the right answer.

Starch is hydrolyzed into maltose by enzymes from the I. salivary glands II. brush border III. pancreas

I and III Starch is hydrolyzed to maltose by two enzymes: salivary amylase (secreted by the salivary glands) in the mouth and pancreatic amylase (secreted by the pancreas) in the duodenum. Brush-border disaccharides can further break down maltose, but do not break down starch.

Which of the following are steroid-based molecules? I. Testosterone II. Triglycerides III. Progesterone IV. DNA

I and III Triglycerides consist of three fatty acids bound by ester linkages to a glycerol backbone. DNA is not a steroid-based molecule; DNA consists of deoxyribose, a phosphodiester linkage, and a nitrogenous base. one=steroid

Resting membrane potential depends on: I. differential distribution of ions across the axon membrane II. the opening of voltage-gated calcium channels III. active transport of ions across the membrane

I and III only The polarization of the neuron at rest is the result of an uneven distribution of ions between the inside and outside of the cell. This difference is achieved through the active pumping of ions into and out of the neuron (using Na+/K+ ATPase). Voltage-gated Ca+ channels are important in the nerve terminal, where the influx of Ca+ triggers the fusion of vesicles continent neurotransmitter with the membrane, but not in maintaining resting membrane potential

The epithelial lining of the small intestine forms from which germ layer? I. Endoderm II. Mesoderm III. Ectoderm A. I only B. II only C. III only D. I and II

I only As a guideline for the MCAT, the ectoderm forms outer coverings of body, e.g. skin/hair/nails, and the nervous system. The endoderm forms the lining of the digestive tract and most of the liver and pancreas. Mesoderm forms everything in-between, e.g. muscle, bone, circulatory system, heart, spleen, gonads. A. I only, correct. B. II only, incorrect: The mesoderm forms the tissues between the body's outer coverings and the lining of the digestive tract. C. III only, incorrect: The ectoderm forms the body's outer coverings and nervous system. D. I and II, incorrect: On the MCAT, organs and tissues will derive from one layer only.

Cancer cells are cells in which mitosis occurs continuously, without regard to quality or quantity of the cells produced. For this reason, most chemotherapies attack rapidly dividing cells. At which point (s) in the cell cycle could chemo effectively prevent cancer in cell division? I. S stage II. Prophase III. Metaphase

I, II, and III prevent DNA synthesis during the S stage. without DNA being replicated, the two viable daughter cells would not be formed. prevent the mitotic cycle from forming altogether in prophase by preventing spindle apparatus formation, preventing the nuclear membrane from dissolving, or interfering with other processes during this phase. similarly, a treatment that would act on cells in the metaphase stage of the cell cycle would also interfere with the mitotic cycle

Idiopathic pulmonary fibrosis (IPF) is a disease in which scar tissue forms in the alveolar walls, making the lung tissue significantly more stiff. Which of the following findings would likely be detected through spirometry in a patient with IPF? I. Decreased total lung capacity II. Decreased inspiratory reserve volume III. Increased residual volume

II only In a patient with IPF, the inc stiffness of the lungs would likely decrease the volume of air the individual could inhale, which would decrease both the total lung capacity and inspiratory reserve volume. However, spirometry can't measure the total lung capacity accurately b/c it can't determine the residual volume-the volume of air left in the lungs when an individual has maximally exhaled. Finally, inc stiffness of the lungs would be expected to decrease the residual volume.

Explain the response to viral (intracellular pathogen) infections.

In a viral infection, the virally infected cell will begin to produce interferons, which reduce the permeability of nearby cells (decreasing the ability of the virus to infect these cells), reduce the rate of transcription and translation in these cells (decreasing the ability of the virus to multiple), and cause systemic symptoms (malaise, muscle aching, fever, and so on). These infected cells also present intracellular proteins on their surface in conjunction with MHC-I; in a virally infected cell, at least some of these intracellular protein will be viral proteins. CD8+ T-cells will recognize the MHC-I and antigen complex as foreign and will inject toxins into the cell to promote apoptosis. In this way, the infection can be shut down before it is able to spread to nearby cells. In the even that the virus downregulates the production and presentation of MHC-I molecules, natural killer cells will recognize the absence of MHC-I and will accordingly cause apoptosis of these cells. Once the pathogen has been cleared, memory T-cells will be generated that can allow a much faster response to be mounted upon a second exposure.

What is the difference between mRNA in eukaryotes and prokaryotes?

In eukaryotes, mRNA is monocistronic, meaning that each mRNA molecule translates into only one protein product. Thus, in eukaryotes, the cell has a different mRNA molecule for each of the thousands of different proteins made by that cell. In prokaryotes, mRNA may be polycistronic, and starting the process of translation at different locations in the mRNA can result in different proteins.

Explain how fatty acid oxidation relates to acetyl-CoA

In the cytosol, a process called activation causes a thioester bond to form between carboxyl groups of fatty acids and CoA-SH. Because this activated fatty acyl-CoA cannot cross the inner mitochondrial membrane, the fatty acyl group is transferred to carnitine via a transesterification reation. Carnitine is a molecules that can cross the inner membrane with a fatty acyl group in tow. Once acyl-carnitine crosses the inner membrane; it transfers the fatty acyl group to a mitochondrial CoA-SH via another transesterificaiton reaction. In other words, carnitine's function is merely to carry the acyl group from a cytosolic CoA-SH to a mitochondrial CoA-SH. Once acyl-CoA is formed in the matrix, B-oxidation can occur, which removes two carbon fragments from the carboxyl end.

what is glycogen? where do glycogen synthesis and degradation occur? What are the different functions of these places?Where is it stored and what is it stored as? Described what it is stored as including the different type of chains it can be composed of.

It is a stored formed of glucose/energy. Primarily in the liver and skeletal muscle, although other tissues store smaller quantities. Liver glycogen is broken down to maintain a constant level of glucose in the blood; muscle glycogen is broken down to provide glucose to the muscle during vigorous exercise. stored in the cytoplasm as granules. Each granule has a central protein core with polyglucose chains radiating outward to form a sphere. Glycogen granules composed entirely of linear chains have the highest density of glucose near the core. if the chains are branched, glucose density is highest at the periphery of the granule, allowing more rapid release of glucose on demand.

Explain Mendel's first law. What part of mitosis/meiosis does this refer to?

Law of segregation 1. Genes exist in alternative forms 2. An organism has two alleles for each gene-one inherited from each parent 3. The two alleles segregate during meiosis, resulting in gametes that carry only one allele for any inherited trait 4. If two alleles of an organism are different, only one will be fully expressed and the other will be silent. The expressed allele is dominant while the silent allele is recessive (keep in mind that codominance and incomplete dominance are exceptions to this rule). The key correlate to draw here is the separation of homologous chromosomes during anaphase I of meiosis. By separating - segregating - these chromosomes into different cells, each gamete carries only one allele for any given trait.

What are the two types of leukocytes? Explain the differences and similarities. where are they produced? How are they produced?

Leukocytes are produced in the bone marrow through hematopoiesis. They are divided into two groups of cells: granulocytes and agranulocytes. These names refer to the presence or absence of granules in the cytoplasm. These granules contain toxic enzymes and chemicals, which can be releast by exocytosis, and are particularly effective against bacterial, fungal, and parasitic pathogens. Both granulocytes and agranulocytes come from a common precursor: hematopoietic stem cells, which also give rise to RBCs.

What happens during prolonged fasting (starvation)?

Levels of glucagon and epinephrine are markedly elevated during starvation. Increased levels of glucagon relative to insulin result in rapid degradation of glycogen stores in the liver. As liver glycogen stores are depleted, glyconeogenic activity continues and plays an important role in maintaining blood glucose levels during prolonged fasting; after about 24 hours, glyconeogenesis is the predominant source of glucose for the body. Lipolysis is rapid, resulting in excess acetyl-CoA that is used in the synthesis of ketone bodies. Once levels of fatty acids and ketones are high enough in the blood, muscle tissue will utilize fatty acids as its major fuel source and the brain will adapt to using ketones for energy. After several weeks of fasting, the brain derives approx 2/3rds of its energy from ketones and 1/3 from glucose. The shift from glucose to ketones as the major fuel reduces the quantity of amino acids that must be degraded to support gluconeogenesis, which spares proteins that are vital for other functions. Cells that have few, if any, mitochondria, like RBCs, continue to be dependent on glucose for their energy.

What are the major organs of the immune system? Give their basic functions. List the minor organs as well.

Major: Lymph nodes filter lymph and are a site where immune responses can be mounted Bone marrow is the site of immune cell production The thymus is the site of T cell maturation The spleen is a storage area for blood, filters blood and lymph, and is a site where immune responses can be mounted. Minor: Adenoids, tonsils, Peyer's patches in the small intestine, appendix.

Explain the M stage of cell division.

Mitosis + cytokinesis. Mitosis is divided into 4 phases: prophase, metaphase, anaphase, and telophase. Occurs in somatic cells, cells that are not involved in sexual reproduction. Cytokinesis is the splitting of the cytoplasm and organelles between the 2 daughter cells

How does the signal get from one neuron to the next?

Neurons are not physically connected to each other. Between the neurons, there is a small space into which the terminal portion of the axon releases neurotransmitters, which bind to the dendrites o the adjacent neuron (the postsynaptic neuron). This space is known as the synaptic cleft. Together, the nerve terminal, synaptic cleft, and postsynaptic membrane are known as a synapse. Neurotransmitters released from the axon terminal traverse the synaptic cleft and bind to receptors on the postsynaptic neuron.

What are the functions of NADPH?

NADH produced from this reduction of NAD+ can feed into the electron transport chain to indirectly produce ATP. Conversely, NADPH primarily acts as an electron donor ina number of biochemical reactions. It can be thought of as a potent reducing agent because it helps other molecules be reduced (and thus oxidized itself during the process). Cells require NADPH for a variety of functions including: -Biosynthesis, mainly of fatty acids and cholesterol. -Assisting in cellular bleach production in certain WBCs, thereby contributing to bactericidal activity -Maintenance of a supply of reduced glutathione to protect against reactive oxygen species (acting as the body's natural antioxidant). This last function is important in protecting cells from free radical oxidative damage caused by perodixes. Hydrogen peroxide is produced as a byproduct in aerobic metabolism and can break apart to form hydroxide radicals. Free radicals can attack lipids, including those in the phospholipids of the membrane. When oxidized, these lipids lose their function and can weaken the membrane, causing lysis. This is especially true in RBCs, which contain high levels of oxygen, which, ehwn oxidized by other free radicals, becomes the superoxide radical O2 radical. Free radicals can also damage DNA, potentially causing cancer.

Explain Complex I of the electron transport chain

NADH-CoQ oxidoreductase NADH + H+ + FMN --> NAD+ + FMNH2 FMHN2 + 2-Fe-Soxidized --> FMN + 2-Fe-Sreduced + 2H+ 2Fe-Sreduced + CoQ + 2H+ --> 2Fe-Soxidized + CoQH2 Net: NADH + H+ + CoQ --> NAD+ + CoQH2 The transfer of electrons from ANDH to coenzyme Q (CoQ) is catalyzed in this first complex. This complex has over 20 subunits, but the two highlighted here include a protein that has an iron-sulfur cluster and a flavoprotein that oxidizes NADH. The flavoprotein has a coenzyme called flavin mononucleotide (FMN) covalently bonded to it. FMN is quite similar in structure to FAD, flavin adenine dinucleotide. The first step in the reaction involves NADH transferring its electrons to FMN, thereby becoming oxidized to NAD+ as FMN is reduced to FMNH2. Next, the flavoprotein becomes reoxidized while the iron-sulfur subunit is reduced. Finally, the reduced iron-sulfur subunit donates the electrons it received from FMNH2 to coenzyme Q (also called ubiquinone). Coenzyme Q becomes CoQH2. This first complex is one of three sites where proton pumping occur, as four protons are moved to the intermembrane space.

What is PCR and how does it work? What is needed to make it work?

Polymerase chain reaction and produces virtually unlimited copies of a very small DNA sample PCR is used to identify criminal suspects, familial relationship, and disease-causing bacteria and viruses. Knowing the sequences that flank the desired region of DNA allows for the amplification of the sequence in between. A PCR reaction requires primers that are complementary to the DNA that flanks the region of interest, nucleotides (dATP, dTTP, dCTP, and dGTP), and DNA polymerase. The primer has high GC content, as the additional H bonds between G and C confer stability. The reaction also needs heat to cause the DNA double helix to denature. Unfortunately, the DNA polymerase found in the human body doesn't work at high temps. Thus, the DNA polymerase from Thermus aquaticus, a bacteria that thrives in the hot springs of Yellowstone National Park is used. During PCR, the DNA of interest is denatured, replicated, and then cooled to allow reannealing of the daughter strands with the parent strands. This process is repeated several times, doubling the amount of DNA with each cycle, until enough copies of the DNA sequence are available for further testing.

Compare prokaryotic and eukaryotic ribosomes

Prokaryotes have 50S and 30S large and small subunits, which assemble to create the complete 70S ribosome. Eukaryotes have 60S and the 40S subunits; these subunits join during protein synthesis to form the whole 80S ribosome.

What is a notable gene on the Y chromosome

SRY (sex- determining region Y), which codes for a transcription factor that initiates testis differentiation and thus, the formation of the male gonads. So in the absence of the Y chromosome, all zygotes will be female.

Explain the second step in muscle contraction.

Shortening of the sarcomere. The free globurular heads of the myosin molecules move toward and bind with the exposed sites on actin. The newly formed actin-myosin cross bridges then allow myosin to pull on actin, which draws the thin filaments toward the M-line, resulting in shortening of the sarcomere. Myosin carrying hydrolyzed ATP (ADP and an inorganic phosphate, Pi) is able to bind with the myosin-binding site. The release of the inorganic phosphate and ADP in rapid succession provides the energy for the powerstroke and results in sliding of the actin filament over the myosin filament. Then, ATP binds to the myosin head, releasing it from actin. This ATP is hydrolyzed to ADP and Pi, which recocks the myosin head so that it is in position to initiated another cross-bridge cycle. The repetitive binding and releasing of myosin heads on actin filaments allows the thin filament to slide along the thick filament, causing sequential shortening of the sarcomere. This is known as the sliding filament model.

Explain the preoperational stage of cognitive development

Stage 2 2-7 years old characterized by symbolic thinking, egocentrism, and centration symbolic thinking refers to the ability to pretend, play make-believe, and have an imagination egocentrism refers to the inability to imagine what another person may think or feel centration is the tendency to focus on only one aspect of a phenomenon, or inability to understand the concept of conservation. Ex: a kid is presented with 2 identical quantities of pizza: on one plate a single large slice, while the other plate has the exact same quantity in two slices. A child in this stage will be unable to tell that the quantities are equal and will focus mainly on the number of slices on the plate rather than the actual quantity.

Explain the S stage of cell division.

Synthesis Stage. The cell replicates its genetic material so each daughter cell has 2 identical copies. After replication, the chromosome consists of 2 identical chromatids that are bound together at a centromere. Note-the ploidy of the cell does not change even though the number of chromatids has doubled. Humans at this stage still only have 46 chromosomes, even though 92 chromatids are present. The term chromosome can be used to refer to a single chromatid before S phase or the pair of chromatids attached at the centromere after S phase. Cells entering G2 have 2x as much DNA as cells in G1

What is gluconeogenesis and where does it occur? What is this promoted and inhibited by?

Synthesis of glucose from non-carbohydrate substrates. Primarily in the liver. The kidney can also carry out gluconeogenesis, although its contribution is much smaller. These pathways are promoted by glucagon and epinephrine, which act to raise blood sugar levels, and are inhibited by insulin, which acts to lower blood sugar levels.

Explain the function of T cells in the immune system and where they are located.

T-cells, another class of adaptive immune cells, mature in the thymus, a small gland located just in front of the pericardium, the sac that protects the heart. T-cells are agents of cell-mediated immunity because they coordinate the immune system and directly kill virally infected cells.

Explain the fetal shunts and why they exist

The fetus doesn't depend on its own lungs and liver. These two organs are both underdeveloped and sensitive to high blood pressure, thus the fetus constructs 3 shunts to actively direct blood away from these organs while they develop. Two shunts are used to reroute blood from the lungs. -Foramen oval: a one-way valve that connects the right atrium to the left atrium. This allows blood entering the right atrium from the inferior vena can to flow into the left atrium instead of the right ventricle, and thereby be pumped through the aorta into systemic circulation directly. Unlike in adult circulation, the right side of the heart is at higher pressure in the developing fetus than the left side, which pushes blood through the opening. After birth this pressure differential reverses, shutting the foramen oval. -Ductus arteriosus: shunts leftover blood from the pulmonary artery to the aorta. The pressure differential between the right and left sides of the heart pushes blood through this opening and into systemic circulation. The liver is bypassed via the ductus venous, which shunts blood returning from the placenta via the umbilical vein directly into the inferior vena cava. The liver still receives some blood supply from smaller hepatic arteries in the systemic circulation.

How does the immune function of the lungs work?

The first line of defense is the nasal cavity, which has small hairs (vibrissae) that help to trap particulate matter and potentially infectious particles. The nasal cavity also contains an enzyme called lysozyme. Also found in tears and saliva, lysozyme is able to attack the peptidoglycan walls of gram-positive bacteria. The internal airways are lined with mucus, which traps particulate matter and larger invaders. Underlying cilia then propel the mucus up the respiratory tract to the oral cavity, where it can be expelled or swallowed; this mechanism is called the mucociliar escalator. The lungs, esp the alveoli, contain numerous immune cells, including macrophages. Macrophages can engulf and digest pathogens and signal to the rest of the immune system that there is an invader. Mucosal surfaces also contain IgA antibodies that help to protect against pathogens that contact the mucous membranes. Finally, mast cells also populate the lungs. These cells have preformed antibodies on their surfaces. When the right substance attaches to the antibody, the mast cell releases inflammatory chemicals into the surrounding area to promote an immune response. Unfortunately, these antibodies are often reactive to substances such as pollen and molds, so mast cells also provide the inflammatory chemicals that mediate allergic reactions.

What are the first steps in glucose metabolism? Explain the roles of Hexokinase and Glucokinase in glycolysis: What tissues are each present in? What is the Km of each of them? What are each inhibited or induced by?

The first steps in glucose metabolism in any cell are transport across the membrane and phosphorylation by kinase enzymes inside the cell to prevent glucose from leaving via the transporter. Kinases attach a phosphate group from ATP to their substrates. Glucose enters the cell by facilitated diffusion or active transport; in either case, these kinases convert glucose to glucose-6-phosphate. Because the GLUT transporters are specific for glucose (not phosphorylated glycose), the glucose gets "trapped" inside the cell and cannot leak out. Hexokinase is present in most tissues, has a low Km (reaches max velocity at low [glucose]), and is inhibited by glucose 6-phosphate. Glucokinase is present in hepatocytes and pancreatic B-islet cells (along with GLUT 2, acts as the glucose sensor), has a high Km (acts on glucose proportionally to its [ ]), and is induced by insulin in hepatocytes.

Explain the secretion function of the kidneys.

The nephrons are able to secrete salts, acids, bases, and urea directly into the tubule by either active or passive transport. The quantity and identity of the substances secreted into the nephron are directly related to the needs of the body at that time. Ex: a diet heavy in meat results in large amounts of protein, which contains lots of nitrogen.Ammonia is a byproduct of the metabolism of nitrogen-containing compounds, and, as a base, can disturb the pH of blood and cells. The liver converts the ammonia to urea, a neutral compound, which travels to the kidney and is secreted into the nephron for excretion in the urine. The kidneys are capable of eliminating ions or other substances when present in relative excess in the blood, such as K cations, H ions, or metabolites of medications. Secretion is also a mechanism for excreting wastes that are too large to pass through glomerular pores.

Explain the difference between the inner and outer mitochondrial membrane.

The outer mitochondrial membrane is highly permeable due to many large pores that allow the passage of ions and small proteins. The outer membrane completely surrounds the inner mitochondrial membrane, with the presence of a small intermembrane space in between the two layers. The inner mitochondrial membrane has a much more restricted permeability compared to the outer mitochondrial membrane. Structurally, the inner mitochondrial membrane contains numerous infoldings, known as cristae, which increase the available surgace area for the integral proteins associated with the membrane. The inner mitochondrial membrane also encloses the mitochondrial matrix, where the citric acid cycle produces high-energy electron carriers used in the electron transport chain. The inner membrane contains a very high level of cardiolipin and does not contain cholesterol.

Explain chemiosmotic coupling.

The proton-motive force interacts with the portion of ATP synthase that spans the membrane, which is called the F0 portion. F0 functions as an ion channel, so protons travel through F0 along their gradient back into the matrix. As this happens, a process called chemiosmotic coupling allows the chemical energy of the gradient to be harnessed as a means of phosphorylating ADP, forming ATP. In other words the ETC generates a high [ ] of protons in the intermembrane space; the protons then flow through the F0 ion channel of ATP synthase back into the matrix. As this happens, the other portion of ATP synthase, which is called the F1 portion, utilizes the energy released from this electrochemical gradient to phosphorylate ADP to ATP.

Explain the Avery, MacLeod, and McCarty experiment

They started out with IIIS (virulent pneumonia), then heat killed them, and then obtained a filtrate that retained the ability to induce transformation of IIR (avirulent) cells (they tested this in their control to make sure that the filtrate could still induce transformation in the IIR cells). They extracted carbohydrates, lipids, and proteins, because these are some of the things that all organisms contain, and after just this step, they realized that the IIR cells were still transformed, so neither carbohydrates, lipids, nor proteins were the transforming principle. Then they treated the filtrate with protease to destroy any remaining protein activity, and it was found that transformation still occurred, so protein was not the transforming principle. Then the filtrate was treated with ribonuclease to destroy any RNA, and it was found that transformation still occurred, meaning that RNA was not the transforming principle. After the filtrate was treated with deoxyribonuclease, it was found that no transformation occurred, meaning that DNA was the transforming principle and was the genetic material because the transformation was heritable.

parallel evolution

Two related species that have made similar evolutionary adaptations after their divergence from a common ancestor

How are thymine dimers formed? Explain how nucleotide excision repair works.

UV light induces the formation of dimers between adjacent thymine residues in DNA. The formation of thymine dimers interferes with DNA replication and normal gene expression, and distorts the shape of the double helix. Thymine dimers are eliminated from the DNA by a nucleotide excision repair (NER) mechanism which is a cut-and-patch process. First, specific proteins scan the DNA molecule and recognize the lesion because of a bulge in the strand. An excision endonuclease then makes nicks in the phosphodiester backbone of the damaged strand on both sides of the thymine dimer and removes the defective oligonucleotide. DNA polymerase can then fill in the gap by synthesizing DNA in the 5' to 3' direction, using the undamaged strand as a template. Finally, the nick in the strand is sealed by DNA ligase.

What happens during birth?

Vaginal childbirth (parturition) is accomplished by rhythmic contractions of uterine smooth muscle, coordinated by prostaglandins and the peptide hormone oxytocin. Birth consists of 3 basic phases: 1. The cervix thins out and the amniotic sac ruptures (water breaking) 2. strong uterine contractions result in the birth of the fetus 3. The placenta and umbilical cord are expelled (afterbirth)

What are mineralocorticoids and what is their function? Where are they secreted from? What is the most noteworthy mineralocorticoid and what does it do in the body?

a class of corticosteroids, secreted from the adrenal cortex used in salt and water homeostasis; their most profound effects are on the kidneys aldosterone **- increases sodium reabsorption in the distal convoluted tubule and collecting duct of the nephron. Water follows the sodium cations in to the bloodstream, increasing blood volume and pressure. Since water and sodium ions flow together, plasma osmolarity remains unchanged (Contrast to ADH, which only increases water reabsorption, decreasing plasma osmolarity). -also decreases the reabsorption of K+ and H+ ions in these same segments of the nephron, promoting their excretion in to the urine

What are G-protein coupled receptors? How are they characterized? In order for GPCRs to transmit signals to an effector in the cell, they utilize what? What does the binding of a ligand to to the G protein?

a family of integral membrane protenis involved in signal transduction characterized by their seven membrane spanning alpha-helices. they utilize a heterotrimeric G protein binding of a ligand increases the affinity of the receptor for the G protein

Explain IQ tests

a founding concept behind these tests is Spearman's "g factor" or general intelligence factor. IQ=(mental age/chronological age) x 100

Explain the role of pyruvate carboxylase in gluconeogenesis.

a mitochondrial enzyme that is activated by acetyl-CoA (from B-oxidation). The product, oxaloacetate(OAA), is a citric acid cycle intermediate and cannot leave the mitochondrion. Rather, it is reduced to malate, which can leave the mitochondrion via the malate-aspartate shuttle. Once in the cytoplasm, malate is oxidized to OAA. The fact that acetyl-CoA activates pyruvate carboxylase is an important point. Acetyl-CoA inhibits pyruvate dehydrogenase because a high level of acetyl-CoA implies that the cell is energetically satisfied and need not run the citric acid cycle in the forward direction; in other words, the cell should stop burning glucose. Rather, pyruvate will be shunted through pyruvate carboxylase to help generate additional glucose through gluconeogenesis. Note that the source of acetyl-CoA is not from glycolysis and pyruvate dehydrogenase in this case, but from fatty acids. Thus, to produce glucose in the liver during gluconeogenesis, fatty acids must be burned to provide this energy, store the forward flow of the citric acid cycle, and produce massive amounts of OAA that can eventually lead to glucose production for the rest of the body.

What is secretin and what is its role in digestion?

a peptide hormone that causes pancreatic enzymes to be released into the duodenum. It regulates the pH of the digestive tract by reducing HCl secretion from parietal cells and increasing bicarbonate secretion from the pancreas. Secretin is also an enterogastrone, a hormone that slows motility through the digestive tract. Slowing of motility allows increased time for digestive enzymes to act on chime- especially fats.

Phospholipids contain what elements? How are they classified?

a phosphate and alcohol that comprise the polar group, joined to a hydrophobic fatty acid tail by phosphodiester linkages One or more fatty acids are attached to a backbone to form the hydrophobic tail region. Phospholipids can be further classified according to the backbone on which the molecule is build. Ex: glycerol, a 3-carbon alcohol, forms phosphoglycerides or glycerophospholipids, and sphingolipids have a sphingosine backbone.

Explain intron/ exon splicing. What is the evolutionary function of introns?

a posttranscriptional modificiation maturation of the hnRNA includes splicing of the transcript to remove noncoding sequences (introns) and ligate coding sequences (exons) together. Splicing is accomplished by the spliceosome. In the spliceosome, small nuclear RNA (snRNA) molecules couple with proteins known as small nuclear ribonucleoproteins (also known as snRNPs or "snurps"). The snRNP/snRNA complex recognizes both the 5' and 3' splice sites of the introns. The introns are excised in the form of a lariat (lasso shaped structure) and then degraded. The evolutionary function of introns in eukaryotic cells is hypothesized that introns play an important role in the regulation of cellular gene expression levels and in maintaining the size of our genome. The existence of introns has also been hypothesized to allow for rapid protein evolution.

what are chemoreceptors? how can they enact changes in BP?

a receptor that can sense when the osmolarity of the blood is too high, which could indicate dehydration. This promotes the release of antidiuretic hormone (ADH or vasopressin), a peptide hormone made in the hypothalamus and stored in the posterior pit, which increases the reabsorption of water, thereby increasing blood volume and pressure (while also dilating the blood). Low perfusion to the juxtaglomerular cells of the kidney stimulates aldosterone release through the renin-angiotensin-aldosterone system; aldosterone increases the reabsorption of sodium and, by extension, water, thereby increasing the blood volume and pressure

Describe the structure and function of the cell (plasma) membrane.

a semipermeable phospholipid bilayer As a semipermeable barrier, it chooses which particles can enter and leave the cell at any point in time. This selectivity is mediated not only by the various channels and carriers that pole holes in the membrane, but also by the membrane itself. Composed of primarily two layers of phospholipids, the cell membrane permits fat-soluble compounds to cross easily, while larger and water-soluble compounds must seek alternative entry. The phospholipid bilayer also includes proteins and distinct signaling areas within lipid rafts. Carbohydrates associated with membrane-bound proteins create a glycoprotein coat. The cell wall of plants, bacteria, and fungi contain higher levels of carbohydrates. The main function of the cell membrane is to protect the interior of the cell from the external environment. Cellular membranes selectively regulate traffic into and out of the cell and are involved in both intracellular and intercellular communication and transport. Cell membranes also contain proteins embedded within the lipid bilayer that act as cellular receptors during signal transduction. These proteins play an important role in regulating and maintaining overall cellular activity.

What is a sleep cycle?

a single complete progression through the sleep stages. The makeup of a sleep cycle changes during the course of the night. Early in the night, SWS predominates as the brain falls into deep sleep and then into more wakeful stages. Later in the night, REM sleep predominates. Over the lifespan the length of the sleep cycle increases from approx 50 minutes in kids to 90 minutes in adults. Children also spend more time in SWS than adults. May heath problems such as memory disruption and diminished cognitive performance can result from sleep cycle changes/disruption.

What is isoelectric focusing? How does it work?

a specialized form of electrophoresis where proteins are separated on the basis of isoelectric pH (pI). The pI is the pH at which the protein or amino acid is electrically neutral, with an equal number of positive and negative charges. The mixture of proteins is placed in a gel with a pH gradient. An electric field is then generated across the gel. Proteins that are positively charged migrate towards the cathode and ones that are negatively charged migrate toward the anode. As the protein reaches the portion of the gel where the pH is equal to the protein's pI, the protein takes on a neutral charge and will stop moving. Ex: if you have a protein with a pI=9, when the protein is in an environment where pH=9 then it will carry no net charge. If we place this protein onto the gel at pH=7, there will be more protons around the protein. These protons will attach to the available basic sites on the protein, creating a net positive charge on the molecule. This charge will then cause the protein to be attracted to the negatively charged cathode, which is located on the basic side of the gradient. As the protein moves closer to the cathode, the pH of the gel slowly increases. Eventually, as the protein nears a pH of 9, the protons creating the positive charge will dissociate, and the protein will become neutral again. .

What is the importance of cholesterol? What is it?

a steroid (lipid). - a major component of the phospholipid bilayer and is responsible for mediating membrane fluidity. -like a phospholipid, is an amphipathic molecule - interactions with both the hydrophobic tails and hydrophilic heads of phospholipids allows cholesterol to maintain relatively constant fluidity in cell membranes. - At low temps, it keeps the cell membrane from solidifying, at high temps it holds the membrane intact and prevents it from becoming too permeable.

Silent mutation. Most common type

a type of point mutation that occurs when the change in nucleotide has no effect on the final protein synthesized from the gene. This most commonly occurs when the changed nucleotide is transcribed to be the third nucleotide in a codon because there is degeneracy (wobble) in the genetic code

What is a vitamin? What are the categories? What is the difference?

an essential nutrient that cannot be adequately synthesized by the body and therefore must be consumed in the diet water-soluble and lipid-soluble. - lipid soluble vitamins can accumulate in stored fat, whereas excess water-soluble vitamins are excreted through the urine

what is the founder effect

an extreme case of genetic drift in which a small populatin of a species finds itself in reproductive isolation from other populations as a result of natural barriers, catastrophic events, or other bottlenecks that drastically and suffenly reduce the size of the population available for breeding. Because the breeding group is small, inbreeding may occur in later generations.

What is Ketolysis?

acetoacetate picked up from the blood is activated in the mitochondria by succinyl-CoA acetoacetyl-CoA transferase (commonly called thiophrase), an enzyme present only in tissues outside the liver. During this reaction, 3-hydroxybutyrate is oxidized to acetoacetate. The liver lacks this enzyme, so it cannot catabolize the ketone bodies that it produces.

what is the role of acetyl-CoA carboxylase?

acetyl-CoA is activated in the cytoplasm for incorporation into fatty acids by acetyl-CoA carboxylase, the rate-limiting enzyme of fatty acid biosyntheis. It requires biotin and ATP to function and adds CO2 to acetyl-CoA to form malonyl-CoA. The enzyme is activated by insulin and citrate. The CO2 added to form malonyl-CoA is never actually incorporated into the fatty acid because it is removed by fatty acid synthase during addition of the activated acetyl group to the fatty acid.

Explain sleep apnea

an inability to breathe during sleep can be either obstructive or central. Obstructive sleep apnea occurs when a physical blockage in the pharynx or trachea prevents airflow; central sleep apnea occurs when the brain fails to send signal so the diaphragm to breathe

Explain what the liver does after a meal? what about in the well fed state? what about between meals and during prolonged fasts?

after a mean, [glucose] in the portal blood is elevated. the liver extracts excess glucose and uses it to replenish its glycogen stores. any glucose remaining in the liver is then converted to acetyl-CoA and used for fatty acid synthesis. the increase in insulin after a meal stimulates both glycogen synthesis and fatty acid synthesis in the liver. the fatty acids are converted to triacylglycerols and released into the blood as VLDLs. In the well-fed state, the liver derives most of its energy from the oxidation of excess amino acids. between meals and during prolonged fasts, the liver releases glucose into the blood. Between meals and during prolonged fasts, the liver releases glucose into the blood. The increase in glucagon during fasting promotes both glycogen degradation and gluconeogenesis. Lactate from anaerobic metabolism, glycerol from triacylglycerols, and amino acids provide carbon skeletons for glucose synthesis.

What are MHC molecules? What are the 2 classes? Explain them.

after digesting an invader, macrophages present little pieces of it (mostly peptides) to other cells using a protein called major histocompatibility complex (MHC). MHC binds to a pathogenic peptide (antigen) and carries it to the cell surface, where it can be recognized by cells of the adaptive immune system. Class I and class II. All nucleated cells in the body display MHC class I molecules. Any protein produced within a cell can be loaded onto MHC-1 and presented on the surface of the cell. This allows the immune system to monitor the health of these cells and to detect if the cells have been infected with a virus or another intracellular pathogen; only those cells that are infected would be expected to present an unfamiliar (nonself) protein on their surfaces. Therefore, the MHC-1 pathways is often called the endogenous pathways because it binds antigens that come from inside the cell. Cells that have been invaded by intracellular pathogens can then be killed by a certain group of T-cells (cytotoxic T-lymphocytes) to prevent infection of other cells. MHC class II molecules are mainly displayed by professional antigen-presenting cells like macrophages. Remember that these phagocytic cells pick up pathogens from the environment, process them, and then present them on MHC-II. An antigen is a substance (usually a pathogenic protein) that can be targeted by an antibody. While antibody production is the domain of the adaptive immune system, cells of the innate immune system also present antigens. Because these antigens originated outside the cell, the MHC-II pathways is often called the exogenous pathway. The presentation of an antigen by an immune cell may result in the activation of both the innate and adaptive immune systems. Professional antigen-presenting cells include macrophages, dendritic cells in the skin, some B-cells, and certain activated epithelial cells.

what are lipoproteins? How are they named? What are the different types?

aggregates of apolipoproteins and lipids. Lipoproteins are named according to their density, which increases in direct proportion to the percentage of protin in the particle. Chylomicrons are the lease dense, with the highest fat-to-protein ratio. VLDL (very low density lipoprotein) is slightly more dense, follwed by IDL (intermediate-density), LDL (low density), and HDL (high density).

What are apolipoproteins?

aka apoproteins. they form the protein component of lipoproteins. They are receptor molecules and are involved in signaling.

What is the postprandial state? When does it occur? Explain this state. how long does it last? Explain the three major target tissues of insulin and its method of action and purpose. What types of cells are insensitive to insulin?

aka the absorptive or well-fed state. This state occurs shortly after eating. It is marked by greater anabolism (synthesis of biomolecules) and fuel storage than catabolism. Nutrients flood in from the gut and make their way via the hepatic portal vein to the liver, where they can be stored or distributed to other tissues. the postprandial state generally lasts 3-5 hours after eating a meal. Just after eating, blood glucose levels rise and stimulate the release of insulin. The three major target tissues for insulin are the liver, muscle, and adipose tissue. Insulin promotes glycogen synthesis in liver and muscle. After the glycogen stores are filled, the liver converts excess glucose to fatty acids and triacylglycerols. Insulin promotes triacylglycerol synthesis in adipose tissue and protein synthesis in muscle, as well as glycose entry into both tissues. After a meal, most of the energy needs of the liver are met by the oxidation of excess amino acids. Two types of cells-nervous tissue and RBCs- are notably insensitive to insulin. Nervous tissue derives energy from oxidizing glucose to CO2 and water in both the well-fed and normal fasting states. Only in prolonged fasting does this situation change. RBCs can only use glucose anaerobically for all their energy needs, regardless of the individual's metabolic state.

As monosaccharides switch between anomeric configuratinos, the hemiacetal rings spend a sport period of time in the open-chain aldehyde form. Just like other aldehydes, they can be oxidized to carboxylic acids; these oxidized aldoses are called ___________________. Because aldoses can be oxidized, they are considered ___________________. Therefore, any monosaccharide with a hemiacetal ring is considered a ____________________. When the aldose in question is in ring form, oxidation yields a ________ instead. What is this?

aldonic acids reducing agents reducing sugar lactone- a cyclic ester with a carbonyl group persisting on the anomeric carbon

What are the states of consciousness?

alertness, sleep, dreaming, altered states of consciousness

What does recombinant DNA technology do?

allows a DNA fragment from any source to be multiplied by either gene cloning or PCR. This provides a means of analyzing and altering genes and proteins. It also provides the reagents necessary for genetic testing, such as carrier detection and prenatal diagnosis of genetic diseases; it is also useful for gene therapy.

What are glial cells?

also called neuroglia. cells in the nervous system that support, nourish, and protect neurons

what is the proton-motive force?

an electrochemical proton gradient generated by the complexes of the electron transport chain

What is enteropeptidase and what is its role in digestion?

an enzyme critical for the activation of trypsinogen, a pancreatic protease, to trypsin. Trypsin then initiates an activation cascade. Enteropeptidase can also activate procarboxypeptidases A and B to their active forms.

when antibodies bind to their targets, called _____, they can cause one of three outcomes. What are they?

antigens 1. neutralizing the antigen, making the pathogen or toxin unable to exert its effect on the body 2. marking the pathogen for destruction by other white blood cells immediately; this marking function is also called opsonization 3. clumping together (agglutinating) the antigen and antibody into large insoluble protein complexes that can be phagocytize and digested by macrophages

catecholamines are secreted by what and include what two hormones? How do they affect the activity of the liver and muscle glycogen phosphorylase? What are the effects of this? How do catecholamines effect glycogenolysis and lipolysis?

by the adrenal medulla and include epi and norepi. They increase the activity of the liver and muscle glycogen phosphorylase, thus promoting glycogenolysis. this increases glucose output by the liver. Glycogenolsis also increases in skeletal muscle, but because muscle lacks glucose-6-phosphatase, glucose cannot be released by skeletal muscle into the bloodstream; instead, it is metabolized by the muscle tissue itself. Catecholamines act on adipose tissue to increase lipolysis by increasing the activity of hormone-sensitive lipase. Glycerol from triacylglycerol breakdown is a minor substrate for gluconeogenesis. Epi also directly acts on target organs like the heart to increase the basal metabolic rate through the sympathetic nervous system. This increase in metabolic function is often associated with an adrenaline rush.

What is the job of mRNA? How is it transcribed? What happens after that?

carries the information specifying the amino acid sequence of the protein to the ribosome. mRNA is transcribed from the template DNA strands by RNA polymerase enzymes in the nucleus of the cells. Adter that, mRNA may undergo a host of posttranscriptional modifications prior to its release from the nucleus.

Explain the role of Glyceraldehyde 3-phosphate dehydrogenase in glycolysis.

catalyzes an oxidation and addition of inorganic phosphate (Pi) to its substrate, glyceraldehyde 3-phosphate. This results in the production of a high-energy intermediate 1,3-bisphosphoglycerate and the reduction of NAD+ to NADH. If the glycolysis is aerobic, the NADH can be oxidized by the mitochondrial electron transport chain, providing energy for ATP synthesis by oxidative phosphorylation.

What are cell adhesion molecules (CAMs)? what are the three major families? explain them

cell-surface proteins that bind the cell to an adjacent cell and to components of the extracellular matrix (ECM). cadherins, integrins, and selectins cadherins- a group of glycoproteins that mediate calcium-dependent cell adhesion. Cadherins often hold similar cell types together, such as epithelial cells. integrins-a group of proteins that all have two membrane-spanning chains called alpha and beta. These chains are v. important in binding to and communicate with the extracellular matrix. Integrins also play a very important role in cellular signaling and can greatly impact cellular function by promoting cell division or apoptosis or other processes. Selectins-unique because they bind to carbohydrate molecules that project from other cell surfaces. These bonds are the weakest formed by the CAMs discussed here. Like integrins, they play an important role in host defense, including inflammation and white blood cell migration

Explain the structure and function of cholesterol in the cell membrane

cholesterol not only regulates membrane fluidity but it is also necessary in the synthesis of all steroids, which are derived from cholesterol. The structure of cholesterol is similar to that of phospholipids in that cholesterol contains both a hydrophilic and hydrophobic region. Membrane stability is derived from interactions with both the hydrophilic and hydrophobic regions that make up the phospholipid bilayer. While cholesterol stabilizes adjacent phospholipids, it also occupies space between them. This prevents the formation of crystal structures in the membrane, increasing fluidity at lower temperatures. At higher temperatures, cholesterol has the opposite effect: by limiting movement of phospholipids within the bilayer, it decreases fluidity and helps hold the membrane intact. By mass, cholesterol composes about 20% of the cell membrane; by mole fraction, it makes up about half. This large ratio of cholesterol to phospholipid ensures that the membrane remains fluid.

What is chromatin? Explain heterochromatin vs euchromatin.

chromatin is how DNA gets packaged in the nucleus. Heterochromatin is tightly coiled DNA that appears dark under a microscope; its tight coiling makes it inaccessible to the transcription machinery, so these genes are inactive. Euchromatin is looser and appears light under the microscope; the transcription machinery can access the genes of interest, so these genes are active.

What are the two classes and 5 basic types of leukocytes?

classes: granulocytes and agranulocytes Granulocytes: Neutrophils, eosinophils, basophils Agranulocytes: lymphocytes and monocytes

osmotic pressure is a ________ property. what does this mean?

colligative a physical property of solutions that is dependent on the [ ]s of the dissolved particles but not on the chemical identity of those dissolved propertiesgfgg

what are zymogens?

contain a catalytic (active) domain and a regulatory domain. The regulatory domain must be either removed or altered to expose the active site. Apoptotic enzymes (capases) exhibit similar regulation most zymogens have the suffix -ogen,

Explain Complex IV of the electron transport chain.

cytochrome c oxidase 4 cytochrome c [with Fe2+] + 4H+ + O2 --> 4 cytochrome c [with Fe3+] + 2H2O This complex facilitates the culminating step of the electron transport chain: transfer of electrons from cytochrome c to oxygen, the final electron acceptor. This complex includes subunits of cytochrome a, cytochrome a3, and Cu2+ ions. Together, cytochromes a and a3 make up cytochrome oxidase. Through a series of redox reactions, cytochrome oxidase gets oxidized as oxygen, becomes reduced, and forms water. This is the final location on the transport chain where proton pumping occurs, as two protons are moved across the membrane.

Explain the malate-aspartate NADH shuttle

cytosolic oxaloacetate, which cannot pass through the inner mitochondrial membrane, is reduced to malate, which can. This is accomplished by cytosolic malate dehydrogenase. Accompanying this reduction is the oxidation of cytosolic NADH to NAD+. Once malate crosses into the matrix, mitochondrial malate dehydrogenase reverses the reaction to form mitochondrial NADH. Now that NADH is in the matrix, it can pass along its electrons ot the ETC via Complex I and generate 2.5 ATP per molecule of NADH. Recycling the malate requires oxidation to oxaloacetate, which can be transaminated to form aspartate. Aspartate crosses into the cytosol, and can be reconverted to oxaloacetate to restart the cycle.

Where is the pineal gland located and what does it secrete?

deep in the brain, secretes melatonin

What are the different groups of consciousness-altering drugs?

depressants, stimulants, opiates, and hallucinogens

adaptive radiation

describes the rapid rise of a number of different species from a common ancestor. the benefit is that it allows for various species to occupy different niches.

Explain the problem-solving dream theory

dreams are a way to solve problems while you are sleeping. Dreams are untethered by the reules of the world, and thus allow interpretation of obstacles differently than during waking hours.

Explain the activation-synthesis dream theory.

dreams are caused by widespread, random activation of neural circuitry. This activation can mimic incoming sensory info, and may also consist of pieces of stored memories, current and previous desires, met and unmet needs, and other experiences. The cortex then tries to stitch this unrelated info together, resulting in a dream that is both bizarre and somewhat familiar.

Explain the cognitive process dream theory

dreams are merely the sleeping counterpart of stream-of-consciousness. Just as you may be thinking about an upcoming weekend trip when your consciousness quickly shifts to your upcoming MCAT, so too does the content of a dream rapidly shift an dchange.

What are stimulants?

drugs that excite neural activity and speed up body functions

Explain male sexual development

during the fetal period (9 wks after fertilization until birth), presence of Y chromosome leads to production of androgens, resulting in male sexual differentiation. For infancy and childhood, androgen production is low. Testosterone, produced by the testes, inc dramatically during puberty and sperm production begins. In order to achieve this, there is a delicate interplay of FSH and LH stimulation on two cell types in the testes. FSH stimulates the Sertoli cells and triggers sperm maturation LH causes the interstitial cells to produce testosterone. Testosterone not only develops and maintains the male reproductive system, but also results in the development of secondary sexual characteristics such as facial and axillary hair, deepening of the voice, inc muscle and bone mass. Testosterone exerts neg feedback on the hypothalamus and ant pit

Explain endocytosis and exocytosis.

endocytosis: transporting of materials into the cell using vesicles cell membrane invaginates and engulfs material to bring it into the cell. The material is encased in a vesicle, which is important because cells will sometimes ingest toxic substances. Pinocytosis is the endocytosis of fluids and dissolved particles, whereas phagocytosis si the ingestion of large solids such as bacteria. Substrate binding to specific receptors embedded within the plasma membrane will initiate the process of endocytosis. Invagination will then be initiated and carried out by vesicle-coating proteins, most notably clathrin. exocytosis: transporting of materials out of the cell using vesicles. Occurs when secretory vesicles fuse with the membrane, releasing material from inside the cell to the extracellular environment. Exocytosis is important in the nervous system and intercellular signaling. For instance, exocytosis of neurotransmitters from synaptic vesicles is a crucial aspect of neuron physiology.

What are retroviruses? How do they work? Give an example.

enveloped, single-stranded RNA viruses able to transcribe their RNA into double stranded DNA. HIV is an example They carry an enzyme called reverse transcriptase which synthesized DNA from single stranded RNA. The DNA then integrates in to the host cell genome, where it is replicated and transcribed as if it were the host cell's own DNA. The only way to remove the infection is to kill the infected cell.

What are restriction enzymes (restriction endonucleases) and how do they work? Where do they come from? What is their original purpose?

enzymes that recognize specific double-stranded DNA sequences. These sequences are palindromic, meaning that the 5' to 3' sequence of one strand is identical to the 5' to 3' sequence of the other strand (in antiparallel orientation). Restriction enzymes are isolated from bacteria, which are their natural source. In bacteria, they act as part of a restriction and modification system that protects the bacteria from infection by DNA viruses. Once a specific sequence has been identified, the restriction enzyme can cut through the backbones of the double helix. Some restriction enzymes produce offset cuts, yielding sticky ends on the fragments. Sticky ends are advantageous in facilitating the recombination of a restriction fragment with the vector DNA. The vector of choice can also be cut with the same restriction enzyme, allowing the fragments to be inserted directly into the vector.

What are waxes structurally made of? Why are they important?

esters of long-chain fatty acids with long-chain alcohols - form pliable solids at room temp (wax) - biologically, they function as protection for both plants and animals. In plants, waxes are secreted as a surface coating to prevent excessive evaporation and to protect against parasites. In animals, waxes are secreted to prevent dehydration, as a water-repellant to keep skin and feathers dry, and as a lubricant.

What is Vitamin A? Explain its importance.

fat soluble vitamin Vitamin A (carotene)- unsaturated hydrocarbon that is important in vision, growth and development, and immune function. - most significant metabolite of vitamin A is retinal (the storage form of vitamin A), which is a component of the light-sensing molecular system in the human eye.

What is Vitamin D? Explain its importance.

fat soluble vitamin. Cholecalciferol -can be consumed or formed in a UV-light driven reaction in the skin - in the liver and kidneys, vitamin D is converted to calcitriol, the biologically active form of vitamin D. - calcitriol increases calcium and phosphate uptake in the intestines, which promotes bone production * a lack of Vitamin D can result in rickets, a condition seen in kids and characterized by underdeveloped, curved long bones as well as impeded growth

What is Vitamin E? Explain its importance.

fat-soluble vitamin -characterizes a group of closely related lipids called tocopherols and tocotrienols - these are characterized by a substituted aromatic ring with a long isoprenoid side chain and are characteristically hydrophobic - Tocopherols are biological antioxidants. The aromatic ring reacts w/ free radicals, destroying them. This in turn prevents oxidative damage, an important contributor to the development of cancer and aging

what is directional selection

favors individuals at one end of the phenotypic range

What is stabilizing selection?

favors intermediate variants and acts against extreme phenotypes

Explain the difference between fluid and crystallized intelligence. How do relate to what age you are?

fluid intelligence is problem solving skills crystallized intelligence is related to use of learned skills and knowledge. Fluid intelligence peaks in early adulthood, while crystallized intelligence peaked in middle adulthood. Both types have been shown to decline with age.

What are centrioles? What are they made of? Where are they found? What activity are they involved in?

found in a region of the cell called the centrosome. They are the organizing centers for microtubules and are structured as 9 triplets of microtubules with a hollow center. During mitosis, the centrioles migrate to opposite poles of the dividing cell and organize the mitotic spindle. The microtubules emanating from the centrioles attach to the chromosomes via complexes called kinetochores and exert force on the sister chromatids, pulling them apart.

explain the role of glucose-6-phosphatase in gluconeogenesis.

found only in the lumen of the ER in liver cells. It is transported into the ER and free glucose is transported back into the cytoplasm, from where it can diffuse out of the cell using GLUT transporters. The absence of G6P in skeletal muscle means that muscle glycogen cannot serve as a source of blood glucose and rather is for use only within the muscle. G6P is used to circumvent glucokinase and hexokinase, which convert glucose to G6P.

how are free fatty acids, triacylglycerol, and cholesterol transported in the blood, respectively?

free fatty acids are transported in association with albumin, a carrier protein. Triacylglycerol and cholesterol are transported in the blood as lipoproteins

explain micelle formation, especially in relation to digestion.

free fatty acids, cholesterol, 2-monoacylglycerol, and bile salts contribute to the formation of micelles, which are clusters of amphipathic lipids that are soluble in the aqueous environment of the intestinal lumen. Essentially, micelles are water-soluble spheres with a lipid-soluble interior. Micelles are vital in digestion, transport, and absorption of lipid-soluble substances starting form the duodenum all the way to the end of the ileum. At the end of the ileum, bile salts are actively reabsorbed and recycled; any fat that remains in the intestine will pass into the colon and ultimately ends up in the stool.

The endocrine system consists of organs, known as _______ that secrete _______.

glands that secrete hormones

what is the van't Hoff factor of glucose? what about NaCl?

glucose is 1 because glucose does not dissociate in solution NaCl is 2 because it dissociates into Na+ and Cl- in solution van't Hoff factor is simply the number of particles obtained from the molecule when in solution.

What is the rate limiting enzyme of the pentose phosphate pathway? What is it induced and inhibited by?

glucose-6-phosphate dehydrogenase (G6PD). it is induced by insulin because the abundance of sugar entering the cell under insulin stimulation will be shunted into both fuel utilization pathways (glycolysis and aerobic respiration) as well as fuel storage pathways (fatty acid synthesis, glycogenesis, and the PPP). The shunt is also inhibited by its product, NADPH, and is activated by one of its reactants, NADP+.

What are basophils?

granulocytes. a type of WBC. contain large purple granules and are involved in allergic responses. They are the least populous leukocyte in the bloodstream under normal conditions. Mast cells are closely related to basophils, but have smaller granules and exist in the tissues, mucosa, and epithelium. Both basophils and mast cells release large amounts of histamine in response to allergens, leading to inflammatory responses.

What are neutrophils?

granulocytes. a type of WBC. the most populous leukocyte in the blood and are very short-lived (a bit more than 5 days). These cells are phagocytic, like macrophages, and target bacteria. Neutrophils can follow bacteria using chemotaxis-the movement of an organism according to chemical stimuli; in this case, the neutrophil senses products given off by the bacteria, moving up the [ ] gradient to the source. Neutrophils can also detect bacteria once they have been opsonized (marked with an antibody from a B-cell) Other cells, like NK cells, macrophages, monocytes, and eosinophils, also contain receptors for antibodies and can attack opsonized bacteria. Dead neutrophil collections are responsible for the formation of pus during an infection.

what are allosteric enzymes?

have multiple binding sites. The active site is present, as well as at least one other site that can regulate the availability of the active site. the type of enzymes that act as the rate limiting steps for an entire metabolic pathway they alternate between active and inactive forms inactive forms can't carry out the enzymatic reaction molecules that bind to the allosteric site may be either allosteric activators or allosteric inhibitors. binding of either causes a conformational shift in the protein

What are the 3 major types of T cells?

helper T-cells, suppressor T-cells, and killer (cytotoxic) T-cells

What are HDLs? Where are they synthesized? How are they released? What are their functions?

high density lipoprotein The highest density of lipoprotein. synthesized in the liver and intestines and released as dense, protein-rich particles into teh blood. HDL contains apolipoproteins and used for cholesterol recovery-that is, cleaning up of excess cholesterol from blood vessels for excretion. HDL also delivers some cholesterol to steroidogenic tissues and transfers necessary apolipoproteins to some of the other lipoproteins.

What is melatonin? What is it secreted by?

hormone secreted by the pineal gland in the brain. known to be involved in circadian rhythms.

Decline in intellectual abilities in adulthood has been linked with what?

how long an older adult retains the ability to function in what is known as activities of daily living (eating, bathing, dressing, etc). Certain characteristics, such as higher levels of education, more frequent performance of intellectual activities, socialization, and a stimulating environment have been found to be protective against intellectual decline.

explain syntax in terms of language

how words are put together to form sentences. a child must notice the effects of word order on meaning: Nathan has only 3 pieces of candy has a difference in meaning than only Nathan has 3 3 pieces of candy.

what are the layers of the skin starting form the deepest working outward?

hypodermis (subcutaneous layer), dermis, and epidermis

What are the three types of solutions? Explain them and what happens to the water in them via osmosis.

hypotonic, hypertonic, and isotonic Hypotonic: [ ] of solutes inside the cell is higher than the surrounding solution. Cell will swell as water rushes in, sometimes to the point of lysis. Hypertonic: [ ] of solutes outside the cell is higher than in side the cell. Water moves out. Isotonic: [ ]s are equal inside and outside the cell. Isotonicity does not prevent movement, it prevents the net movement of particles. Water molecules will continue to move but the cell will neither gain nor lose water overall.

What are some similarities between meiosis and mitosis?

in both processes, genetic material must be duplicated, chromatin is condensed to form chromosomes, and microtubules emanating from centrioles are involved in dividing genetic material.

how does piloerection work? how about shivering?

in cold conditions, arrector pili muscles contract, causing the hairs of the skin to stand up on end (piloeretion). This helps to trap a layer of heated air near the skin. The arterioles that feed the capillaries of the skin contstrict, limiting the quantity of blood reaching the skin. Skeletal muscle may also begin to contract rapidly, causing shivering. shivering requires a sizable amount of ATP; however, a significant portion of the energy from ATP is converted into thermal energy. In addition to these mechanisms, humans possess a layer of fat just below the skin. This fat helps to insulate the body. In addition to this fat, which is called white fat, brown fat may also be present, especially in infants. Brown fat has a much less efficient electron transport chain, which means that more heat energy is released as fuel is burned.

describe inducible operon systems. Give an example.

in inducible systems, the repressor is bonded tightly to the operator system and thereby acts as a roadblock. RNA polymerase is unable to get from the promoter to the structural gene because the repressor is in the way. Such systems-in which the binding of a protein reduces transcriptional activity-are called negative control mechanisms. To remove that block, an inducer must bind the repressor protein so that RNA polymerase can move down the gene. Inducible systems operate on a principle analogous to competitive inhibition for enzyme activity: as the [ ] of the inducer increases, it will pull more copies of the repressor off of the operator region, freeing up those genes for transcription. This system is useful because it allows gene products to be produced only when they are needed. A classic example is the lac operon, which contains the gene for lactase. Bacteria can digest lactose, but it is more energetically expensive than digesting glucose. Therefore, bacteria only wants to use this option if lactose is high and glucose is low. The lac operon is induced by the presence of lactose; thus, these genes are only transcribed when it is useful to the cell. The lac operon is assisted by binding of the catabolite activator protein (CAP). CAP is a transcriptional activator used by E. coli when glucose levels are low to signal that alternative carbon sources should be used. Falling levels of glucose cause an increase in the signaling molecule cAMP, which binds to CAP. This induces a conformational change in CAP that allows it to bind the promoter region of the operon, further increasing transcription of the lactase gene. Such systems-in which the binding of a molecule increases transcription of a gene-are called positive control mechanisms.

what are chromosomal mutations? What are the categories?

larger-scale mutations in which large segments of the DNA are effected. Deletion, duplication, inversion, insertion, and translocation

What is fermentation? What is the key fermentation enzyme in mammalian cells? Explain what it does and how it works. How does fermentation work in yeast cells?

in the absence of oxygen, fermentation will occur. The key fermentation enzyme in mammalian cells is lactate dehydrogenase, which oxidizes NADH to NAD+, replenishing the oxidized coenzyme for glyceraldehyde-3-phosphate dehydrogenase. Without mitochondria and oxygen, glycolysis would stop when all the available NAD+ had been reduced to NADH. By reducing pyruvate to lactate and oxidizing NADH to NAD+, lactate dehydrogenase prevents this potential problem from developing. There is no net loss of carbon in this process: pyruvate and lactate are both three-carbon molecules. In aerobic tissues, lactate does not normally form in significant amounts. However, when oxygenation is poor (during strenuous exercise in skeletal muscle, a heart attack, or a stroke), most cellular ATP is generated by anaerobic glycolysis, and lactate production increases. In yeast cells, fermentation is the conversion of pyruvate (three carbons) to ethanol (two carbons) and carbon dioxide (one carbon). While the end products are different, the result of both mammalian and yeast fermentation is the same: replenishing NAD+//

Explain natural killer cells.

in the arms race between the human immune system and pathogens, some pathogens have found ways to avoid certain defenses. For example, some viruses cause downregulation of MC molecules, making it harder for T-cells to recognize the presence of an infection. Natural Killer (NK) cells, a type of nonspecific lymphocyte, are able to detect the downregulation of MHC and induce apoptosis in these virally infected cells. Cancer cells may also down-regulate MHC production, so NK cells also offer protection from the growth of cancer as well.

where are all cells of the immune system created?

in the bone marrow

How does insulin affect amino acid uptake?

insulin increases amino acid uptake by muscle cells, thereby increasing levels of protein synthesis and decreasing breakdown of essential proteins.

What is humoral immunity? How long does it take?

involves the production of antibodies. may take as long as a week to become fully effective after the initial infection. These antibodies are specific to the antigens of the invading microbe. Antibodies are produced by B cells, which are lymphocytes that originate and mature in the bone marrow and are activated in the spleen and lymph nodes.

what is absorption? (digestion related)

involves the transport of products of digestion from the digestive tract into the circulatory system for distribution to the body's tissues and cells.

is pyruvate dehydrogenase complex (PDH) reversible or irreversible? What is pyruvate dehydrogenase activated by in the liver vs the nervous system? What is pyruvate dehydrogenase inhibited by?

irreversible! cannot be used to convert acetyl CoA to pyruvate or glucose. in the liver it is activated by insulin whereas in the nervous system, the enzyme is not responsive to hormones. This makes sense because high insulin levels signal to the liver that the individual is in a well-fed state; thus, the liver should not only burn glucose for energy, but shift the fatty acid equilibrium toward production and storage rather than oxidation (fatty acid synthesis). Inhibited by its product, acetyl-CoA.

Explain skeletal muscle. Responsible for what type of movement? Innervated by what? How does it appear when viewed microscopically? Why? How many nuclei does skeletal muscle have? Why is this? Explain the different types of fibers within skeletal muscle.

is responsible for voluntary movement and is therefore innervated by the somatic nervous system. Due to the arrangement of actin and myosin into repeating units called sarcomeres, it appears striped or striated when viewed microscopically. Skeletal muscle is multinucleated because it is formed as individual muscle cells fuse into long rods during development. There are multiple different types of fibers within skeletal muscle. Red fibers, also known as slow-twitch fibers, have high myoglobin content and primarily derive their energy aerobically. Myoglobin is an oxygen carrier that uses iron in a heme group to bind oxygen, imparting a red color. Red fibers also contain many mitochondria to carry out oxidative phosphorylation. White fibers (fast-twitch fibers) contain much less myoglobin. Because there is less myoglobin, and therefore, less iron, the color is lighter. These two types of fibers can be mixed in muscles. Muscles that contract slowly, but that can sustain activity (such as the muscles that support posture), contain a predominance of red fibers. Muscles that contract rapidly but fatigue quickly contain mostly white fibers.

Where does acetyl-CoA accumulate after a large meal? where does it need to be for fatty acid biosynthesis? How does this happen?

it accumulates in the mitochondrial matrix and needs to be moved to the cytosol. Acetyl-CoA is the product of the pyruvate dehydrogenase complex and it couples with oxaloacetate to form citrate at the beginning o fthe citric acid cycle . Remember that isocitrate dehydrogenase is the rate-limiting enzyme of citric acid cycle, which causes citrate accumulation. Citrate can then diffuse across the mitochondrial membrane. In the cytosol, citrate lyase splits citrate back into acetyl-CoA and oxaloacetate. The oxaloacetate can then return to the mitochondrion to continue moving acetyl-CoA

What does aldosterone do? What type of hormone is it? Where is it secreted from?

it is a corticosteroid, more specifically, a mineralocorticoid, secreted by the adrenal cortex increases sodium reabsorption in the distal convoluted tubule and collecting duct of the nephron. Water follows the sodium cations in to the bloodstream, increasing blood volume and pressure. Since water and sodium ions flow together, plasma osmolarity remains unchanged (Contrast to ADH, which only increases water reabsorption, decreasing plasma osmolarity). -also decreases the reabsorption of K+ and H+ ions in these same segments of the nephron, promoting their excretion in to the urine primarily under the control of the renin-angiotensin-aldosterone system. -decreased pressure causes the juxtaglomerular cells of the kidney to secrete renin, which cleaves an inactive plasma protein, angiotensinogen, to its active form, angiotensin I. Angiotensin 1 is then converted to angiotensin II by angiotensin coverting enzyme (ACE) in the lungs. Angiotensin II stimulates the adrenal cortex to secrete aldosterone. Once BP is restored, there is a decrease drive to stimulate renin release, thus serving as negative feedback.

what is calcitonin and how does it affect bone metabolism?

it is a peptide hormone released by the parafollicular cells of the thyroid in response to high blood calcium. it promotes bone formation, lowering blood calciul levels.

How does Vitamin D affect bone metabolism?

it is activated by parathyroid hormone and like it, promotes reabsorption of bone. The reabsorption of bone in response to vitamin D actually encourages the growth of new, stronger bone, thus overcompensating for the effect of resorbing bone in the first place.

what is the job of transfer RNA (tRNA)? what does each tRNA molecule contain? What is a charged tRNA? Where is mature tRNA found? How are amino acids actiated?

it is responsible for converting the language of nucleic acids to the language of amino acids and peptides. each tRNA molecules contains a folded strand of RNA that includes a three-nucleotide anticodon. This anticodon recognizes and pairs with the appropriate codon on an mRNA molecule while in the ribosome. There are 20 amino acids in eukaryotic proteins, each of which is represented by at least one codon. To become part of a nascent polypeptide in the ribosome, amino acids are connected to a specific tRNA molecule; such tRNA molecules are said to be charged or activated with an amino acid. Mature tRNA is found in the cytoplasm. Each type of amino acid is activated by a different aminoacyl-tRNA synthetase that requires two high energy bonds from ATP, implying that the attachment of the amino acid is an energy-rich bond.

What is cholecystokinin (CCK) and what is its role in digestion?

it is secreted in response to the entry of chime (specifically, amino acids and fat in the chime) into the duodenum. This peptide hormone stimulates the release of both bile and pancreatic juices and also acts in the brain, where it promotes satiety.

what is the wobble position and what is it for?

it is the third base in the codon. It is an evolutionary development designed to protect against mutations in the coding regions of our DNA. Mutations in the wobble position tend to be called silent or degenerate, which means there is no effect on the expression of the amino acid and therefore no adverse effects on the polypeptide sequence.

What does the degeneracy of the genetic code mean?

it means that more than one codon can specify a single amino acid. Allows for mutations in DNA that do not always result in altered protein structure or function.

what is Edman degradation?

method to analyze the amino acids of small proteins. uses cleavage to sequence proteins of up to 50-70 amino acids. selectively and sequentially removes the N-terminal amino acid of the protein, which can be analyzed by mass spectroscopy

What are the sources of cholesterol? Describe the de novo synthesis of cholesterol. How is this regulated?

most cells derive their cholesterol from LDL or HDL, but some cholesterol may be synthesized de novo. De novo synthesis of cholesterol occurs in the liver and is driven by acetyl-CoA and AP. The citrate shuttle carries mitochondrial acetyl-CoA into the cytoplasm, where synthesis occurs. NADPH (from the pentose phosphate pathway) supplies reducing equivalents. Synthesis of mevalonic acid in the smooth ER is the rate-limiting step in cholesterol biosynthesis and is catalyzed by 3-hydroxy-3-methylglutaryl (HMG) CoA reductase. Cholesterol synthesis is regulated in several ways. First, increased levels of cholesterol can inhibit further synthesis by a feedback inhibition mechanism. Next, insulin promotes cholesterol synthesis. Control over de novo cholesterol synthesis is also dependent on regulation of HMG-CoA reductase gene expression in the cell.

explain the effects of pH on enzyme-catalyzed reactions

most enzymes depend on pH in order to function properly, b/c pH affects the ionization of the active site and bc changes in pH can lead to denaturation of the enzyme. human blood optimal pH=7.4 pepsin (in stomach) has optimal pH around 2. pancreatic enzymes (in small intestine) have optimal pH around 8.5

Explain oncogenes and proto-oncogenes

oncogenes-normal genes that are mutated and become cancer causing proto-oncogenes- before oncogenes are mutated, they are referred to as this

What is the path an egg takes through the female reproductive system?

one egg per month is ovulated into the peritoneal sac, which lines the abdominal cavity. It is then drawn into the Fallopian tube or oviduct, which is lined with cilia to propel the off forward. The Fallopian tubes are connected to the muscular uterus, which is the site of fetal development. The lower end of the uterus, known as the cervix, connects to the vaginal canal, where sperm are deposited during intercourse. The external female anatomy is known collectively as the vulva.

Explain helper T-cells

one of the 3 types major of T cells. also called CD4+ T-cells. Coordinate the immune response by secreting chemicals known as lymphokines. These molecules are capable of recruiting other immune cells (such as plasma cells, cytotoxic T-cells, and macrophages) and increasing their activity. The loss of these cells occurs in HIV infection, prevents the immune system from mounting an adequate response to infection. CD4+ T-cells (helper T-cells) respond to antigens presented on MHC-II molecules. Because MHC-II presents exogenous anitgens, CD4+ T cells are most effective against bacterial, fungal, and parasitic infections.

what's a kinase? what class of enzyme is it in?

one of the 6 classes of enzymes. Kinases catalyze the transfer of a phosphate group, generally from ATP, to another molecule. Transferases

Explain ligases.

one of the 6 classes of enzymes. catalyze addition or synthesis reactions, generally between large similar molecules, and often require ATP. Synthesis reactions with smaller molecules are generally accomplished by lyases. Ligases are most likely to be encountered in nucleic acid synthesis repair on Test Day.

Explain hydrolases

one of the 6 classes of enzymes. catalyze the breaking of a compound into two molecules by adding water. In common usage, many hydrolyses are named only for their substrate. Ex: one of the most common hydrolyses on the MCAT is a phosphatase, which cleaves a phosphate group from another molecule. Other hydrolases include peptidases, nucleases, and lipases, which break down proteins, nucleic acids, and lipids, respectively

What is the lock and key theory for enzyme catalysis?

opposed to the induced fit model. this is the less accepted theory of the 2. suggests that the the enzyme's active site (lock) is already in the appropriate conformation for the substrate (key) to bind. the substrate can then easily fit into the active site, like a key into a lock or a hand into a glove. no alteration of the tertiary or quaternary structure necessary

what are osteoblasts and osteoclasts?

osteoblasts = bone Builders (increase bone material) osteoclasts = bone destroyers (Chew bone) (decrease bone material)

In most organs, epithelial cells constitute the _____ (___)

parenchyma, functional parts of the organ (Ex: nephrons in the kidney are composed of epithelial cells, and hepatocytes in the liver, and acid-pouring cells of the stomach)

What is the most important controller of insulin secretion? How does this work?

plasma glucose! above a threshold of 100mg/dL or about 5.6mL glucose, insulin secretion is directly proportional to plasma glucose. For glucose to promote insulin secretion, it must not only enter the B-cell but also be metabolized, increasing intracellular ATP [ ]. Increased ATP leads to calcium release in the cell, which promotes exocytosis of preformed insulin from intracellular vesicles. Insulin secretion is also affected by signaling by other hormones such as glucagon and somatostatin.

Explain the 5'cap and 3' poly-A tail. What happens after the cap and tail are added?

posttranscriptional modifications at the end of the 5' of the hnRNA molecules, a 7-methylguanylate triphosphate cap is added. The cap is actually added during the process of transcription and is recognized by the ribosome as the binding site. It protects the mRNA from degradation in the cytoplasm. A polyadenosyl (poly-A) tail is added to the 3' end of the mRNA transcript and protects the message against rapid degradation. it is composed of adenine bases. Think of the poly-A tail as a fust for a "time bomb" for the mRNA transcript: as soon as the mRNA leaves the nucleus, it will start to get degraded from its 3' end. The longer the poly-A tail, the more time the mRNA will be able to survive before being digested in the cytoplasm. The poly-A tail also assists with export of the mature mRNA from the nucleus. At this point, when only the exons remain and the cap and tail have been added, the cell has created the mature mRNA that can now be transported into the cytoplasm for protein translation. Untranslated regions of the mRNA (UTRs) will still exist at the 5' and 3' edges of the transcript because the ribosome initiates translation at the start codon (AUG) and will end at a stop codon(UAA, UGA, UAG).

how does cell differentiation work?

primarily it is by selective transcription of the genome. Only the genes needed for that particular cell type are transcribed. Selective transcription is often related to the concept of induction, which is the ability of one group of cells to influence the fate of nearby cells. This process is mediated by chemical substances called inducers which diffuse from the organizing cells to the responsive cells. These chemicals are responsible for processes such as the guidance of neuronal axons. Induction also ensures the proximity of different types that work together within an organ.

What are oligodendrocytes (CNS) and Schwann cells (PNS)?

produce myelin around axons

What is calcitonin and what does it do? How is it stimulated?

produced by C-cells (parafollicular cells) in the thyroid gland. it decreases plasma calcium levels in 3 ways: 1. increasing calcium excretion from the kidneys 2. decreasing calcium absorption from the gut 3. increasing storage of calcium in the bone high levels of calcium in the blood stimulate secretion of calcitonin from the C-cells

what happens in the nucleolus?

rRNA (ribosomal RNA) is synthesized (ribosomes are made)

What are the effects of peptide hormones usually like? How do they travel in the bloodstream? compare both of these with steroid hormones.

rapid but short-lived b/c these hormones act though second messenger cascades, which are transient. It is quicker to turn them on and off compared w/ steroid hormones, but their effects don't last w/o relatively constant stimulation. Because peptides are generally water-soluble, they can travel freely in the bloodstream and usually don't require carriers. This is in stark contrast to steroid hormones, which are lipid-soluble.

Explain passive immunity

results from the transfer of antibodies to an individual. The immunity is transient because only the antibodies and not the plasma cells that produce them are given to the individual. Natural examples are the transfer of antibodies across the placenta during pregnancy to protect the fetus and the transfer of antibodies from a mother to her nursing infant through breast milk. In some cases of exposure, such as to the rabies virus or tetanus, intravenous immunoglobulin may be given to prevent the pathogen from spreading.

explain mixed inhibition

results when an inhibitor can bind to either the enzyme or the enzyme-substrate complex, but has a different affinity for each. (if it had the same affinity for both it would be a noncompetitive inhibitor) bind at an allosteric site, not the active site. alters Km. if the inhibitor preferentially binds to the enzyme, it inc Km(lowers affinity). if it binds to the ES complex, it lowers Km(inc affinity)

What are the two major functions of the thyroid? How does it do these things?

setting basal metabolic rate and calcium homeostasis it mediates the basal metabolic rate by releasing triiodothyronine (T3) and thyroxine (T4) it mediates calcium homoeostasis through the release of calcitonin

what are hormones?

signaling molecules that are secreted directly into the bloodstream to distant target tissues.

What are heuristics? What is the availability heuristic? What is the representativeness heuristic?

simplified principles used to make decisions; rules of thumb the availability heuristic is used when we try to decide how likely something is. When we use this, we make our decisions based on how easily similar instances can be imagined. the representativeness heuristic involves categorizing items on the basis of whether they fit the prototypical, stereotypical, or representative image of the category. Ex: consider a standard coin that is flipped 10 times in a row and lands on heads every time. What is the probability of the coin landing on heads the next time? Mathematically, it has to be 50%, but most individuals will either overestimate the probability with the logic that the number of heads and tails must "even out."

What are viroids?

single strandedcircular RNA molecules that have no surrounding capsids and infect plants.

What is the body's first line of defense? How does it protect the body?

skin. it provides a physical barrier between the outside world and our internal organs, preventing most bacterial, viruses, fungi, and parasites from entering the body. Additionally, antibacterial enzymes called defensins can be found on the skin. Sweat also has antimicrobial properties.

What is the difference between steroids and steroid hormones?

steroid refers to a group defined by a particular chemical structure. steroid hormones are steroids that act as hormones, meaning that they are secreted by endocrine glands into the bloodstream and then travel on protein carriers to distant sites, where they can bind to specific high-affinity receptors and alter gene expression levels. - Steroid hormones are potent biological signals that regulate gene expression and metabolism, affecting a wide variety of biological systems even at low [ ]s.

Explain amphetamines

stimulants cause increased arousal by increasing release of dopamine, norepi, and serotonin and the synapse and decreasing their reuptake. This increases arousal and causes a reduction in appetite and decreased need for sleep. Physiological effects include an increase in HR and BP. Psychological effects include euphoria, hypervigilance, anxiety, delusions of grandeur, and paranoia.

What does collagen do? What is it?

structural protein. has a characteristic trihelical fiber and makes up most of the extracellular matrix of connective tissue. It is found throughout the body and is important in providing strength and flexibility

what does elastin do? What is it?

structural protein. it is an important component of the extracellular matrix of connective tissue. Its primary role is to stretch and then recoil like a spring, which restores the original shape of the tissue

How does the hypothalamus control the anterior pituitary? Specifics.

the hypophyseal portal system (a blood vessel system that directly connects the hypothalamus with the ant pit). Hormones released from the hypothalamus travel directly to the ant pit and cannot be found in appreciable [ ]s in the systemic circulation. Note: hypophysis is an alternative term for the pituitary Once hormones have been released from the hypothalamus into this portal bloodstream, they travel down the pituitary stalk and bind to receptors in the ant pit, stimulating the release of other hormones

What is the last point in the meiotic cycle in which the cell has a diploid number of chromosomes?

telophase I

What do tendons and ligaments do?

tendons attach muscle to bone and ligaments hold bones together at joints.

Explain divided attention

the ability to perform multiple tasks at the same time. Most new or complex tasks require undivided attention, and utilize controlled (effortful) processing. In contrast, familiar or routine actions can be performed with automatic processing, which permits the brain to focus on other tasks with divided attention.

What is the lytic cycle of a virus?

the bacteriophage maximizes the use of the cell's machinery with little regard for the survival of the host cell. Viruses in the lytic phase are termed virulent.

What is the menstrual cycle? What are the 4 events?

the endometrial lining growing and shedding the follicular phase, ovulation, the luteal phase, and menstruation

How does prolactin affect lactation?

the high levels of estrogen and progesterone allow for the development of milk ducts in preparation for lactation, but it is not until shortly after the expulsion of the placenta, when estrogen, progesterone, and dopamine levels drop, that the block on milk production is removed and lactation actually begins - milk ejection occurs when the newborn infant latches onto the breast. Nipple stimulation causes activation of the hypothalamus, resulting in two different reactions. First, oxytocin is released from the post pit, resulting in contraction of the smooth muscle of the breast and ejection of milk through the nipple. Second, the hypothalamus stops releasing dopamine ont o the anterior pit, which allows prolactin release, causing the production of milk and regulation of the milk supply

Explain active immunity.

the immune system is stimulated to produce antibodies against a specific pathogen. The means by which we are exposed to this pathogen may either be natural or artificial. Through natural exposure, antibodies are generated by B-cells once an individual becomes infected. Artificial exposure (through vaccines) also results in the production of antibodies; however, the individual never experiences a true infection. Instead, he or she receives an injection or intranasal spray containing an antigen that will activate B-cells to produce antibodies to fight the specific infection. The antigen may be a weakened or killed form of the microbe, or it may be a part of the microbe's protein structure.

explain the role of pyruvate kinase in glycolysis.

the last enzyme in aerobic glycolysis, pyruvate kinase catalyzes a substrate-level phosphorylation of ADP using the high-energy substrate phosphoenolpyruvate (PEP). Pyruvate kinase is activated by fructose 1,6-bisphosphate from the PFK-1 reaction. This is referred to as feed-forward activation, meaning that the product of an earlier reaction of glycolysis (fructose 1,6-bisphosphate) stimulates, or prepares, a later reaction in glycolysis (by activating pyruvate kinase).

what is inbreeding depression? What about outbreeding (outcrossing)?

the loss of genetic variation may cause reduced fitness of the population is inbreeding depression. on the opposite end of the spectrum, outbreeding is the introduction of unrelated individuals into a breeding group.

What is needed for a basic DNA sequencing reaction?

the main players from replication including a template DNA, primers, an appropriate DNA polymerase, and all 4 deoxyribonucleotide triphosphates. In addition, a modified base called a dideoxyribonucleotide is added in lower [ ]s. Dideoxyribonucleotides (ddATP, ddGTP, ddCTP, ddTTP) contain a hydrogen C-3', rather than a hydroxyl group; thus, once one of these modified bases has been incorporated, the polymerase can no longer add to the chain. Eventually, the sample will contain many fragments (as many as the number of nucleotides in the desired sequence), each one of which terminates with one of the modified bases. These fragments are then separated by size using gel electrophoresis. The last base for each fragment can be read, and because gel electrophoresis separates the strands by size, the bases can easily be read in order.

Explain the role of resting skeletal muscle after a meal and in the fasting state.

the major fuels of skeletal muscle are glucose and fatty acids. because of its enormous bulk, skeletal muscle is the body's major consumer of fuel. after a meal, insulin promotes glucose uptake in skeletal muscle, which replenishes glycogen stores and amino acids used for protein synthesis. both excess glucose and amino acids can also be oxidized for energy. in the fasting state, resting muscle uses fatty acids derived from free fatty acids circulating in the bloodstream. ketone bodies may also be used if the fasting state is prolonged.

What are terpenes? What are their physical properties? How are they grouped?

the metabolic precursors to steroids and other lipid signaling molecules. They are a class of lipids built from isoprene (C5H8) moieties and share a common structural patter with carbons grouped in multiples of 5. They are produced mainly by plants and also by some insects. They are generally strong scented. Grouped according to the number of isoprene units present; a single terpene units contains two isoprene units.

Explain the oral cavity's role in digestion.

the oral cavity plays a role in mechanical and chemical digestion of food. Mechanical digestion in the mouth involves the breaking up of large food particles into smaller particles using the teeth, tongue, and lips. This process is called mastication. Chewing helps to increase the surface area-to-volume ratio of the food, creating more surface area for enzymatic digestion as it passes through the gut tube. It also moderates the size of food particles entering the lumen of the alimentary canal; food particles that are too large create an obstruction risk in the tract. Chemical digestion is the breakdown of chemical bonds in the macromolecules that make up food. This relies on enzymes from saliva produced by the three pairs of salivary glands. Saliva also aids mechanical digestion by moistening and lubricating food. The salivary glands, like all glands of the digestive tract, are innervated by the parasympathetic NS. The presence of food in the oral cavity triggers a neural circuit that ultimately leads of increased parasympathetic stimulation of these glands. Salivation can also be triggered by signals that food is near, such as smell or sight. Saliva contains salivary amylase (aka ptyalin) and lipase. Salivary amylase is capable of hydrolyzing starch into smaller sugars (maltose and dextrins) while lipase catalyzes the hydrolysis of lipids. The amount of chemical digestion that occurs in the mouth is minimal, though, because the food does not stay in the mouth for long. Our muscular tongue forms the food into a bolus, which is forced back to the pharynx and swallowed.

What are the functions of the pancreas?

the pancreas has both exocrine and endocrine functions. Exocrine tissues secrete substances directly into ducts; the pancreas produces a number of digestive enzymes. Endocrine: small clusters of hormone-producing cells are group together into islets of Langerhans throughout the pancreas. Islets contain 3 distinct types of cells: alpha (a), beta (B), and delta (d) cells. Each cell type secretes a different hormone: alpha cells- glucogon beta cells- insulin delta cells- somatostatin

What are the accessory organs of the digestive system?

the pancreas, liver, and gallbladder.

What is the pharynx? Explain the pharynx's role in digestion

the pharynx is the cavity that leads from the mouth and posterior nasal cavity to the esophagus. The pharynx connects not only to the esophagus, but also the the larynx, which is a part of the respiratory tract. The pharynx can be divided into three parts: the nasopharynx (behind the nasal cavity), the oropharynx (at the back of the mouth), and the laryngopharynx (above the vocal cords). Food is prevented from entering the larynx during swallowing by the epiglottis(trap door), a cartilaginous structure that folds down to cover the laryngeal inlet. Failure of this mechanism can lead to aspiration of food and choking. The chemical digestion of carbohydrates and fats is initiated in the mouth, but not proteins.

Where does nutrient, gas, and waste exchange occur in the fetus? What type of transport is used for nutrients and waste products? Explain fetal hemoglobin (what does this do and how does it help?).

the placenta. Fetal lungs do not function until birth. It is crucial that maternal and fetal blood to not mix b/c they may be different blood types. The simplest method to move nutrients and waste products is by diffusion, the preferred method for water, glucose, amino acids, and inorganic salts. -Diffusion requires a gradient, which implies that there is a higher partial pressure of oxygen in maternal blood that in fetal blood. -To further enhance the transfer of oxygen from maternal to fetal circulation, fetal blood cells contain fetal hemoglobin (HbF), which has a greater affinity for oxygen than maternal (adult) hemoglobin. This also assists with the transfer and retention of oxygen into the fetal circularity system. Waste material and carbon dioxide move in the opposite direction.

What are brush border enzymes? Where are they located and what is their function?

the presence of chime in the duodenum causes the release of brush-border enzymes like disaccharides (maltase, isomaltase, lactase, and sucrase) and peptidases (including dipeptidase). Brush border enzymes are present on the luminal surface of cells lining the duodenum and break down dimers and trimers of biomolecules into absorbable monomers.

explain the metabolic role of active skeletal muscle

the primary fuel used to support muscle contraction depends on the magnitude and duration of exercise as well as the major fibers involved. a very short-lived source of energy (2-7 seconds) comes from creatine phosphate, which transfers a phosphate group to ADP to form ATP. Skeletal muscle has stores of both glycogen and some triacylglycerols. Blood glucose and free fatty acids may also be used. Short bursts of high-intensity exercise are also supported by anaerobic glycolysis drawing on stored muscle glycogen. During moderately high-intensity, continuous exercise, oxidation of glucose and fatty acids are both important, but after 1 to 3 hours of continuous exercise at this level, muscle glycogen stores become depleted, and the intensity of exercise declines to a rate that can be supported by oxidation of fatty acids.

what is myosin? What does it do?

the primary motor protein that interacts with actin. In addition to its role as the thick filament in a myofibril, myosin can be involved in cellular transport. Each myosin subunit has a single head and neck; movement at the neck is responsible for the power stroke of sarcomere contraction

What is a simple twitch response?

the response of a single muscle fiber to a brief stimulus at or above threshold. It consists of a latent period, contraction period, and relaxation period. The latent period is the time between reaching threshold and the onset of contraction. It is during this time that the action potential spreads along the muscle and allows for calcium to be released from the sarcoplasmic reticulum. The muscle then contracts, and, assuming calcium is cleared from the sarcoplasm, it then relaxes.

Explain pregnancy, what happens if fertilization has occurred.

the resulting zygote will develop into a blastocyst that will implant in the uterine lining and secrete human chorionic gonadotropin (hCG), an analog of LH-looks very similar to LH and can stimulate LH receptors. This maintains the corpus lute. hCG is critical during first trimester development bc the estrogen and progesterone secreted by the corpus lutem keep the uterine lining in place. By the second trimester, hCG levels decline because the placenta has grown to a sufficient size to secrete enough progesterone and estrogen by itself. the high levels of estrogen and progesterone continue to serve as neg feedback on GnRH secretion.

what is glycogenesis? What is the rate limiting enzyme? What stimulates and inhibits it?

the synthesis of glycogen granules. glycogen synthase is the rate-limiting enzyme of glycogen synthesis and forms the a-1,4 glycosidic bond found in the linear glucose chains of the granule. It is stimulated by glucose 6-phosphate and insulin. it is inhibited by epinephrine and glucagon through a protein kinase cascade that phosphorylates and inactivates the enzyme.

what is confirmation bias?

the tendency to interpret new evidence as confirmation of one's existing beliefs or theories.

what is functional fixedness?

the tendency to think of things only in terms of their usual functions the inability to consider how to use an object in a nontraditional manner.

In males, the primitive gonads develop into what? What are the two functional components of this?

the testes. seminiferous tubules and the interstitial cells of Leydig

Explain oogenesis.

there is no underlying supply of stem cells analogous to spermatogonia in females, all of the oogonia a woman will ever have are formed during fetal development. By birth, all of the oogonia have already undergone DNA replication and are considered primary oocytes. These cells are 2n and are arrested in prophase 1. Once a woman reaches menarche, one primary oocyte per month will complete meiosis I, producing a secondary oocyte and a polar body. The division is characterized by unequal cytokinesis, which distributes ample cytoplasm to one daughter cell (the oocyte) and nearly none to the other (the polar body). The polar body generally does not divide any further and will never produce functional gametes. The second oocyte remains arrested in metaphase II and does not complete the remainder of meiosis II unless fertilization occurs. Meiosis II is triggered when a sperm cell penetrates the corona radiate and zona pellucida (two layers surrounding the oocyte) with the help of acrosomal enzymes. The secondary oocyte undergoes the second meiotic division to split into a mature ovum and another polar body, which will eventually be broken down. A mature ovum is a very large cell consisting of large quantities of cytoplasm and organelles. The ovum contributes nearly everything to the zygote (half of DNA, all of cytoplasm, organelles, and RNA) whereas sperm only contribute half the DNA. Upon completion of meiosis II, the haploid pronuclei of the sperm and the ovum join creating a diploid zygote.;

Explain step 4 of the citric acid cycle, succinyl-CoA and CO2 formation

these reactions are carried out by the alpha-ketoglutarate dehydrogenase complex, which is similar in mechanism, cofactors, and coenzymes to the pyruvate dehydrogenase (PDH) complex. In the formation of succinyl-CoA, alpha-ketoglutarte and CoA come together and produce a molecule of carbon dioxide. This carbon dioxide represents the second and last carbon lost from the cycle. Reducing NAD+ produces another NADH.

What are macrophages? What are they derived from? What do they have the potential to do? What are some examples? How are they activated? Explain what happens after the macrophages are activated.

they are a type of agranulocyte, residing within the tissues. These cells derive from blood-borne monocytes and can become a resident population within a tissue (becoming a permanent, rater than transient, cell group in the tissue). Ex: migroglia in the CNS, Langerhans cells in the skin, osteoclasts in the bone. When a bacterial invader enters a tissue, the macrophages become activated. The activated macrophages does three things. First, it phagocytizes the invader through endocytosis. Then, it digests the invader using enzymes. Finally, it presents little pieces of the invader (mostly peptides) to other cells using a protein called major histocompatibility complex (MHC). MHC binds to a pathogenic peptide (antigen) and carries it to the cell surface, where it can be recognized by cells of the adaptive immune system. In addition, macrophages release cytokines, chemical substances that stimulate inflammation and recruit additional immune cells to the area. Macrophages and dendritic cells also have special receptors known as pattern recognition receptors (PRR), the best-described of which are toll-like receptors (TLR). PRRs are able to recognize the category of the invader (bacterium, virus, fungus, or parasite). This allows for the production of appropriate cytokines to recruit the right type of immune cells; each immune cell has different weapons that can target particular groups of pathogens.

what are lymphocytes? what are they important for? Where do they mature?

they are agranulocytes, a class of white blood cell they are important in the specific immune response, the body's targeted fight against particular pathogens such as viruses and bacteria. Some lymphocytes act as primary responders against an infection while others function to maintain a long-term memory bank of pathogen recognition. Many vaccines work by training these cells. lymphocyte maturation takes place in one of three locations. Lymphocytes that mature in the bone marrow are B-cells. Lymphocytes that mature in the thymus are T-cells. B-cells generate antibodies. T-cells kill virally infected cells and activate other immune cells.

What are monocytes and what do they do?

they are agranulocytes, a class of white blood cell they phagocytize foreign matter such as bacteria. Most organs of the body contain a collection of these phagocytic cells; once they leave the bloodstream and enter an organ, monocytes are renamed macrophages. Each organ's macrophage population may have a specific name as well. In the CNS they are called microglia. In the skin they are called Langerhans cells. In the bone they are called osteoclasts.

What are the roles of 1,3-bisphosphoglycerate (1,3-BPG) and phosphoenolpyruvate (PEP) in glycolysis?

they are high energy intermediates used to generate ATP by substrate-level phosphorylation. This is the only ATP gained in anaerobic respiration.

During interphase, why are individual chromosomes not visible with light microsopy? Why is this happening? What is form are the chromosomes in?

they are in a less condensed form known as chromatin. This is because the DNA must be available to RNA polymerase so that genes can be transcribed. During mitosis, it is preferable to condense the DNA into tightly coiled chromosomes to avoid losing any genetic material during cell division

what are alpha-linolenic acid and linolenic acid? why are they importat?

they are polyunsaturated fatty acids that are important in maintaining cell membrane fluidity, which is critical for proper functioning of the cell.

what are ion channels? what are the three main groups? what type of diffusion do they each permit?

they are proteins that create specific pathways for charged molecules. undated channels, voltage-gated channels, ligand-gated channels they all three permit facilitated diffusion of charged particles

What are peptidases and what is their role in digestion? What about aminopeptidase and dipeptidases?

they break down proteins (or peptides, as the name implies). Aminopeptidase is a peptidase secreted by glands in the duodenum that removes the N-terminal amino acid from a peptide. Dipeptidases cleave the peptide bonds of dipeptides to release free amino acids. Unlike carbohydrates, which must be broken down into monosaccharides for absorption, proteins can be broken down into di and even tripeptides and can be absorbed across the small intestine wall.

How do the lungs fill with air?

they can't fill passively, they require skeletal muscle to generate the negative pressure for expansion. The most important of these muscles is the diaphragm, a thin, muscular structure that divides the thoracic cavity from the abdominal cavity. The diaphragm is under somatic control, even though breathing itself is under autonomic control.

What is the pyloric glands role in digestion?

they contain G cells that secrete gastrin, a peptide hormone. Gastrin induces the parietal cells in the stomach to secrete more HCl and signals the stomach to contract, mixing its contents. The digestion of solid food in the stomach results in an acidic, semifluid mixture known as chime. The combined mechanical and chemical digestive activities of the stomach result in a significant increase in the surface area of the now unrecognizable food particles, so when the chime reaches the small intestine, the absorption of nutrients from it can be maximized. There are a few substances that are absorbed directly from the stomach (such as alcohol and aspirin), but the stomach is mainly an organ of digestion.

When fatty acids are metabolized, what first happens? Then what happens? How does this differ between short, medium, long, and very long chain fatty acids?

they first become activated by attachment to CoA, which is catalyzed by fatty acil CoA synthetase. The product is generically referred to as a fatty acyl-CoA or acyl-CoA. Then, short chain fatty acids (two to four carbons) and medium chain fatty acids (six to twelve carbons) diffuse freely into mitochondria, where they are oxidized. In contrast, while long chain fatty acids (14 to 20 carbons) are also oxidized in the mitochondria, they require transport via a carnitine shuttle. Carnitine acyltransferase I is the rate limiting enzyme of fatty acid oxidation. Very long chain fatty acids (20+ carbons) are oxidized elsewhere in the cell.

What do the seminiferous tubules do? What are they?

they one of two functional components of the testis. They produce sperm.

What is a frameshift mutation? What are some examples? Are they more or less serious than point mutations?

they shift the reading frame of the mRNA sequencey insertion or deletion. more serious effects than point mutations.

What are glycerophospholipids? How are they named? How does this differ?

those phospholipids that specifically contain a glycerol backbone bonded by ester linkages to two fatty acids and by a phosphodiester linkage to a highly polar head group Because the head group determines the membrane surface properties, glycerophospholipids are named according to their head group. Ex: phosphatidylcholine is the name of a glycerophospholipid with a choline head group. The head groups can be positively charged, negatively charged, or neutral. The membrane surface properties of these molecules make them very important to cell recognition, signaling, and binding. Within each subtype, the fatty acid chains can vary in length and saturation, resulting in an astounding variety of function.

Explain how the citric acid cycle is regulated upstream from its starting point.

through phosphorylation of PDH, which is facilitated by the enzyme pyruvate dehydrogenase kinase. Thus, whenever levels of ATP rise, phorphorylating PDH inhibits acetyl-CoA production. Conversely, the pyruvate dehydrogenase complex is reactivated by the enzyme pyruvate dehydrogenase phosphatase in response to high levels of ADP. By removing a phosphate from PDH, pyruvate dehydrogenase phosphatase is able to reactivate acetyl-CoA production. Acetyl-CoA also has a negative feedback effect on its own production. When using alternative fuel sources such as fats, the acetyl-CoA production is sufficient to make it redundant to continue producing acetyl-CoA from carbohydrate metabolism- that's part of why eating a high fat meal fills you up so quickly! ATP and NADH, as markers of the cell being satisfied energetically, also inhibit PDH

What is thymosin and what secretes it? what does it do? what happens to the levels of thymosin throughout ones life?

thymosin is secreted by the thymus (located behind the sternum). important for proper T-cell development and differentiation. The thymus atrophies by adulthood, and thymosin levels drop accordingly.

What is the small intestine lined with? Specific parts of the endothelial cells.

villi, which are small, fingerlike projections from the epithelial lining. Each villus has many microvilli, drastically increasing the surface area available for absorption. In addition, at the middle of each villus there is a capillary bed for the absorption of water-soluble nutrients and a lacteal, a lymphatic channel that takes up fats for transport into the lymphatic system.

Explain interferons.

to protect against viruses, cells that have been infected with viruses produce interferons, proteins that prevent viral replication and dispersion. Interferons cause nearby cells to decrease production of both viral and cellular proteins. They also decrease the permeability of these cells, making it harder for a virus to infect them. In addition, interferons upregulate MHC class I and class II molecules, resulting in increased antigen presentation and better detection of the infected cells by the immune system. Interferons are responsible for many "flu-like" symptoms that occur during viral infection, including malaise, tiredness, muscle soreness, and fever.

What is the job of connective tissue? Connective cells contribute to the ______, meaning what? What are some examples of connective tissues? Most connective tissues produce and secrete materials such as collagen and elastin to form what?

to support the body and provide a framework for the epithelial cells to carry out their functions. stroma-support structure (as opposed to parenchyma) examples: bone, cartilage, tendons, ligaments, adipose tissue, blood the extracellular matrix

What are transcription factors? Explain their structure.

transcription-activating proteins that search the DNA looking for specific DNA-binding motifs. They tend to have two recognizable domains: a DNA binding domain and an activation domain The DNA binding domain binds to a specific nucleotide sequence in the promoter region or to a DNA response element (a sequence of DNA that binds only to specific transcription factors) to help in the recruitment of transcriptional machinery. The activation domain allows for the binding of several transcription factors and other important regulatory proteins, such as RNA polymerase and histone acetylases, which function in the remodeling of the chromatin structure.

What is a transposon?

transposons - mobile elements. pieces of DNA that hop around in the genome. Can land in an area and knockout a gene.

Which ion channels are responsible for maintaining the resting membrane potential?

ungated channels. ungated or "leak" channels permit limited free flow of ions, while the sodium-potassium pump is also active and corrects for this leakage.

what is the Jacob-Monod model?

used to describe the structure and function of operons. In this model, operons contain structural genes, an operator site, a promoter site, and a regulator gene. The structural gene codes for the protein of interest. Upstream of the structural gene is the operator site, a non-transcribable region of DNA that is capable of binding a repressor protein. Further upstream is the promoter site, which is similar in function to promoters in eukaryotes: it provides a place for RNA polymerase to bind. Furthest upstream is the regulator gene, which codes for a protein known as the repressor. There are two types of operons: inducible systems and repressible systems.

What is the Michaelis-Menten equation?

v = (vmax [S])/(Km + [S])

How does the DNA get unwound? What happens to it once it's opened? Explain this process of the unwinding DNA

via enzyme Helicase generates two single-stranded template strands ahead of the polymerase. Once opened, the unpaired strands of DNA are very sticky, in a molecular sense. The free purines and pyrimidines seek out other molecules with which to hydrogen bond. Proteins are therefore required to hold the strands apart. single-stranded DNA-binding proteins will bind to the unraveled strand, preventing both the reassociation of the DNA strands and the degradation of DNA by nucleases. As the helixes unwinds the DNA, it will cause positive supercoiling that strains the DNA helix. Supercoiling is a wrapping of DNA on itself as its helical structure is pushed ever further toward the telomeres during replication; picture an old-fashioned telephone cord that's become tangled on itself. To alleviate this torsional stress and reduce the risk of strand breakage, DNA topoisomerase introduce negative supercoils. They do so by working ahead of helices, nicking one or both strands, allowing relaxation of the torsional pressure, and then resealing the cut strands.

How do RBCs get energy? Is this the only way they can get energy? Why?

via glycolysis. yes it is the only way because they don't have mitochondria, which are required for the citric acid cycle, electron transport chain, oxidative phosphorylation, and fatty acid metabolism (B-oxidation).

what is the discomformation principle?

when evidence obtained from testing demonstrates that the solution does not work

what is the solvation layer?

whenever a solute dissolves in a solvent, the nearby solvent molecules form a solvation layer around that solute. From an enthalpy standpoint, even hydrocarbons are more stable in aqueous solution than in organic ones. However when a hydrophobic side chain is placed in the aqueous solution, the water molecules in the solvation layer cannot form hydrogen bonds with the side chain. This forces nearby water molecules to rearrange themselves into specific arrangements to maximize H bonding-which means a negative change in entropy, delta S. This makes the overall process non spontaneous, delta G > 0. On the other hand, putting hydrophilic residues on the exterior of the protein allows the nearby water molecules more latitude in their positioning, increasing their entropy and making the overall solution process spontaneous. Thus, by moving hydrophobic residues away from the water molecules and hydrophilic residues toward water molecules, a protein achieves maximum stability.

Explain the leukocyte.

white blood cells. usually less than 1% of total blood volume. WBCs are a part of the immune system, acting as our defenders against pathogens, foreign cells, cancer, and other materials not recognized as self.


Ensembles d'études connexes

Final Review: Managerial Accounting

View Set

Unit 2: Study Guide Bio121 Johnstone

View Set

Lesson 3 , Philip ii of Macedonia

View Set

fundamentals of electricity A.C.

View Set

ACSM Exercise NEW guidelines 2018

View Set

Mass Comm Law Quiz 7: chap 11 and 12

View Set